MCQ Orthopedics

Download as pdf or txt
Download as pdf or txt
You are on page 1of 433

The Fourth Floor

MCQs in
Orthopaedics

Edited By
Mohamed B. RASHED, FRCSI
Hadba-Khadra Hospital

1
Tripoli-Libya 2015

ISBN 978-9959-1-0886-9

Libyan Book House/ Benghazi-Libya

Tel:00218-61-9097074,9096379,9090509

Fax; 00218-61-9097073

Emal; [email protected]

2
N.o INDEX: Page

1 Introduction 5

2 Forward 9

3 General Orthopaedics MCQs 11

4 Trauma MCQs 121

5 Paediatrics Orthopaedics MCQs 169

6 Bone Tumors MCQs 189

7 Problem Solving MCQ 211

8 General Orthopaedics EMQs 275

9 Biomechanics& Biomaterials MCQ 301

10 Sports Medicine MCQs 331

11 Libyan Board of Orthopaedics MCQ Entry Exam 351

12 Libyan Board of Orthopaedics MCQ Part I 359

13 Libyan Board of Orthopaedics MCQ Part II (A) 377

14 Libyan Board of Orthopaedics MCQ Part II ( B) 397

15 SI COT Dilpoma Exam 427

16 References 432

3
4
INTRODUCTION:

Multiple choice questions have evolved from a stage of studied dis-


dain to grudging acceptance to a stage where they have become ubiq-
uitous. Their value in objective testing of the student’s grasp of the
subject has been widely acknowledged by academicians the world
over.

So it is not surprising to see that they are now so entrenched in


the system that the Under-graduate medical student becomes famil-
iar with them from his first year itself and is rid of them only after his
super-specialty days are over. As it has always been my endeavour to
help students learn the fascinating subject of Orthopaedics from all
angles. I scoured the various books available for MCQs in Orthopae-
dics, but the dearth of a single, authentic question bank for MCQs was
so glaring that it prodded me to bring out this book. I hope, will ad-
dress the long-felt need for an authentic, researched book on MCQs
for Orthopaedics.

MCQs included in this book are of similar modalities, to the ones


used for the Libyan Board Orthopaedic Examination of the entrance
as well as the first and second exams. This book is aimed to satisfy the
needs of not only the Undergraduates preparing for their Orthopae-
dic Paper but also the Post-graduates who are preparing for their ex-
ams. The purpose of this book would have been achieved if it kindles
in the students an abiding interest in orthopedics

A much greater depth and breadth of knowledge are needed to be


successful. There is no easy preparation for the exam the books have
to be read. To re-emphasis advice in the book the MCQ/EMI books

5
should be used as a last minute test and boost of confidence rather
than as a major learning tool. This is a useful test for candidates to re-
fine their MCQ techniques in a less hostile environment than the real
exam and also gives objective feedback about level of knowledge and
need for further study.

 The multiple choice books do not cover the syllabus in sufficient


depth for the different orthopaedic exams. Also the style of question
and level of knowledge in these books do not always match that of
the real exam and may give candidates a false sense of security. Some
questions are poorly constructed. It is easy to construct an MCQ but
very difficult and time consuming to construct a good quality MCQ
question that will blue print the syllabus appropriately, differentiate
good quality from poor quality candidates and be educationally valid.

One different type of gamble mentioned by several candidates was


to go through thousands or so MCQs on orthobullets as a means of
learning the syllabus for any exam. This was in preference to formal-
ly reading a mixture of orthopaedic textbook to acquire the required
knowledge If needed candidates could occasionally supplement their
orthopaedic knowledge from a textbook but this would usually be the
exception rather than the rule. I also think this is a bit of a risky strategy
and remain unsure how successful it would be.

This book will help the orthopaedic surgeon preparing for the
written part of the examination to be ready to face the task ahead.
The MCQs and Enhanced MCQ (EMQs) appear in the same format as
the examination and cover the syllabus topics. Divided into differ-
ent specialties, including general orthopaedics, trauma, paediatric
orthopaedics and bone tumors, this book is ideal for use alongside
a revision plan. The questions have answers to approach the topic
correctly. It will be ideal preparation for the Libyan Board different

6
examinations as well as being helpful for other postgraduate ortho-
paedic exams.

A MESSAGE FROM THE SOLE AND THE HEART

The candidate who is reading this book is living with desire to de-
vote his life to practice orthopaedics in this country. I feel responsible
to support him in his venture, thus presenting this book permitting
him to take a shot not only in Orthopaedics questions but also other
subjects.

The selection of the MCQs are not based upon intelligence and
knowledge but based on revision of recent topics and proper planning
so keep revising. Please do not count the weeks or days before the
exam, let the days count.

Prof. M.B. RASHED

7
8
FORWARD

It is a pleasure to introduce to our


young doctors seeking a promising
future in the field of trauma and or-
thopaedics a comprehensive revision
manual which should prove to be a
practical guide to their preparation
for their future Board evaluations.
Such manual should also be a practical
guidetothosecandidatesundertraining
who will confront several issues during
their daily work in casualty and ortho-
paedic units which may demand appropriate decision mak-
ings consistent with the appropriate basic scientific principles.
May I convey my appreciation to prof. Rashed for this out-
standing work which should prove an asset to our efforts
in building our trauma and orthopaedic services in Libya

Prof.M.Lenghi
M.Ch.(orth.),FRCS(Eng.)
Tripoli- LIBYA

9
10
General Orthopaedics MCQs:

1- bone growth is influenced maximally by ?


a- estrogen b- thyroxine
c- growth hormone d- testesterone
2 - Albert schonberg disease is also called as ?
a – osteoporosis b- osteodystrophy
c- osteopetrosis d- osteitis punctata
3 - Trident hand is seen in
a- achondroplasia b- scurvy
c- mucopolysaccaridosis d- none of the above
4 - Rocker bottom foot is NOT seen in?
a- congenital vertical talus b- CTEV over correction
c- grice procedure d- Diabetic foot
5 - ganglion is most commonly seen over ?
a- dorsal aspect of wrist b- volar aspect of wrist
c- over forehead d- dorsum of the tongue
6 - calcitonin is secreted by ?
a- thyroid gland b- parathyroid gland
c- adrenal gland d- ovaries
7 - all of the following can cause osteoporosis except
a- hyperparathyroidism b- steroids use
c- fluorosis d- thyrotoxicosis
8 - Rugger jersey spine is seen in ?

11
a- fluorosis b- achondroplasia
c- renal osteodystrophy d- marfan’s syndrome
9 - spina ventosa is seen in ?
a- tuberculosis dactylitis b- meningo myelocele
c- malignant melanoma d- all of the above
10 - Osgood schlatters disease affects ?
a- upper tibia b- lower tibia
c- distal femur d- proximal femur
11 - stress fracture is most commonly seen in ?
a- 2nd / 3rd metatarsal bone b- talus
c- calcaneum d- metacarpals
12 - definitive diagnosis for blow out fracture of orbit is by ?
a- CSF rhinorrhea b- loss of vision
c- tear drop sign d- fracture of the sphenoid
13 - Which of the following is the fracture of atlas verte-
brae?
a- Jefferson’s fracture b- Bennet’s fracture
c- Essex loprestti frx d- Green stick fracture
14 - Bankart’s lesion involves the of the glenoid labrum ?
a- anterior lip b- superior lip
c- anterosuperior lip d- anteroinferior lip
15 - Which bone tumor occurs in the epiphysis ?
a- osteoclastoma b- Ewing’s sarcoma
c- chondromyxoid fibroma d- osteosarcoma
16 - Paget’s disease of bone
(a) May present with neurological complications or a pathological fracture
(b) Causes an increase in both serum calcium and phosphate

12
(c) Causes an decrease in serum alkaline phosphatase
(d) Malignant change occurs in 25% of patients
(e) The commonest malignant tumour in patients with Paget’s dis-
ease is a chondrosarcoma
17. Regarding osteomalacia all are false except;
(a) Is due to vitamin A deficiency
(b) Can cause a distal myopathy
(c) May present with pseudo-fractures
(d) Serum calcium is increased
(e) A bone biopsy would show an increase in mineralised osteoid
18. Perthe’s disease all are true except;
(a) Usually presents before 10 years of age
(b) Is due to avascular necrosis of the distal femoral epiphysis
(c) Is more common in boys
(d) Plain x-ray may show the capital femoral epiphysis to be smaller,
denser and flatter
(e) May require surgical containment with a subtrochanteric oste-
otomy
19. Regarding upper limb peripheral nerve injuries all are
true except;
(a) Injury to the median nerve results in a wrist drop
(b) Injury to the median nerve results in loss of sensation over the
palmar aspect of the index finger
(c) Injury to the radial nerve results in loss of sensation in the ana-
tomical snuffbox
(d) Injury to the ulnar nerve results in a claw hand
(e) Injury to the ulnar nerve results in loss of sensation over the little
finger

13
20. Regarding osteomyelitis, all are true except;
(a) Is most commonly due to a staphylococcus aureus infection
(b) Can be due to salmonella infection in patients with sickle cell
anaemia
(c) Infection usually involves the metaphysis of long bones
(d) It may cause increase or decrease of limb length
(e) New bone forming beneath the periosteum is known a the se-
questrum
21. Regarding bone metastases all are true except;
(a) Less than 5% of patients with malignant disease develop bone
metastases
(b) 10% of patients with bone metastases develop a pathological
fracture
(c) Breast cancer is the commonest cause of bone metastases in
women
(d) Isotope scan can help early in the disease process
(e) Prostate cancer can cause osteosclerotic lesions
22. Regarding osteoid osteoma all are true except;
(a) It is a premalignant bone tumour
(b) The femur and tibia are the commonest bones involved
(c) Presents with severe pain that is typically relieved by aspirin
(d) Radiologically appears as a radiolucent lesion surrounded by
dense bone
(e) Local excision or currettage is curative
23. Regarding osteosarcomas all are true except;
(a) Affects the metaphyses of long bones
(b) Are most commonly seen around the knee and in the proximal
humerus

14
(c) Haematogenous spread can result in pulmonary metastases
(d) Is exclusively a disease of adolescence and early adult life
(e) X-ray shows a ‘sunburst’ appearance due to soft tissue involve-
ment
24. Regarding rheumatoid arthritis all are true except;
(a) Is a disease primarily of the articular cartilage
(b) Is associated with the HLA antigens DR4 and DW4
(c) Occurs more often in women
(d) The hand, elbows, knees and cervical spine are the commonest
joints involved
(e) Extra-articular manifestations occur in 20% of patients
25) Which fracture description does not match its name?
A) Lisfranc = dorsal dislocation of tarso-metatarsal joint +/- # 1st
cuneiform or 2nd MT
B) Monteggia = # proximal third ulna and dislocation of radial head
C) Clay-shoveller’s = avulsion of C3, 4,5 spinous processes
D) Bennett’s = intra-articular # base of thumb metacarpal and proxi-
mal, radial and dorsal displacement of distal fragment
26)The following statements are true EXCEPT
A) Volkmann’s ischaemic contracture of the fore-arm and hand may
develop despite wrist pulses being present and the forearm skin being
well perfused
B) ‘Fat pad’ sign is helpful for diagnosing occult #’s outside the el-
bow joint capsule
C) On the lateral XR of the elbow, the radio-capitellar line should
intersect the middle 1/3 of the capitellum
D) 25 % of supra-condylar #’s are undisplaced and is the most com-
mon # involving the elbow in children

15
27) In the child with Slipped Upper Femoral Epiphysis
(SUFE), which statement is FALSE?
A) SUFE is a Salter Harris type fracture.
B) Bilateral SUFE in a child under 10 years of age is more likely to be
associated with endocrine / metabolic disorders.
C)The ‘Klein line’ is helpful in diagnosing SUFE
D) A ‘Frog leg’ plain XR view of the hips is a helpful investigation to
diagnose SUFE
28) In the child with Leg-Calve-Perthes Disease (LCPD),
which statement is TRUE?
A) It is a form of capital femoral epiphysis avascular necrosis sec-
ondary to trauma
B) CT imaging is preferred above plain ‘frog leg’ XR’s of the hips
C) Patients with suspected LCPD disease require admission for fur-
ther inpatient management.
D) Is clinically indistinguishable from ‘acute synovitis’
29) Features seen on a child’s cervical spine XRay that are
different from an adult’s include all of the following EXCEPT
A) Anterior pseudosubluxation C2 on C3 can be normal until 8 years
B) Increased depth of anterior soft tissue space
C) Predental space (C1) up to 5mm
D) Prominent lordosis
30) Which of the following is FALSE Regarding the sys-
temic form of Juvenile Chronic Arthritis, (Stills disease)?
A) A temperature of 39.5 for two weeks is required for the diagnosis
of the systemic form
B) Pericarditis is an association
C) Lymphadenopathy is common
D) The child generally appears well

16
31) Regarding Paediatric Arthritis,
A) The arthritis of psoriatic arthritis always follows the skin lesions
B) Enthesitis is associated with HLAB-27
C) Iritis is associated particularly with the polyarticular form
D) Constitutional features are common with the pauci-articular form
32) Regarding hip injuries, which is TRUE?
A) Garden I and II fractures are non displaced fractures and can be
managed conservatively
B) Intracapsular fractures are more common than extracapsular
fractures
C) The hip is least stable when it is flexed and abducted
D) Superior femoral head fractures are often associated with ante-
rior hip dislocations
33) Which of the following is FALSE in regard to Plaster
of Paris?
A) Created by heating Gypsum at 350 0C to form Calcium sulphate
hemihydrate
B) Hardens in an exothermic reaction when exposed to water
C) A forearm cast will be fully dry by 36 hours post-application
D) A Forearm cast will develop 50% of the final full strength by one
hour post-application
34) Which of the following is FALSE in regard to the Man-
gled Extremity Severity Score (MESS)?
A) The presence and degree of major injuries is a parameter that
contributes to the final score
B) Limb ischaemia is a parameter that contributes to the final score
C) A score of 7 or more is associated with 100% chance of amputation
D) The degree of soft tissue injury is a parameter that contributes
to the final score

17
35) Which of the following is TRUE ?
A) Clinical examination is more sensitive than xray in picking up a
fracture
B) A ‘fat pad’ sign of a fracture is less reliable in obese individuals
C) x-ray is less specific than clinical examination in regards to frac-
ture diagnosis
D) X-rays with 2 views is 95% sensitive and specific for major joint
dislocation
36) Which philosopher is MOST likely to be useful to Or-
thopaedics surgery in Libya?
a) M. Langhi b) S. Makhlouf
c) M. Gabroun d) All of the above
37) Which of the following pairs is CORRECT
A) Femoral head fractures –Pipkin classification
B) Femoral head fractures -Gardiner classification
C) Patellar fractures –Hohl classification
D) Ankle factures –Boehler classification
38) Regarding Nerve Injuries that characteristically ac-
company certain Orthopaedic Injuries which
pairing is CORRECT?
A) Lunate dislocation -Ulnar nerve
B) Femoral Shaft Fracture-Femoral Nerve
C) Shoulder Dislocation-Radial Nerve
D) Knee Dislocation-Peroneal Nerve
39) Regarding Compartment Syndrome which bone
fractlicated?
A) Femur B) Radius
C) Tibia D) Humerus

18
40) Which is a CORRECT indication for re-plantation post
amputation?
A) Single digit amputations proximal to the FDS insertion
B) Sharp amputations with minimal to moderate avulsion proximal
to the elbow
C) Multiple level amputations D) Self-Inflicted amputations
41) Which of the following regarding elbow x rays is
FALSE EXCEPT:
A) The anterior humeral line passing through the middle third of the
capitellum indicates a supracondylar fracture
B) A visible posterior fat indicates an intracapsular fracture
C) A small anterior fat pad is normal
D) The radio-capitellar line should pass through the middle 1/3of
the capitellum
42) All of the following is true with regards to Perthes dis-
ease, EXCEPT:
A) All children with irritable hips should be referred to an orthopae-
dic surgeon
B) It is mostly unilateral
C) It occurs more often in malnourished children
D) When followed up 20 years later, most patients are pain free and
active
43) Origin of bone is from:
a. Ectoderm. b. Mesoderm.
c. Endoderm. d. All of the above.
44) Acute osteomyelitis is commonly caused by:
a. Staph aureus. b. S. pyogenes.
c. H. influenzae. d. Salmonella.

19
45) Acute osteomyelitis usually begins at:
a. Epiphysis. b. Metaphysis.
c. Diaphysis. d. Any of the above.
46) What is not True of acute pyogenic osteomyelitis:
a. Trauma is a predisposing factor.
b. Common infecting agent is Staph. Aureus.
c. Infection is usually blood borne.
d. All are true.
47) What is not True of Brodie’s abscess:
a. A form of chronic osteomyelitis. b. Intermittent
pain and swelling.
c. Common to diaphysis. d. Excision is
very often required.
48) Tuberculosis of the spine most likely originates from:
a. Intervertebral disk.
b. Cancellous vertebral body.
c. Ligamentous structures.
d. Paravertebral soft tissue.
49) In Pott’s spine, the disease starts in the:
a. Intervertebral disk.
b. Anterior vertebral margin.
c. Posterior vertebral margin.
d. Paravertebral soft tissue.

50) Melon seed bodies in joint fluid are characteristic of:


a. Rheumatoid arthritis. b. Tuberculous arthritis.
c. Septic arthritis. d. None of the above.

20
51) The earliest sign of TB hip in X-ray is:
a. Narrow joint space.
b. Irregular moth eaten femoral head.
c. Periarticular osteoporosis.
d. Dislocation.
52) Healing of tuberculous arthritis can lead to:
a. Calcification. b. Fibrous ankylosis.
c. Boney ankylosis. d. None of the above.
53) The synonym for Paget’s disease is:
a. Osteitis fibrosa. b. Osteitis proliferans.
c. Osteitis deformans. d. None of the above.
54) Multiple myeloma tumor cells resemble:
a. Granulocytes. b. Plasma cells.
c. Lymphocytes. d. Chondrocytes.
55) An adamantinoma historically contains:
a. Squamous cell rests. b. Pallisading cells.
c. Cells resembling basilar cells. d. All of the above.
56) Osteomalacia predominantly affects the:
a. Spine. b. Pelvis.
c. Skull bones. d. Metatarsals.
57) The enzyme found in osteoclasts but not in osteo-
blastsis:
a. Alkaline phosphatase. b. Acid phosphatase.
c. Elastase. d. Cytochrome oxidase.
58) A Gigli saw is:
a. An electrically driven circular bone saw.
b. A pneumatically driven bone saw.

21
c. A short straight bone saw.
d. A long twisted wire bone saw.
59) Osteoclasis can be used to:
a. Correct deformity of the tibia due to rickets.
b. Curette an osteoclastoma.
c. Correct deformity.
d. Correct a ricketery rosary.
60) In Dupuytren’s contracture which one of the following
statements incorrect:
a. It is a contracture of the flexor tendons to the ring and little fin-
gers.
b. It is a contracture of the palmar fascia.
c. It may occur in the plantar fascia.
d. There is an association with cirrhosis of the liver
61) In an adult patient with a fracture of the shaft of the
femur:
a. No blood can be lost without obvious swelling.
b. No blood can be lost without obvious bruising.
c. Two liters of blood can be lost without obvious swelling or bruising.
d. There is no possibility of death from hemorrhagic shock.
62) A greenstick fracture:
a. Occurs chiefly in the elderly.
b. Does not occur in children.
c. Is a spiral fracture- of tubular bone.
d. Is a fracture where part of the cortex is intact and partis crumpled
or cracked.

22
63) Spiral fracture is due to:
a. Blunt trauma. b. Axial compression.
c. Twist. d. Direct impact.
64) The single most important factor in fracture healing is:
a. Correct bone alignment. b. Accurate reduction.
c. Immobilization. ]d. Organization of clot.
65) Which one of these statements is True in diagnosis of
congenital hip dislocation in the first few days of life:
a. It is impossible to diagnose it.
b. The sign of telescoping is the best way of diagnosing it.
c. It is possible to diagnose it by the Van Rosen/BarlowTest.
d. The Trendelenberg test is the most useful.
66) Trendelenburg’s sign is used in the diagnosis of:
a. Varicose veins.
b. Congenital dislocation of the hip.
c. Carcinoma of the stomach.
d. Pulmonary embolism.
67) If an unstable hip is detected at birth the management
policy is:
a. Do nothing and re-examine every six months as only a minority of
hips develop into a persistent dislocation.
b. Use a splint to keep the hip joint in 45° flexion and adduction.
c. Use a splint to keep the hip joint in 90° flexion and abduction.
d. Advise operative stabilization.
68) The essential examination of the hip in order to clinch
the diagnosis of chronic slipped femoral epiphysis is:
a. Measuring for shortening of the leg.

23
b. Palpation of the femoral head.
c. A-P plain x-ray view of the hip.
d. Lateral x-ray view of the hip.
69) Shenton’s line is a sign applicable to:
a. The detection of shortening of the leg on physicalexamination.
b. A radiological feature of the pelvis applied to thediagnosis of con-
genital dislocation of the hip.
c. A radiological feature of the lungs applied to thediagnosis of pul-
monary vein thrombosis.
d. A physical sign applied to the diagnosis of adrenaldeficiency.
70) Perthes’ disease is common to age group of:
a. 1-5. b. 6-10. c. 11-15. d. 16-20.
71) The average duration of Perthes’ disease is:
a. 1-2 years. b. 3- 4 years.
c. 1 month - 6 months. d. 6 months - 1 year.
72) In Perthes’ disease the hip movements restricted are:
a. Abduction and external rotation.
b. Abduction and internal rotation.
c. Adduction and external rotation.
d. All of the above.
73) The sequestrum in X-ray appears:
a. Dense. b. Light.
c. Isodense as surrounding bone. d. Any of the above.
74) The term delayed union is employed when the fracture
fails to unite within:
a. 1.5 times the normal union time.
b. Twice the normal union time.

24
c. 2.5 times the normal union time.
d. None of the above.
75) 2. Which of the following muscles are stance phase
muscles?
(a) Quadriceps. {b) Hamstring muscles.
(c) Soleus - gastrocnemius. d. Peroneus longus.
76) The joint most likely to have recurrent dislocation is:
a. Ankle. b. Knee. c. Shoulder. d. Patella.

77) Myositis ossificans commonly occurs around:


a. Shoulder. b. Elbow. c. Wrist. d. Knee.

78) Volkman’s ischemia commonly occurs following:


a. Fracture shaft humerus. b. Supracondylar fracture.
c. Colles’ fracture. d. Monteggia fracture.
79) The single dependable sign of early
Volkmann’scontracture is:
a. Cyanosis of fingers.
b. Obliteration of radial pulse.
c. Paralysis of flexor muscles of forearm.
d. Pain.
80) Cubitus valgus of elbow commonly follows fracture of:
a. Lateral condyle.
b. Medial condyle.
c. Capitalum.
d. Lower third of humerus.

25
81) Fracture involving which part of humerus can cause
delayed ulnar palsy:
a. Shaft. b. Surgical neck.
c. Medial epicondyle. d. Lateral epieondyle.
82) The deformity of wrist in Colles’ fracture is:
a. Madelung’s deformity. b. Dinner fork deformity.
c. Buttonaire deformity. d. None of the above.
83) What is True of Sudeck’s atrophy of hand:
a. Hand is painful and swollen.
b. Osteoporosis of carpals and metacarpals.
c. There is increased blood flow to para-articular areas.
d. All are true.
84) The carpal bone most commonly fractured is:
a. Triquetrum. b. Hamate. c. Capitate. d. Scaphoid.
85) Which nerve is compressed in carpal tunnel syn-
drome?
a. Ulnar. b. Median. c. Radial. d. All of the above. 
86) Nelaton›s line joins anterior superior iliac spine to:
a. Xiphisternum. b. Pubic tubercle.
c. Ischial tuberosity. d. Ischial spine.
87) Bryant›s triangle helps to assess:
a. Fracture neck of femur. b. Iliac crest displacement.
c. Trochanteric displacement d. None of the above.
88) Normal neck-shaft angle of femur is:
a. 90°. b. 120°. c. 150°. d. 170°.
89) Fracture femoral neck can be diagnosed from:
a. Limb shortening. b. External rotation.

26
c. Abduction. d. A + B.
90) Spontaneous bleeding into joints in haemophilia oc-
curs when factor VI level is less than:
a. 50%. b. 25%. c. 10%. d. 5%.
91) Recurrence of Baker›s cyst should make the surgeon
suspect:
a. Neoplastic change.
b. Undiagnosed pathology within knee.
c. Incomplete removal of the cyst.
d. The communication to the joint is persisting.
92) Flexion of distal interphalangeal joint with fixing the
proximal interphalangeal joint (PIP) tests:
a. Flexor digitorum profundus.
b. Flexor digitorum superficials.
c. Palmaris longus.
d. All of the above.
93) Stenosing tenovaginitis commonly affects:
a. Abductor pollicis.
b. Flexor pollicis longus.
c. Opponens pollicis.
d. All of the above.

94) A sequestrum is
a. a piece of soft dead tissue
b. a piece of dead skin
c. a dead tooth
d. a piece of dead bone

27
95) Union of a simple uncomplicated transverse fracture
of the tibia in an adult normally takes;
a. 6 weeks b. 8 weeks
c. 12 weeks d. 18 weeks e. 26 weeks
96) Malunion of a fracture is;
a. a fracture which unites in a position of deformity b. de-
layed union of a fracture
c. non-union of a fractured followed by pseudoarthrosis d. due
to tuberculosis
e. seen in scaphoid fractures
97) Volkmann›s contracture is;
a. affects the palmar fascia
b. develops at the ankle in a case of chronic venous ulcer 
c. follows ischemia of the forearm muscles
d. is due to excessive scarring of the skin of the axilla following a
burn
e. follows ulnar nerve palsy
98) A Brodie›s abscess is;
a. a subperiosteal abscess due to infection of the mastoid air cells
b. a type pf breast abscess
c. a chronic abscess of the bone
d. an abscess arising in the inguinal lymph nodes
e. an abscess forming in an infected varicose vein
99) The initial abnormality in primary osteoarthritis is;
a. in the synovial membrane
b. sclerosis of cartilage
c. fibrillation of cartilage

28
d. an osteophyte
e. a pannus
100) The initial abnormality in rheumatoid arthritis is;
a. fibrillation of cartilage
b. sclerosis of cartilage
c. in the synovial membrane
d. in the capsule
e. proliferation of bone
101) Pott›s paraplegia is due to;
a. hematomyelia following trauma
b. damage to the cord by a piece of bone when vertebraecollapse in
tuberculosis of the spine
c. tuberculous pus and angulation in tuberculosis of the spine
d. damage to die corda equina after a fall
e. fracture dislocation of cervical vertebrae
102) Still›s disease is;
a. spastic diplegia b. rheumatoid ar-
thritis in childhood
c. rheumatoid arthritis in the elderly
d. post-traumatic bone formation in the lateral ligament of  the knee
e. synonymous with Reiter›s disease
103) A benign tumor forming osteoid is;
a. a synovioma b. a chondroma
c. an osteoma d. a fibroma e. an adenoma
104) Ewing›s tumor affecting the humerus
a. is a metastasis from carcinoma of the thyroid b. should be
treated by immediate amputation

29
c. looks like a cut onion on x-ray d. has a soap-
bubble appearance on x-ray
e. displays sun-ray spicules on x-ray
105) Barlow›s sign is related to the diagnosis of 
a. talipes equinus varus b. congenital dislo-
cation of the hip
c. ulnar nerve palsy d. genu varum
e. fractured neck of femur 
106) The reported incidence of unstable hips per 1000 at
birth is as much as;
a. 0.5 b. 2-5 c. 8-20 d. 25-30
e. 35-40
107) The word talipes refers to
a. long feet with spidery toes b. knock knee
c. flat feet d. hammer toes
e. club feet
108) Bone dysplasia is due strictly to
a. faulty nutrition b. osteomyelitis
c. parathyroid tumor  d. trauma
e. faulty development
109) Idiopathic scoliosis is a
a. lateral curvature of the spine
b. rotation of the spine
c. lateral curvature with rotation of the spine
d. flexion deformity of the spine
e. congenital disease with hemivertebrae
110) A Milwaukee brace can be used in
a. sacro-iliac strain

30
b. derangement of the teeth
c. a patient with an above knee amputation
d. scoliosis
e. fractured clavicle
111) Legg-Calve-Perthe›s disease is
a. osteochondritis of the spine
b. tuberculosis of the hip joint
c. slipped proximal femoral epiphysis
d. osteochondritis of the proximal femoral epiphysis
e. osteomalacia
112) The name associated with joint neuropathy is that of 
a. Cushing b. Osier
 c. Moon d. Charcot e. Addison
113) Adrenocorticosteroids administered in excess cause
a. osteoporosis b. osteosclerosis
c. osteochondritis d. endochondral ossification
e. osteosarcoma
114) ‹Tennis elbow› is the term used for 
a. olecranon bursitis
b. ‹non-articular rheumatism› of the extensor muscles of forearm
attached to lateral epicondyle of the humerus
c. myositis ossificans of the supinator muscle
d. a fractured head of radius
e. ulnar nerve neuritis
115) An adventitious bursa is
a. an anatomical bursa overlying any joint
b. a type of degeneration of adventitia of popliteal artery

31
c. an acquired bursa generated from connective tissue
d. a pseudocyst in the lesser sac (omental bursa)
e. an infected knee
116) A trigger finger is;
a. an inflamed index finger
 b. an atrophic index finger in a median nerve palsy
c. due to stenosing tenovaginitis affecting one of the flexor tendons
in the palm
d. an essential feature of the carpal tunnel syndrome
e. a component of syndactyly
117) A Baker›s cyst is
a. an implantation dermoid cyst occurring in the palms of those
who work in a bakery
b. a synovial cyst of the wrists of those who make bread c.
a prepatellar bursa
d. a synovial cyst of the ankle e. a synovial cyst
of the popliteal fossa
118) Immobilization of fractures of long bones should in-
clude
a. Fractured bone only
b. Joint involved in the fracture
c. Proximal joint
d. Both proximal and distal joints
e. Distal joint
119) The most severe growth disturbance results from
which of the following types of epiphyseal injuries:
a. Separation of the epiphysis at the metaphyseal side of the epiph-
yseal plate

32
b. Separation of the epiphysis with a triangular fragment of the
metaphysic
c. Intra-articular fracture involving the articular cartilage epiphysis
and epiphyseal plate
d. Intra-articular fracture extending from the joint surface through
the epiphysis and epiphyseal plate to the metaphysis
e. Crashing injuries compressing the epiphyseal plate without dis-
placement
120) Non-union in closed fractures may due to any of the
following except:
a. Inadequate immobilization b. Interposition of
soft parts
c. Impaired blood supply d. Inpsction of the
fragments
e. Wide separation of the fragments
121) Causes of gangrene after fracture in a limb do not  in-
clude:
a. Direct crushing of the tissues b. Injury to the
main vessels
c. Tight plasters d. Septic infection
e. Clostridial infection

122) The correct treatment of traumatic myositis ossifi-


cans is by:
a. Prolonged immobilization
b. Active exercises
c. Passive stretching and massage
d. Both A and B
e. Both B and C

33
123) The incorrect statement about anterior dislocation of
the shoulder joint is that:
a. shoulder loses its rounded contour & becomes flattened
b. The elbow is abducted from the side
c. All movements of the shoulder are limited and painful
d. The anterior and posterior folds of the axilla are elevated
e. The hand cannot be placed on the opposite shoulder (Duga›s test)
124) Recent dislocations of shoulder joint are best treated by:
a. Hippocrates› method of closed reduction b. Kocher›s manipula-
tion
c. Modified Milch›s manoevre
d. Open reduction
e. Putti-platt1s operation
125) Recurrent shoulder joint dislocation is best treated
by:
a. Physiotherapy
b. Nicola›s operation
c. Bankart›s operation
d. Putti-Platt›s operation
e. Arthrodesis of the joint
126) Posterior dislocation of elbow joint is characterized
by the following except:
a. Gross swelling of the elbow region
b. Loss of all movements at the elbow joint
c. Shortening of the upper arm
d. Absence of crepitus
e. Loss of the normal relationship of the olecranon with the two
epicondyles

34
127) 3 Tears of the meniscus of the knee result from which
of the following strain:
a. Hyperextension b. Abduction
c. Adduction d. Rotation
e. Combined flexion and rotation
128) A lateral blow at the level of the knee joint may cause:
a. Rupture of anterior cruciate ligament
b. Rupture of medial collateral ligament
c. Avulsion of medial meniscus
d. Bumper fracture of tibia
e. All of the above
129) A march fracture most frequently results from:
a. Direct trauma b. Jumping
from a height
c. Muscle fatigue from prolonged walking d. Use of
high-heeled shoes
e. Osteoporosis

130) Intestinal absorption of calcium is dependent upon:


a. Vitamin D b. Parathoraone
c. Calcitonin d. All of the above
e. None of the above
131) An irregular epiphyseal line with calcifying periosteal
haematc found on X-ray examination is indicative of:
a. Infantile rickets b. Scurvy
c. Hemophilia d. Hypoparathyroidism
e. Hypervitaminosis A

35
132) Osteoporosis is a deficiency in:
a. Calcium metabolism b. Calcium deposition
c. Protein supporting tissue d. All of the above
e. None of the above
133) Which of the following statements is untrue concern-
ing the enzyme alkaline phosphatase:
a. Has a normal serum concentration of 3-13 KA units
b. Is present in high concentrations in liver cells
c. Is excreted in the bile
d. Is elevated in the serum of patients with healing fractures
e. Is↑in the serum of patients with rickets andosteomalacia
134) Localized bone sclerosis may be due to:
a. Syphilis b. Sclerosing osteoperiostitis
c. Osteoarthritis d. Bone tumors
e. All of the above
135) Enlarged tender epiphyses with bowing of long bones
and X-ray evidence of delayed carpal ossification suggest
the diagnosis of:
a. Scurvy b. Infantile rickets
c. Syphilitis epiphysitis d. Osteogenesis imperfect
e. Achondroplasia
136) Osteomalacia may be due to the following factors
except;
a. Starvation
b. Repeated pregnancies 
c. Idiopathic steatorrhea
d. Increased renal excretion of calcium and phosphorus

36
e. Prolonged recumbency
137) Achondroplasia is characterized by the following
features except; 
a. Short extremities b. Normal trunk length
c. Normal intelligence d. Enlarged head
e. Normal ossification of cartilage
138) Osteogenesis imperfecta is characterized by thefol-
lowing features except ;
a. Blue sclera b. Brittle shell-like bones
c. Multiple fractures d. Osteoporosis
e. Familial tendency
139) Concerning Ollier›s disease, which of the following
statements is incorrect:
a. There is a strong hereditary predisposition
b. Multiple enchondromata occur in small long bones of hands and
feet
c. The affected bones are stunted
d. Deformities may arise from unequal affection of metaphyses
e. Chonrosarcoma may develop in one of the enchonromata
140) The following statements about diaphyseal aclasis
are true except  that it:
a. Is a common hereditary condition
b. Never affects membrane bones
c. Is characterized by multiple exostoses
d. May be associated with dwarfism
e. Spares the metaphyses of long bones

37
141) Concerning Osgood-Schlatter›s disease, the follow-
ing statements are true except  that it:
a. Is an avascular necrosis of the epiphysis of the tibiatubercle
b. Usually occurs between ages of 10 and 16 years
c. Is due to traction of the patellar tendon on the tibialtubercle
d. Is associated with no radiological signs
e. Causes pain and swelling over the tubercle
142) Generalized osteitis fibrosa cystica is characterized
by the following features except: 
a. Deficiency of parathormone
b. Diffuse decalcification and softening of bones
c. Cysts containing Brwonish fluid
d. Multiple gaint-cell tumors
e. Urinary symptoms due to renal calculi
143) The following statements about Paget›s disease
of bones are true except  that it:
a. Affects elderly subjects, particularly males
b. Is a generalized bone dystrophy of obscure etiology
c. Causes deformities in the skull, spine, pelvis and lower limbs
d. Produces no pressure symptoms
e. Is often associated with cardiovascular complications
144) Which of the following statements is untrue in Pag-
et’s disease:
a. The bones are greatly thickened and very vascular 
b. Cranial nerve palsies may occur 
c. Spontaneous fractures are rare
d. X-ray examination of the skull is diagnostic
e. Osteogenic sarcoma occurs in over 5% of cases

38
145) Which statement is untrue in renal rickets:
a. Results from renal insufficiency in infancy
b. Is due to deficient phosphorus, excretion
c. Manifests itself by marked dwarfism
d. Causes no deformities in the limbs
e. Ends fatally from uraemia at puberty

146) Solitary bone cyst is characterized by the following


features except  that it:
a. Occurs most often in children and adolescents
b. Usually arises in the diaphysis of a long bone
c. Often remains symptomless until complicated by pathological
fracture
d. Appears as clear ovoid expanding cavity in the X-ray
e. May be associated with new-bone formation

147) The most common tumor of the small bones of the


hands and feet is:
a. Enchondroma b. Osteochondroma
c. Osteoclastoma d. Cancellous osteoma
e. Bone sarcoma

148) Which of the following is most common in the small


bones of the hands and feet:
a. Osteochondroma b. Enchondroma
c. Osteoid osteoma d. Osteochondritis juvinelis
e. Tuberculous osteitis

39
149) Concerning osteoclastoma, the following statements
are correct except  that it:
a. Usually occurs between the ages of 15 and 40 years
b. Always arises in metaphyseal region of cartilaginous bones
c. Consists of large giant cells in a very vascular stroma of spindle cells
d. Presents as a painless globular swelling with well-defined edge
e. Produces diagnostic radiological signs
150) The radiological signs of osteoclastoma include the
following except: 
a. Abrupt expansion of the bone b. Characteristic
soap-bubble appearance
c. Presence of an operculum obliterating the medullary cavity d.
Absence of any new-bone formation
e. Presence of areas of bone destruction
151) The treatment of osteoclastoma includes the follow-
ing measures except:  
a. Curettage of tumor tissue & packing cavity with bone chips
b. Excision with safety margin of bone
c. Amputation
d. Radiotherapy
e. Chemotherapy
152) The following statements about multiple myeloma
are true except  that it::
a. Is a primary malignant tumor of bone marrow
b. Occurs between the ages of 40 and 60 years
c. Usually presents with bone pain especially in the back d. Is rarely
associated with fever and anemia
e. May cause paraplegia with girdle pains

40
153) The following statements about bone sarcoma are-
true except  that it:
a. Arises from osteoblasts of the periosteum or bone cortex
b. Forms a fusiform mass ensheathing the bone
c. Often invades the epiphyseal cartilage and neighbouring joint
d. Produces characteristic new bone formation in theX-ray
e. Disseminates rapidly by the blood stream
154) Which one of the following statements is untrue con-
cerning chondro-sarcoma?
a. Occurs most often between the ages 20 and 60 yrs
b. Is always a primary malignant tumor of bone
c. Most commonly affects scapula, pelvis, ribs &sternum
d. Causes bone expansion and destruction with irregular opacities
in the X-ray
e. Is radioresistant
155) Ewing’s sarcoma is characterized by the following
except  that it:
a. Is a common tumor of children
b. Always arises in the metaphysis of a long bone
c. Presents as a fusiform swelling with inflammatory changes in the
overlying soft tissues
d. May be associated with leucocytosis
e. Produces characteristic radiological signs
156) The most important DD of Ewing’s tumor is:
a. Chondrosarcoma b. Osteogenic sarcoma
c. Acute osteomyelitis d. Malignant metastasis
e. Reticulum cell sarcoma

41
157) The most common osteolytic metastases in bones
are derived from the:
a. Lung b. Breast
c. Stomach d. Kidney e. Prostate
158) The most pain-sensitive structure in a joint is the:
a. Bone end b. Articular cartilage
c. Joint capsule d. Synovial membrane
e. Skin and subcutaneous tissues
159) Ostaomalacia is characterized by the following fea-
tures except: 
a. Deficient protein metabolism
b. Demineralization of the bones
c. Slow epiphyseal closure
d. Skeletal deformities
e. X-ray Looser ‘s zones
160) The following statements about solitary bone cyst
are true except  that it:
a. Occurs most often in children and adolescents
b. Is commonest in the humerus , femur and tibia
c. Arises in the diaphysis of the bone
d. Assumes an avoid shape and may cause bone expansion
e. May remain in-noticed until complicated by pathological fracture
161) The differential diagnosis of osteoid osteoma in-
cludes all the following except: 
a. Brodies’ abscess b. Ossifying fibroma
c. Ewing’s tumor  d. Metastatic thyroid nodule
e. Bone sarcoma

42
162) Ivory osteomata occur most often in the:
a. Skull b. Spine c. Humerus
d. Femur  e. Tibia
163) Sensory end organs are absent from the:
a. Muscle b. Periostium
c. Synovial membrane d. Articular cartilage
e. Joint capsule
164) The viscosity of synovial fluid is mainly due to:
a. Chondroitin sulphate b. Hyaluronidase
c. Albumin and laucin d. Alpha and beta globulins
e. Beta lipoproteins
165) Septic arthritis of infancy usually affects which of the
following joints:
a. Shoulder  b. Elbow
c. Wrist d. Hip
e. Knee
166) Trenderburg’ s sign can be elicited in all of the fol-
lowing except : 
a. Congenital dislocation of the hip. b. Infantile paralysis
of the gluteal muscles.
c. Coxa vara. d. Tuberculous arthritis
of the hip joint.
e. Non-united fracture of the femoral neck.
167) The clinical features of coxa vara include the follow-
ing except: 
a. Shortening, addution and eversion of the limb.
b. Raising of the greater trochanter above Nelaton’sline.
c. Limitation of all movements of the hip.

43
d. Positive Trendelenburg’s sign.
e. Limping and difficulty in kneeling, riding and separating the legs.
168) Treatment of genu valgum includes the following ex-
cept:  
a. Physiotherapy. b. Wedged shoes.
c. Night splints. d. Osteoclasis of the tibia.
e. McEwen’s osteotomy of the femur.
169) The most common congenital deformity of the hind
foot is talipes :
a. Calcaneus. b. Equinus.
c. Eguinovarus. d. Varus. e.
Valgus.
170) Paralytic talipes is differentiated from, congenital
talipes by the following features except that:
a. The deformity appears later after birth.
b. The limb is atrophied, cyanosed and cold.
c. The muscles are wasted and flabby.
d. Usually both sides are affected.
e. The deformity can be corrected easily by manipulation.
171) Syme’s amputation is better than a below-knee am-
putation except  that it:
a. Is less “catastrophic” to the patient.
b. Allows the patient to walk around in his room without prosthesis.
c. Maintains, the pleasure of “earth feeding”.
d. Requires a cheap stump boot.
e. Is not attended with serious complications.

44
172) Reimplantation of a traumatically amputated limb re-
quires all of the following except:  
a. Limb preservation.
b. Shortening of bone.
c. Immediate arterial and venous repair.
d. Routine angiograms.
e. Delayed repair of nerves.
173- Regarding hallux valgus, the following statements
are true except that it:
a- Consists of outward deviation of the great toe at the metatarso-
phalangeal joint
b- Is usually due to badly fitting shoes.
c- Is not progressive.
d- Causes hammer-toe deformity in the other toes.
e- Predisposes to several painful complications.
174) The causes of hammer-toe include the following ex-
cept: 
a. Overcrowding of the toes by ill-fitting shoes.
b. Hallux valgus.
c. Pes cavus.
d. Talipes equinus.
e. Rupture of the extensor expansion.
  175) The following statements about provisional amputa-
tion for infective gangrene are true except that it:
a. May be urgently needed to control infection and toxemia.
b. Should be made through the healthy limb above the infected area.
c.Shouldbedoneaslowaspossibletoallowreamputation at the optimum
level.

45
d. Should provide free drainage.
e. May be carried out by the guillotine or flap method without clo-
sure.
176) The ideal amputation should fulfill the following re-
quirements except that it should:
a. Be as long as possible
b. Have a smoothly rounded cone-shaped end
c. Not include muscle over the bone end
d. Have a linear freely movable scar not exposed to pressure.
e. Be painless with a freely movable joint above and a smooth bone
end below.

177) Which of the following assertions on Perthe´s dis-


ease is not correct:
a. It usually presents with a painful limp
b. It may present with pain in the knee
c. It frequently presents with limitation of all hip movements, espe-
cially internal rotation and abduction
d. It is associated with an intermittent pyrexia in the early stages
178) In the treatment of osteoarthritis of the knee:
a. physiotherapy has no value even in mild cases
b. a high osteotomy is indicated when it affects mainly the medial
compartment providing deformity is not severe
c. knee arthrodesis is the treatment if choice in bilateral cases
d. total knee replacement is the treatment of choice in the under
40 year age group

46
179) Which statement is true about the “three-column
concept” of spinal fracture stability?
A. An unstable spine consists of bone or soft tissue injury in a single
column.
B. An unstable spine involves injury to all three columns.
C. Instability results from injury to two columns plus evidence of
compression of the dural tube.
D. Instability results from significant bone and/or soft tissue injury
in two columns.
180) All of the following statements are true of Jefferson’s
fracture of the atlas except:
A. The injury results from an axial load to the cervical spine.
B. The fracture fragments characteristically displace into the spinal
canal.
C. Neurologic injury is uncommon.
D. Computed tomography (CT) best demonstrates the fracture’s
configuration.
181) The neurovascular structure most commonly injured
as a result of an anterior dislocation of the shoulder is the:
A. Musculocutaneous nerve.
B. Axillary nerve.
C. Axillary artery.
D. Median nerve.
182) The classification of fractures of the proximal humer-
us is based on:
A. The number of fracture segments and amount of displacement.
B. The mechanism of injury.
C. Presence or absence of associated dislocations.
D. All of the above

47
183) The radial nerve is at greatest risk for injury with
which fracture?
A. Fracture of the surgical neck of the humerus.
B. Fracture of the shaft of the humerus.
C. Supracondylar fracture of the humerus.
D. Olecranon fractures.
184) The best method of treating a supracondylar fracture
of the humerus in a child that is unstable when the elbow is
flexed to 90 degrees is:
A. Hyperflexion of the elbow to 130 degrees and casting.
B. Open reduction and internal fixation.
C. Percutaneous pinning.
D. Skeletal traction.
185) Both-bone forearm fractures in adults are best man-
aged by:
A. Closed reduction and casting.
B. Closed reduction and application of an external fixator.
C. Open reduction and placement of intramedullary rods.
D. Open reduction and internal fixation with compression plates.
186) The most consistent sign of a fracture of the carpal
scaphoid is:
A. Wrist pain during attempted push-ups.
B. Diffuse swelling on the dorsum of the wrist.
C. Localized tenderness in the anatomic snuffbox.
D. Wrist popping on movement.
187) Total interruption of the radial nerve at midarm pro-
duces specific findings on physical examination. The most
complete description of the neurologic deficit includes:

48
A. Paralysis of the thumb extensors, interphalangeal joint extensors,
extensor carpi radialis, and extensor carpi ulnaris.
B. Paralysis of the extensor carpi radialis longus and brevis, abductor
pollicis longus, extensor pollicis brevis, and extensor  pollicis longus.
C. Paralysis of the brachioradialis, extensor carpi radialis longus and
brevis, extensor carpi ulnaris, thumb extensors, and metacarpopha-
langeal (MCP) joint extensors, and loss of cutaneous sensibility at the
dorsal aspect of the thumb and indexfingers.
D. Paralysis of the brachioradialis, extensor carpi radialis longus and
brevis, radialis, thumb extensors, finger MCP jointextensors, and flexor
carpi radialis, and loss of sensation in the cutaneous distribution over
the dorsal aspect of thumb andindex fingers.
188) The most common physical findings in a patient with
median nerve compression at the wrist (carpal tunnel syn-
drome)are:
A. Diminished two-point discrimination and dryness of the index
and long fingers.
B. Atrophy of the abductor pollicis brevis and opponens pollicis.
C. A positive percussion test at the wrist and a positive wrist flexion
test producing paresthesias at the thumb, index, andlong fingers.
D. A weak grip in addition to hand cramping and difficulty writing.
189) Which of the following describes the most desirable
position in which to immobilize the hand?
A. Wrist is flexed, MCP joints are extended, and IP joints are flexed.
B. Wrist is flexed, MCP joints are flexed, and IP joints are extended.
C. Wrist is extended, MCP joints are extended, and IP joints are
flexed.
D. Wrist is extended, MCP joints are flexed, and IP joints are ex-
tended

49
190) An early sign of compartment syndrome in the hand
includes:
A. Pain with passive stretch of the digits.
B. Absent radial pulse.
C. Motor paralysis.
D. Swelling of the digits.
191) Palmar dislocation of the PIP joint with fracture:
A. Is more common than dorsal dislocation.
B. Is treated by splinting with the PIP joint in flexion.
C. Is treated by splinting with the PIP joint and DIP joints in exten-
sion.
D. If not splinted properly, will cause a boutonniere deformity.
192) Fracture of the fifth metacarpal neck:
A. Usually requires open reduction and internal fixation.
B. Must be reduced anatomically and stabilized with pins.
C. Is called a “boxer’s fracture.”
D. Will result in significant functional disability if angulated 30 de-
grees dorsally.
193) A Bennett’s fracture is:
A. An extra-articular fracture of the base of the thumb metacarpal.
B. Displaced by the pull of the abductor pollicis longus and adductor
pollicis.
C. Displaced by the pull of the abductor pollicis longus and extensor
pollicis longus
D. Usually successfully treated with closed reduction and casting.
194) The Ilizarov device aids in management of tibial frac-
tures because of its ability to:
A. Stabilize acute fractures.

50
B. Correct angular deformities in cases of malunion.
C. Transport bone by distraction callotasis.
D. All of the above.
195) An 8 cm. by 10 cm. soft tissue defect over the proxi-
mal third of the tibia with exposed bone devoid of perios-
teum is best treated with:
A. Skin graft.
B. Gastrocnemius rotational myoplasty.
C. Soleus rotational myoplasty.
D. Free tissue transfer.
196) Prognosis of healing in tibial fractures correlates
best with:
A. Energy absorption at the time of fracture.
B. Amount of soft tissue damage.
C. Location of the fracture (i.e., in the proximal, middle, or distal third).
D. Age of patient.

197) Management of a III-b tibia fracture is best treated


initially by:
A. Plaster immobilization.
B. Immediate plating.
C. Reamed intramedullary nailing.
D. External fixation.
198) The most frequent forces acting on the foot that
cause ankle fractures are:
A. External rotation.
B. Internal rotation.
C. Plantar flexion.

51
D. Dorsiflexion.
199) Patients who have abduction injuries to the foot are
prone to injure the following structures:
A. Medial malleolus and deltoid.
B. Lateral malleolus and deltoid ligament.
C. Interosseous ligament.
D. Posterior tibiofibular ligament.
200) Of the following bones in the foot, the tarsal bone
that is most prone to vascular compromise is the:
A. Calcaneus. B. Navicular.
C. Talus. D. Cuboid.
201) A Lisfranc fracture is a fracture-dislocation involving:
A. Calaneocuboid joint.
B. Tarsometarsal joint.
C. Metatarsophalangeal joint
D. Talocalcaneal dislocation.
202) The most common reason for surgical amputation in
the general population is:
A. Trauma. B. Tumor.
C. Infection. D. Ischemia.
203) A radical margin in the resection of a musculoskel-
etal tumor removes:
A. The entire limb.
B. A 5-cm. margin of normal tissue around the neoplasm.
C. The anatomic compartment in which the tumor arises.
D. The joint adjacent to the neoplasm.

52
204) Isolated flexor digitorum superficialis tendon func-
tion is determined by assessing:
A. Flexion of the metacarpophalangeal joint.
B. Flexion of the proximal interphalangeal joint with the adjacent
digits held in extension.
C. Flexion of the distal interphalangeal joint.
D. Flexion of the proximal interphalangeal joint
205) The zone of flexor tendon injury that carries the poor-
est prognosis following injury and repair is:
A. Zone I. B. Zone II.
C. Zone III. D. Zone IV.
206) Medial meniscus is injured more frequently than the
lateral meniscus because:
a- it is more mobile. b-it is less mobile.
c- it is thinner. d- it is attached lightly to the femur.
207) twisting injury to knee frequently results in all of the
following except:
a- meniscal tear. b- capsular tear.
c- ACL tear. d- fibular collateral ligament tear.
208) twisting injury to knee during weight bearing results
most frequently in an injury to :
a- ACL. b- medial meniscus.
c- PCL. d- fibular collateral ligament.
209) positive pivot shift test in knee is due to injury of:
a- posterior cruciate ligament.
b- ACL. c- MCL.
d-postero lateral complex tear (PIC).

53
210)most common site of osteochondritis dessicans
(OCD) of the knee is:
a- medical side of the medical femoral condyle .
b- lateral side of the medial femoral condyle.
c- lateral side of lateral femoral condyle.
d- inter condyle notch.
211) finkelstein’s test is done in:
a-tennis elbow. b- golfer’s elbow.
c- de quervain’s teosynovitis. d- carpal tunnel syndrome.
212) after repair of tendoachilles, the appropriate position
of immobilisation:
a- dorsi flexion. b- neutral.
c- planter flexion. d- varies according to the repair.
213) Bankarts lesion is seen in:
a- post surface of glenoid labrum.
b- anterior surface of glenoid labrum
c- anterior part of humeral head.
d- posterolateral part of humeral head.
214) true regarding mallet finger:
a- avulsion of tendon at the base of middle phalynx.
b- avulsion of extensor tendon at base of distal phalynx.
c- fracture of distal phalynx. d-
fracture of proximal phalynx.
215) the type of crystals found in pseudo gout are:
a-mono sodium urate. b- calcium pyrophosphate.
c- calcium thiosulfate. d- sodium tartarate.

54
216) charcot joint is seen in :
a- tuberculosis. b- diabetes.
c- rickets. d- survey.
217) morquio’s syndrome is diagnosed by detection in
the urine of :
a- dermatan sulphate. b- keratin sulphate.
c- heparin sulphate. d- chondroitin sulphate.

218) Mafucci’s syndrome is a condition characterised by:


a- enchondromatosis . b- multiple hemangiomas.
c- both a and b. d- none of the above.
219) the commonest nerve injured in posterior disloca-
tion of the elbow in children is:
a- median nerve. b- ulnar nerve.
c- posterior interosseous nerve. d- radial nerve.
220) the acetabular index is an angle formed between:
a- the hilgenreiners line and a line drawn along the acetabular sur-
face.
b- the perkins line and a line drawn along the acetabular surface.
c- the shenton’s line and a line drawn along the acetabular surface.
d- the shoemakers line and a line drawn along the acetabular sur-
face.
221) the graf classification of DDH is based on:
a- ultrasound of the hip.
b- combined radiography and ultrasound.
c- combined arthrography and radiography.
d- combined arthrography and ultrasound.

55
222) the treatment of stable SCFE is
a- observation.
b- traction to the hip.
c- hip spica immobilisation.
d- in situ pinning.
223) the most common etiological factor in recurrent pa-
tellar dislocation is:
a- shallow intercondylar notch. b- lateral soft
tissue contarctures.
c- lateral malalignment of the quadriceps mechanism. d- ex-
cessive external tibial torsion.
224) accessory navicular is also known as:
a- prehallux.
b- OS trigonum.
c- OS odontoidium.
d- OS fibulare.
225) kohler’s disease is an osteochondrosis of :
a- cuboid. b- navicular.
c- medial cuneiform. d- talus.
226) which is not acceptable for a humeral shaft fracture?
30 degrees varus
20 degrees apex anterior angulation
15 degrees rotation
3 cm shortening
227) what is true when comparing ulnar nerve transposi-
tion versus simple decompression for cubital tunnel?
a. lower recurrence for transposition

56
b. better outcomes for transpostion
c. similar functional outcomes and complication rates
d. similar functional outcomes with fewer complication
228) which would benefit LEAST from a IMN?
reverse obliquity
large posteromedial fragment
subtroch extension
4 part intertroch
229) varus malreduction of an intertroch fracture treated
with a DHS does what?
makes placement of the lag screw too inferior
aids in achieving the correct tip apex distance
Increase the lever arm of the construct
reduces lag screw cut out
230) you use an extramedullary guide for the tibial cut of a
total knee. If you don’t consider rotation of the tibia by plac-
ing the guide in the center of the ankle what would happen?
no change in coronal alignment
Varus coronal alignment
valgus coronal alignment
increased posterior slope in the sagital plane
231) which is true about achilles tendinitis?
it happens 10:1 times more in the general population than in run-
ners
pain is because of inflammation in paratenon
eccentric loading physsio is helpful
if conservative measures fail repair achilles tendon with plantaris
augmentation

57
232) How to fix a butterfly fragment of humeral shaft # in
a poly trauma pt.?
A-cast b-IMN c-LCP plate
d-Lag Screw and neutralization plate
233) most common presentation of necrotizing fasci-
itis;
a-fever b-erythema c-Pain d-blisters
234) When should you not operate on a polytrauma pa-
tient
a-T 35 b-Lactate 4.5
c-HR 110 BP 110/70 d-can’t remember

235) which is right about club foot casting


first start by correcting cavus by pronating the midfoot
final correction is progressively dorsiflex foot to 15 degrees
navicular is neutralized on talus , fore foot is inverted
correct adduction by abducting forefoot while in pronation and
counterpressure on talus
236) which is not an indication to prophylactically nail a
pathologic lesion
a-> 2mo life expectancy
b-2.5 cm lesion in proximal femur
c-avulsion LT
d-multiple lytic lesions
237) you decide to treat a lady with plantar fascitis surgi-
cally which should you not do?
release the distal tarsal tunnel
release the Abductor hallucis fascia

58
release 100 of plantar fascia
release 75 - 80% of the plantar fascia
238) what is not part of the test for posteromedial instabil-
ity of the elbow
a-flexion b-varus load
c-axial load d-supination

239) The principle of tension band can be applied by us-


ing all of the following fixation methods except?
a-ex-fix b-IMN
c-K-wires and cerclage d-plate
240) All increase stability for an ex-fix except
increase the number of bars
decrease the distance of the bar to the bone
increase the distance between pins on the same side of the fracture
increase the distance between the fracture and the pins
241) which is not true about ceramic on ceramic total
hips?
up to 10% can squeak
rate of breaking is 1/15000
there are multiple options for head and liner
you should never revise just the head
242) what is true about titanium plates
see less callous
they are stronger than stainless steel
cortical bone under the plate is less porous
they have greater tensile strength than stainless steel

59
243) which ankle fracture should you fix?
isloated lat mal that is lat displaced 3 mm
isolated lateral mallulus that is 3mm short
undisplaced medial mal
fibular avulsion with intact mortise
244) which of the following will reduce polyethelene wear?
ethylene oxide irradiation
congruent (concave on concave)tibial polyethelene ie joint conformity
metal backed polyethelene
use screws in the tibia
245) When data are combined from smaller studies into
a larger sample size, which can then be statistically evalu-
ated in a more robust fashion than the smaller samples, the
following term is applied
a-prospective study b-case-control study
c-cohort study d-meta-analysis
e-double-blind clinical trial
246) One of the following statements in the patellofemoral
joint reaction force (PJRF) is not correct
PJRF is the force compressing the patella against the femur
PJRF becomes greater with increasing quadriceps tension activity
PJRF becomes greater with increased knee flexion
during squatting, PJRF increases to twice body weight
in the weight-bearing leg, PJRF increases on climbing stairs
247) In TBI subjects the Functional Independence Mea-
sure (FIM) is inadequate in which of the following areas:
a-neuropsychological b-self care c-sphincter control
d-mobility e-locomotion

60
248) Which among the following statements concerning
the Barthel index is incorrect? 
It allows comparison between services
It has predictive value
It takes cognitive function into account
It assesses 10 aspects of daily life
Its validity has been studied extensively 
249) In cervical spine disorders a cervicothoracic stabili-
sation programme is designed to limit pain,maximize func-
tion and prevent further injury. Which of the following treat-
ment modalities should not be included in this rehabilitation
programme? 
postural training
eccentric isokinetic exercises of the neck extensors
training and coordination of the muscles in the neck area
isometric and isotonic resistance exercises of the cervicothoracic
muscles
proprioceptive feedback
250) Posttraumatic Myositis Ossificans (PMO) is a compli-
cation that can develop after a muscular contusion. Which
of the following statements is incorrect?
myositis ossificans is the formation of non-neoplastic cartilage or
bone in connective tissue and muscle
the quadriceps muscle is the most common site of involvement
PMO can be seen within 1 week on plain radiographic films
early detection is most sensitive with bone scan or ultrasonography
active PMO tends to stabilize in size in 3 to 6 months

61
251) Which of the following treatments is ineffective in
longstanding plantar fasciitis in a young man?
a-Local injection of hydorcortisone
b-Immobilisation of the foot and ankle
c-Protective heel insoles d-Iontophoresis
e-Extracorporal short wave therapy
252) Which one of the following features is not character-
istic of a fibromyalgia syndrome?
Diffuse muscle discomfort and pain are found
A disturbed, non restful sleep is found
Multiple discrete areas of localised tenderness are found
Gastro-intestinal symptoms are often present
The symptoms are accentuated by warmth
253) Which of the following is not true in spasticity follow-
ing traumatic brain injury (TBI) ?
spasticity of cerebral origin characteristically presents with lesser
extensor tone in the lower extremities and more tendency to spasms
compared with spinal cord spasticity
Diazepam and oral baclofen are recommended for spasticity
liver enzymes need to be monitored when using Dantrolene sodium
splinting techniques can be used both to decrease tone and to
stretch soft tissues
Botulinum toxin A is recommended for focal spasticity
254) Which of the following statements is not correct re-
garding the prognosis of traumatic brain injury?
duration of post-traumatic amnesia is a good indicator
school age children and young adults achieve better outcomes than
infants or older adults (> 45 years)

62
one or both non-reactive pupillary light reflexes is associated with
a poorer outcome
combined severe musculo-skeletal injuries predict worse outcomes
findings on CT scan of the brain are more sensitive comparedto MRI
as good predictor of outcome following severe traumatic brain injury
255) What initial action should one take for a paraplegic
patient suddenly presenting with increased spasticity?
Intensify rehabilitation
Rapidly increase the dose of antispastic medication
Diminish the ambient temperature of the patient’s room
Carry out a complete clinical examinaton
Measure the alkaline phosphatase
256) Which of the following is not a potential complication
of tracheostomy ventilation?
increased risk of respiratory tract infection
tracheal stenosis
paralysis of the posterior aspect of the palate
trache-oesophageal fistula
swallowing problems
257) The most likely diagnosis in this case of leg pain is
A-Periostitis b-Thrombophlebitis
c-Chronic compartment syndrome d-Stress fracture
e-Compression neuropathy of the superficial peroneal nerve
258) Which of the following conditions is unlikely to pro-
duce anterolateral leg pain ?
a. Periostitis of the fibula
b. Fibular stress fracture
c. Peroneal nerve entrapment

63
d. Fascial defect with muscle herniation
e.Popliteal artery entrapment syndrome
259- tuberculosis of the spine first starts in ?
a- vertebral body b- nucleolus pulposus
c- annulus fibrosis d- paravertebral joints
260- commonest site of skeletal tuberculosis is ?
a- tibia b- radius
c- humerus d- vertebrae
261- Jefferson’s fracture
a- c1 b- c2 c- c1 and c2 d- c2 and c3
262 - blount’s disease ?
a - genu valgus b- tibia vara c- flat foot
d- genu recurvatum
263 - bunion is ?
a – exostosis
b - adventitious bursitis lateral to first metatarsal
c - base of 1st metatarsal d - head of first metatarsal
264 - distal interphalangeal joint is not involved in ?
a- rheumatoid arthritis b- psoriatic arthritis
c- multicentric histiocytosis d- neuropathic arthropathy
265 - syme’s amputation involves which joint ?
a- ankle b- midtarsal
c- tarsometatarsal d- all of the above
266 - march fracture most commonly involves ?
a- 1st metatarsal b- 2nd metatarsal
c- 3rd metatarsal d- fibula

64
267- Regarding Consent and Capacity - who is able to
provide consent?
a. parents for an intellectually disabled adult child
b. verbal as good as written
c. intoxicated with ethanol
d. Hospital Administration
268- Regarding Triage what is true?
a. Urgency of a patient with intra pelvic Hemorrhage
b. Urgency of a patient with GCS of 4
c. Treatment of multiple fractured first
d. all of the above
269 - BOUCHARD’S NODES are seen in ?
a- proximal IP joints b- distal IP joints
c- sternoclavicular joints d- knee joints
270 - positivity of HLA B 27 in ankylosing spondylitis is ?
a- 96 % b- 10 % c- 78 % d- 100 %
271 - osteoarthritis involves all except ?
a- hip joint b- knee joint
c- distal interphalangeal joints
d- metacarpophalangeal joint of the thumb
e- shoulder joint
272 - metabolic bone disease is caused by excess intake
of which vitamin ?
a- vit A b- vit B c- vit C d- vit E e- all of the above
273 - osteoporosis is associated with ?
a- smoking b- reduced weight for height
c- family history present d- late child birth

65
274 - Trendelenburg test is positive due to injury of which
nerve ?
a- superior gluteal nerve b- inferior gluteal nerve
c- pudendal nerve d- obturator nerve
275 - Sever’s disease refers to ?
a- calcaneum b- radius
c- talus d- capitulum
276 - Lisfranc dislocation is ?
a- tarsometatarsal dislocation b- lunate dislocation
c- scaphoid dislocation d- posterior dislocation of the elbow
277 - pathological changes in caissons disease is due to
?
a - N2 b- O2 c- CO2 d- CO
278 - Housemaids knee is inflammation of which bursa?
a- subpatellar b- suprapatellar
c- infrapatellar d- pre patellar
279 - Which of the following structures is at risk of injury
while carrying out a posterior approach to the hip joint?
a. The superior gluteal nerve
b. The inferior gluteal nerve
c. The sciatic nerve
d. The superior gluteal artery
e. The obturator nerve
280 - . Which one of the following has the highest osteo-
genic potential?
a. Cancellous autograft
b. Cancellous allograft

66
c. Cortical autograft
d. BMP
e. Synthetic bone graft
281- Which of the following can lead to an increase in
physeal growth?
a. Vitamin A intoxication
b. Division of sympathetic nerve supply
c. Meningococcal septicaemia
d. Chemotherapy
e. Thalassaemia
282 - Rocker bottom foot is NOT seen in?
a- congenital vertical talus b- CTEV over correction
c- grice procedure d- Diabetic foot
283 - ganglion is most commonly seen over ?
a- dorsal aspect of wrist b- volar aspect of wrist
c- over forehead d- dorsum of the tongue
284 - calcitonin is secreted by ?
a- thyroid gland b- parathyroid gland
c- adrenal gland d- ovaries
285 - all of the following can cause osteoporosis except
a- hyperparathyroidism b- steroids use
c- fluorosis d- thyrotoxicosis
286 - Rugger jersey spine is seen in ?
a- fluorosis b- achondroplasia
c- renal osteodystrophy d- marfan’s syndrome

67
287 - spina ventosa is seen in ?
a- tuberculosis dactylitis b- meningo myelocele
c- malignant melanoma d- all of the above
288 - osgood schlatters disease affects ?
a- upper tibia b- lower tibia
c- distal femur d- proximal femur
e- patella
289 - stress fracture is most commonly seen in ?
a- 2nd / 3rd metatarsal bone b- talus
c- calcaneum d- metacarpals
290 - definitive diagnosis for blow out fracture of orbit is
by ?
a- CSF rhinorrhea b- loss of vision
c- tear drop sign d- fracture of the sphenoid
291 - Heberdens nodes are found in ?
a- PIP in osteoarthritis b- DIP in OA
c- PIP joints in RA d- DIP joints in RA
292 - Von Rosen splint is used in ?
a- Congenital dislocation of hip
b- congenital talipes equino varus
c- shaft of femur
d- scoliosis
293 - commonest cervical vertebral fracture
a- c2 b- c3 c- c4 d- c5
294- Which of the following is NOT a feature of an ideal
routine screening test?
a. High sensitivity B. HIGH SPECIfiCITY

68
c. Low incidence of disease d. Test has low morbidity
e. Treatment already available
295- A laceration of the radial nerve high in the axilla inter-
feres with all of the following functions, except:
a. Pronation of the forearm b. Extension of the elbow
c. Extension of the wrist d. Supination of the forearm
E. EXTENSION OF THE fingers
296- Which of the following factors is least predictive of a
poor outcome after whiplash injury?
a. Headache b. Immediate pain
C. PAIN ISOLATED TO NECK D. SEVERE PAIN IN FIRST WEEK
e. Shoulder pain
297- Which of the following statements concerning hemi-
arthroplasty of the hip for fracture is not correct?
a. Cemented implants are associated with a lower overall reopera-
tion rate
b. Cemented implants are associated with a lower peri-operative
fracture rate
c. Cemented implants carry the same overall complication rate as
uncemented implants
d. Cemented implants have the same survivorship as uncemented
implants
e. Cemented implants in general are associated with less pain on
mobilisation
298- In the rheumatoid hand with an intact extensor ten-
don mechanism, replacement of the metacarpophalangeal
joints might be considered if there is?
a. Gross joint destruction
b. Palmar and proximal dislocation of the base of the phalanx

69
C. Marked ulnar drift combined with (A) AND (B)
d. All of these e. None of these
299- Which of the following organisms is currently least
likely to be responsible for septic arthritis of the hip in a
child?
a. Enterobacter species
b. HAEMOPHILUS INflUENZAE
c. Methicillin resistant Staphylococcus aureus
d. Staphylococcus aureus
e. Streptococcus group A
300- Which of the following quadrants of the acetabulum
are most at risk for injury by screws DURING fiXATION OF
AN UNCEMENTED TOTAL HIP replacement?
a. Anterior-posterior and posterior-inferior b. Posterior-
superior and posterior-inferior
c. Anterior-superior and posterior-superior d. Anterior-
inferior and posterior-superior
e. Anterior-superior and anterior-inferior
301- Most post-operative deep infections in total hip re-
placement result from?
a. Airborne bacteria in the operating room
b. Post-operative wound contamination
c. Intra-operative septicaemia
d. Contaminated acrylic cement
e. Inappropriate antibiotic cover
302- Eliciting the impingement test in the hip involves
which of the following manoeuvres?
a. Flexion, abduction and external rotation

70
b. Flexion, adduction and internal rotation
c. Flexion, adduction and external rotation
d. Flexion, abduction and internal rotation
e. Extension, adduction and internal rotation
303-ANTERIOR DISLOCATION OF THE PROXIMAL inter-
phalangeal joint in the hand is associated with a great risk
of permanent impairment because of:
a. Rupture of the volar plate
b. Development of a Boutonniere deformity
c. Damage to the digital nerve
d. Damage to the digital artery
E. RUPTURE OF THE flEXOR TENDONS
304-.Supracondylar fractures in children are commonly
associated with:
a. Median nerve palsy b. Ulnar nerve palsy
c. Radial nerve palsy d. Anterior interosseous nerve palsy
e. Posterior interosseous nerve palsy
305- The most common organism responsible for an epi-
dural abscess is:
a. Streptococcus species b. Enterobacter species
c. Staphylococcus d. HAEMOPHILUS INflUENZAE
e. None of the above
306. Which carpal bone fracture causes Median nerve in-
volvement?
(a) Scaphoid (b) Lunate (c) Trapezium (d) Trapezoid
307. Biomechanics of internal fixation all are true except;
a. It should be the strongest
b. It should be the stiffest

71
c. Bone is strong but breaks under strong deformation
d. It provides temporary support.
308. The fixation construct shown to provide optimal fixa-
tion of an unstable vertically displaced transforminal sacral
fracture is:
a) An iliosacral screw into the S1 vertebral body
b) Iliosacral screws in S1 and S2
c) Tension band plate fixation
d) Triangular osteosynthesis
309. Which one of the following traumatic spinal fractures
places the neurologic structures at greatest risk?
a) compression fracture b) translational fracture
c) flexion-distraction fracture d) extension-distraction fracture
310. Which of the following statements is true regarding
neurogenic shock:
a) Neurogenic shock is due to severe blood loss associated with a
spinal cord injury.
b) Neurogenic shock can be diagnosed when there is hypotension
and tachycardia.
c) Neurogenic shock is due to increased parasympathetic tone.
d) Neurogenic shock is best treated with judicious use of fluids and
vasopressors.
e) Neurogenic shock is a sign of an incomplete spinal cord injury.
311. Spondylolysis (pars defect) is most widely believed
to be caused by what?
a. A congenital defect in the pars
b. An acute traumatic defect in the pars
c. A stress fracture of the pars

72
d. A benign neoplasm involving the pars
e. None of the above
312. Cauda equina compression (CEC) syndrome is most
typically manifested by:
a. Foot-drop
b. Severe back pain
c. Parasthesias in a nonanatomic distribution
d. Urinary retention
313. A sequestered disk herniation refers to:
a) Bulging of the nucleus through a weakened annulus
b) Rupture of the nucleus through the annulus
c) Rupture of the nucleus through the posterior longitudinal liga-
ment
d) Separation of a herniated fragment from the disk
314. Which of the following is the most common source of
infection in vertebral osteomyelitis:
a. Trauma b. Iatrogenic
c. Hematogenous spread d. Unknown mechanism
375. A Tillaux fracture of the distal tibia is the result of
what mechanism of injury:
(A) Supination (B) Pronation
(C) Dorsiflexion (D) External rotation
(E) Internal rotation
315. In performing a lateral approach to the calcaneus for
open reduction, internal fixation, the structure at risk is:
a) Lateral plantar artery b) Lateral plantar nerve
c) Dorsalis pedism artery d) Sural nerve
e) Superficial peroneal nerve

73
315. Union rates of the femur after antegrade or retro-
grade reamed intramedullary nailing are:
A) Higher for antegrade nailing
B) Higher for retrograde nailing
C) Identical
D) Dependent on location in the bone
E) None of the above
316.Which of the following tibial fractures is most likely to
have residual angulation (more than 5 degrees) after treat-
ment with a statically locked intramedullary rod:
a) A distal third oblique fracture
b) A proximal third metaphyseal fracture
c) A comminuted midshaft fracture
d) A transverse midshaft fracture
e) An open transverse fracture with a large butterfly fragment
317. The most important factor in predicting cutout of an
implant to repair intertrochanteric fractures of the hip is:
a) Size of the chosen screw
b) Posterior/inferior placement
c) Tip/apex distance
d) Pitch of the chosen screw
e) Angle of the plate
318. The optimal number of screws to repair displaced
fractures of the femoral neck is:
A) Three B) Five
C) Two D) Four E) Six
319. Angiography should be used in dislocations of the knee:
a) In the presence of asymmetric pulses

74
b) In all cases
c) With absent pulses only
d) Does not need to be used as long as Doppler pulses are audible
e) Has been replaced by magnetic resonance image scanning
320. When using the lesser trochanteric profile to assess
femoral rotation, a smaller lesser trochanter compared to
the uninjured side indicates:
a) External rotation deformity of the distal fragment
b) Internal rotation deformity of the distal fragment
c) A prior injury of the lesser trochanter
d) A varus deformity of the proximal segment
e) A valgus deformity of the proximal segment
321. The safest distance below the knee for placement of
external fixation wires is:
A) 5 mm B) 9 mm
C) 14 mm D) 19 mm E) 25 mm
322. Which of the following is not required for use of the
dynamic condylar screw (DCS) in a supracondylar femur
fracture
A) 4 cm of intact distal femur or easily reconstructable distal femur
B) Intact medial condyle
C) Healthy, nonosteoporotic bone
D) Intact lateral soft tissue envelope
E) Fracture without intracondylar extension
323. Clinical variables associated with a poor outcome
following calcaneal fractures include all of the following ex-
cept:
a) Age older than 50 years b) Increased body weight

75
c) History of heavy labor d) On workers’ compensation
e) Decreased body weight
324. Which of the following tibial plateau fractures is of-
ten associated with vascular injury:
a) Schatzker type I b) Schatzker type II
c) Schatzker type III d) Schatzker type IV
e) Bilateral fractures
325. Which of the following is an advantage of lateral posi-
tioning over supine positioning when performing antegrade
intramedullary nailing of a subtrochanteric femur fracture
with an intact lesser trochanter:
a) Provides improved pulmonary ventilation
b) Eliminates valgus sag at fracture site
c) Allows faster setup and positioning
d) Provides easier alignment of the distal segment to the flexed
proximal segment
e) Provides more accurate rotational alignment
326. The primary goal of the Neer classification of proxi-
mal humeral fractures is to:
a) Determine the position of fragments
b) Determine the best surgical approach
c) Delineate the number of fragments
d) Determine the vascularity of the articular segment
e) Determine whether the fracture has dislocated
327. Implants protruding beyond the medial humeral neck
can impinge on which of the following structures:
A) Axillary nerve B) Axillary vein
C) Radial nerve D) Axillary artery

76
E) Musculocutaneous nerve
328. Osteoporosis results in:
a. Increase in skeletal mass
b. Decrease in the amount of mineralized bone
c. No change in the mineral to matrix ratio
d. Decrease in the cortical diameter of a long bone
e. Increase in bone mineral density by DEXA scanning
329. Paget’s disease of bone is:
a. A disease of decreased bone turnover
b. Common in young males
c. Rarely seen in the skull
d. Is most common in the foot
e. None of the above
330. Bone as a tissue:
a. Is largely devoid of cells
b. Contains more water than cartilage
c. Contains type II collagen in its ground substance
d. Depends on its lamellar structure for strength
e. Has amorphous calcium phosphate as its major mineral constitu-
ent
331. Hyaline cartilage:
a. Is a hypocellular tissue
b. Depends on the glycosaminoglycans for resiliency
c. Is primarily water by weight
d. Is found in diarthrodial joints
e. All the above

77
332. Achondroplasia:
a. Is an aberration of epiphyseal growth
b. Is the most common skeletal dysplasia
c. Is characterized by knock knees
d. Is often associated with mental retardation
e. Is fatal in infancy
333. Rheumatoid arthritis:
a. Is a disease of the articular cartilage
b. Typically results in hyperostosis around the joints
c. Commonly causes early fibrosis of the synovium
d. Is considered a noninflammatory arthritis
e. Radiographically is characterized by juxtaarticular osteopenia
334. All the following are characterized by generalized os-
teopenia on the
radiograph except:
a. Gout b. Hyperparathyroidism
c. Osteomalacia d. Multiple myeloma e. Osteoporosis
335. Neuropathic arthritis is the result of proprioceptive
sensory loss in conjunction with microtrauma and a compo-
nent of vasomotor instability. All the following can cause a
neuropathic arthritis except:
a. Syphilis b. Polio c. Diabetes
d. Heavy metal intoxication e. Leprosy
336. Avascular necrosis of bone:
a. Can result from exogenous steroid administration
b. Is characterized by relative radiodensity
c. Can involve the femoral head and talus among other bones

78
d. Can be associated with sickle cell disease
e. All the above
337. Collagen:
a. Is a proteoglycan
b. Is important for the compressive strength of cartilage
c. Is synthesized by chondroblasts and osteoblasts
d. Is a large ring molecule
e. Is not a cross-linked molecule
338. Regarding osteomalacia;
(a) It is due to vitamin A deficiency
(b) Can cause a distal myopathy
(c) May present with pseudo-fractures
(d) Serum calcium is increased
(e) A bone biopsy would show an increase in mineralised osteoid
339. Perthe’s disease all are true except;
(a) Usually presents before 10 years of age
(b) Is due to avascular necrosis of the distal femoral epiphysis
(c) Is more common in boys
(d) Plain x-ray may show the capital femoral epiphysis to be smaller,
denser and flatter
(e) May require surgical containment with a subtrochanteric oste-
otomy
340. Regarding upper limb peripheral nerve injuries all
are true except;
(a) Injury to the posterior interosseus nerve results in a wrist drop
(b) Injury to the median nerve results in loss of sensation over the
palmar aspect of the index finger

79
(c) Injury to the radial nerve results in loss of sensation in the ana-
tomical snuffbox
(d) Injury to the ulnar nerve results in a claw hand
(e) Injury to the ulnar nerve results in loss of sensation over the
thumb
341. Regarding eponymous fractures;
(a) Bennett’s fractures involves the thumb
(b) Colles’ fracture involves the distal radius
(c) Galeazzi’ s fracture involves the radial shaft & dislocation of the
proximal radioulnar joint
(d) Monteggia’s fracture involves the proximal ulna & anterior dislo-
cation of the head of the radius.
(e) Pott’s fracture is a general term applied to fractures around the knee
342. Regarding rheumatoid arthritis all are true except;
(a) Is a disease primarily of the articular cartilage
(b) Is associated with the HLA antigens DR4 and DW4
(c) Occurs more often in women
(d) The hand, elbows, knees and cervical spine are the commonest
joints involved
(e) Extra-articular manifestations occur in 20% of patients
343. Paget’s disease of bone all are true except ;
(a) May present with neurological complications or a pathological
fracture
(b) Causes an increase in both serum calcium and phosphate
(c) Causes an increase in serum alkaline phosphatase
(d) Malignant change occurs in 1% of patients
(e) The commonest malignant tumour in patients with Paget’s dis-
ease is an osteosarcoma

80
344. The leading cause of perioperative death in hip ar-
throplasty surgery is
A. blood loss
B. bone cement implantation syndrome
C. myocardial infarction
D. pneumonia
E. pulmonary embolus
345. Each of the following statements concerning surgery
for correction of scoliosis is true EXCEPT
A. one third of the blood loss occurs postoperatively
B. major blood loss is frequently accompanied by a consumptive
coagulopathy
C. surgery will halt progression of the restrictive lung deficit
D. the major neurological deficits that occur are usually due to dam-
age to the posterior columns of the spinal cord
E. the use of aprotinin reduces blood loss
346. Acute compartment syndrome in the lower limb
A. can be reliably detected by loss of peripheral pulses
B. does NOT occur after open fractures of the tibia
C. is a contraindication to regional anaesthesia
D. is often associated with pain on passive stretching of the affected
compartment
E. occurs more commonly in patients over 35 years of age
347. The single most important treatment in preventing
acute renal failure following crush injury is
A. maintenance of an alkaline urine (pH > 6) to prevent cast forma-
tion
B. maintenance of adequate urine output with mannitol

81
C. vigourous intravenous fluid replacement
D. administration of xanthine oxidase inhibitors to prevent hyper-
uricemia
E. emergent dialysis to remove myoglobin from the circulation
348. Fat embolus:
A. 30% mortality
B. Only with orthopaedic injuries
C. Petechial rash from thrombocytopenia
D. 5% incidence with pelvic and long bone fractures
E. Decreased incidence with early fixation of fractures
349. The commonest organism cultured from septic ar-
thritis (?knee) is:
A. Neisseria gonorrhea B. Strep pyogenes
C. Staph aureus D. Staph epidermidis
E. E. coli
350. Meralgia paraesthetica is relieved by nerve block of
the:
A. Lingual nerve
B. Trigeminal nerve
C. Lateral femoral cutaneous nerve
D. Lumbar sympathetic chain
E. Femoral nerve
351. The sign causing the most concern in a patient with
fat embolism is:
A. Dyspnoea B. Fat globules in the urine
C. fatty patient D. Fat in sputum
E. Petechial rash over the upper chest and shoulders

82
352. Which of the following is the correct position to
splint a hand after injury?
a. Wrist 0° , MPJ flexion 0° , IPJ flexion 20°, thumb abduction
b. Wrist 0o, MPJ extension, IPJ flexion 30o, thumb abduction
c. Wrist 20° flexion, MPJ flexion 10°, IPJ flexion 30°, thumb abduc-
tion
d. Wrist 30° extension, MPJ flexion 70°, IPJ flexion 0°, thumb abduc-
tion
e. Wrist 30° extension, MPJ flexion 90°, IPJ flexion 0°, thumb adduc-
tion
353. Why is the wrist joint usually splinted in 30° exten-
sion?
a. To ensure distal glide of the extensor hood over the proximal pha-
lanx
b. To help maintain the length of the extrinsic extensors
c. To help maintain the length of the extrinsic flexors
d. To maintain the length of the collateral ligaments
e. To prevent an extensor tendon lag
354. Which of the following investigations is the best di-
agnoses bone mineral density assessment in a patient at
risk of osteoporosis?
a. 24 hour urine calcium excretion
b. Dual-energy X-ray absorptiometry
c. Plain radiograph of the spine
d. Technetium radioisotope bone scan
e. Serum parathyroid hormone

83
355-Tibial tubercle osteotomy has proven to be an effec-
tive treatment in the management of articular cartilage inju-
ries of the patella. It is contraindicated for chondral injuries
in what location on the patella?
a-Lateral b-Distal c-Proximal d-Central e-Medial
356-For potential injury to the S2 nerve root following a
sacral fracture, the physician should test for
a-perianal sensation.
b-great toe dorsiflexion.
c-ankle toe plantarflexion.
d-sensation on the medial border of the foot.
e-sensation in the first web space of the foot
357-For the purposes of retractor and screw placement,
the anterior and posterior zones of the acetabulum are di-
vided by a line
a-directly along the body axis.
b-between the posterior superior iliac spine and the obturator foramen.
c-between the anterior superior iliac spine and the ischial tuberosity.
d-between the anterior superior iliac spine and the center of the
acetabulum.
d-from the center of the acetabulum to the transverse acetabular
ligament.
358-The anterolateral approach to the distal tibia and an-
kle for open reduction and internal fixation of pilon fractures
places which of the following nerves at most risk?
a-Sural b-Saphenous c-Deep peroneal
d-Medial plantar e-Lateral plantar

84
359- Secondary osteosarcomas have been observed most
frequently in which of the following conditions?
a-Melorheostosis b-Paget’s disease
c-Cortical desmoids d-Fibrous dysplasia
e-Osteoporosis in patients treated with bisphosphonates
360-Femoral nailing through the piriformis fossa starting
portal is contraindicated in adolescents with open physes
because of the risk of ;
a-injury to the sciatic nerve.
b-injury to the greater trochanteric apophysis.
c-injury to the medial femoral circumflex artery.
d-injury to the lateral femoral circumflex artery.
e-increased intramedullary pressure.
361-Which of the following patients would be the best can-
didate for surgical “guided growth technique” using a tension
band plate(s) across the medial distal femoral growth plate(s)?
a-Bone age 5-year-old girl with bilateral genu varum with wide ir-
regular growth plates
b-Bone age 11-year-old girl with bilateral rachitic genu varum
c-Bone age 11-year-old girl with bilateral severe genu valgum
d-Bone age 12-year-old boy with unilateral posttraumatic genu val-
gum and a lateral femoral growth arrest
e-Bone age 12-year-old boy with unilateral posttraumatic genu var-
um and a medial femoral growth arrest
362-Neuroblastoma is the most common metastatic ma-
lignant solid tumor in childhood and arises from which of
the following?
a-Eosinophils b-Histiocytes c-Macrophages
d-Nucleus pulposus cells e-Neural-crest derived cells

85
363-The muscle contraction characterized by constant
muscle tension through the range of motion is called
a-isotonic b-isometric. c-isoelastic.
d-isokinetic. e-isoconcentric.
364-What is the most common complication of the Weil
distal metatarsal osteotomy when combined with a proximal
interphalangeal joint resection arthroplasty?
a-Floating-toe deformity
b-Nonunion of the metatarsal osteotomy
c-Recurrent proximal interphalangeal joint deformity
d-Recurrent dislocation of the metatarsophalangeal joint
e-Intractable plantar keratosis at the metatarsophalangeal joint
365-Which of the following has been shown to be a risk
factor for pseudarthrosis following long adult spinal defor-
mity surgery?
a-Thoracoabdominal approach
b-Kyphosis of less than 20 degrees
c-Preexisting knee osteoarthritis
d-Positive sagittal balance of less than 2 cm
e-Instrumentation of the upper thoracic spine
366-When comparing outcomes between hemiarthroplas-
ty and total shoulder arthroplasty for patients with symptom-
atic osteoarthritis with an intact rotator cuff and a concen-
tric joint, hemiarthroplasty results in which of the following?
a-Equivalent pain relief
b-Increased blood loss
c-Increased rate of revision
d-Increased rate of instability

86
e-Increased incidence of subscapularis insufficiency
367-Which of the following fluoroscopic views is required
for the proper introduction of iliosacral screws within the
pelvis?
a-Anteroposterior b-Iliac oblique
c-Obturator oblique d-Obturator inlet
e-Sacral lateral
368-Lateral retinacular release is most indicated in the
treatment of
a-patella alta.
b-patellar subluxation.
c-patellofemoral instability.
d-symptomatic medial synovial plica.
e-lateral facet compression syndrome.
369-The highest rate of associated nerve injury in sacral
fractures occurs in
a-Denis 1, which is a fracture lateral to the foramen.
b-Denis 1, which is a fracture medial to the foramen.
c-Denis 2, which is a fracture through the foramen.
d-Denis 3, which is a fracture lateral to the foramen.
e-Denis 3, which is a fracture medial to the foramen.
370-Approximately how many milligrams of calcium are
in a standared cup of milk?
a-75 b-150 c-300 d. 500 e-
1000
371--A nerve palsy involving what nerve is most common
with halo cervical traction?
a-Cranial nerve V b-Cranial nerve VI

87
c-Cranial nerve VII d-Cranial nerve X
e-Cranial nerve XII
372-What type of femoral fracture is best suited for locked
plate fixation?
a-Lateral femoral condyle b-Comminuted diaphyseal
c-Comminuted metaphyseal d-Short oblique diaphyseal
e-Short oblique metaphyseal
373-What type of tissue has the greatest blocked expres-
sion by selective Cox-2 inhibitors?
a-Platelet b-Periosteum c-Vascular endothelium
d-Tracheal endothelium e-Gastrointestinal epithelium
374- What characteristic(s) do calcium sulfate and calci-
um phosphate cements share in common?
a-Both have similar porosity
b-Both have similar resorption rate
c-Both have similar compressive strength
d-Both have osteoinductive properties
e-Both resist tension and shear stresses poorly
375-What tract is the main descending motor pathway in
the cervical spinal cord?
a-Posterior column b-Lateral corticospinal
c-Anterior corticospinal d-Lateral spinothalamic
e-Anterior spinothalamic
376-What is the most important predictor of infection af-
ter an open type III tibial fracture?
a-Time to transfer to definitive trauma center
b-Having an antiseptic dressing placed in the field
c-Fracture stabilization within 6 hours

88
d-Getting to the operating room within 6 hours
e-Receiving broad-spectrum antibiotics within 12 hours
377-Which of the following conditions is associated with
renal osteodystrophy?
a-Hypoparathyroidism b-Hyperparathyroidism
c-Hypothyroidism d-Hyperthyroidism
e-25 (OH) vitamin D2 deficiency
378-Which of the following is the most common neurolog-
ic complication following surgical reduction of a high-grade
spondylolisthesis at L5-S1 in a pediatric patient?
a-L4 nerve root injury b-L5 nerve root injury
c-S1 nerve root injury d-S2 nerve root injury
e-Cauda equina syndrome
379-In which of the following scenarios is intraosseous
infusion considered most appropriate?
a-Adult in a hypovolemic state
b-Required infusion rate in excess of 40 mL/min
c-Required rapid delivery of blood products and antibiotics
d-Previous attempted access at same site within 24 hours
e-Normotensive child older than 8 years with poor peripheral IV ac-
cess
380-Bone absorption that occurs with multiple myeloma
is associated with which of the following?
a-Increased RANKL/osteoprotegerin ratio
b-Decreased levels of macrophage colony stimulating factor
c-Decreased parathyroid-related hormone (PTrH)
d-Decreased TGF beta receptor
e-Suppression of 1,25 dihydroxy vitamin D

89
381-With the forearm in full supination, the superficial ra-
dial nerve exits the muscular fascia to become subcutane-
ous along the
a-midportion of the supinator.
b-volar side of the brachioradialis.
c-dorsal side of the brachioradialis.
d-radial side of the flexor carpi radialis.
e-ulnar side of the flexor carpi radialis.
382-When does the medial epicondyle apophysis of the
elbow close in a male?
a-4 to 7 years b-7 to 10 years c-10 to 13 years
d-13 to 16 years e-16 to 19 years
383-Quinolone antibiotics, such as ciprofloxacin, exert
their effect through the inhibition of which bacterial enzyme?
a-DNA gyrase b-DNA synthase c-RNA methylase
d-RNA polymerase e-Beta-lactamase

384-After fracture of the articular surface, free devascu-


larized segments of cartilage and bone are created. What is
the best way to provide nutrition to surviving chondrocytes?
a-Hyperbaric oxygen therapy
b-Joint range of motion
c-Static positioning of the articulation
d-Slight distraction of the articular surface
e-Immediate weight bearing through opposing articular surface

90
385-Orthopaedic surgeons performing history and exami-
nations on patients receive the lowest patient satisfaction
scores concerning what part of the interaction?
a-Not examining the patient
b-Not dealing with pain issues
c-Not spending enough time with the patient
d-Not getting a good history of the problem
e-Failure to offer empathetic responses
386-During cyclical loading of tendons and ligaments, the
decrease in peak loads over time with the same amount of
elongation is referred to as which of the following?
a-Creep b-Wolff’s law c-Hueter-Volkmann law
d-Load relaxation e-Elastic elongation

387-Which of the following diagnoses has the best 5-year


survival if no metastatic disease is found at the time of di-
agnosis?
a-Ewing’s sarcoma
b-Parosteal osteosarcoma
c-Periosteal osteosarcoma
d-High-grade surface osteosarcoma
e-Secondary osteosarcoma from radiation therapy
388-While revising the acetabulum, a screw is placed in
the posterior superior quadrant exiting the cortex. What
neurovascular structure is at risk?
a-Sciatic nerve b-Obturator nerve
c-Femoral artery d-External iliac vein
e-Superior gluteal vein

91
389- During the deceleration phase of throwing a ball,
which of the following is the principal decelerator and is
susceptible to tensile failure due to eccentric loading?
a-Deltoid b-Brachialis c-Rotator cuff
d-Pectoral major e-Latissimus dorsi
390-Which of the following is considered an advantage
of the deltopectoral approach compared with a transdeltoid
(“deltoid splitting”) approach when managing a proximal
humerus fracture with plate fixation technique?
a-Less risk to the axillary nerve
b-Less disruption to remaining osseous blood supply
c-Less necessity for partial detachment of the deltoid insertion
d-Enhanced access to a greater tuberosity fracture
e-Exposuredirectly overlies the application site of the proximal por-
tion of the plate
391-A child with a Pulled or nursemaids’ elbow postures
the arm in what position?
a-Full elbow extension and forearm pronation
b-Full elbow extension and forearm supination
c-Slightly flexed elbow and forearm pronation
d-Elbow flexion of at least 130 degrees and forearm pronation
e-Elbow flexion of between 100 and 130 degrees and forearm su-
pination
392-In the stress-strain curve, the irreversible change in
length after removing the load during the plastic range is
called
a- elastic limit. b- proportional limit.
c- breaking point. d- ultimate strength.
e- plastic deformation.

92
393-Protraction of the scapula between the preparing to
throw and acceleration phases of throwing is mediated by
what nerve?
a-Long thoracic b-Thoracodorsal
c-Dorsal scapular d-Spinal accessory
e-Medial pectoral
394-What is the most common anatomic variant in the re-
lationship of the short external rotators and the sciatic nerve
at the hip joint?
a- It passes, unsplit, through the piriformis muscle.
b-It travels between the piriformis and capsule, exiting posteriorly
above the superior gemellus.
c-It travels between the capsule and the short external rotators exit-
ing below the superior gemellus.
d- It splits above the piriformis with one branch passing through the
muscle and the other below it.
e-It splits by the piriformis muscle so that a portion of the nerve is
anterior and a portion is posterior.
395-The principles of tension band fixation are most ap-
plicable to which of the following fractures?
a-Comminuted patella
b-Comminuted olecranon
c-Non-comminuted tibial tubercle
d-Non-comminuted subtrochanteric femoral
e-Non-comminuted mid-femoral with vascular compromise requir-
ing plate fixation medially
396-With respect to the following, in what manner have com-
puter-assisted navigation systems proved advantageous?
a-Improved fracture healing times for humeral shaft fractures treat-
ed with medullary nails

93
b-Improved long-term patient outcomes after percutaneous iliosa-
cral screw fixation for pelvic ring disruptions
c-Reduced overall surgical time for medullary nailing of femoral
shaft fractures
d-Reduced bladder perforation after percutaneous posterior col-
umn fixation for acetabular fractures
e-Reduced radiation exposure for surgeons performing freehand in-
terlocking of medullary nails
397-Which of the following describes the phenomenon
that occurs when a material is subjected to numerous load-
ing cycles and then fails at a stress level lower than the ulti-
mate stress for that material?
a-Creep b-Fatigue c-Toughness
d-Viscoelasticity e-Plastic deformation
398-Olecranon fracture fragment excision and triceps ad-
vancement is best indicated in which of the following sce-
narios?
a-A 30-year-old woman with a closed comminuted fracture involv-
ing more than 50% of the joint surface
b-A 30-year-old woman with an open olecranon fracture which is
proximal to the “bare spot”
c-A 55-year-old woman with an oblique olecranon fracture through
the coronoid process
d-An 85-year-old man with a comminuted fracture involving less
than 50% of the joint surface
e-An 85-year-old man with an oblique fracture through the coro-
noid process
399-In the evaluation of a patient, which of the following is
a nonorganic sign of low back pain?
a-Proportional verbal or physical reaction

94
b-Positive straight-leg raise at 30 degrees
c-Back symptoms with axial loading
d-Hyperactive reflexes with patient distraction
e-Dermatomal sensory loss in the lower extremities
400-The progressive permanent deformation of materials
in response to a constant force applied over an extended
period of time is called
a-creep. b-ductility. c-anisotropy.
d-fatigue failure. e-viscoelasticity.
401-What factor differentiates a Pipkin I from a Pipkin II
femoral head fracture?
a-Degree of comminution
b-Direction of the hip dislocation
c-Location of the femoral head fracture
d-Presence of an associated femoral neck fracture
e-Size of the femoral head fracture fragment
402-The proton energy produced by a 3.O Tesla machine
is how many times greater than a 1.5 Tesla machine?
a-2 b-4 c-6 d-9 e-12
403-Internal impingement of the shoulder may be best described as
contact between the
a-lesser tuberosity and the coracoid.
b-supraspinatus and the coracoacromial arch.
c-long head of the biceps and the coracoacromial ligament.
d-undersurface of the teres minor and the posterior glenoid labrum.
e-posterosuperior glenoid labrum and the greater tuberosity.

95
404-A comminuted olecranon fracture is best treated with
which of the following constructs?
a-Bridge plating
b-Compression plating
c-A 7.3-mm lag screw alone
d-A 7.3-mm lag screw and tension band
e-Kirschner wire and tension band construct
405-Fanconi anemia is associated with what congenital
deformity?
a-Brachydactyly b-Preaxial polydactyly
c-Postaxial polydactyly d-Ulnar longitudinal deficiency
e-Radial longitudinal deficiency
406-Core decompression of the distal radius for the treat-
ment of Kienböck’s disease is thought to work through
which of the following mechanisms?
a-Unload the lunate fossa
b-Increase force distribution
c-Decrease distal radius stiffness
d-Decrease excessive intraosseous pressure
e-Incite local vascular healing response
407-What is the single most important predictive factor
for successful healing of a Syme ankle disarticulation?
a-Serum albumin level greater than 2.5
b-Ischemic index level greater than 0.45
c-Transcutaneous O2 levels of greater than 30
d-Total lymphocyte count of greater than 1,000
e-Posterior tibial artery Doppler signal

96
408-Which of the following nerves may travel within 1 mm
of the inferior shoulder capsule and glenoid rim and is at
risk during suture passage at the posterior inferior glenoid
for shoulder stabilization procedures?
a-Suprascapular b-Medial pectoral
c-Spinal accessory d-Lower subscapular
e-Posterior branch of the axillary
409-During placement of an iliosacral screw into the body
of S1, it is important to obtain what pelvic radiographic view
to help avoid injury to the S1 nerve root?
a-AP b-Lateral c-Inlet
d-Outlet e-Obturator oblique
410-What nerve is most frequently injured during a two-
incision distal biceps repair?
a-Ulnar b-Median c-Anterior interosseous
d-Posterior interosseous e-Lateral antebrachial cutaneous
412-What gene product is most likely associated with mul-
tiple osteoblastic lesions due to metastatic prostate cancer?
a-BMP 4 b-Matrilin 3 c-Collagen Ib
d-Endothelin 1 e-ACVR1 receptor
413-Which of the following methods minimizes risk of fix-
ation cut-out in an unstable 4-part intertrochanteric femur
fracture fixed with a compression hip screw?
a-Use of a supplemental anti-rotation screw
b-Use of an extra large screw and deep thread design
c-Use of a Dimon-Hughston medialization to allow load sharing
d.Screw tip placement with a tip-apex distance of less than 25 mm
e.Screw tip placement in the inferior hemisphere of the femoral
head

97
414-Cardiomyopathy is one of the dose-limiting toxicities
of which of the following chemotherapy drugs?
a-Cisplatin b-Doxorubicin c-Iphosphamide
d-Methotrexate e-VP-16
415-A hip arthroplasty performed through the anterior ap-
proach uses the plane between what nerves?
a-Superior gluteal and femoral
b-Inferior and superior gluteal
c-Obturator and superior gluteal
d-Obturator and lateral femoral cutaneous
e-Lateral femoral cutaneous and superior gluteal
416-Which anatomic factor best differentiates an APC-II
from an APC-III pelvic fracture pattern?
a-Amount of pubic symphysis diastasis
b-Disruption of the sacrotuberous ligament
c-Disruption of the anterior sacroiliac ligament
d-Disruption of the posterior sacroiliac ligament
e-Associated “crescent” fracture of the posterior iliac wing
417-Fibrodysplasia ossificans progressiva, characterized
by massive formation of spontaneous heterotopic bone, is
caused by an altered signal transduction of which of the fol-
lowing proteins?
a-BMP-2 b-BMP-4 c-BMP-5
d-BMP-6 e-BMP-7
418-Teriparatide (Forteo) is contraindicated in patients
with which of the following medical conditions?
a-Hypocalcemia b-Renal failure c-Osteoarthritis
d-Paget’s disease e-Rheumatoid arthritis

98
419- Which of the following is predictive of a poor out-
come following latissimus dorsi transfer?
a-Deltoid tear b-Teres minor tear
c-Subscapularis tear d-Supraspinatus tear
e-Long head of the biceps tear
420- In designing a posterior cruciate ligament (PCL) re-
construction rehabilitation program, which of the following
activities generates the least amount of stress within the
PCL graft?
a-Isokinetic knee flexion
b-Non-weight-bearing resisted knee flexion
c-Non-weight-bearing resisted knee extension (60 to 0 degrees)
d-Non-weight-bearing knee extension from full flexion to about 70
degrees
e-Weight-bearing exercises with depths of greater than 70 degrees
of knee flexion
421- In the preparation and administration of platelet-rich
plasma, what is the role of calcium chloride?
a-Reduced viscosity b-Reduced immunogenicity
c-Initiation of platelet activation d-Increased angiogenesis
e-Increased expression of prostaglandin E2
422-Tumorigenesis, or the development of a tumor with
the ability to metastasize, is a multi-step process character-
ized as which of the following?
a-Genomic stability
b-Sustained angiogenesis
c-Programmed cell death (apoptosis)
d-Protease activity down-regulation
e-Limited replicative potential (telomerase activity)

99
423-Which of the following statements is true about the
use of a cane?
a-Use in the ipsilateral hand reduces the abductor angle.
b-Use in the contralateral hand may reduce joint reactive force of
up to 50%.
c-Cane use reduces the body weight leverarm.
d-Cane use will prolong the life of total hip arthroplasty.
e-Cane use cannot change the joint reactive force, only improve bal-
ance.
424-Which of the following treatments has been shown
to improve bone density, decrease fracture incidence, and
improve function when administered to young children with
Sillence type 3 osteogenesis imperfecta?
a-Etanercept b-Chondroitin
c-Bisphosphonates d-Collagen type A
e-1,25-dihydroxy vitamin D
425-Which of the following findings are typical of femoral
fractures that occur in patients after long-term alendronate
therapy?
a-Low energy, spiral, atrophic cortices
b-Low energy, transverse, atrophic cortices
c-Low energy, transverse, hypertrophic cortices
d-High energy, transverse, hypertrophic cortices
e-High energy, transverse, normal cortices
426- During a lateral approach to the shoulder, the axillary
nerve can be found at approximately what distance from the
tip of the acromium?
a-3 cm b-5 cm c-7 cm d-9 cm e-11 cm

100
427-When considering implant options for managing a
displaced subtrochanteric fracture of the proximal femur,
what is the principal benefit of intramedullary nailing in con-
trast with plating?
a-Diminished injury to the abductors
b-Implant insertion facilitates reduction
c-More ideally manages preexisiting deformity
d-Biomechanic superiority and diminished risk of implant failure
e-Uniquely suited to indirect reduction and minimally invasive fixa-
tion
428-The anterior interosseous nerve innervates which of
the following muscles?
a-Flexor digitorum profundus to the index and middle fingers, flexor
pollicis longus, pronator quadrates
b-Flexor digitorum profundus to the index, middle, ring, little fin-
gers, flexor pollicis longus, pronator quadratus
c-Flexor digitorum profundus to the ring, little fingers, flexor pollicis
longus, flexor pollicis brevis
d-Flexor digitorum profundus to the index, middle fingers, flexor
pollicis longus, abductor pollicis brevis
e-Flexor digitorum profundus to the index, middle fingers, flexor
pollicis longus, lumbricals to the index and middle fingers
429-Where does the piriformis muscle originate?
a-Inner table of the iliac wing
b-Inner side of the obturator membrane
c-Outer table of the iliac wing
d-Outer side of the obturator membrane
e-Ventral surface of the sacrum

101
430- Which of the following best describes the technique
used to obtain a false profile view of the hip?
a-Standing AP position with the hip flexed 65 degrees
b-Standing position at an angle of 65 degrees between the pelvis
and the film
c-Supine position with the hip abducted 65 degrees
d-Supine position with an oblique angle of 65 degrees between the
pelvis and the film
e-Sitting position with an oblique angle of 65 degrees between the
pelvis and the film
431- Compared with standard infrapatellar nailing of a
proximal-third tibia fracture, the semi-extended technique
for intramedullary nailing is beneficial because it
a-does not violate the intra-articular space of the knee.
b-minimizes the apex anterior deformity at the fracture site.
c-reduces the risk of iatrogenic injury to the menisci.
d-reduces the risk of saphenous nerve injury.
e- increases the length of the nail that can safely be inserted.
432- Which of the following is most commonly associated
with the use of calcium sulfate alone in the treatment of frac-
ture nonunions?
a-90% rate of healing of the nonunion
b-Increased osteoinductivity at the nonunion site
c-Increased rate of infection at the surgical site
d-Increased rate of serous drainage at the surgical site
e-Increased likelihood of refracture after successful healing of the
nonunion

102
433- A disorder transmitted as an autosomal-dominant
trait with genetic anticipation implies which of the follow-
ing?
a-25% of offspring, both males and females, will have the disorder
and the ones who inherit the disease will be likely to have a less severe
disease than the involved parent.
b-50% of male and female offspring will have the disorder and the
ones who inherit it will be likely to have a more severe disease than the
involved parent.
c-50% of male offspring will have the disorder and females who in-
herit the disease will have fewer tandem repeats than the involved
parent.
d-50% of female offspring will have the disorder and males who in-
herit the disease will have fewer tandem repeats than the involved
parent.
e-The percentage of offspring who inherit the disorder increases
with each subsequent generation.
434-When comparing multiply injured patients with and
without foot injuries, the SF-36 scores in those patients with
foot injuries shows a:
Greater mean score b-Lower mean score
c-Lower general health score d-Greater vitality score
e-Lower mental health score
435-Which of the following amputation levels has the low-
est energy cost of walking?
a-Transtibial b-Transfemoral
c-Midfoot d-Syme( through the ankle)
e-Through the knees

103
436-Which of the following structures is the primary an-
tagonist to the tibialis anterior tendon?
a-Flex. Hall. Longus b-Peroneus longus c-Peroneus brevis
d-Posterior tibial tendon e-TendoAchilis
437-Which nerve is most likely injured in the anterior por-
tal for THR?
a-Lateral femoral cutaneous b-Superior gluteal
c-Inferior gluteal d-Femoral
e-Sciatic
438-A low grade soft tissue sarcoma of thigh is completely
excised. What is the first common location for recurrence?
a-Thigh b-Lymph nods c-Skeleton
d-Liver e-Lung
439-Bisphosphonates are clinically used for osteoporo-
sis or osteolytic metastatic bone cancers. What is the com-
mon mechanism of pharmacologic action?
Increased mineralization of bone matrix
Increased apoptosis of osteoclasts
Increased production of alpha-v-beta-3 integrin by osteoclasts
Potentiation of PTM-related peptide effect on osteoblasts
Amplification of BMP-2 signal
440-Submuscular plating techniques, when compared to
conventional plating techniques, offer;
Less compromise to medullary and periosteal perfusion.
Greater compromise to medullary and periosteal perfusion.
No difference with regard to periosteal perfusion only.
No difference with regard to medullary perfusion only.
No difference with regard to periosteal and medullary perfusion.

104
441-Hip pointers are contusion injuries to what area of the
hip?
a-Iliac wing b-Greater trochanter c-Hip joint
d-Quadriceps muscle e-Femoral shaft
442-A bone marrow biopsy is a routine part of the staging
work-up for what type of sarcoma?
a-Osteosarcoma b-Synovial sarcoma
c-Ewing’s sarcoma d-Fibrosarcoma
e-Chondrosarcoma
443-What is the most appropriate treatment method for a
displaced vertical medial malleolus fracture?
a-MUA POP b- Medial antiglide plate c-Mallular screw
d-TBW e-Percutaneuos k-wire
444-A stress fracture is most prone to development of a
nonunion or delayed union in which of the following areas?
a-Anterior tibial cortex Stress fractures
b-Metatarsals c-Navicular d-Neck of femur
e-Great toe sesamoid
445-The strength of flexor tendon repair can be increased
by all of the following techniques except;
repair of the flexor tendon sheath
increasing the size of the core suture
increasing the number of core suture strands that cross the repair
site
adding epitendonous suture
adding locking loops to the core suture

105
446-Demineralized bone matrix contains all of the follow-
ing except
a-collagen
b-bone morphogenetic proteins
c-transforming growth factor-beta
d-mesenchymal precursor cells
e-residual calcium
447-The Pathophysiology of age-related disc degenera-
tion involves all of the following except;
a relative change in the proportions of keratin sulfate and chondro-
tin sulfate with an increase in the latter
fragmentation of large aggregated proteoglycans and an increase in
small and nonaggregated proteoglycans
unchanged absolute quantity of collagen
a decrease in water content
a decrease in the absolute number of cells in the nucleus pulposus
448-All of the following are risk factors for wound compli-
cations following Achilles tendon repair except;
a-tobacco use b-steroid use c-female gender
d-diabetes mellitus e-timing of surgery
449-Factors contributing to an apex anterior sagittal plane defor-
mity when introducing an intermedullary nail to manage a proximal
tibial fracture include all the following except;
a-anterior starting hole b-interlocking in flexion
c-posterior cortical comminution d-posterior directed tibial nail
e-posterior placed blocking screw

106
450- Herniated discs are associated with a spontaneous
increase in production of all of the following, except;
nitric oxide
b-prostaglandin E2
c-interleukin-6
d-transforming growth factor-beta
e-tumor necrosis factor-alpha
451-The concept of “periodization” or scheduling the fre-
quency, duration, and intensity of work an athlete does dur-
ing various periods of an athletic season, is an integral part
of this program. The phases have been described in this
program are except:
active rest (brief period of total rest followed by cross-training);
off season (emphasizes general athletic fitness, flexibility, and
weight training);
preseason (sports-specific skills are practiced and refined);
early in-season (maintenance of condition with recovery periods af-
ter competition);
late in-season minimal efficiency of performance
452-Which of the following associated injuries is most
commonly noted during arthroscopy after acute shoulder
dislocation?
a-Anterior labral tears b-Hill-sach lesion
c-Avulsed greater tubrosity d-Avulsed lesser tubrosity
e-Fracture head of humerus
453.  Which is true about joint mobility testing?
A. The joint becomes more painful as you approach the open pack
position.
B. Movements are not under voluntary control.

107
C. You must have normal osteokinematrics before you have normal
arthrokinematics.
D. Joint play movements are performed in the closed pack position.
454.  Which is not true of isthmic spondylolisthesis?
A. Usually L5-S1 B -Usually L4-L5
C. Mostly in young people D. LBP
E. True instability and HNP are rare
455.  Which is NOT true of degenerative spondylolisthe-
sis?
A. L4-L5 mostly
B. more in women
C. pars defect
D. stenosis associated
E. uaully >50% slip
456.  Which is not a common cause of spinal stenosis?
a. DISH
B. Degenerative
C. Rheumatoid Arthritis
D. Paget›s
E. Congenital
457. Which movement opens the spinal canal? 
a. Flexion b. Extension c. Side rotation d. rotation
458.  When the annulus is not completely torn, the hernia-
tion is termed:
A. Sequestrated
B. Extruded
C. Prolapse

108
d. Bulging
459.  Which is NOT true about herniated NP?
A. Patients want to sit more.
B. Leg pain in radicular symptoms.
C. 40s-50s.
D. 60-70% get better without surgery.
460.  Lumbar Degenerative Disc Disease:
A. always requires surgery.
B. causes back pain .
C. causes leg pain.
D. occurs to about 25% of older adults.
461. Osteoarthrosis can be primary or secondary. Primary
OA is caused by trauma, post-operatively, or by AVN.
A. True C. by both
B. False D. by non of them
462. What is pannus?
A. inflamed skin
B. inflamed synovium
C. inflamed joints
D.Inflamed tendon
463. Obesity is only a factor for what type of OA?
A. hip B. ankle C. Knee D. more than 2
464.Ligamentous instability is present in which of the fol-
lowing:
A. RA C. ankylosing spondylitis
B. OA D.Non of the above

109
465. Which is not a radiographic finding of OA?
A. symmetric loss of joint space B. osteophytes
C. increased subchondral sclerosis D. cysts
466. Is the medial or lateral knee of the knee most com-
monly affected by OA? What is the postural deformity as-
sociated with it?
A. medial, genu valgum B. lateral, genu valgum
C. medial, genu varum D. lateral, genu valgum
467- With regards to radioulnar limb formation and the
zone of polarizing activity, defects in which protein will re-
sult in duplication of digits?
a.Fibroblast growth factor. b. Sonichedgehog protein.
c. LMX1. d. Transforming growth factor.
e. Cartilage-derived morphogenetic protein.
468- All of the following contribute to the wrist and hand
deformity in rheumatoid arthritis except?
a.Volar subluxation of the extensor carpi ulnaris (ECU).
b. Radio-scapho-capitate ligament failure.
c. Scaphoid extension.
d. Supination of the carpus on the forearm.
e. Distal radio ulnar joint (DRUJ) destruction.
469- . Which of the following is not a recognized treatment
for carpal tunnel syndrome?
a. Nerve stimulation therapy. b. Steroid injection.
c. One-portal endoscopic surgical release.
d. Two-portal endoscopic surgical release.
e. Yoga.

110
470- Which of the following is true regarding a Mayfield
Stage I injury?
a. There is not always a scaphoid fracture. b. There is a
lunotriquetral ligament injury.
c. The lunate is extruded. d. There is a radio-scapho-
capitate ligament detachment.
e. There is a perilunate dislocation.
471- Which of the following is not true of
Dupuytren’sdisease?
a. The long-term recurrence rate is 50%.
b. Painful nodules are an indication for surgery
c. Metacarpophalangeal joint (MCPJ) contracture of greater than
30º is an indication for surgery.
d. Myofibroblasts are the offending cells in the etiology of the dis-
ease.
e. Concomitant carpal tunnel release increases incidence of post-
operative flare
472- Which of the following is not a poor prognostic indi-
cator in traumatic brachial plexus injury?
a. Horner’s sign. b. Transverse process fracture.
c. Empty sheaths on MRI scan. d. Diaphragmatic flattening
on inspiration/expiration X-rays.
e. No sensation from tip of acromion to tip of fingers.
473- All of the following make up the spiral cord except?
a. Grayson’sligaments. b. Spiralband. c. Lateral sheet.
d. Natatory ligament. e. Pre-tendinous band.

111
474- The following are all good prognosis after nerve in-
jury except?
a. Young age. b. Low velocity injury.
c. Sharp (knife) injury. d. Proximal injury.
e. Early exploration
475- In Wartenburg syndrome the compression takes
place between?
a. Brachioradialis and extensor carpiradialis longus (ECRL) inprona-
tion.
b. Brachioradialis and ECRL insupination.
c. ECRL and extensor carpiradialis brevis (ECRB).
d. Abductor pollicis longus (APL), extensor pollicis brevis (EPB) and
ECRL, ECRB.
e. Brachioradialis and flexor carpiradialis (FCR).
476- Which of the following is not a sign of an unstable
scaphoid fracture?
a. Vertical oblique fracture. b. Comminuted fracture.
c. >1mm displacement. d. Associated perilunate injury.
e. Scapholunate angle<60º.
477- When reducing a Smith’s or volar Barton’s fracture,
the reduction manoeuvre should include?
a. Supination only. b. Extension only.
c. Extension and supination. d. Extension and pronation.
e. Flexionandsupination.
478- Which of the following regarding metacarpal neck
fractures is true?
a. Up to 35º of angulation of the index and middle finger can be ac-
cepted.

112
b. Up to 40º of angulation of the little and ring finger can be ac-
cepted.
c. Metacarpal neck fractures should never be operated upon unless
it is an open injury
d.The Jahss position is the correct position to immobilize a manipu-
lated metacarpal neck fracture.
e. Up to 15ºof angulation of the index and middle finger can be ac-
cepted.
479- When performing a replant of an amputated finger,
which of the following is the correct order of surgery?
a. Bone, Artery, Extensor, Flexor, Nerve, Vein.
b. Artery, Bone, Vein, Extensor, Flexor, Nerve.
c. Artery, Bone, Extensor, Flexor, Vein, Nerve.
d. Bone, Extensor, Flexor, Artery, Nerve, Vein
e. Bone, Extensor, Flexor, Artery, Vein, Nerve.
480- When performing flexor tendon repair, which of the
following pulleys must be preserved?
a.A2 and A4 .b.A2only .c.A2andC2.
d.A2andA3. e.A3only.
481- A Stener lesion is significant because?
a. Adductor aponeurosis interposition between the proximally
based avulsed ligament impairs ligament healing.
b. Adductor aponeurosis interposition between the distally based
avulsed ligament impairs ligament healing.
c. Skiing is an increasingly popular sport.
d. It involves partial and complete ulnar collateral ligament rupture.
e. It is associated with a fleck sign on the X-ray

113
482- Which of the following is not a cause of a swan neck
deformity?
a. Mallet deformity. b. Flexor tendon tenosynovitis.
c. Volar plate rupture. d. Central slip rupture.
e. Lateral band subluxation.
483- Which of the following is a recognized complication
of extra-articular corticosteroid injections?
a- Acne b- Depression c- Fat atrophy
d- Hypogonadism e- Manic episode
484-Optimal management of osteoporosis diagnosed af-
ter a hip fracture includes
urgent medical consultation.
administration of bisphosphonates with follow-up as needed.
referral to the patient’s primary care physician within a year of sur-
gery.
initiation of vitamin D and calcium supplementation with follow-up
as needed.
initial evaluation by the orthopaedic team with early postoperative
referral to an osteoporosis clinic.
485- Which of the following diagnostic modalities is most
useful for the diagnosis of proximal thigh deep venous
thrombosis?1
D-dimer assay b- Physical examination
c- Venous ultrasonography d- Computed tomography
e-Impedance plethysmography
486- Paget’s disease of bone results from a defect in
which of the following processes?
a- Bone resorption
b- Bone regeneration

114
c- Osteodifferentation
d-Matrix maturation and mineralization5
e-Coupling of bone formation and resorption
487- Paget’s disease is best characterized as increased
bone turnover that results:
from a sequenced missense genetic mutation.
from an autosomal-recessive inheritance pattern.
in significant deformity but is rarely painful.
in subtle radiographic findings that necessitateadvanced imaging.
in increased urinary N-telopeptide and alpha-C-telopeptide
488- Which of the following factors is the best predic-
tor of successful non-operative management of an osteo-
chondritis dissecans lesion in the knee?
a. Open femoral physis
b. Location in the knee
c. High signal behind the lesion on MRI
d. Articular cartilage thickness
e. Body mass index
490- What is the rate of bone loss per year at menopause?
a. 0.3% to 0.5% for 3 to 5 years
b. 0.3 % to 0.5 % until age 65 years
c. 2% to 3% for 2 to 3 years
d. 2% to 3% for 6 to 10 years
e. 4% to 5% for 2 to 3 years
491- Which of the following diseases is characterized by a
defect in type I collagen metabolism?
a. Diastrophic dwarfism b. Osteogenesis imperfecta

115
c. Mucopolysaccharidosis d. Pseudoachondroplasia
a. Multiple epiphyseal dysplasia
492- Normal mineralization of bone is seen in which of the
following conditions?
a-Rickets
b- Seizure disorder treated with phenytoin
c- Renal osteodystrophy
d- Fanconi syndrome type II.
e- Osteoporosis
493- Which of the following is considered a risk factor for
osteoporosis?
a-Obesity
b- Mediterranean heredity
c- Fair skin and hair
d- A history of manual labor
e- Late-onset menopause
494- Normal cortical bone has which of the following MR
signal characteristics?
Low on T1-weighted images and low on T2-weighted images
Low on T1-weighted images and high on T2-weighted images
Moderate on T1-weighted images and low on T2-weighted images
High on T1-weighted images and low on T2-weighted images
High on Tl-weighted images and high on T2-weighted images
495- Which of the following organisms can produce gas
gangrene and may necessitate open amputation above the
level of infection?
a-Pseudomonas aeruginosa
b-Staphylococcus aureus methicillin-resistant

116
c-Staphylococcus aureus coagulase-negative
d-Clostridium perfringens
e- Group D enterococcus
496- A form of renal osteodystrophy that is characterized
by pure osteomalacia is caused by
Secondary hyperparathyroidism
Phosphate retention secondary to uremia
Insufficient renal synthesis of 1, 25 dihydroxy vitamin D
Aluminum deposition in bone from oral phosphate binders
Persistent acidosis aggravating the negative calcium balance
497- Which of the following radiographic findings would
be characteristic of the knee jointsof a patient with neuro-
pathic osteoarthropathy of the knee?
Fragmentation and subluxation of the normal joint articulation
Varus deformity with medial subchondral sclerosis
Preferential narrowing of the medial tibiofemoral compartment
Narrowing of the medial, lateral, and patellofemoral compartments
Bone proliferation at the patellar tendon and ligament insertions
498- What is the most common clinical indicator of reflex
sympathetic dystrophy of theknee?
a-Effusion b- Muscle atrophy c- Atrophic hair changes
d- Disproportionate pain e- Decreased range of motion
499- Regarding rheumatoid arthritis
a. Is a disease primarily of the articular cartilage
b. Is associated with the HLA antigens DR4 and DW4
c. Occurs more often in women
d. The hand, elbows, knees and cervical spine are the commonest
joints involved

117
e. Extra-articular manifestations occur in 20% of patient
500- Total interruption of the radial nerve at mid-arm pro-
duces specific findings on physical examination. The most
complete description of the neurologic deficit includes:
a. Paralysis of the thumb extensors, interphalangeal joint exten-
sors, extensor carpi radialis, and extensor carpi ulnaris.
b. Paralysis of the extensor carpi radialis longus and brevis, abduc-
tor pollicis longus, extensor pollicis brevis, and extensor pollicis longus.
c. Paralysis of the brachioradialis, extensor carpi radialis longus and
brevis, extensor carpi ulnaris, thumb extensors, and metacarpopha-
langeal (MCP) joint extensors, and loss of cutaneous sensibility at the
dorsal aspect of the thumb and index fingers.
d. Paralysis of the brachioradialis, extensor carpi radialis longus
and brevis, radialis, thumb extensors, finger MCP joint extensors, and
flexor carpi radialis, and loss of sensation in the cutaneous distribution
over the dorsal aspect of thumb and index fingers.

118
ANSWERS:

1-c 2-c 3-a 4-c 5-a 6-a 7-c 8-c 9-a 10-a 11-a 12-c
13-a 14-a 15-a 16-a 17-c 18-b 19-a 20-e 21-a 22-a 23-d
24-a 25-c 26-b 27-d 28-d 29-d 30-d 31-b 32-d 33-d
34-a 35-a 36-d 37-a 38-c 39-b 40-a 41-a 42-a 43-b
44-a 45-b 46-d 47-c 48-b 49-b 50-b
51-c 52-b 53-c 54-b 55-d 56-a 57-b 58-d 59-a 60-a
61-c 62-d 63-c 64-c 65-c 66-b 67-c 68-d 69-b 70-b
71-a 72-b 73-a 74-a 75-c 76-c 77-b 78-b 79-d 80-a 81-d
82-b 83-d 84-d 85-b 86-c 87-c 88-b 89-d 90-d 91-b
92-a 93-a 94-d 95-c 96-a 97-c 98-c 99-c 100-c
101-c 102-b 103-c 104-c 105-b 106-c 107-e 108-e
109-c 110-d 111-d 112-d 113-a 114-b 115-c 116-c 117-
e 118-d 119-e 120-d 121-d 122-a 123-d 124-c 125-c
126-b 127-e 128-d 129-c 130-a 131-b 132-b 133-e 134-e
135-e 136-d 137-e 138-d 139-a 140-e 141-d 142-a 143-d
144-c 145-d 146-e 147-a 148-b 149-b 150-e
151-e 152-c 153-c 154-b 155-b 156-c 157-b 158-b
159-a 160-c 161-d 162-a 163-c 164-b 165-d 166-d 167-
c 168-d 169-b 170-d 171-e 172-b 173-b 174-e 175-b
176-a 177-d 178-b 179-d 180-b 181-b 182-a 183-b 184-
c 185-d 186-c 187-c 188-c 189-d 190-a 191-d 192-c
193-b 194-d 195-b 196-a 197-d 198-a 199-a 200-c
201-b 202-d 203-c 204-b 205-b 206-b 207-b 208-b
209-b 210-b 211-c 212-c 213-b 214-b 215-b 216-b 217-
b 218-c 219-a 220-a 221-a 222-d 223-b 224-a 225-b
226-c 227-d 228-d 229-c 230-b 231-c 232-d 233-c 234-
b 235-b 236-d 237-c 238-d 239-b 240-d 241-c 242-c
243-b 244-b 245-d 246-d 247-a 248-c 249-b 250-c
251-d 252-e 253-a 254-e 255-d 256-c 257-c 258-e

119
259-a 260-d 261-a 262-a 263-b 264-a
265-a 266-b 267-a 268-a 269-a 270-a 271-d 272-a
273-a 274-a 275-a 276-a 277-a 278-d 279-c 280-a 281-
b 282-c 283-a 284-a 285-c 286-c 287-a 288-a 289-a
290-c 291-b 292-a 293-d 294-c 295-a 296-d 297-a 298-d
299-b 300-e
301-a 302-b 303-b 304-d 305-c 306-b 307-c 308-d
309-b 310-d 311-c 312-d 313-d 314-c 315-c 316-b 317-c
318-a 319-a 320-b 321-c 322-a 323-e 324-d 325-d 326-
d 327-a 328-a 329-e 330-d 331-e 332-b 333-e 334-a
335-b 336-e 337-c 338-c 339-b 340-e 341-e 342-a 343-b
344-c 345-d 346-c 347-c 348-e 349-c 350-c
351-a 352-d 353-c 354-b 355-c 356-a 357-d 358-c
359-b 360-b 361-c 362-e 363-d 364-a 365-e 366-a 367-e
368-c 369-a 370-c 371-b 372-a 373-c 374-e 375-b 376-d
377-b 378-c 379-e 380-a 381-a 382-d 383-b 384-a 385-e
386-e 387-a 388-e 389-d 390-b 391-c 392-e 393-a 394-d
395-b 396-d 397-b 398-d 399-d 400-a

401-c 402-d 403-d 403-e 404-e 405-a 406-e 407-c 408-


e 409-a 410-d 411-a 412-a 413-b 414-b 415-b 416-d 417-b
418-d 419-b 420-d 421-b 424-a 423-c 424-b 425-e 426-a
427-b 428-c 429-d 430-a 431-b 432-b 433-b 434-b 435-
436-b 437-a 438-c 439-b 440-a 441-a 442-c 443-b 444-a
445-a 446-d 447-b 448-e 449-d 450-d

451-e 452-a 453-b 454-b 455-e 456-c 457-a 458-c


459-a 460-b 461-b 462-b 463-c 464-a 465-a 466-c 467-
b 468-c 469-a 470-a 471-b 472-e 473-d 474-d 475-a
476-e 478-c 479-d 480-d 481-a 482-d 483-d 484-c 485-e
486-c 487-e 488-e 489-a 490-a 491-a 492-a 493-c 494-a
495-d 496-d 497-a 498-d 499-a 500-c

120
TRAUMA MCQS

1. Regarding eponymous fractures, all are true except;

(a) Bennett’s fractures involves the distal ulna


(b) Colle’s fracture involves the distal radius
(c) Galeazzi’ s fracture involves the radial shaft & dislocation of the
proximal radioulnar joint
(d) Monteggia’ s fracture involves the proximal ulna & anterior dis-
location of the head of the radius.
(e) Pott’s fracture is a general term applied to fractures around the
ankle
2) Which fracture description does not match its name?
A) Lisfranc = dorsal dislocation of tarso-metatarsal joint +/- # 1st
cuneiform or 2nd MT
B) Monteggia = # proximal third ulna and dislocation of radial head
C) Clay-shoveller’s = avulsion of C3, 4,5 spinous processes
D) Bennett’s = intra-articular # base of thumb metacarpal and proxi-
mal, radial and dorsal displacement of distal fragment
3 ) Which of the following is FALSE regarding C-spine in-
juries?
A)Hangman’s # is a hyperextension injury
B)Anterior wedge # is usually mechanically stable
C) Anterior tear-drop # is nearly always mechanically unstable
D) C2 and C3 are the most commonly injured vertebra.

121
4) Regarding knee injuries, which of the following is
TRUE?
A ) Segond fractures are important markers of PCL disruption
B) Fracture types I, II and III involve the medial tibial plateau with
increasing articular depression
C) Medial tibial plateau fractures are twice as common as lateral
fractures
D)Lateral plateau fractures are associated with ACL and MCL disrup-
tion
5) Regarding forearm fractures, which of the following is
TRUE?
A) Radial neck fractures with up to 30 degrees tilt can be managed
conservatively
B) Post interosseous nerve injury is common with Monteggia frac-
ture dislocation
C) X-rays in Galeazzi injury shows a radial fracture with dorsolateral
tilt
D) Radial nerve injury is common in Colles fracture
6) Regarding forearm fractures
A) Bartons fracture involves the distal radio-ulnar joint
B) Monteggia fracture dislocation involves fracture of the radius
proximally and distal radio-ulnar joint disruption
C) Monteggia fracture requires POP application in supination
D) Galeazzi fractures require fixation of the related joint disruption
in most cases as well as ORIF of the bone fracture
7) In which area do clavicular fractures most commonly
occur?
A) Inner 1/3.
B) Junction of inner 1/3 and middle 1/3.

122
C) Proximal end
D) Junction of middle 1/3 and outer 1/3.
8) Which of the following regarding scapular fractures is
FALSE?
A) Most commonly involve the blade.
B) Are commonly overlooked.
C) Are associated with thoracic injury in 80% of cases.
D) Commonly managed non-operatively.
9) Which of the following is TRUE with regards to knee
injuries?
A) Usually a varus force with axial loading will produce a lateral con-
dylar fracture of the tibia
B) Elderly patients with a lateral tibial plateau fracture usually re-
quire a bone scan as part of their investigation
C) A Segond fracture is a marker of collateral ligament injury and
ACL disruption
D) Popliteal artery is relatively protected from injury due to it’s ana-
tomical position
10) In regard to elbow dislocation, which of the following
is FALSE?
A) It is associated with significant soft tissue injury and resultant
instability.
B) It is one of the three most common large joint dislocations –the
others being glenohumeral and patellofemoral
C) Patients typically present with their arm held in near full exten-
sion
D) The commonest associated neurovascular injury is to the Ulnar
nerve.

123
11) Which of the following statement regarding Femoral
neck fractures is TRUE?
A) They are more common in men
B) Intracapsular fractures are more common than extracapsular
fractures
C) Displaced fractures should be treated with skin traction in the ED
D) Intermittent catheterization is preferable to indwelling catheters
12) Radial head fractures are commonly associated with
all of the following EXCEPT
A) Lateral collateral ligament injury
B) Medial collateral ligament injury
C) Coronoid fracture
D) Olecranon fracture
13) With regards to isolated medial epicondyle fractures,
which is FALSE?
A) Patients present with pain on flexion of the digits B) They are
extraarticular injuries
C) The medial epicondyle may become entrapped in the joint space
in Medial Collateral ligament tears.
D) They affect adults more commonly than children
14) Which association is the least likely?
A) Brachial plexus injury and fracture of the coracoid process
B) Pseudosubluxation in proximal humeral fractures
C) Wrist drop in Type III supracondylar fractures
D) Volkmanns Ischaemic Contracture in supracondylar fractures
15) Which is FALSE regarding supracondylar fractures?
A) Posterior displacement of the distal fragment are more common
than anterior displacement

124
B) Fat pad sign may be negative in severe supracondylar fractures
involving capsular rupture
C) Injuries to the anterior interosseous nerve are easily missed and
are detected by an inability to use FDP on the radial side
D) An absent radial pulse as an isolated finding generally warrants
urgent surgical exploration
16) Regarding arm injuries which of the following is TRUE?
A) Radial neck fractures with >10° angulation should be reduced.
B) Colles fractures in post-menopausal women require only ortho-
paedic follow-up.
C) A Barton fracture with minimal displacement will require opera-
tive reduction.
D) Galeazzi fracture should not be diagnosed when there is <4mm
of distal radio-ulnar separation.
17) In knee injuries which of the following is TRUE?
A) A common triad of injuries is ACL/MCL and medial meniscus.
B) Proximal tib-fib dislocation occurs with a twisting force to the
extended knee.
C) Tears of the patellar tendon need repair due to the high stress of
quadriceps contraction.
D) 60% of adolescents with a knee haemarthrosis on x-ray have an
osteochondral fracture
18) Regarding hip injuries, which is TRUE?
A) Garden I and II fractures are non displaced fractures and can be
managed conservatively
B) Intracapsular fractures are more common than extracapsular
fractures
C) The hip is least stable when it is flexed and abducted
D) Superior femoral head fractures are often associated with ante-
rior hip dislocations

125
19) Regarding knee injuries, which is FALSE?
A) Lateral tibial plateau fractures are associated with MCL and ACL
injury
B) The knee joint is the most commonly injured joint in the body
C) Patella often dislocated laterally due to the weakening of the vas-
tus lateralis tendon insertion
D) Dislocation of the knee is associated with popliteal artery and
tibial nerve injury
20) Which of the following statements is FALSE?
A) The tibia shaft is the most commonly fractured long bone in
adults
B) Lateral malleolar fractures are the commonest ankle fracture
C) The knee is the largest and most complicated joint
D) Non-union is the most common complication of hip fractures
21) Regarding forearm fractures which is FALSE?
A) All Galeazzi fracture-dislocations require surgery
B) All Monteggia fracture-dislocations require ORIF
C) Radial head fractures with up to 200 tilts can be managed con-
servatively
D) Smith’s fracture is managed with below elbow plaster with wrist
in pronation and full dorsiflexion
22) Regarding Segond fractures which is TRUE?
A) Small avulsio-fracture proximal medial tibia
B) Signifies tear of the menisco-tibial attachment of the middle 1/3
of the medial capsular ligament
C) All have anterior cruciate tears D) Few have
meniscal tears

126
23) Which of the following pairs is CORRECT
A) Femoral head fractures –Pipkin classification
B) Femoral head fractures -Gardiner classification
C) Patellar fractures –Hohl classification D) Ankle factures
–Boehler classification
24) Which of the following is CORRECTwhen treating neck
of femur fractures
A) Alignment maintained by pre operative skin traction helps post
operative mobility
B) Indwelling urinary catheters are not associated with post opera-
tive infection
C) Antibiotics at time of anaesthetic induction reduce post opera-
tive infection rates
D) Patients do not need nursing on pressure mattresses in the pre
operative period where surgery is conducted within 24 hrs
25) Which of the following fracture-eponym combinations
is NOT correct?
A) Tillaux # -avulsion # of tibia at inferior tibiofibular joint
B) Essex-Lopresti –fractured radial head and dislocated distal radio-
ulnar joint
C)Hume # -fractured distal radius with dislocation of distal radioul-
nar joint
D) Bennet # ,-# dislocation at the base of the 1stmetacarpal
26) With regard to scaphoid #’s which statement is TRUE?
A) Risk of # if not seen on initial X-ray is 10-20%
B) Most common location is proximal pole ≈50%
C) Can get AVN of tubercle #s
D) MRI sensitivity approaches 100%

127
27) Which of the following is TRUE
A) Most tibial plateau fractures involve the medial plateau
B) A Segond fracture suggests PCL disruption and anterolateral rota-
tory instability
C) The younger the age of onset of Perthes disease, the better the
prognosis
D) The more proximal the scaphoid fracture, the lower the inci-
dence of avascular necrosis
28) Which of the following is TRUE;
A) Most tibial plateau fractures involve the medial plateau
B) A Segond fracture suggests PCL disruption and anterolateral rota-
tory instability
C) The younger the age of onset of Perthes disease, the better the
prognosis
D) The more proximal the scaphoid fracture, the lower the inci-
dence of AVN
29) Which of the following is TRUE
A) An Ipsilateral Pelvic fracture is commonly associated with an L5
transverse process fracture
B) The posterior column of the thoracolumbar spine involves the
posterior longitudinal ligament
C) Intracapsular hip fractures are more common than extracapsular
fractures
D) The Hawkins classification can be used for hip fractures
30) In regards to Slipped Upper Femoral Epiphysis (SUFE),
which is CORRECT?
A) More common in females
B) In chronic slip passive flexion is associated with external rotation
C) In unstable SUFE patient can still weight bear

128
D) More common in children under the age of 10
31) What is the key event required for a Triplane Ankle
Fracture to Occur?
A) Inversion Injury
B) Ossification of Epiphysis
C) AxialLoading Injury
D) Partial Fusion of Growth Plate
32) Which of the following statements regarding ankle
fractures is TRUE except;:
A) A Maisonneuve fracture is produced by forced external rotation
B) The Pott’s system is based on the level of the fibula fracture
C) The Weber classification is based on the number of malleoli in-
volved
D) The Ottawa ankle rules have not been prospectively validated in
children
33) A greenstick fracture:
a. Occurs chiefly in the elderly.
b. Does not occur in children.
c. Is a spiral fracture- of tubular bone.
d. Is a fracture where part of the cortex is intact and partis crumpled
or cracked.
34) Spiral fracture is due to:
a. Blunt trauma.
b. Axial compression.
c. Twist.
d. Direct impact.

129
35) Internal reduction and fixation is considered in pres-
ence of:
a. Reduction impossible to the achieved or maintained.
b. Healing is expected to be delayed.
c. Pathological fracture.
d. All of the above.
36) What is True of clavicle fracture:
a. Non-union is rare.
b. Malunion is of no functional significance.
c. Reduction even if achieved is difficult to maintain.
d. All are true.
37) The joint most likely to have recurrent dislocation is:
a. Ankle. b. Knee. c. Shoulder. d. Patella.
38) Anterior dislocation of shoulder may be complicated
by:
a. Brachial plexus injury. b. Tear of rotator cuff.
c. Fracture head of humerus. d. All of the above.
39) What is not True about fracture surgical neck of hu-
merus:
a. Occurs due to fall on outstretched hand.
b. Common to children.
c. Osteoporosis is an important risk factor.
d. Non-union is uncommon.
40) Radial nerve palsy may occur in fr of humerus involv-
ing:
a. Surgical neck. b. Shaft.
c. Lower end. d. At all of the above locations.

130
41) The most common form of supracondylar fracture hu-
merus in children is of which type:
a. Flexion. b. Extension.
c. Combination of A & B d. None of the above.
42) Fracture involving which part of humerus can cause
delayed ulnar palsy:
a. Shaft. b. Surgical neck.
c. Medial epicondyle. d. Lateral epieondyle.
43) The deformity of wrist in Colles’ fracture is:
a. Madelung’s deformity. b. Dinner fork deformity.
c. Buttonaire deformity. d. None of the above.
44) Colles’ fracture can be complicated by late rupture of:
a. Extensor pollicis longus. b. Abductor pollicis longus.
c. Adductor pollicis longus. d. Flexor pollicis longus.
45) The carpal bone most commonly fractured is:
a. Triquetrum. b. Hamate. c. Capitate. d. Scaphoid
46) The most common injury following pelvic fracture is
of:
a. Bladder. b. Urethra. c. Rectum. d. Vagina.
47) Fracture femoral neck can be diagnosed from:
a. Limb shortening. b. External rotation.
c. Abduction. d. A + B.
48) Which of the following is not True of intertrochanteric
fracture of femur:
a. Limb shortening.
b. Malunion.
c. Avascular necrosis of femoral head.
d. Internal fixation is preferred.

131
49) Most common complication of fracture shaft femur is:
a. Malunion. b. Nonunion.
c. Knee stiffness. d. Fat embolism.
50) Fractures which do not impact include
a. fracture of tile vault of the skull
b. a compression fracture
c. a simple fracture
d. a transverse fracture of the patella
e. fracture of the neck of the femur 
51) Colles› fracture is
a. a fracture of the clavicle
b. a fracture about the ankle joint
c. common in elderly women
d. a fracture of the head of the radius
e. fracture of scaphoid
52) Bennett›s fracture is
a. reversed Colles› fracture
b. fracture of the scaphoid bone in the wrist
c. fracture of the radial styloid (chauffeur›s fracture)
d. fracture dislocation of the first metacarpal
e. cause of mallet finger 
53) Supracondylar fracture of the humerus in a child
a. is due to a fall on the point of the elbow
b. is usually compound
c. requires admission of the patient after reduction
d. requires immediate open reduction
e. is a fracture dislocation

132
54) A fracture of the midshaft of the clavicle is best treated by;
a. clavicle rings b. a figure-of-eight bandage
c. open reduction and plating d. an intramedullary nai
e. a broad arm sling and analgesics
55) A Pott›s fracture is a type of fracture of the
a. wrist b. ankle c. spine d. foot e. skull
56) Treatment of a severe comminuted fracture of the pa-
tella includes
a. physiotherapy alone
b. insertion of a figure-of-eight tension band
c. patellectomy
d. inserting screws or wire
e. skin traction
57) Malunion of a fracture is;
a. a fracture which unites in a position of deformity
b. delayed union of a fracture
c. non-union of a fractured followed by pseudoarthrosis
d. due to tuberculosis
e. seen in scaphoid fractures
58) Immobilization of fractures of long bones should in-
clude
a. Fractured bone only
b. Joint involved in the fracture
c. Proximal joint
d. Both proximal and distal joints
e. Distal joint

133
59) Non-union is common in fractures of the following
bones except the:
a. Carpal scaphoid b. Neck of the femur 
c. Lower third of the tibia d. Talus
e. Tuberosity of the fifth metatarsal
60) The signs of fractured shaft of a bone do not include:
a. Swelling b. Deformity
c. Loss of all movements in the limb
d. Acute localized bone tenderness
e. Abnormal mobility in the line of the bone
61) In the following types of fractures of long bones, crep-
itus can be elicited only in:
a. Fissures b. Subperiosteal cracks
c. Greenstick fractures d. Spiral and oblique fractures
e. Impacted fractures
62) The most severe growth disturbance results from
which of the following types of epiphyseal injuries:
a. Separation of the epiphysis at the metaphyseal side of the epiph-
yseal plate
b. Separation of the epiphysis with a triangular fragment of the
metaphysic
c. Intra-articular fracture involving the articular cartilage epiphysis
and epiphyseal plate
d. Intra-articular fracture extending from the joint surface through
the epiphysis and epiphyseal plate to the metaphysis
e. Crashing injuries compressing the epiphyseal plate without dis-
placement

134
63) Local complications of closed fractures do not include:
a. Malunion b. Non-union c. Infection
d. Sudek›s atrophy e. Joint stiffness
64) Non-union in closed fractures may due to any of the
following except:
a. Inadequate immobilization b. Interposition of soft parts
c. Impaired blood supply d. Inpsction of the fragments
e. Wide separation of the fragments
65) Causes of gangrene after fracture in a limb do not  include:
a. Direct crushing of the tissues b. Injury to the main vessels
c. Tight plasters d. Septic infection
e. Clostridial infection
66) concerning fracture of the shaft of the clavicle, it is
untrue that it:
a. Is usually due to direct trauma
b. Commonly involves the middle third
c. Is often associated with overriding of fragments
d. Causes dropping and deformity of shoulder 
e. Is usually treated by figure-of-eight bandage
67) A child with midclavicular fracture and overriding
of the fragments is best treated by:
a. Supine bed rest with interscapsular sandbag support
b. Open reduction and internal fixation
c. Figure-of-eight bandage
d. Closed reduction and plaster fixation
e. Manipulative reduction and abduction splint

135
68) In shoulder dislocations, the humeral head usually
dislocates primarily in which of the following directions:
a. Inferiorly b. Superiorly c. Anteriorly
d. Posteriorly e. Laterally
69) The incorrect statement about anterior dislocation of
the shoulder joint is that:
a. shoulder loses its rounded contour & becomes flattened
b. The elbow is abducted from the side
c. All movements of the shoulder are limited and painful
d. The anterior and posterior folds of the axilla are elevated
e. The hand cannot be placed on the opposite shoulder (Duga›s test)
70) Recent dislocations of shoulder joint are best treated by:
a. Hippocrates› method of closed reduction
b. Kocher›s manipulation
c. Modified Milch›s manoevre
d. Open reduction
e. Putti-platt1s operation
71) Recurrent shoulder joint dislocation is best treated
by:
a. Physiotherapy b. Nicola›s operation
c. Bankart›s operation d. Putti-Platt›s operation
e. Arthrodesis of the joint
72) Fractures of the shaft of the humerus are best treated
by:
a. Closed reduction and shoulder spica
b. Continuous skeletal traction
c. Open reduction and internal fixation

136
d. Hanging plaster cast
e. Coaptation plaster splint with a Velpeau dressing
73) The most vulnerable structure in supracondylar frac-
ture of the humerus is the:
a. Median cubital vein b. Brachial artery
c. Median nerve d. Ulnar nerve e. Radial nerve
74) Posterior dislocation of elbow joint is characterized
by the following except:
a. Gross swelling of the elbow region b. Loss of all move-
ments at the elbow joint
c. Shortening of the upper arm d. Absence of crepitus
e. Loss of the normal relationship of the olecranon with the two
epicondyles
75) In fracture of the olecranon process of the ulna, the
following statements are true except  that it:
a. Is usually due to a fall on the elbow
b. Can be felt as a gap between the olecranon and the shaft
c. Is rarely associated with hemarthrosis
d. May be complicated by anterior dislocation of the elbow joint
e. Always requires surgical treatment
76) Concerning extension Monoteggia›s fracture-disloca-
tion, it is untrue that it:
a. Consists of fracture of the upper third of the ulna andanterior
dislocation of the radial head
b. Is usually due to a severe blow on the back of theforearm
c. Can be treated by manipulative reduction in children
d. Always requires surgical treatment in adults
e. Is rarely associated with complications

137
77) Tears of the meniscus of the knee result from which of
the following strain:
a. Hyperextension b. Abduction
c. Adduction d. Rotation
e. Combined flexion and rotation
78) A lateral blow at the level of the knee joint may cause:
a. Rupture of anterior cruciate ligament
b. Rupture of medial collateral ligament
c. Avulsion of medial meniscus
d. Bumper fracture of tibia
e. All of the above
79) A march fracture most frequently results from:
a. Direct trauma
b. Jumping from a height
c. Muscle fatigue from prolonged walking
d. Use of high-healed shoes
e. Osteoporosis
80) Which statement is true about the “three-column con-
cept” of spinal fracture stability?
A. An unstable spine consists of bone or soft tissue injury in a single
column.
B. An unstable spine involves injury to all three columns.
C. Instability results from injury to two columns plus evidence of
compression of the dural tube.
D. Instability results from significant bone and/or soft tissue injury
in two columns.

138
81) All of the following statements are true of Jefferson’s
fracture of the atlas except:
A. The injury results from an axial load to the cervical spine.
B. The fracture fragments characteristically displace into the spinal canal.
C. Neurologic injury is uncommon.
D. Computed tomography (CT) best demonstrates the fracture’s
configuration.
82) The neurovascular structure most commonly injured
as a result of an anterior dislocation of the shoulder is the:
A. Musculocutaneous nerve. B. Axillary nerve.
C. Axillary artery. D. Median nerve.
83) The classification of fractures of the proximal humer-
us is based on:
A. The number of fracture segments and amount of displacement.
B. The mechanism of injury.
C. Presence or absence of associated dislocations.
D. All of the above
84) The radial nerve is at greatest risk for injury with which
fracture?
A. Fracture of the surgical neck of the humerus.
B. Fracture of the shaft of the humerus.
C. Supracondylar fracture of the humerus.
D. Olecranon fractures.
85) The best method of treating a supracondylar fracture
of the humerus in a child that is unstable when the elbow is
flexed to 90 degrees is:
A. Hyperflexion of the elbow to 130 degrees and casting.
B. Open reduction and internal fixation.

139
C. Percutaneous pinning.
D. Skeletal traction.
86) Both-bone forearm fractures in adults are best man-
aged by:
A. Closed reduction and casting.
B. Closed reduction and application of an external fixator.
C. Open reduction and placement of intramedullary rods.
D. Open reduction and internal fixation with compression plates.
87) The most consistent sign of a fracture of the carpal
scaphoid is:
A. Wrist pain during attempted push-ups.
B. Diffuse swelling on the dorsum of the wrist.
C. Localized tenderness in the anatomic snuffbox.
D. Wrist popping on movement.
88) Palmar dislocation of the PIP joint with fracture:
A. Is more common than dorsal dislocation.
B. Is treated by splinting with the PIP joint in flexion.
C. Is treated by splinting with the PIP joint and DIP joints in exten-
sion.
D. If not splinted properly, will cause a boutonniere deformity.
89) Fracture of the fifth metacarpal neck:
A. Usually requires open reduction and internal fixation.
B. Must be reduced anatomically and stabilized with pins.
C. Is called a “boxer’s fracture.”
D. Will result in significant functional disability if angulated 30 de-
grees dorsally.

140
90) A Bennett’s fracture is:
A. An extra-articular fracture of the base of the thumb metacarpal.
B. Displaced by the pull of the abductor pollicis longus and adductor
pollicis.
C. Displaced by the pull of the abductor pollicis longus and extensor
pollicis longus
D. Usually successfully treated with closed reduction and casting.
91) Prognosis of healing in tibial fractures correlates best
with:
A. Energy absorption at the time of fracture.
B. Amount of soft tissue damage.
C. Location of the fracture (i.e., in the proximal, middle, or distal
third).
D. Age of patient.
92) Management of a III-b tibia fracture is best treated ini-
tially by:
A. Plaster immobilization. B. Immediate plating.
C. Reamed intramedullary nailing. D. External fixation.
93) The most frequent forces acting on the foot that cause
ankle fractures are:
A. External rotation. B. Internal rotation.
C. Plantar flexion. D. Dorsiflexion.
94) Patients who have abduction injuries to the foot are
prone to injure the following structures:
A. Medial malleolus and deltoid.
B. Lateral malleolus and deltoid ligament.
C. Interosseous ligament.
D. Posterior tibiofibular ligament.

141
95) A Lisfranc fracture is a fracture-dislocation involving:
A. Calaneocuboid joint. B. Tarsometarsal joint.
C. Metatarsophalangeal joint D. Talocalcaneal dislocation.
96) Medial meniscus is injured more frequently than the
lateral meniscus because:
a- it is more mobile. b-it is less mobile.
c- it is thinner. d- it is attached lightly to the femur.
97) twisting injury to knee frequently results in all of the
following except:
a- meniscal tear. b- capsular tear.
c- ACL tear. d- fibular collateral ligament tear.
98) twisting injury to knee during weight bearing results
most frequently in an injury to :
a- ACL. b- medial meniscus.
c- PCL. d- fibular collateral ligament.
99) positive pivot shift test in knee is due to injury of:
a- posterior cruciate ligament. b- ACL.
c- MCL. d-postero lateral complex tear (PIC).
100) the commonest nerve injured in posterior dislocation
of the elbow in children is:
a- median nerve. b- ulnar nerve.
c- posterior interosseous nerve. d- radial nerve
101) which is not acceptable for a humeral shaft fracture?
a-30 degrees varus b-20 degrees apex anterior angulation
c-15 degrees rotation d-3 cm shortening
102) which would benefit LEAST from a IMN?
a-reverse obliquity b-large posteromedial fragment
c-subtroch extension d-4 part intertroch

142
103) varus malreduction of an intertroch fracture treated
with a DHS does what?
a-makes placement of the lag screw too inferior
b-aids in achieving the correct tip apex distance
c-Increase the lever arm of the construct
d-reduces lag screw cut out
104) How to fix a butterfly fragment of humeral shaft # in
a poly trauma pt.?
a-cast b-IMN
c-LCP plate d-Lag Screw and neutralization plate
105) When should you not operate on a polytrauma patient
a-Temp. 35 b-Lactate 4.5
c-HR 110 BP 110/70 d-can’t remember
106) which ankle fracture should you fix?
a-isloated lat mal that is lat displaced 3 mm
b-isolated lateral mallulus that is 3mm short
c-undisplaced medial mal
d-fibular avulsion with intact mortise
107) Which of the following statements is not correct re-
garding the prognosis of traumatic brain injury
a-duration of post-traumatic amnesia is a good indicator
b-school age children and young adults achieve better outcomes
than infants or older adults (> 45 years)
c-one or both non-reactive pupillary light reflexes is associated with
a poorer outcome
d-combined severe musculo-skeletal injuries predict worse outcomes
e-findings on CT scan of the brain are more sensitive comparedto MRI
as good predictor of outcome following severe traumatic brain injury

143
108- Jefferson’s fracture
a- c1 b- c2
c- c1 and c2 d- c2 and c3
109- Regarding Triage what is true?
a. Urgency of a patient with intra pelvic Hemorrhage
b. Urgency of a patient with GCS of 4
c. Treatment of multiple fractured first
d. all of the above
110 - commonest cervical vertebral fracture
a- c2 b- c3
c- c4 d- c5
111- Which of the following factors is least predictive of a
poor outcome after whiplash injury?
a. Headache b. Immediate pain
C. PAIN ISOLATED TO NEC D. SEVERE PAIN IN fiRST WEEK
e. Shoulder pain
112- Which of the following statements concerning hemi-
arthroplasty of the hip for fracture is not correct?
a. Cemented implants are associated with a lower overall reopera-
tion rate
b. Cemented implants are associated with a lower peri-operative
fracture rate
c. Cemented implants carry the same overall complication rate as
uncemented implants
d. Cemented implants have the same survivorship as uncemented
implants
e. Cemented implants in general are associated with less pain on
mobilisation

144
113-ANTERIOR DISLOCATION OF THE PROXIMAL inter-
phalangeal joint in the hand is associated with a great risk
of permanent impairment because of:
a. Rupture of the volar plate
b. Development of a Boutonniere deformity
c. Damage to the digital nerve
d. Damage to the digital artery
E. RUPTURE OF THE flEXOR TENDONS
114-.Supracondylar fractures in children are commonly
associated with:
a. Median nerve palsy
b. Ulnar nerve palsy
c. Radial nerve palsy
d. Anterior interosseous nerve palsy
e. Posterior interosseous nerve palsy
115- The most common organism responsible for an epi-
dural abscess is:
a. Streptococcus species b. Enterobacter species
c. Staphylococcus d. HAEMOPHILUS INflUENZAE
e. None of the above
116. Which carpal bone fracture causes Median nerve in-
volvement?
(a) Scaphoid (b) Lunate (c) Trapezium (d) Trapezoid
117. The fixation construct shown to provide optimal fixa-
tion of an unstable vertically displaced transforminal sacral
fracture is:
a) An iliosacral screw into the S1 vertebral body
b) Iliosacral screws in S1 and S2

145
c) Tension band plate fixation
d) Triangular osteosynthesis
118. Which one of the following traumatic spinal fractures
places the neurologic structures at greatest risk?
a) compression fracture
b) translational fracture
c) flexion-distraction fracture
d) extension-distraction fracture
119. In performing a lateral approach to the calcaneus for
open reduction, internal fixation, the structure at risk is:
a) Lateral plantar artery b) Lateral plantar
nerve
c) Dorsalis pedism artery d) Sural nerve
e) Superficial peroneal nerve
120. Union rates of the femur after antegrade or retro-
grade reamed intramedullary nailing are:
A) Higher for antegrade nailing
B) Higher for retrograde nailing
C) Identical
D) Dependent on location in the bone
E) None of the above
121.Which of the following tibial fractures is most likely to
have residual angulation (more than 5 degrees) after treat-
ment with a statically locked intramedullary rod:
a) A distal third oblique fracture
b) A proximal third metaphyseal fracture
c) A comminuted midshaft fracture
d) A transverse midshaft fracture

146
e) An open transverse fracture with a large butterfly fragment
122. The most important factor in predicting cutout of an
implant to repair intertrochanteric fractures of the hip is:
a) Size of the chosen screw b) Posterior/inferior
placement
c) Tip/apex distance d) Pitch of the chosen
screw
e) Angle of the plate
123. The optimal number of screws to repair displaced
fractures of the femoral neck is:
A) Three B) Five
C) Two D) Four E) Six
124. Angiography should be used in dislocations of the knee:
a) In the presence of asymmetric pulses
b) In all cases
c) With absent pulses only
d) Does not need to be used as long as Doppler pulses are audible
e) Has been replaced by magnetic resonance image scanning
125. When using the lesser trochanteric profile to assess
femoral rotation, a smaller lesser trochanter compared to
the uninjured side indicates:
a) External rotation deformity of the distal fragment
b) Internal rotation deformity of the distal fragment
c) A prior injury of the lesser trochanter
d) A varus deformity of the proximal segment
e) A valgus deformity of the proximal segment
126. The safest distance below the knee for placement of
external fixation wires is:

147
A) 5 mm B) 9 mm C) 14 mm
D) 19 mm E) 25 mm
127. Which of the following is not required for use of the
dynamic condylar screw (DCS) in a supracondylar femur
fracture
A) 4 cm of intact distal femur or easily reconstructable distal femur
B) Intact medial condyle
C) Healthy, nonosteoporotic bone
D) Intact lateral soft tissue envelope
E) Fracture without intracondylar extension
128. Clinical variables associated with a poor outcome
following calcaneal fractures include all of the following ex-
cept:
a) Age older than 50 years b) Increased
body weight
c) History of heavy labor d) On work-
ers’ compensation
e) Decreased body weight
129. The most important factor in fracture healing is:
a. Good alignment b) Organization of blood clot
c. Accurate reduction and 100% apposition of fracture fragments
d. Immobilization
130. Which of the following tibial plateau fractures is of-
ten associated with vascular injury:
a) Schatzker type I b) Schatzker type II
c) Schatzker type III d) Schatzker type IV
e) Bilateral fractures

148
131. Which of the following is an advantage of lateral
positioning over supine positioning when performing an-
tegrade intramedullary nailing of a subtrochanteric femur
fracture with an intact lesser trochanter:
a) Provides improved pulmonary ventilation
b) Eliminates valgus sag at fracture site
c) Allows faster setup and positioning
d) Provides easier alignment of the distal segment to the flexed
proximal segment
e) Provides more accurate rotational alignment
132. The primary goal of the Neer classification of proxi-
mal humeral fractures is to:
a) Determine the position of fragments
b) Determine the best surgical approach
c) Delineate the number of fragments
d) Determine the vascularity of the articular segment
e) Determine whether the fracture has dislocated
133. Implants protruding beyond the medial humeral neck
can impinge on which of the following structures:
A) Axillary nerve B) Axillary vein
C) Radial nerve D) Axillary artery
E) Musculocutaneous nerve
134. Acute compartment syndrome in the lower limb
A. can be reliably detected by loss of peripheral pulses
B. does NOT occur after open fractures of the tibia
C. is a contraindication to regional anaesthesia
D. is often associated with pain on passive stretching of the affected
compartment

149
E. occurs more commonly in patients over 35 years of age
135. The single most important treatment in preventing
acute renal failure following crush injury is
A. maintenance of an alkaline urine (pH > 6) to prevent cast formation
B. maintenance of adequate urine output with mannitol
C. vigourous intravenous fluid replacement
D. administration of xanthine oxidase inhibitors to prevent hyper-
uricemia
E. emergent dialysis to remove myoglobin from the circulation
136. The sign causing the most concern in a patient with
fat embolism is:
A. Dyspnoea B. Fat globules in the urine
C. fatty patient D. Fat in sputum
E. Petechial rash over the upper chest and shoulders
137. Which of the following is the correct position to
splint a hand after injury?
a. Wrist 0° , MPJ flexion 0° , IPJ flexion 20°, thumb abduction
b. Wrist 0o, MPJ extension, IPJ flexion 30o, thumb abduction
c. Wrist 20° flexion, MPJ flexion 10°, IPJ flexion 30°, thumb abduc-
tion
d. Wrist 30° extension, MPJ flexion 70°, IPJ flexion 0°, thumb abduction
e. Wrist 30° extension, MPJ flexion 90°, IPJ flexion 0°, thumb adduction
138. Why is the wrist joint usually splinted in 30° exten-
sion?
a. To ensure distal glide of the extensor hood over the proximal pha-
lanx
b. To help maintain the length of the extrinsic extensors
c. To help maintain the length of the extrinsic flexors

150
d. To maintain the length of the collateral ligaments
e. To prevent an extensor tendon lag
139 - Which of the following is the fracture of atlas verte-
brae?
a- Jefferson’s fracture b- Bennet’s fracture
c- Essex loprestti frx d- Green stick fracture
140- Factors help to augment fracture healing are except;
a. Bone grafts b. Electromagnatic fields
c. Osteo-inductive agent d. Mechanical methods
e. physiotherapy
141- Absolute stability Results in:
a. no fracture site movement
b. Direct healing via osteons
c. improved vascularity
d. all of the above
142- Damage control Orthopaedics aims at:
Control of bleeders
Achievement of stabilisation of fractures:
Reduction of blood loss in primary surgery by applying closed versus
open surgical techniques
No secondary surgeries
143- Damage control Orthopaedics helps in:
a. Prevention of fat-/thromboembolis
b. orevention of renal failure, ARDS, MOF
c. promote Systemic Inflammatory Responce Syndrom (SIRS)
d. prevention of compensatory anti-inflammatory response syn-
drome` = CARS

151
144. Early trauma care (ETC) all are true except;
a. After the admission of a polytrauma patient to the hospital
b. Immediate, definitive operative treatment of all fractures
c. Including im-nailing of femoral/tibial fractures
d. Higher rate of complications
e. Injury Severity Score (ISS ) less than 18 will benifit from ETC
145. Early trauma care (ETC) these statements are true
except;
a. Early long bone, pelvic stabilization and mechanical ventelation
b. Less thromboembolism, fat embolism syndrome
c. Pressure sore, muscle wasting and death are less
d. Quicker mobilisation
e. late discharge from hospital
146. Abbreviated Injury Scale Score (ISS);
a. Each injury is allocated to one of six body regions such as; Head,
Face, Chest, Abdomen, Extremities including Pelvis
b. The ISS score takes values from 0 to 75
c. If an injury is assigned an ISS of 6 it is survivable injury
d. The ISS score is virtually the only anatomical scoring system in use
e. It correlates linearly with mortality, morbidity, hospital stay and
other measures of severity
147. All of the following are causes of compartment syn-
dromes EXCEPT
A. Crush injuries B. Electrocution
C. Vigorous exercise D. Decompression illness
148. Which is the least likely to be affected by compart-
ment syndrome?
A. Peroneal compartment of the leg

152
B. Extensor compartment of forearm
C. Anterior compartment of thigh
D. Intrinsic muscle compartments of the hand
149. The compartmental pressure at which muscle isch-
aemia first occurs is?
A. 5-10 mmHg B. 10-25 mmHg
C. 35 –45 mm Hg mmHg D. 55 – 100 mmHg
150. Penetrating Neck Injuries, all the following are true
EXCEPT
A. The patient with Hemodynamic instability or obvious aero-diges-
tive injury requires urgent Operating Theatre management
B. Esophageal injuries are at risk of being missed clinically with sub-
sequent delayed presentation and very high mortality
C. Zone 3 injuries are best assessed using Angiography
D. Exploration of penetrating neck injuries beyond the platysma
does have a role in the ED
151. In the secondary survey of the pregnant trauma
patien;t
A. Abdominal examination is unreliable and may cause premature
labour in the irritable uterus so should not be performed
B. A vaginal examination is contraindicated if there is evidence of
vaginal blood loss
C. A stethoscope is inadequate for auscultation of the fetal HR.
Doppler ultrasound should be performed
D. The uterine fundus can reliably be palpated for contractions
152. Regarding pelvic fracture classification, all the fol-
lowing are true EXCEPT;
A. Lateral compression type I has 4% rate of bladder rupture
B. Mortality is roughly 25% for vertical shear injuries

153
C. Malgaigne fractures show symphyseal diastasis
D. Antero-posterior compression type III have 75% chance of se-
vere haemorrhage
153. In assessment of penetrating neck wounds;
A. Zone 1 refers to the region from the angle of the jaw to the lower
mandible
B. Zone 3 injuries can be observed if there is no clinical evidence of
major structure damage
C. All should have a full cervical spine XRay series and CXR
D. Combined oesophagoscopy and oesophagram for Zone 2 injuries
has a >90% sensitivity for detecting oesophageal injuries
154. Which statement is TRUE regarding Fractures of the
Pelvis?
A. Associated Hemodynamic instability carries a 25% mortality
B. CT is not helpful to evaluate suspected injuries to the Sacro-Iliac
Joint, Sacrum or Acetabulum.
C. Angiography with intervention / embolisation can be utilised for
uncontrolled haemorrhage from pelvic vessels
D. An ipsilateral double pubic ramus fracture is a ‘stable’ fracture
155. In spinal trauma it is true that;
A. The thoracolumbar junction is vulnerable due to its poor mobility.
B. The lower cervical spine is particularly vulnerable in children.
C. The dorsal column fibres decussate at the level of entry.
D. Spinal cord injury most often occurs between C5 an C7.
156. With regards to spinal cord injuries, which of the fol-
lowing is correct?
A. Brown-Sequard syndrome manifests as loss of motor function
and positionsense on the same side, and loss of pain and temperature
on the oppositeside.

154
B. Transverse spinal cord syndrome has sparing of the dorsal columns.
C. Cervical spine injuries occur most commonly at C4/5
D. In patients with a cervical spine fracture, it is unnecessary to look
for further fractures elsewhere.
157. Thromboelastography (TEG)
a. TEG reaction time value, representing the time of initial clot for-
mation,
b. The only hematologic marker predictive of mortality in patients
with trauma.
c. Delay in reaction time was associated with a significantly de-
creased death rate, independent of injury severity.
d. The death rate association was not observed with traditional
markers of clotting.
e. Patients with pelvic trauma should undergo screening TEG to
evaluate for coagulopathy
158. Compensatory anti-inflammatory response syn-
drome`( CARS);
a. Inflammation can be triggered in two main ways, either by infec-
tions, or by the products of tissue destruction
b. The CARS response Stimulate these processes by reduction of
lymphocytes by means of apoptosis
c. The CARS decreases cytokine response of monocytes to stimulation
d. The Cars decreases numbers of human leukocyte antigen (HLA)
antigen-presenting receptors on monocytes
e. It expresses cytokines such as IL-10 that suppress TNF expression
159. Systemic Inflammatory Response Syndrome (SIRS);
a. Sepsis evokes a systemic inflammatory response
b. Systemic Inflammatory Response Syndrome (SIRS) symptoms in-
cluding fever
c. Systemic Inflammatory Response Syndrome (SIRS) symptoms in-

155
cluding hypothermia, tachycardia
d. No change in blood leucocyte count
e. The relationship between SIRS symptoms and morbidity and
mortality in medical emergency ward patients is unknown
160. The polytrauma patient with a pelvic ring injury, all
are true except;
a. Detection and treatment of life-threatening situations by emer-
gency algorithm
b. Diagnosis, classification of the osteoligamentous injury, and op-
erative planning by Young-Burgess
c. The initial resuscitation according to ATLS protocols
d. The hemodynamic stability should be immediately evaluated
e. Resuscitation volume as required in hemodynamically unstable
patient is very productive in the long run without an adequate control
of the bleeding site
161. The across-the-knee application of external fi xation
is useful for
a. Stable bony segments around the knee
b. Indications include knee fracture-dislocations, or the floating knee
c. Its Components include two bars with a bar-to-bar clamp (or al-
ternatively one long bar) and two pin clamps
d. Options include pin clusters with either one double-pin clamp on
either the joint or, for a larger leg, one double clamp plus a single-pin
clamp on either side of the joint for multiplanar fixation.
e. Neurovascular damage, pin insertion, iatrogenic joint capsule
penetration, and resultant theoretical risk of joint sepsis are complica-
tions
162. Vacuum-assisted wound closure (VAC) is;
a. an application of negative pressure wound therapy which been
used for treating open fracture wounds
b. VACs were previously termed topical negative pressure (TNP)

156
c. VACs were previously termed supra-atmospheric pressure(SPD)
d. VACs were previously termed vacuum sealing technique (VST)
e. VACs were previously termed negative pressure wound therapy (NPWT)
163. The VAC all are true except:
a. Appears to increase the rate of granulation tissue formation com-
pared with saline dressing-treated wounds
b. The VAC may also reduce bacterial counts in wounds
d. The VAC may also decrease the need for future free flaps or ro-
tational flaps
c. The essential part of the setup is the special pressure distributing
dressing made up of open cell polyurethane foam
e. The VAC has to be applied after the wound has been debrided
164. The requirement of three conditions for triage in
emergency practice:
a. At least modest scarcity of resources exists.
b. A health care worker assesses each patient’s medical needs based
on a brief examination.
c. Equal respect for all, the principle of non-maleficence ( Not to do
any harm), and the principle of non-abandonment
d. The triage officer uses an established system or plan, usually
based on an algorithm or a set of criteria to determine a specific treat-
ment or priority for each patient
e. The fundamental point of triage is the following: everyone who
needs a particular form of health care, such as medicine, therapy, surgery,
transplantation, intensive care bed, can gain immediate access to it

165. Emergency Severity Index all are true except;


a. The ESI is a Ten-level triage scale developed by ED physicians
R.Wuerz and D. Eitel in the U. S
b. It facilitate the prioritization of patients based on the urgency of
treatment for the patients’ conditions

157
c. ESI was simple to use
d. It reduces the subjectivity of the triage decision
e. It is more accurate than other triage systems
166. Complication of Implant failures in retrograde nail-
ing;
a. associated with the insertion of a small diameter nail or use of
an interlocking nail
b. plastic deformation of the IM rod mainly occurs with nails that
are more than 10 mm in diameter
c. failures with interlocking nails, tendency to use smaller diameter
nails nail may fail at hole site
d. weak part of the nail is proximal of the 2 distal holes
e. Fractures located w/ in 5 cm of this hole will be stressed above
endurance limit w/ ambulation
167. fracture blisters appears after;
a. 1-3 days b. 3-5 days c. 5-7 days d. 7-9 days
e. Two weeks
168. What is the most appropriate treatment method for a
displaced vertical medial malleolus fracture?
a- MUA POP b- Medial antiglide plate
c-Mallular screw d-TBW e-Percutaneuos k-wire
169- All of the following are risk factors for wound compli-
cations following Achilles tendon repair except;
a- tobacco use
b- steroid use
c- female gender
d- diabetes mellitus
e- timing of surgery

158
170- Which of the following associated injuries is most
commonly noted during arthroscopy after acute shoulder
dislocation?
a-Anterior labral tears
b-Hill-sach lesion
c- Avulsed greater tubrosity
d- Avulsed lesser tubrosity
e- Fracture head of humerus
171- The strength of flexor tendon repair can be increased
by all of the following techniques except;
a- repair of the flexor tendon sheath
b- increasing the size of the core suture
c- increasing the number of core suture strands that cross the repair site
d- adding epitendonous suture
e- adding locking loops to the core suture
172- Submuscular plating techniques, when compared to
conventional plating techniques, offer;
a- Less compromise to medullary and periosteal perfusion.
b- Greater compromise to medullary and periosteal perfusion.
c- No difference with regard to periosteal perfusion only.
d- No difference with regard to medullary perfusion only.
e- No difference with regard to periosteal and medullary perfusion.

173- When comparing multiply injured patients with and


without foot injuries, the SF-36 scores in those patients with
foot injuries shows a:
a- Greater mean score
b- Lower mean score

159
c- Lower general health score
d- Greater vitality score
e- Lower mental health score
174- Tendoachilis rupture all are true EXCEPT:
a. Secondary to eccentric contraction of gastrocnemius-sloeus com-
plex
a- Occur 4-6 cm proximal to the insertion on the calcaneum
b- Thompson test ( squeezing the muscle) is negative
c- Patient can not do heel raise on the affected side
d- Patient can do dorsiflexion of his ankle
175- The goals of proper fracture reduction include the
following except?
a. Providing patient comfort and analgesia.
b. Allowing for restoration of length of the extremity.
c. Correcting angular deformity and rotation.
d. Enabling immediate motion of all fractured extremities.
e. Providing a foundation for bone healing and union.
176- Which of the following statement is NOT true con-
cerning the treatment of diaphyseal fractures?
a. The use of nailing is better than placting
b. The use of intramedullary rods allows early weight bearing and
minimal immobilization
c. The infection rate using intramedullary fraction fixation devices
is minimal
d. Results of intramedullary rods are better for fractures of the fem-
oral shaft than the tibia
e. Loss of limb length is inevitable with segmented or comminuted
fractures

160
177- A Patient has avascular necrosis of the femoral head
with collapse. Which treatment provides the MOST predict-
able outcome?
a- Core decompression b-Vascularized fibula
c- Arthroplasty d- Bone grafting
e-All of the above
178- The most common epiphyseal injury to the ankle is:
A- Salter Harres typeI of distal tibia
b-Salter Harres gradeII distal tibia
c-Triplanar fracture
d-Salter Harres type I distal fibula
e-Salter Harres type II distal fibula
179- With respect to cervical spine injuries ;
a- Clay shoveller’s # classically involves the upper thoracic spine
b- Jefferson’s # is a hyperextension injury
c- Regarding odontoid fractures, type 2 is the most common
d- Predental space should be less than 3 mm in children
e- In flexion injuries the posterior vertebral body bears most of
the force, sustaining simple wedge compression posteriorly
180- With regard to Anterior Spinal Cord injury which of
the following is INCORRECT:
a- Results from either direct blunt injury to the cord itself or com-
pression of the anterior spinal artery by disc, bone or hematoma
causing ischaemic damage to the anterior cord.
b- Patient presents with loss of motor and pain sensation bilater-
ally below the level of the lesion
c- Posterior cord function is tested with a tuning fork for vibratory
sensation or by testing of proprioception using dorsi and plantar
flexion of the great toe.

161
d- It is also known as “Beck’s syndrome
e- Prognosis after this injury is good
181- With regard to Cauda Equina Syndrome which is IN-
CORRECT:
a- Involves injury to the lumbosacral nerve roots
b- Characterized by an areflexic bowel and/or bladder
c- Motor and sensory loss is predictable
d- The affected limbs are areflexic
e- The symptoms may also appear as a temporary side-effect of a
sacral extra-dural injection
182- With regard to trauma in pregnancy which of the fol-
lowing is TRUE?
a- Diagnostic peritoneal lavage is contraindicated
b- Uterine rupture only occasionally results in foetal mortality
c- The most common source of abdominal haemorrhage is splenic
injury
d- The foetus is unlikely to be compromised if the mother does not
appear hypovolaemic
e-Which trimester has no difference in the management
183- With regard to SCIWORA (spinal cord injury without
radiological abnormality)
a- If recovery of neurological deficit has occurred prior to arrival
then no further spinal precautions are required
b- Can only be diagnosed if neurological deficits persist without
change
c- Occurs in children with incompletely calcified vertebral columns
de Has a good prognosis regardless of level of injury
e- Delayed onset of permanent paralysis, up to four days after in-
jury, occurred in approximately all of patients.

162
184- Which statement is TRUE regarding Fractures of the
Pelvis ?
a- Associated Hemodynamic instability carries a 25% mortality
b- CT is not helpful to evaluate suspected injuries to the Sacro-
Iliac Joint, Sacrum or Acetabulum.
c- Angiography with intervention / embolisation can be utilised
for uncontrolled haemorrhage from pelvic vessels
d- An ipsilateral double pubic ramus fracture is a ‘stable’ fracture
e- The Tile classification system is based on the integrity of the
symphysis pubis
185- Which is true regarding trauma in the elderly?
a. Cervical spine injuries are commonest in mid to lower Cervial
spine levels
b. 10% of patients hospitalised for falls die within 1 year
c. Patient medications rarely affect management of injuries.
d. Most deaths from trauma in that age are a result from motor
vehicle crashes.
e. Web lash injuries are the commonest

186- Which of the following is used in the Injury Severity


Score?
a- GCS b- Anatomical region injured c- Heart rate
d- Blood pressure e-Breathing rate
187- Penetrating Neck Injuries, all the following are true
EXCEPT
a-The patient with Hemodynamic instability or obvious aero-di-
gestive injury requires urgent Operating Theatre management
b- Esophageal injuries are at risk of being missed clinically with
subsequent delayed presentation and very high mortality

163
c- Zone 3 injuries are best assessed using Angiography
d- Exploration of penetrating neck injuries beyond the platysma
does have a role in the ED
e- Full neurological assessment is needed
189- Signs of major pelvic fractures include all of the fol-
lowing EXCEPT:
a-Destot’s sign b- Roux’s sign c- Earle’s sign
d-Bucholz’s sign e-Breuil’ sign
190- Which of the following is false with regards to cervi-
cal spine fractures:
a- Clay shoveller’s fracture is a flexion injury
b- Bilateral facet joint dislocation is a rotation type-injury
Anterior tear drop fracture is mechanically unstable
c- A type II fracture is the most common type of odontoid fracture
d-The cranial nerves examination should always be performed.
191- With regard to facial trauma you would usually ex-
pect to find…
a- An adjoining wound in mandibular fractures.
b- A hanging teardrop sign of orbital floor fracture on CT.
c- Maxillary fractures in children under 6 years old.
d- A Le Fort type 2 fracture when the eyes are mobile with maxil-
lary traction.
e- Loss of sensation under the eye denotes fracture zygoma
192- With regards to spinal cord injuries, which of the
following is correct?
a- Brown-Sequard syndrome manifests as loss of motor function
and position sense on the same side, and loss of pain and tempera-
ture on the opposite side.
b- Transverse spinal cord syndrome has sparing of the dorsal columns.
c- Cervical spine injuries occur most commonly at C4/5

164
d- In patients with a cervical spine fracture, it is unnecessary to
look for further fractures elsewhere
e- As days goes by there will be no additional damage to the spine
193- Which of the following is not a complication of crush
injury?
a-Hyperkalemia b- Hypercalcemia c- Coagulopathy
d-Hyperphosphatemia e-Sodium bicarbonate
194- about Central cord syndrome;
a- Is usually seen in older patients who sustain a hyperflexion in-
jury of their neck
b- Affects laterally placed nerve fibres of the lower extremity
c- Prognosis for recovery of function is good
d- The dorsal columns are affected
e- causing hematomyelia injury rather than edematous
195- The NEXUS criteria for cervical spine imaging in-
cludes all of the following EXCEPT;
a-No painful distracting injury
b- Normal level of alertness
c- No evidence of intoxication
d-The patient is able to actively rotate their neck 45 degrees left
and right
e-Mid line spinal tenderness
196- Unstable cervical spine injuries include all of the
following EXCEPT ;
Anterior tear drop fracture
b- Extension tear drop fracture
c- Bilateral facet joint dislocation
d- Anterior wedge fracture

165
e- 1-column injuries are relatively stable, whereas 3-column injuries
are unstable and typically require halo immobilization and/or surgery
197- Complications of spinal cord injury include:
a- Autonomic Instability b- Neurogenic shock
c-uncontrolled spasticity muscle tone or flaccidity
d- Pulmonary Oedema e- D All of the above
198- Complications of pelvic fractures; all are true except;
a- High incidence of nerve, bladder etc complications at revision
surgery
b- Infection increased incidence associated with visceral injury
c- 6% incidence and increased with ilio-inguinal approach
d- Nerve palsy usually peroneal component of sciatic nerve in an-
terior fractures
e- Ectopic bone formation
199- Classification AO for fracture acetabulum;
a- Type A one column fractured, the other intact, post wall, post
column, ant wall and/or ant column
b- Type B; transverse types with portion of roof attached to intact
ilium, transverse +/- post wall
c- T types, anterior type with post hemitransverse
d- Type C; both columns are fractured and all articular segments ,
including the roof are detached from the remaining segment of the
intact ilium
e- Type C; the floating acetabulum, ant column fracture extends
to the iliac crest, ant column fracture extends to ant border of
ilium,fracture enters the SI joint
f- All of the above

166
200- Non-operative treatment for fracture acetabulum, all
are true except;
a- If displacement less than 2-5 cm in dome, depending on loca-
tion of fracture and patient factors
b- High anterior column fractures
c- Low transverse fractures
d- Association both column fracture with secondary congruence
e- If roof angle more than 45 deg rarely need operative treatment

167
ANSWERS:

1-a 2-c 3-d 4-d 5-b 6-c 7-d 8-c 9-c 10-c 11-d
12-d 13-a 14-a 15-d 16-c 17-d 18-d 19-c 20-d 21-d
22-c 23-a 24c 25-c 26-d 27-c 28-a 29-b 30-d 31-d
32-a 33-d 34-c 35-d 36-d 37-c 38-d 39-b 40-b 41-b
42-d 43-b 44-a 45-d 46-b 47-d 48-c 49-c 50-d
51-c 52-d 53-c 54-e 55-b 56-b 57-a 58-d 59-e 60-
c 61-d 62-e 63-c 64-d 65-d 66-a 67-c 68-a 69-d 70-c
71-c 72-e 73-b 74-b 75-c 76-e 77-e 78-d 79-c 80-d
81-b 82-b 83-a 84-b 85-c 86-d 87-c 88-d 89-c 90-b
91-a 92-d 93-a 94-a 95-b 96-b 97-b 98-b 99-b 100-a
101-c 102-d 103-c 104-d 105-b 106-b 107-e 108-a
109-a 110-d 111-d 112-a 113-b 114-d 115-c 116-b 117-
d 118-b 119-d 120-c 121-b 122-c 123-a 124-a 125-b
126-c 127-a 128-e 129-d 130-d 131-d 132-d 133-a 134-c
135-c 136-a 137-d 138-c 139-a 140-e 141-d 142-d 143-
c 144-e 145-e 146-c 147-d 148-c 149-c 150-d 151-d
152-d 153-d 154-d 155-d 156-c 157-c 158-b 159-d 160-e
161-a 162-c 163-e 164-e 165-a 166-b 167-b 168-b
169-e 170-a 171-a 172-a 173-b 174-c 175-d 176-e
177-c 178-b 179-c 180-e 181-c 182-c 183-c 184-d 185-
e 186-b 187-d 188-d 189-d 190-b 191-a 192-a 193-b
194-c 195-d 196-d 197-e 198-d 199-e 200-b

168
PAEDIATRIC ORTHOPAEDICS MCQs
1. The Term Orthopaedics was coined by
(a) Nicholas Audrey (b) Hugh Owen Thomas.
(c) Thomas Bryant. (d) Sir Robert Jones.
2. First bone to ossify in foetal life is:
a. Femur. b. Tibia. c. sternum d. clavicle
3. Ortolani’s test is done for 
(a) Congenital dislocation hip (b) Dislocation patella
(c) Meniscal injury of knee joint (d) Penhes’ disease
4. Waddling gait is due to
(a) Gluteal muscle weakness
(b) Paravertebral muscle weakness
(c) Obturator nerve palsy
(d) Adductor muscle weakness
5. All of the following statements about a positiveTren-
delenberg’s sign are true, except
It occurs with a) coxa vara
(b) It occurs with paralysis of hip adductors
(c) If it is present on both the sides, the gait can look normal
(d) If it is present on one side, the patient has a lurching gait down-
wards towards the unsupported side
6. Antalgic hip gait is related to which of the following
(a) Waddling gait (b) Trendelenberg gait
(c) Painful hip gait {d) Shon leg gait

169
7- Which of the following is seen in bilateral congenital
dislocation of hip?
(a) Waddling Gait. (b) Shenton’s line is broken
(c) Trendelenberg test positive. (d) Allis test positive
8. Treatment of CTEV should begin :
(a) Soon after birth. (b) After discharge from hospital.
(c) After one month. (d) At 2 years.
9. Most important pathology in club foot is :
(a) Congenital talonavicular dislocation.
(b)Tightening of Tendoachilles.
(c) Calcaneal fracture.
(d) Lateral derangement.
10.In a newborn child, abduction and internal rotation pro-
duces a click sound. It is known as :
(a) Otorolani’s sign. (b) Telescoping sign.
(c) Mc Murray’s sign. (d) Lachman’s sign.
11. Club foot seen in a 15 year old could be treated suc-
cessfully by a :
(a) Appropriate footwear. (b) Soft tissue operation.
(c) Triple arthrodesis. (d) Quadrple fusion.
12. Sprengel’s deformity of scapula is :
(a) Undescended / Elevated scapula.
(b) Undescended neck of scapula
(c) Exostosis scapula.
(d) None of the above.
13. Treatment of club foot should begin:
(a) As soon as possible after birth (b) 1 month after birth
(c) 1 year after birth. (d) None of the above.

170
 14. Treatment for chronic cases of club foot is :
(a) Triple arthrodesis. (b) Dorso medial release.
(c) Amputation. (d) None.
15. Phocomelia is best described as;
(a) Defect in development of long bones.
(b) Defect in development of flat bones
(c) Defect of intramembranous ossification
(d) Defect of cartilage replacement by bone
16. In correction of clubfoot by manipulation, which de-
formity should be corrected first :
(a) Forefoot adduction. (b) Varus.
(c) Upper end tibia. (d) Calcaneum
17. Child 3’/4 years is treated for CTEV by
(a) Triple arthrodesis
(b) Postero medial soft tissue release
(c) Lateral wedge resection
(d) Tendo Achilles lengthening and posterior capsulatomy.
18. Von Rosen Splint in used in
(a) CTEV. (b) CDH.
(c) Fracture shaft of femur. (d) Fracture tibia.
 19. Commonest deformity in congenital dislocation of
hip.
(a) Small head of femur. (b) Angle of torsion.
(c) Decreased neck shaft angle. (d) Shallow acetabulum
20. Club foot in a new born is treated by
(a) Surgery (b) Manipulation by the mother 
(c) Dennis Brown splint (d) Strapping

171
 
21. Treatment of clubfoot in new born is
(a) Manipulation and corrective splint
(b) Corrective splint
(c) Nothing to be done for 6 months
(d) Surgical
22. In a newborn child, abduction and internal rotation pro-
duces a click sound. It is;
(a) Otorolani›s sign (b) Telescoping sign
(c) Lachman›s sign (d) Mc Murray›s sign
23. Sprengel’s shoulder is due to deformity
(a) Scapula (b) Humerus
(c) Clavicle (d) Vertebra
24. Barlow’s sign is related to the diagnosis of 
(a) Talipes equino varus (b) Ulnar nerve palsy
(c) Genu varum (d) Congenital dislocation of the hip.
25. The word ‘Talipes” refers to
(a) Long feet with spidery toes (b) Flat feet
(c) Club feet (d) Hammer toes
26. In congenital dislocation of hip, not true is;
(a) Real shortening (b) Telescoping
(c) Trendelenburg test (d) Head of femur
downwards
27. Congenital dislocation of hip is more commonly seen in
(a) Caucasians (b) Negroes
( c) Japanese (d)Eskimos

172
28. Rocker bottom foot is due to
(a) Under corrected club foot (b) Poliomyelitis
(c) Vertical talus (d) Deformities of spine
29. Concerning the diagnosis of congenital dislocation of
the hip at the earliest possible moment in life all are true
except;
(a) All obstetricians, midwives and general practitioners can be able
to diagnose congenital dislocation of the hip at birth
(b) The condition is diagnosed at birth by eliciting a’click’ or a ‘clunk’
from the hip
(c) The sign is known as Barlow’s or Von Rosen’s Sign
(d) Trendelcnberg’s sign is positive
30. Club foot is commoner among
(a) Males (b) Binovular
twins
(c) Females (d) Uniovular
twins
31. In a newborn child, abduction and internal rotation
produces a click sound. It is
(a) Otorolani’s sign (b) Telescoping sign
(c) Lachman’s sign (d) Mc Murray’s sign
32. Causes of a painless limp since infancy includes
(a) Congenital dislocation of hip (b) Infantile coax vara
(c) Poliomyelitis (d) All of the above.
33. Barlow’s test is done for testing
(a) CDH in child (b) CDH in
infancy
(c) Femoral neck fracture (d) Slipped
femoral epiphysis
34. Early CTEV is treated by

173
(a) CTEV cast from 1st postnatal day
(b) Manipulation
(c) Manipulation & Strapping
(d) Operative repair as early as possible
35. Which of the following test is useful in diagnosis
of congenital dislocation of hip?
(a) Barlow’s test (b) Thomas test
(c) Hibb’s test (d) Laguerres test
36. Sprengel’s deformity is
(a) Absence of clavicle
(b) Acomioclavicular dislocation
(c) Congenital elevation of scapula
(d ) Recurrent dislocation of shoulder 
37. A club foot is
(a) A consequence of placenta praevia
(b) Characteristically associated with breech presentation
(c) Most commonly of the equino valgus variety
(d) None of the above.
38. The most common congenital anomaly among the fol-
lowing is encountered in our country
(a) Congenital Pseudoarthrosis of Tibia
(b) Congenital dislocation of hip
(c) Congenital talipes equino varus
(d) Multiple congenital contractures
39. Madelung’s deformity involves the following;
(a) Knee (b) Wrist
(c) Hip (d) Elbow

174
40. Spina bifida occulta is
(a) Infection of the spine
(b) Traumatic fracture of the spine
(c) Congenital fusion of the body of the Vertebra
(d) Congenital non fusion of the spinal lamina
  41. The club foot characteristically involves
(a) Foot and ankle (b) Foot, ankle and leg
(c) Foot only (d) Foot, ankle, leg and knee joint
42. The most common deformity seen in club foot is
(a) Talipes equino varus (b) Equinu
(c) Equino valgus (d) Calcaneo equino varus
43. Various congenital deformity of the ankle joint occurs at;
(a) Calcanionavicular joint (b) Talocalcaneal joint
(c) Talonavicular joint (d) Tarsonavicular joint
44. All of the following are absolute indications for radio-
logical evaluation of pelvis for congenital dislocation of hip,
except
(a) Positive family history (b) Breech presentation
(c) Shortening of limb (d) Unstable hip
45. Talipes equinovarus is ;
(a) Equinus, inversion, abduction
(b) Equinus, inversion, adduction
(c) Equinus, eversion, abduction
(d) Equinus, eversion, abduction
46. CDH is due to
(a) Shallow acetabulum (b) Rotation of femur 
(c) Small neck femur  (d) Small femoral head

175
47. Phocomelia is
(a) Absence of short bones
(b) Complete absence of extremities
(c) Defects of long bones of limb
(d) Partial absence of extremities
 48. ‘Pseudoarthrosis* in Triple fusion is seen at the joint of 
(a) Calcaneocuboid (b) Calcaneonavicular 
(c) Navieulocuboid (d) Talonavicular 
49. Congenital bilateral dislocation of hip shows all true
except;
(a) Waddling gait (b) Lordosis
(c) + ve Trendelenburg test (d) + ve von Rosen’s sign.
50. Most common presentation benign tumor in a 10 year old
a. pathologic fracture b. incidental finding
c. pain d. fever
51- which is right about club foot casting
a- first start by correcting cavus by pronating the midfoot
b- final correction is progressively dorsiflex foot to 15 degrees
c- navicular is neutralized on talus , fore foot is inverted
d- correct adduction by abducting forefoot while in pronation and
counterpressure on talus
52. Congenital vertical talus is associated with:
a. A need for posterior tibialis lengthening
b. Varus and inversion of the hindfoot
c. Achilis tendon contracture
d. Dorsiflexion of the calcaneus

176
53. What are the areas of the physis from the metaphysis
to the joint?
a. Hypertrophic, proliferative, reserve
b. Hypertrophic, reserve, proliferative
c. Reserve, proliferative, hypertrophic
d. Proliferative, hypertrophic, reserve
54. Which is not an indication for surgery for Coxa Vara:
a. Trendelenberg
b. Hip pain
c. Heilgenreiner’s- epiphyseal angle of 60 deg
d. Neck shaft angle of 110 deg----100 or less
55. The graf classification of DDH is based on:
a- ultrasound of the hip.
b- combined radiography and ultrasound.
c- combined arthrography and radiography.
d- combined arthrography and ultrasound.
56) The treatment of stable SCFE is
a- observation.
b- traction to the hip.
c- hip spica immobilisation.
d- in situ pinning.
57- Alpha angle in DDH;
a- Decreases b. Increases
c- Constant d. Variable
58- True about Bilateral DDH;
a-Waddling gait b. increased lordosis
c. Short stature d. Broken Shenton’s lines

177
59. commonest presentation of congenital knee disloca-
tion is;
a. Varus b. flexion
c. hyperextension d. valgus
60. Perthe’s disease all are true except:
a. Usually presents before 10 years of age
b. Is due to avascular necrosis of the distal femoral epiphysis
c. Plain x-ray may show the capital femoral epiphysis to be smaller,
denser and flatter
d. May require surgical containment with a subtrochanteric oste-
otomy
61. The following statements about growth plate injuries
all are true except;
a- Fractures which pass along the line of the physis (Salter–Harris
type 1) commonly cause growth arrest.
b- Fractures which pass along the epiphyseal plate and then devi-
ate off into the metaphysis (Salter–Harris type 2) are the commonest
and rarely cause growth arrest.
c- Fractures which track along the epiphyseal plate and then devi-
ate into the joint (Salter–Harris type 3) may cause arthritis secondary
to joint incongruity.
d- Fractures which compress the growth plate (Salter–Harris type
5) are difficult to diagnose and cause growth arrest.
62. Most common cause of VIC in children is due to:
a- Supracondylar U humerus b- Lateral condyle # humerus
c- Medial condyle # humerus d- both bone forearm #
63- The nerve roots involved in Erb’s palsy are;
a-C3/4 b- C4/5 c- C5/6 d-C6/7

178
64- In children, all are true except:
(a) Dislocations are rare (b) Comminuted fractures are common
(c) Thick periosteum (d) Soft hones
65- Erb’s point is located at the junction of;
 (a) Anterior primary rami of C5 and C6
(b) Anterior primary rami of C8 and T1
(c) Superior and middle trunk of Brachial plexus
(d) None of the above
66. Erb’s palsy is due to involvement of 
(a) Upper trunk of brachial plexus
(b) Middle trunk of brachial plexus
(c) Lower trunk of brachial plexus
(d) Long thoracic nerve
67- Which one of the following statement is wrong about
Erb’s palsy?
(a) Abductors of the shoulder are weak 
b) Upper part of brachial plexus is involved
(c) Supinators are normal
(d) Internal rotation of arm
68- Which of the following is known for Non union in chil-
dren, if left untreated?
a-Fracture shaft of humerus
b- Fracture shaft of femur 
c-Fracture distal 1/3* of tibia
d-Fracture lateral condyle of humerus
e-Supracondylar fracture of humerus

179
69- Which fracture requires open reduction in children?
a-Fracture of both bones of forearm
b- Epiphyseal separation of tibia
c- Intercondylar fracture of femur 
d-Lateral condyle fracture of humerus
70- The mode of injury of Klumpke’s paralysis is;
a-Forcible increase in the neck shoulder angle
b- Forcible rotation of upper arm
c- Forcible hyperabduction of upper arm
d-Forcible External rotation of shoulder
71-The indications for Titanium elastic nail all are true ex-
cept;
a- pediatric femoral shaft fractures
b- Complications of other operative methods of stabilization are
reported.
c- less advantages of mobilization
d- rapid return to function than with non-operative techniques,
e- less soft tissue disruption and smaller scars when compared
with other surgical methods
72- In the child with acute onset diskitis, what is the earli-
est radiographic finding?
a- Narrowing of the intervertebral disk space
b- Scalloping of the superior vertebral body
c- Scalloping of the inferior vertebral body
d- Vertebra Magna
e- Loss of normal lumbar lordosis

180
73- A 10 year-old girl has abdominal pain following a mo-
tor vehicle accident where she was restrained in the back
seat by a lap belt. Bruising is noted across the lower abdo-
men. Radiographs reveal a flexion-distraction injury of T12-
L1. What is the most likely associated nonskeletal injury?
a-Iliac vein tear b-Uterine rupture
c-Colonic perforation d-Cardiac contusion
e-Aortic dissection
74- Which of the following conditions will most likely
eventually develop in a newborn girl with achondroplasia?
a- Degenerative disease of the hips b-Spinal stenosis
c- Atlantoaxial instability d- Cardiomyopathy
e- Patellar subluxation
75- In which of the following pediatric neoplasms can in-
volvement of the synovium result in arthralgias?
a- Ewing’ tumor b- Osteosarcoma
c- Eosinophilic granuloma d- Leukemia
e- Chondrosarcoma
76- An otherwise healthy 10-year-old boy has an erythem-
atous, painful epitrochlear lymph node after visiting a rela-
tive with cats 3 days ago. Which of the following organisms
is the most likely cause of this problem?
a- Bartonella henselae b- Mycobacterium marinum
c-Eikenella corrodens d- Blastomycosis dermatitidis
e- Pasteurella
77- Which of the following conditions will most likely
eventually develop in a newborn girl with achondroplasia?
a- Degenerative disease of the hips b-Spinal stenosis
c- Atlantoaxial instability d- Cardiomyopathy
e- Patellar subluxation

181
78- Compared with adults, trigger fingers in children are
associated with a high incidence of what abnormality?
a- Proximal placement of the A-1 pulley
b- Triggering at the lumbrical origin
c- Anomalous insertion of the dorsal interosseous
d- Distal triggering at the sublimus decussation
e- A-2 pulley triggering
79- What is the most common fracture associated with an
elbow dislocation in a child?
a- Olecranon b- Coronoid
c- Radial neck d- Medial epicondyle
e- Medial humeral condyle
80- A 4-year-old girl who weighs 16 kg is scheduled to un-
dergo a proximal femoral osteotomy. You ask the anesthe-
siologist to alert you if she loses 25% of her blood volume.
What volume of blood would this represent?
a- 200 mL b- 300 mL c- 500 mL d- 625 mL e- 1,000 mL
81- A 5-year-old child presents with a 2-day history of the
atraumatic onset of pain, erythema, and swelling of the right
knee joint. The child is febrile with an elevated white blood
cell count. The differential diagnosis includes:
a. Acute rheumatic fever. b. Leukemia.
c. Acute septic arthritis. d. Acute juvenile rheumatoid arthritis.
e.All of the above
82- About congenital torticollis all are true except :
a) Always associated with breech extraction
b) Spontaneous resolution in most cases
c) 2/3rd cases have palpable neck mass at birth
d) Uncorrected cases develop plagiocephaly

182
83- 7 year old child presents with a lesion in upper tibia.
X-ray shows radiolucent area with Codman’s triangle and
Sunray appearance. Diagnosis is :
a) Ewing’s sarcoma b) Osteosarcoma
c) Osteoid osteoma d) Chondrosarcoma
e) Chronic osteomyelitis
84- Following are associated with craneo-vertebral (CV)
junction anomalies except –
a) Basilar invagination b) Odontoid dysgenesis
c) Ankylosing Spondylosis d) Klipp-fill syndrom
e) None of the above
85- Rickets in infancy is characterised by the following
except:
a) Craniotabes b) Rachitic rosary
c) Wide open fontanelles d) Bow legs
e) wrist swelling
86- The malunion of supracondylar fracture of the humer-
us most commonly leads to:
a) Flexion deformity b) Cubitus varus
c) Cubitus valgus d) Extension deformity
e) non-union
87- Transient synovitis (toxic synovitis) of the hip is char-
acterized by all of the following, except:
a) May follow upper respiratory infection
b) ESR and white blood cell counts are usually normal
c) Ultrasound of the joint reveals widening of the joint space
d) The hip is typically held in adduction and internal rotation
e) Irritable hip syndrome

183
88- Which of the following tumors arise from epiphysis
a) Ewing’s sarcoma b) Osteoclastoma (GCT)
c) Chondromyxoid fibroma d) Osteosarcoma
e) Esonophilc granuloma
89- A 15 year old boy is injured while playing football. X-
rays of the leg rule did out of a possible fracture. The radi-
ologist reports the boy has an evidence of aggressive bone
tumor with both bone destruction and soft tissue mass. The
bone biopsy reveals a bone cancer with neural differentia-
tion. Which of the following is the most likely diagnosis?
a) Chondroblastoma b) Ewing‘s sarcoma
c) Neuroblastoma d) Osteosarcoma
e) rabdomyosrcoma
90- All the statements are true about exotosis, except;
a. It occurs at the growing end of bone
b. Growth continues after skeletal maturity
c. It is covered by cartilaginous cap
d. Malignant transformation may occur
e.occurrence of neurological deficit
91- Which one of the following is the investigation of
choice for evaluation of suspected Perthes disease?
a. Plain X-ray
b. Ultrasonography (US)
c. Computed Tomography (CT)
d. Magnetic Resonance Imaging (MRI
e. Arthrography

184
92- Most common nerve injured in supracondylar fracture
humerus?
a. Median b. Radial c. Ulnar
d. Anterior interosseous nerve
e. posterior interosseous nerve
93- Commonest site of fracture in childhood ?
a. Femur b. Distal humerus c. Clavicle
d. Radius e. ulna
94- In Klippel-Feil syndrome, the patient has all of the fol-
lowing clinical features except:
a. Low hair line
b. Bilateral Neck webbing
c. Bilateral shortness of sterno-mastoid muscles
d. Gross limitations of neck movements
e. No neurological deficits
95-Triad of Klippel Feil syndrome consists of all except?
a. Low hair line b. Elevated scapula
c. Short neck d. Limited neck movements  
e. normal strenomastoid muscles
96- Which of the following is a characteristic feature of
Blount’s disease?
a. Genu valgum b. Genu varum c. Coxa vara
d. Coxa valga e. recurvatum knees
97- Neuroblastoma is the most common metastatic malig-
nant solid tumor in childhood and arises from which of the
following?
a-Eosinophils b-Histiocytes c-Macrophages
d-Nucleus pulposus cells e-Neural-crest derived cells

185
98- In regards to Slipped Upper Femoral Epiphysis (SUFE),
which is CORRECT?
a) More common in females
b) In chronic slip passive flexion is associated with external rotation
c) In unstable SUFE patient can still weight bear
d) Occur in underweight patients
d) More common in children under the age of 10
99- Supracondylar fracture of the humerus in a child
a. is due to a fall on the point of the elbow
b. is usually compound
c. requires admission of the patient after reduction
d. requires immediate open reduction
e. is a fracture dislocation
100- The most severe growth disturbance results from
which of the following types of epiphyseal injuries:
a. Separation of the epiphysis at the metaphyseal side of the epiph-
yseal plate
b. Separation of the epiphysis with a triangular fragment of the
metaphysic
c. Intra-articular fracture involving the articular cartilage epiphysis
and epiphyseal plate
d. Intra-articular fracture extending from the joint surface through
the epiphysis and epiphyseal plate to the metaphysis
e. Crashing injuries compressing the epiphyseal plate without dis-
placement

186
ANSWERS:
1-a 2-d 3-a 4-a 5-c 6-c 7-a 8-a 9-b 10-a 11-c 12-a
13-a 14-a 15-a 16-a 17-c 18-b 19-d 20-b 21-a 22-a
23-a 24-d 25-c 26-d 27-a 28-c 29-b 30-a 31-a 32-d
33-b 34-c 35-a 36-c 37-d 38-b 39-b 40-d 41-b 42-a
43-b 44-c 45-b 46-a 47-c 48-d 49-d 50-b
51-b 52-c 53-a 54-d 55-a 56-d 57-a 58-b 59-d 60-b
61-a 62-a 63-c 64-c 65-a 66-a 67-c 68-d 69-d 70-a
71-b 78-d 79-d 80-b 81-e 82-b 83-b 84-d 85-d 86-b
87-d 88-b 89-b 90-b 91-d 92-d 93-c 94-c 95-c 96-b
97-e 98-b 99-c 100-e

187
188
BONE TUMOURS MCQs

1. All areTrue about Chondrosarcoma Except;;


a. It is a malignant tumor
b. It is a bone forming tumor
c. 25% of all biopsied malignant bone tumors are chondrosarcomas
d. Synovial chondromatosis can turn into a chondrosarcoma
e. 8-17% of all biopsied primary bone tumors are chondrosarcomas
2. Which of the following can progress to become a chon-
drosarcoma?
a. Enchondroma b. Osteochondroma
c. Chondromyxoid fibroma d. Chondroblastoma
e. all of the above
3. Myxoid chondrosarcoma belongs to which grade of
chondrosarcoma
a. Grade 1 b. Grade 2 c. Grade 3 d. None
4. Find the False statement of these:
a. Chondrosarcoma is common in children
b. 50% of patients are more than 40 years old with a mean peak
incidence of 50 years
c. Male to female ratio is 1.5:1
d. Clinical presentation is usually pain of insidious onset
e. Chondrosarcoma can also present with palpable mass(28%) or
pathological fracture(27%)

189
5. Which of the following is the least commonly affected
site for chondrosarcoma?
a. Pelvis b. Proximal Femur
c. Proximal Humerus d. Hands and Feet
6. Find the false statement of these;
a. A central chondroid tumor with pain or increased growth should
be regarded as chondrosarcoma until proven otherwise by biopsy or
benign clinical progress
b. Chondrosarcoma is suggested if the level of scintigraphic activity
in the lesion is greater than that in the anterior iliac crest
c. Calcification occurs in 29% of chondrosarcomas on CT
d. Deep endosteal cortical scalloping (greater than 2/3rds) is sug-
gestive of chondrosarcoma as opposed to chondroma
7. Radiographic features of Chondrosarcoma are all except
a. 75% of cases show calcification
b. Small low grade chondrosarcoma may not be differentiated from
a chondroma, appearing as a well defined lytic lesion with chondroid
matrix mineralization.
c. Plain radiography may not accurately define medullary involvement
d. Periosteal reaction is a rare feature of chondrosarcomas
8. MRI features of chondrosarcoma are;
a. Lesion is slightly hypointense to muscle on T1 and may show focal
areas of hyperintensity due to trapped areas of marrow
b. Malignant cartilage shows characteristic increased signal inten-
sity on T2 images and a multilobulated appearance
c. Matrix mineralization manifests as focal areas of signal void
d. Most chondrosarcomas are very well vascularized and enhance-
ment after IV contrast is excellent
e. T2 images have proven valuable in the identification of dediffer-

190
entiation, in which case a region of intermediate signal intensity adja-
cent to the typical hyperintense chondral tumor mass is seen
9. Which of the following are true regarding a Second-
ary Peripheral Chondrosarcoma developing from an Osteo-
chondroma
a. Most Osteochondromas have cartilage caps no thicker than 5mm
and a cap in excess of 20mm is likely to be malignant
b. Radiological features of malignant change consist of destruction
of part of the calcified cap or ossified stem of the osteochondroma
c. Malignant change should be suspected clinically if pain develops
or continued growth occurs after skeletal maturity
d. Radiographs are usually used to measure the thickness of the
cartilage cap of the osteochondroma
10. True statement about Periosteal Chondrosarcoma (a
rare form of chondrosarcoma)
a. Typically involves long bones
b. Most commonly involves distal femoral and proximal humeral
metaphyses
c. More common in men with a wide range of presentation
d. Radiologically a calcified juxtacortical mass is seen, with cortical
thickening and periosteal reaction
e. Prognosis is poor.
11. True statement/s about Mesenchymal Chondrosarco-
ma (another rare type of chondrosarcoma)
a. Occurs at a younger age (third and fourth decade)
b. Much more cellular malignant matrix than a normal chondrosar-
coma
c. Some predilection for ribs and mandible noted
d. Indistinguishable from a central chondrosarcoma and often
shows characteristic chondroid calcification

191
e. Local recurrence and disseminated metastases occur late and less
frequently than with conventional chondrosarcoma
12. True statement about Clear-cell Chondrosarcoma (an-
other rare type of Chondrosarcoma)
a. 2% of all chondrosarcomas
b. It is a high grade type of Chondrosarcoma
c. Patient has a long history and a better prognosis
d. Radiographically resembles Chondroblastoma and Chondromyx-
oid fibroma
e. Lesion is usually lytic and may present a loculated or soap-bubble
appearance
12) Osteoid osteoma originates from:
a. Periosteum. b. Cortex.
c. Medullary cavity. d. All of the above.
13) Sun ray appearance of osteosarcoma is because of:
a. Periosteal reaction. b. Osteonecrosis.
c. Calcification along vessels. d. None of the above.
14) Bone metastasis in male commonly arises from can-
cer of:
a. Lung. b. Prostate.
c. Kidney. d. Thyroid.
15) Osteoblastic bone secondaries commonly arise from
cancer of:
a. Breast. b. Lung.
c. Prostate. d. Adrenal.
16) Bone metastasis can be best evaluated by:
a. X-ray b. 99mTC bone scan. c.111Indium scan.
d. Calcium-alkaline phosphatase elevation.

192
17) Multiple myeloma tumor cells resemble:
a. Granulocytes. b. Plasma cells.
c. Lymphocytes. d. Chondrocytes.
18) An adamantinoma historically contains:
a. Squamous cell rests. b. Pallisading
cells.
c. Cells resembling basilar cells. d. All of the above.
19) A benign tumor forming osteoid is;
a. a synovioma b. a chondroma
c. an osteoma d. a fibroma e. an adenoma
20) Ewing’s tumor affecting the humerus
a. is a metastasis from carcinoma of the thyroid
b. should be treated by immediate amputation
c. looks like a cut onion on x-ray
d. has a soap-bubble appearance on x-ray
e. displays sun-ray spicules on x-ray
21) Solitary bone cyst is characterized by the following
features except  that it:
a. Occurs most often in children and adolescents
b. Usually arises in the diaphysis of a long bone
c. Often remains symptomless until complicated by pathological
fracture
d. Appears as clear ovoid expanding cavity in the X-ray
e. May be associated with new-bone formation

22) The most common tumor of the small bones of the


hands and feet is:

193
a. Enchondroma b. Osteochondroma
c. Osteoclastoma d. Cancellous osteoma
e. Bone sarcoma
23) Which of the following is most common in the small
bones of the hands and feet:
a. Osteochondroma b. Enchondroma
c. Osteoid osteoma d. Osteochondritis juvinelis
e. Tuberculous osteitis
24) Concerning osteoclastoma, the following statements
are correct except  that it:
a. Usually occurs between the ages of 15 and 40 years
b. Always arises in metaphyseal region of cartilaginous bones
c. Consists of large giant cells in a very vascular stroma of spindle
cells
d. Presents as a painless globular swelling with well-defined edge
e. Produces diagnostic radiological signs
25) The radiological signs of osteoclastoma include the
following except: 
a. Abrupt expansion of the bone
b. Characteristic soap-bubble appearance
c. Presence of an operculum obliterating the medullary cavity
d. Absence of any new-bone formation
e. Presence of areas of bone destruction
26) The treatment of osteoclastoma includes the follow-
ing measures except:  
a. Curettage of tumor tissue & packing cavity with bonechips
b. Excision with safety margin of bone c. Amputation
d. Radiotherapy e. Chemotherapy

194
27) The following statements about multiple myeloma are
true except  that it::
a. Is a primary malignant tumor of bone marrow
b. Occurs between the ages of 40 and 60 years
c. Usually presents with bone pain especially in the back
d. Is rarely associated with fever and anemia
e. May cause paraplegia with girdle pains
28) The following statements about bone sarcoma aretrue
except  that it:
a. Arises from osteoblasts of the periosteum or bone cortex
b. Forms a fusiform mass ensheathing the bone
c. Often invades the epiphyseal cartilage and neighbouring joint
d. Produces characteristic new bone formation in theX-ray
e. Disseminates rapidly by the blood stream
29) Which one of the following statements is untrue con-
cerning chondro-sarcoma?
a. Occurs most often between the ages 20 and 60 yrs
b. Is always a primary malignant tumor of bone
c. Most commonly affects scapula, pelvis, ribs &sternum
d. Causes bone expansion and destruction with irregular opacities
in the X-ray
e. Is radioresistant
30) Ewing’s sarcoma is characterized by the following ex-
cept  that it:
a. Is a common tumor of children
b. Always arises in the metaphysis of a long bone
c. Presents as a fusiform swelling with inflammatory changes in the
overlying soft tissues

195
d. May be associated with leucocytosis
e. Produces characteristic radiological signs
31) The most important DD of Ewing’s tumor is:
a. Chondrosarcoma b. Osteogenic sarcoma
c. Acute osteomyelitis d. Malignant metastasis
e. Reticulum cell sarcoma
32) The most common osteolytic metastases in bones are
derived from the:
a. Lung b. Breast c. Stomach
d. Kidney e. Prostate
33) The most pain-sensitive structure in a joint is the:
a. Bone end b. Articular cartilage
c. Joint capsule d. Synovial membrane
e. Skin and subcutaneous tissues
34) The following statements about solitary bone cyst are
true except  that it:
a. Occurs most often in children and adolescents
b. Is commonest in the humerus , femur and tibia
c. Arises in the diaphysis of the bone
d. Assumes an avoid shape and may cause bone expansion
e. May remain in-noticed until complicated by pathological fracture
35) The differential diagnosis of osteoid osteoma includes
all the following except: 
a. Brodies’ abscess b. Ossifying fibroma
c. Ewing’s tumor  d. Metastatic thyroid nodule
e. Bone sarcoma
36) Ivory osteomata occur most often in the:
a. Skull b. Spine c. Humerus
d. Femur  e. Tibia

196
37- Which does not need chemotherapy at all?
a- Rhabdomyosarcoma…..
b Angiosarcoma
c- MFH of bone
d- synovial sarcom
38. what helps you differentiate osteosarcoma from Ew-
ings?
a- alk phos
b- LDH
c- Aneuploidy
d- Cytotoxics
39. Most common presentation benign tumor in a 10 year old
a- pathologic fracture
b- incidental finding
c- pain
d- fever
40. What is true about parosteal osteosarcoma?
a- they are usually dedifferentiated
b- they are usually in the lower extremity
c- they never communicate with the medullary cavity
d- the treatment of choice is curettage

41. What is the worse prognostic factor associated with


soft tissue sarcoma?
a) Size of lesion b) Metastatic disease
c) Grade d) Extracompartmental disease

197
42- Onion peel appearance on x ray is seen in?
a- osteosarcoma b- ewings sarcoma
c- osteoclastoma d- osteochondroma
43- Bony metastasis is not seen in which carcinoma ?
a- testis b- breast
c- pelvis d- bronchus
44- According to newer hypothesis Ewing’s sarcoma arises from ?
a- epiphysis b- diaphysis
c- medullary cavity d- cortex
45 - Marker for bone formation ?
a- tartarate resistant acid phosphate b- osteocalcin
c- urinary calcium d- serum nucleo-
tidase
46- sunray appearance on x ray ?
a- osteosarcoma b- osteoclastoma
c- osteomyelitis d- ewings sarcoma
47- Most reliable method for detecting the bone metasta-
sis is ?
a- MRI b- CT scan
c- radiography d- SPECT
48- Which of the following is not a benign tumor ?
a- chondroma b- chordoma
c- osteochondroma d- enchondroma
49 - most common site of osteogenic sarcoma ?
a- femur upper end b- femur lower end
c- tibia upper end d- tibia lower end
50 - soap bubble appearance is the radiological feature of ?
a- osteoclastoma or GCT b- osteosarcoma
c- osteoid osteoma d- chordoma

198
51 - chondroblastomas most commonly occurs in ?
a- epiphysis b- diaphysis
c- metaphysic d- medullary cavity
52 - most common soft tissue tumor in a child ?
a- rhabdomyosarcoma b- histiocytoma
c- fibrosarcoma d- liposarcoma
53 - skeletal metastasis is common in ?
a- cancer stomach b- cancer pancreas
c- hepatoma d- cancer breast
54 - most common benign tumor of the bone ?
a- giant cell tumor b- simple bone cyst
c- osteochondroma d- enchondroma
55 - most common site of osteogenic sarcoma ?
a- femur upper end b- femur lower end
c- tibia upper end d- tibia lower end
56 - soap bubble appearance is the radiological feature of ?
a- osteoclastoma or GCT b- osteosarcoma
c- osteoid osteoma d- chordoma
57. Regarding osteoid osteoma all are true except;
(a) It is a premalignant bone tumour
(b) The femur and tibia are the commonest bones involved
(c) Presents with severe pain that is typically relieved by aspirin
(d) Radiologically appears as a radiolucent lesion surrounded by
dense bone
(e) Local excision or currettage is curative
58. Regarding osteosarcomas all are true except;
(a) Affects the metaphyses of long bones
(b) Are most commonly seen around the knee and in the proximal
humerus

199
(c) Haematogenous spread can result in pulmonary metastases
(d) Is exclusively a disease of adolescence and early adult life
(e) X-ray shows a ‘sunburst’ appearance due to soft tissue involvement
59. Regarding bone metastases all are true except;
(a) Less than 5% of patients with malignant disease develop bone
metastases
(b) 10% of patients with bone metastases develop a pathological
fracture
(c) Breast cancer is the commonest cause of bone metastases in
women
(d) Isotope scan can help early in the disease process
(e) Prostate cancer can cause osteosclerotic lesions
60 - which bone tumor occurs in the epiphysis ?
a- osteoclastoma b- ewing’s sarcoma
c- chondromyxoid fibroma d- osteosarcoma
61- With multiple hereditary exostosis have a greater risk
of the development of what kind of mesenchymal tumor?
a- Hemangioma b- Enchondroma
c- Chondrosarcoma d- Chondroblastoma
e- Extra-abdominal desmoids
62- Which of the following diseases of bone (when non-
metastatic at diagnosis) carries the worst prognosis for
5-year survival)?
a- Lymphoma b-Osteosarcoma
c-Ewing’s sarcoma d-Paget’s sarcoma
e-Conventional chondrosarcoma
63- Osteosarcoma most commonly develops in which of
the following locations?
a- Pelvis b- Distal humerus c-Proximal tibia
d- Proximal femur e- Proximal humerus

200
64- All are true regarding synovial sarcoma except?
a. Knee and foot are commonly involved
b. Usually seen in individuals less than 50 years of age
c. Originates from the synovial lining
d. More common at extra articular sites
e. Highly radiosensitive
65- Brown tumour is characteristic of?
a. Adreno cortical mass b. Hypoparathyroidism
c. Hyperthyroidism d. Hypopituitarism
e.Hyperparathyroidism
66- Which of the following conditions has the highest rate
of malignant change?
A- Ollier’s disease b- Enchondromatosis
c- Maffucci’s syndrome d- Multiple exostoses
e- Solitary osteochondroma
67- Which of the following conditions is least likely to
present as an eccentric osteolytic lesion:
a. Aneurysmal bone cyst b. Giant cell tumor
c. Fibrous cortical defect d. hydatid cyst
e. Simple bone cyst
68- Which of the following is diagnosed by urine protein
electrophoresis?
a- Monoclonal heavy chains of myeloma
b- Monoclonal light chains of myeloma
c- Diffuse B-cell lymphoma
d- Primary lymphoma of bone
e- Amyloidosis

201
69- What predominant cell type is typically seen on a bi-
opsy specimen in a patient with multiple myeloma?
a-Spindle cells b- Giant cells c- Osteoclasts
d- Lymphocytes e-Plasma cells
70- Which of the following soft tissue sarcomas is most
common in children?
a-epitheloid sarcoma b-rhabdomyosarcoma
c- synovial sarcoma d- Liposarcoma
e- extraskeletal Ewing’s sarcoma
71- For which of the following skeletal tumors is radiation
therapy routinely used for definitive local control?
a- Osteogenic sarcoma b- Ewing’s sarcoma
c- Chondrosarcoma d- Pleomorphic sarcoma
e- Paget’s sarcoma
72- What is the most current recommendation for defini-
tive treatment of a 15 year-old boy who has a high grade
osteosarcoma of the distal femur?
a. Surgical resection only
b. Raditation therapy only
c. Radiation therapy and surgical resection
d. Chemotherapy only
e. Chemotherapy and surgical resection
73- Which of the following terms best describe most os-
teosarcomas at the time of diagnosis?
a- Metastatic b- Low-grade, intracompartmental
c-Low-grade, extracompartmental
d- High-grade intracompartmental
e- High-grade extracompartmental

202
74- A 55 year old man with metastatic prostate cancer has
a painful lesion of the mid shaft of the humerus in which
approx. 75% of the cortex is involved. Management should
consist of.
a- an incisional biopsy.
b- a humeral cuff and a sling
c- closed interlocking nailing
d- radiation therapy to the humerus
e- plate fixation with bone fixation
75- A 12 year old girl has a Ewing’s sarcoma of the proxi-
mal fibula with no metastatic disease or neurovascular in-
volvement. Treatment should include
a- radiation therapy.
b- chemotherapy.
c- surgical resection.
d- radiation therapy and surgical resection.
e- chemotherapy and surgical resection.
76- Which of the following conditions has the highest rate
of malignant change?
a-Ollier’s disease b-Enchondromatosis
c- Maffucci’s syndrome d- Multiple exostoses
e- Solitary osteochondroma
77- The differential diagnosis of lesion, histologically re-
sembling giant cell tumour in the small bones of the hands
or feet, includes all of the following except:
a) Fibrosarcoma b) Aneurysmal bone cyst
c) Osteosarcoma d) Hyperparathyroidism
e) osteoid osteoma

203
78- What is the most common metastatic carcinoma to
the hand?
a- Lung b- Renal c- Breast
d- Thyroid e- Prostate
79- Expansile lytic osseous metastases are characteris-
tic of primary malignancy of:
a. Kidney b. Bronchus c. Breast
d. Prostate e. All of the above

80- Which of the following tumors has the greatest poten-


tial to metastasize to the lung?
a- Osteoblastoma b- Enostosis
c- Desmoplastic fibroma d-Giant cell tumor
e- Enchondroma
81- Which of the following bone tumors contains epithe-
lial cells in addition to mesenchymal cells?
a-Osteosarcoma b- Adamantinoma
c- Conventional chondrosarcoma d- Osteoma
e-Giant cell tumor
82- Chemotherapy has been shown to improve survival
for all of the following malignancies EXCEPT1
a- Osteosarcoma b- Ewing’s sarcoma
c- Chondrosarcoma d- lymphoma of bone
e- Rhabdomyosarcoma
83- A low grade soft tissue sarcoma of thigh is completely
excised. What is the first common location for recurrence?
a.Thigh b. Lymph nodes
c. Skeleton d. Liver e. Lung

204
84- Bisphosphonates are clinically used for osteoporosis
or osteolytic metastatic bone cancers. What is the common
mechanism of pharmacologic action?
a Increased mineralization of bone matrix
b.Increased apoptosis of osteoclasts
c.Increased production of alpha-v-beta-3 integrin by osteoclasts
d.Potentiation of PTM-related peptide effect on osteoblasts
e.Amplification of BMP-2 signal
85- Which of the following malignant tumors is radiore-
sistant?
a. Osteosarcoma b.Retinoblastoma
c. Neuroblastoma d. Ewing’s sarcoma .
e. Prostatic Secondaries
86- Which of the following bone lesions arises exclusively
in the epiphysis or apophysis of the long bones?
a-Giant cell tumor b- Aneurysmal bone cyst
c- Unicameral bone cyst d-Fibrous dysplasia
e- Chondroblastoma
87- In which of the following pediatric neoplasms can in-
volvement of the synoviumresult in arthralgias?
a. Ewing’ tumor b. Osteosarcoma
c. Eosinophilic granuloma d. Leukemia e. Chondrosarcoma
88- Which of the following benign lesions may occasion-
ally metastasize to the lungs.
Nonossifying fibroma b- Osteofibrous dysplasia
c- Chondromyxoid fibroma d- Chondroblastoma
e- Periosteal chondroma

205
89- What type of chondrosarcoma has the lowest 5-year
disease-free survival rate?
a- Chondrosarcoma developing in an osteochondroma
b- Chondrosarcoma developing in Olliers disease3
c- Intermediate grade (grade 11)
d- Mesenchymal
e- Clear cell
90- Which of the following bone tumors is typically multi-
focal and involves bones in the same extremity?
a-Osteoblastoma b- Osteosarcoma
c-Chondrosarcoma d- Chondroblastoma
e- Hemangioendothelioma
91- Which of the following soft-tissue lesions is best de-
scribed as a Musculoskeletal Tumor Society stage 3 lesion
(aggressive)?
a- Nodular fasciitis b- Lipoma
c- Malignant fibrous histiocytoma
d- Fibromatosis (extra-abdominal desmoid)
e- Giant cell tumor of the tendon sheath
92- Which of the following processes is related to osteofi-
brous dysplasia (Campanaccis disease)?
a-Nonossifying fibroma b- Osteoid osteoma c- Adamantinoma
d- Fibrosarcoma e- Olliers disease (multiple enchondromatosis
93- Which of the following soft-tissue sarcomas most
commonly shows prominentmineralization within the lesion
on plain radiographs?
a-Rhabdomyosarcoma b- Liposarcoma
c- Epithelioid sarcoma d- Synovial sarcoma
e- Angiosarcoma

206
94- Which of the following is considered the most com-
mon presenting symptom in a patient with a high-grade pri-
mary malignant bone tumor?
a- Weight loss b- Fatigue
c- Fever d- Pain e- Malaise
95- Lymphatic metastasis is a common feature of which
of the following lesions?
a-Liposarcoma b- Nodular fasciitis
c- Rabdomyosarcoma d-Malignant fibrous histiocytoma
e- Extra-abdominal desmoid tumor
96- Which of the following metastatic tumors to bone car-
ries the greatest risk ofcomplications from intraoperative
bleeding?
a- Breast b- Prostate c- Gastrointestinal
d- Kidney e- Multiple myeloma
97- A skeletal survey is more accurate than a bone scan
for detecting skeletal involvementin which of the following
neoplastic diseases?
a-Ewing’s sarcoma b- Osteogenic sarcoma
c- Multiple myeloma d- Metastatic prostate carcinoma
e- Metastatic breast carcinoma

98- Which of the following surgical options after resec-


tion of a sarcoma about the kneewould require a patient to
expend the greatest amount of energy while walking?
a. Arthrodesis b. Rotationplasty
c. Above-knee amputation d. Osteoarticular allograf
e. Endoprosthesis (custom arthroplasty)

207
99- Flow cytometry of tumors measures the
a. Size of cells b.Amount of DNA in cells
c. Nucleus-cytoplasm ratio d. Specific DNA sequences
e. Specific messenger RNA sequences
100- Which of the following types of sarcoma of the bone
is most sensitive to external beam radiation?
a. Ewings tumor
b. Parosteal osteosarcoma
c. Dedifferentiated chondrosarcoma
d. Low grade intramedullary chondrosarcoma
e. High grade intramedullary osteosarcoma

208
ANSWERS:

1-b 2-e 3-b 4-a 5-d 6-c 7-d 8-d 9-d 10-e 11-e 12-b
13-c 14-b 15-c 16-b 17-b 18-d 19-c 20-c 21-b 22-a 23-b
24-d 25-d 26-c 27-d 28-b 29-b 30-b 31-c 32-b 33-c 34-
c 35-c 36-a 37-b 38-d 39-b 40-b 41-b 42-b 43-a 44-c
45-d 46-a 47-d 48-b 49-b 50-a
51-a 52-a 53-d 54-c 55-b 56-a 57-a 58-d 59-a 60-a 61-
c 62-e 63-c 64-c 65-e 66-a 67-e 68-b 69-e 70-b
71-b 72-e 73-e 74-c 75-e 76-b 77-a 78-a 79-a
80-a 81-b 82-c 83-c 84-b 85-a 86-e 87-d 88-d
89-d 90-e 91-d 92-c 93-d 94-d 95-c 96-d 97-c
98-c 99-b 100-a

209
210
MCQ of problem solving type,

1) An elderly♀ sustained Colles’ fracture which was prop-


erly treated. However, she developed severe pain &stiffness
of the wrist with coldness and cyanosis of the hand. X-ray
examination revealed diffuse decalcification of the bones.
She proved to be suffering from:
a. Causalgia b. Tuberculous arthritis of wrist joint
c. Traumatic tenosynovitis d. Sudek’s atrophy
e. Osteoarthritis of wrist joint
2) Following a stumble on stairs, a 70-year-old ♂Felt se-
vere pain in the hip and could not stand up. O/E, there was
shortening of the limb, external rotation deformity and ten-
der thickening of the greater trochanter. X-ray examination
revealed:
a. Intracapsular fracture of the neck of the femur
b. Pertochanteric fracture of the femur 
c. Dislocation of hip
d. Fracture of acetabulum
e. Fracture of greater trochanter 
3. An athlete complains for 3 months of pain induced only
by athletic activity, often arising at a precise point in the
training session. The pain is located at the anterior aspect
of the lower leg, with pain during stretch of the toe and an-
kle dorsal flexors in inversion. There is a slight weakness in
the tibialis anterior and extensor digitorum longus muscles.
There is also some numbness in the dorsal first cleft of the
toes. X-ray of the lower leg is normal.
At this stage which of the following examinations is indicated:
a. Arteriography of the lower extremity

211
b. Venography of the lower extremity
c. Radio-isotope bone scan of the lower extremity
d. Anterior tibial compartment pressure measurement
e. MRI of the lower leg
4) In a football game, an athlete felt severe pain in his Rt
knee while turning to the left side with the joint flexed and tak-
ing the body weight. Soon after, the joint became swollen and
painful but recovery followed rest for 3weeks. Thereafter, the
patient suffered from recurrent locking with pain and a feeling
of “giving way” in the joint. The most probable diagnosis is:
a. Solitary loose body
b. Fracture of the tibial spine
c. Rupture of the medial ligament
d. Rupture of the medial semillunar cartilage
e. Fracture of the patella
5) A 7-year-old child presented with intermittent limp and
pain in the right hip and knee. On examination, flexion and
extension movements were free and there was no tender-
ness and no muscle wasting. X-ray examination confirmed
the diagnosis of:
a. Early tuberculous arthritis of the hip joint
b. Traumatic arthritis
c. Perthes’ disease
d. Slipped upper femoral epiphysis
e. Coxa vara
6) A 9-year-old boy presented with limping and pain in the
right knee two days after a fall in the street. On examination
he looked ill and in severe pain with high fever and swelling
of the knee region extending to the thigh which was warm
and very tender. The most probable diagnosis is:
a. Traumatic synovitis b. Hemarthrosis

212
c. Acute osteomyelitis of the femur  d. Septic arthritis of knee
e. Bone sarcoma
7) A 10-year-old male with neglected congenital disloca-
tion of the hip presented because of increasing pain in the
back with limping and fatigue. The appropriate management
should be :
a. Analgesics and anti-inflammatory drugs.
b. Raising the heel of the right shoe.
c. Open reduction with deepening the acetabulum by ashelf proce-
dure.
d. Colonna’s arthroplasty.
e. Lorenz’s bifurcation osteotomy.
8) A 4-year-old rachitic child developed severe genuvar-
um. The best line of treatment is by :
a. Antirachitic treatment.
b. Repeated moulding.
c. Corrective splints.
d. Osteoclasis of the tibia.
e. Tibial osteotomy.
9) A 13-year-old boy presented for consultation because
his knees tend to knock together and he tends to fall dur-
ing running. Examination revealed separation of the medial
malleoli by 3 inches when standing with the knees in contact
with each other and the feet directed forwards. The correct
diagnosis is :
a. Coxa vara. b. Genu valgum. c. Genu varum.
d. Talipes calcaneovalgus. e. Bilateral flat foot.
10) The best method of treating a supracondylar fracture
of the humerus in a child that is unstable when the elbow is
flexed to 90 degrees is:

213
A. Hyperflexion of the elbow to 130 degrees and casting.
B. Open reduction and internal fixation.
C. Percutaneous pinning.
D. Skeletal traction.
11. A patient describes a fall on the outstretched hand
during sports activities. Multiple radiographic views show
no distinctfracture. He is tender to palpation in the anatomic
snuffbox. The most suitable method of management is:
A. Diagnose “sprained wrist” and apply an elastic bandage.
B. Diagnose suspected scaphoid fracture and apply a short-arm cast to
include the thumb.
C. Apply a canvas wrist splint for immobilization.
D. Prescribe salicylates and permit continued activity.
12) A 24-year-old woman presents to the emergency room with
a dislocated knee. In transferring the patient from stretcher to ex-
amining table, the knee is spontaneously reduced. Physical exam-
ination reveals no palpable or “Dopplerable” pulses inthe foot on
the affected side and booming pulses in the foot on the nonaffect-
ed side. Proper treatment would include whichof the following?
A. Doppler evaluation of the arteries in the lower extremity followed by
arteriography if the Doppler study was abnormal.
B. Magnetic resonance imaging (MRI) of the affected leg.
C. Close follow-up examination.
D. Emergent transfer to the operating room for exploration of the popli-
teal artery.
E. Immobilization of the knee with gentle warming of the extremity and
elevation.
13) In the child with Slipped Upper Femoral Epiphysis
(SUFE), which statement is FALSE?
A) SUFE is a Salter Harris type fracture.
B) Bilateral SUFE in a child under 10 years of age is more likely to be

214
associated with endocrine / metabolic disorders.
C)The ‘Klein line’ is helpful in diagnosing SUFE
D) A ‘Frog leg’ plain XR view of the hips is a helpful investigation to
diagnose SUFE
14) In the child with Leg-Calve-Perthes Disease (LCPD),
which statement is TRUE?
A) It is a form of capital femoral epiphysis avascular necrosis second-
ary to trauma
B) CT imaging is preferred above plain ‘frog leg’ XR’s of the hips
C) Patients with suspected LCPD disease require admission for further
inpatient management.
D) Is clinically indistinguishable from ‘acute synovitis’
15) A 29-year-old female presents at 32 weeks gestation
following a Motor Vehicle Accident. She is asymptomatic
but has a seatbelt sign. A Kleihauer-Betke Test and Group
and Screen are performed. She is A negative Blood Type and
Kleihauer is pending. She has a normal examination. Which
is TRUE in this case?
a. It is recommended that anti-D not be used in the Rh-negative patient if
the Kleihauer-Betke stain test is negative in Pregnant Trauma
b. In this case anti-D should be given within 72 hours
c. If the Kleihauer Test is positive then give 625 IU of anti D
d. When indicated in pregnancy the same dose of anti D should be given
when the criteria for its use are met
15)A tackle injury of a 20 yr footballer results in quadriple-
gia, his x-rays shows burst fracture of C4, the most likely
mechanism of his injury;
a. Flexion and compression
b. Vertical (axial)compression
c.m Extension and compression
d. Distraction and flexion

215
16) A 12 yr boy with T4-T12 kyphosis of 70 degrees. What
is the treatment?
a) PT b) Fusion c) Boston Brace
d) Milwakee Brace
17) The surgeon asks your advice for a 25-year old car-
penter with a severe trauma of the non-dominant hand and
wrist. Surgical limb salvage is considered to be impossible.
Which one is the correct advice to the surgeon?
a. a long forearm residual limb is preferred because an optimal
body-powered prosthetic restoration is the goal
b. a long forearm residual limb is preferred because an optimal ex-
ternally powered prosthetic restoration is the goal
c. a short transradial level is preferred because an optimal body-
powered prosthetic restoration is the goal
d. a short transradial amputation does not limit elbow flexion strength
the long, medium and short transradial amputation levels require
the same rehabilitation levels and prosthetic components and there-
fore the surgeon is allowed to decide on the level himself
18) An orthopaedic surgeon refers a man (age 59 years)
who complains of bilateral calf pain after walking 500 a 600
metres. The pain subsides when the patient stops walk-
ing. Lower extremity pulses are normal, as is the rest of the
physcial examination. Computed tomography scan, lumbo-
sacral spine films and electromyogram are within normal
limits. Non-invasive vascular studies only reveal an ankle-
to-brachial ratio of 0.75. You recommend : 
a. Consideration of trans tibial amputation
b. Angiography of vessels in the lower extremity
c. Referral to vascular surgery for vascular bypass
d. Health education and a rehabilitation program
e. Venography to rule out thrombophlebitis

216
19) An athlete complains for 3 months of pain induced
only by athletic activity, often arising at a precise point in the
training session. The pain is located at the anterior aspect
of the lower leg, with pain during stretch of the toe and an-
kle dorsal flexors in inversion. There is a slight weakness in
the tibialis anterior and extensor digitorum longus muscles.
There is also some numbness in the dorsal first cleft of the
toes. X-ray of the lower leg is normal.
At this stage which of the following examinations is indicated:
a. Arteriography of the lower extremity
b. MRI of the lower limb
c. Venography of the lower extremity
d. Radio-isotope bone scan of the lower extremity
e. Anterior tibial compartment pressure measurement
20) One month later he complains of a sudden increase
in pain causing him to stop running. The pain itself is out of
proportion to the clinical situation. Passive stretching of the
anterior leg muscles precipitate excruciating pain and an in-
ability to generate a significant contraction due to pain inhi-
bition. There is also a slight decrease of the dorsalis pedis
artery pulse. At this moment one of the following investiga-
tions is necessary:
a.Venography of the lower extremity
b. Electromyography of the lower extremity
c. Anterior compartment pressure measurement of the lower leg
d. CT-scan of the lower leg
e. Echography of the lower leg
21) Some hours later, due to inadequate management,
muscular weakness and sensory loss increases. This is
in association with intractable pain. The pulse is again de-
creased. The following urgent treatment is indicated:
a. Surgical decompression of the anterior compartment (fasciectomy)

217
b. Compression bandage
c. Cast immobilisation of the lower leg
d. Exploration of the peroneal nerve
e. Thrombectomy
22) A kid has a knee injury. He has a positive external
rotation recurvatum test and opening to varus stress at full
extension and 30 deg. Which are you most likely to see?
a. segond fracture.(LAT. CAPSULAR AVULSION SEEN WITH ACL)
b. tibial plateau fracture.(IT CAN BE SEEN BUT LESS COMMON)
c. avulsion fibular head
d. avulsion PCL from tibia
23) A guy had a achilles tendon rupture many months ago.
MRI shows 6 cm retraction between edges. Which should
you do?
a. primary repair
b. V-Y Achilles tendon repair
c. FHL augmentation
d. allograft reconstruction
24) A22 year old female. neck pain. No neurology. Sys-
temically well. Xrays show a expansile lytic lesion in the
anterior vertebral body extending into the pedicle.Her bone
scan shows multiple mets with increase uptake in the thy-
roid This is most likely
a. hemangioma-anterior elements not lytic/ not metastatic (Whee-
less)
b. ABC -posterior elements of spine/not metastatic (AAOS Core)
c. Metastatic thyroid malignancy
d. TB- involves vertebral body spares post elements (Wheeless)
25) Guy on dialysis for 10 years. Has neck pain. No neu-
rology. Xrays show narrowing of disk space and end plate

218
errosion. MRI shows dark adjacent vertebral bodies. What
is happening?
a. bacterial discitis
b. fungal spondylosis
c. hemodialysis related spondyloarthropathy
d. something else
26) A week old comes in with fevers and won’t move right
arm. He is otherwise healthy and came from Home. Elbow
has a tense effusion. Aspiration will most likely show?
a. Staph aureus b-GBS c-Neisseria
e-something else
27) lady with known breast CA is hypercalcemic with EKG
changes and hemodynamic instability. What is most effec-
tive and fastest treatment acutely to reduce serum calcium
a. bisphosphonates b-calcitonin
c-forced dieresis d-Vit D3
28) A lady sprained her ankle 6 months ago. It’s still sore.
She has an MRI that shows non specific inflammation in the
sinus tarsi. Clinically this is where she’s sore. What should
you do?
a. physio for aggressive subtalar motion
b. cast x 6 weeks
c. OR for debridement of her sinus tarsi
d. Injection of steroids and lidocaine into the sinus tarsi
29) A5 year old kid with a fully segmented L5 hemiverte-
bra and L5-S1 angle of 50 degrees how to treat?
a. Anterior and posterior hemiepiphysiodesis on convex side
b. Surgery should include hemivertebra resection
c. posterolateral fusion only
d. anterior and posterior fusion is required

219
30) A lady has a MOM THA and says “it never felt good”
which test should you not do?
a. ESR CRP b-synovial biopsy
c-metal allergy testing d-serum Co-Cr levels
31) A volleyball player has weakness while playing. on
exam they have infraspinatus wasting. What do you expect
to find on arthroscopy?
a. Bankart b-RTC tear
c-Lateral tear and adjacent cyst d-hill sachs
32) A 23F with a radial head fracture shown on the XR
(picture of XR showing comminuted, displaced radial head
fracture with one piece lying anterior). A CT scan was done
and shows four fragments. What is the best treatment:
a. ORIF b-Radial head excision
c-Radial head replacement d-Sling and splint at 90 degrees
33) A young F with a C5-6 fracture-dislocation who is a
complete quadriplegic is put in traction and she reduces an-
atomically. She had maxillary and mandibular fratures. You
are called to see her later because she develops a decreased
level of consciousness, tinnitus, dysphagia, diplopia, nys-
tagmus and difficulty swallowing. What is the cause?
a. Subdural hematoma
b. Vertebra basilar insufficiency
c. Retropulsion of disk with ascending myelopathy after your reduction
d. She is coning (Unreactive pupils, and blown up pupils)

34) A Patient has numbness on small and ring finger, and


decreased strength in FDP of those two fingers. Where is
the nerve injury:
a. AIN b-Cubital tunnel c-C8 root,as per Toronto
d-Carpal tunnel with Martin-Gruber anastomosis

220
35) A Young man involved in MVC with flail chest, intubat-
ed and other injuries. Also said he had a radial nerve palsy.
Showed AP and lateral of comminuted extra-articular distal
humerus fracture. What is the best treatment for definitive
management of this injury:
a. Ex-fix
b. b-Open IM nail to visualize the nerve
c-Lag screw and neutralization plating
d-Bridge plate
36) A 68 year-old with Parkinson’s has a displaced femo-
ral neck fracture. No pre-existing hip arthritis. What is the
MOST appropriate treatment?
a. Internal fixation b-Hemiarthroplasty
c-THA d-Large-head THA
37 )A 30-ish year-old male diagnosed with GAS necrotiz-
ing fasciitis of the arm. Treatment should consist of:
a-12 hours of IV antibiotics followed by surgical debridement
b-IV antibiotics and immediate surgical debridement removing affected
skin/subcutaneous tissue/fascia with a re-evaluation of questionable muscle
at 24 hours
c-IV antibiotics and immediate surgical debridement removing all affected
and questionable skin/subcutaneous tissue/fascia and muscle
d-24 hours of IV antibiotics followed by reassessment of clinical status

38) A man gets shot in the right mid arm. He undergoes


ORIF. Post-op he is unable to flex his elbow, has loss of
sensation to the lateral aspect of the forearm and has full
flexion and extension strength of the wrist and hand. The
most likely nerve injury is:
a-Median b-Musculocutaneous
c-Radial d-Ulnar

221
39) A 40 y old man, labourer has extensive symptomatic
hallux rigidis of the left foot. Which of the following is the
MOST appropriate treatment for this patient?
a- Resection of the base of the proximal phalanx (Keller)
b-MTP fusion c-Distal chevron d-¾ molded orthotic
40) A 43 y old man with pain in the right forearm and weak-
ness of forearm pronation. Associated with FPL weakness
and weakness of the FDP to the index and middle finger.
What is the likely pathology?
a.Pronator syndrome, tested by resisted elbow flexion in supination
, resisted pronation with elbow EXTENDED,resisted long finger pip flex-
ion
b.AIN compression syndrome
c.Median nerve entrapment
d.All of the above
41) A 25 y old man suffers a simple posterolateral elbow
dislocation. No associated fractures. When he is reduced,
he is found to be stable in extension, but opens up with val-
gus force. Which would be the best treatment for this man?
a. Directly to the OR to repair the injured ligament
b. Splint the patient’s elbow at 90 degrees x 3-4 weeks
c. Splint the patient’s elbow at 90 degrees x 1-2 weeks (in supination)
d. Hinged external fixator to allow for earlier motion and protect the dam-
aged ligament
42) The surgeon asks your advice for a 25-year old car-
penter with a severe trauma of the non-dominant hand and
wrist. Surgical limb salvage is considered to be impossible.
Which one is the correct advice to the surgeon?
a. a long forearm residual limb is preferred because an optimal
body-powered prosthetic restoration is the goal
b. a long forearm residual limb is preferred because an optimal ex-
ternally powered prosthetic restoration is the goal

222
c. a short transradial level is preferred because an optimal body-
powered prosthetic restoration is the goal
d. a short transradial amputation does not limit elbow flexion
strength
e. the long, medium and short transradial amputation levels require
the same rehabilitation levels and prosthetic components and there-
fore the surgeon is allowed to decide on the level himself
43) A 23 y old girl has an extensively tender hallux val-
gus. She has exhausted all non-operative interventions and
needs surgery. On exam, obvious deformity of the hallux
and she does not correct. She has an IM angle of 15 and an
HVA of 40. (you are shown an x-ray that shows no arthritis
of the joint). Of the following, which would be the most pre-
dictable to help?
a.Distal metatarsal osteotomy
b. b-Proximal metatarsal osteotomy
c-Akin osteotomy d-1st MTP fusion
44) A 50 y old alcoholic presents to your office with a 4
month history of severe ankle pain being treated for an “an-
kle sprain”. Initial radiographs and CT provided – shows a
talonavicular dislocation and no evidence of any arthritis at
the tibiotalar or subtalar joint (ie. missed lateral subtalar dis-
location). It is not possible to reduce this closed. What is
the best option at this time:
a.Operating room for triple arthrodesis
b.Cast only POSSIBLE because alcoholic
c.Operating room to ORIF the talar neck fracture
d.Leave the patient, non-operative treatment, with her symptoms.
45) A 20 year-old male with osteoblastic surface tumour
posterior distal femur. Not involving vessels or medullary ca-
nal and no ST mass (likely Parosteal osteosarcoma) shows
that the neurovascular bundle is free of the lsion, there is no
medullary involvement with the femur, and there is no soft

223
tissue component . What is the BEST treatment?
a- Chemo followed by surgery b- Surgery followed by chemo
c. Surgery only d. Chemo only
46) A70 year-old with femoral neck fracture. What is the
reason to do arthroplasty (hemi/THA) versus internal fixa-
tion?
a. Mortality rate is higher with internal fixation
b. ORIF is associated with increased complication rate from 2-10 years
c. Functional outcome is better with THA
d. No differences
47) An Open midshaft tibia fracture. When treating with
I&D 2 fragments devoid of tissue are found. Discarding them
will leave a segmental defect. What do you do?
a- Discard them
b- Scrub them with chlorhexidine and replant them
c- Scrub them with proviodine and replant them
d- Rinse them with something and replant them
48) A 4 year-old treated previously with Ponseti casting
for clubfoot. Has intoeing gait and dynamic supination de-
formity during swing phase. What is the treatment?
a-Tibialis posterior transfer b-Tibialis anterior transfer
c-Medial column lengthening d-Lateral column shortening

49) A 13 year old boy with bilateral painful flat foot. Able
to heel walk and arch reconstitutes with single leg heel rise.
What could you find on physical exam (exact wording)?
a- Claw toes b- Decreased subtalar motion
c- Peroneal spasm d-Into gait
50) A 65 year old man with renal cell carcinoma. With Lytic
lesion in his femur. Full-length views show it to be isolated.

224
How do you treat this 5CM distal to lt.?
a- Long, distally locked Cephalomedullary nail
b- Short distally locked Cephalomedullary nail
c- Standard antegrade centromedullary nail
d- DCS
51) A16 yrs female with epilepsy, on dilantin. Hx of osteo-
malcia. Why?
a) Failure of hydroxylation of 24-25 vitamin D
b) Failure of hydroxylation of 25- vitamin D
c) Failure of resorption calcium in kidney
d) Gut irritation
52) A 12 yr boy with T4-T12 kyphosis of 70 degrees. What
is the treatment?
a) PT b) Fusion c) Boston Brace d) Milwakee Brace
53) A 24-year-old woman presents to the emergency
room with a dislocated knee. In transferring the patient from
stretcher to examining table, the knee is spontaneously re-
duced. Physical examination reveals no palpable or “Dop-
plerable” pulses inthe foot on the affected side and booming
pulses in the foot on the nonaffected side. Proper treatment
would include whichof the following?
A. Doppler evaluation of the arteries in the lower extremity followed by
arteriography if the Doppler study was abnormal.
B. Magnetic resonance imaging (MRI) of the affected leg.
C. Close follow-up examination.
D. Emergent transfer to the operating room for exploration of the popli-
teal artery.
E. Immobilization of the knee with gentle warming of the extremity and
elevation
54) A previously healthy 20-year-old male is undergoing
open reduction and internal fixation of a fractured tibia. The

225
limb was exsanguinated and the tourniquet appropriately
applied and inflated to 250 mmHg prior to surgery. A small
amount of continued bleeding is noted after surgical expo-
sure. The tourniquet still seems appropriately inflated. The
patient’s blood pressure is 110/70 mmHg. You should
A. accept that a tourniquets does not stop all bleeding
B. check the patient’s coagulation profile
C. inflate the tourniquet to a higher pressure
D. reduce the patient’s blood pressure
E. re-exsanguinate the limb and re-apply the tourniquet
55) A well 65 year old is having a THR under general an-
aesthesia with sevo/N2O/fentanyl. BP is 130/70. Before the
surgeon commences reaming and cementing, the best thing
to do is:
A. Induce hypotension B. Raise BP with vasopressors
C. Turn off nitrous D. Give steroids
E. Give heparin 5000u
56) A tourniquet is being used on the arm of an adult pa-
tient to reduce haemorrhage during surgery on the hand.
The maximum recommended time to leave the tourniquet
continuously inflated is:
A. 60 minutes B. 90 minutes
C. 120 minutes D. 150 minutes
E. 180 minutes
57) A 20-year-old, 80 kg, previously well male is awaiting
surgery for a fractured femur. He was admitted 12 hours ear-
lier following a motor-bike accident. His admission chest X-
ray was normal. Since admission, analgesia has been pro-
vided with a femoral nerve block and 40 mg of intravenous
morphine. He has been placed in leg traction. He is now
drowsy and confused. His blood pressure is 120/70mmHg,
pulse 120 min-1 and respiratory rate 25 min-1. On ausculta-

226
tion of the chest, inspiratory crepitations can be heard. De-
spite supplemental oxygen of 6 1.min-1 via Hudson mask
his arterial oxygen saturation (SaO2) is 85%. The most likely
cause of the low SaO2 is:
A. Aspiration B. fat embolism syndrome
C. hypoventilation D. pneumothorax
E. pulmonary contusions
58) Trauma pt, Head Injury GCS 5, high ICPs, best man-
agement for ortho procedure(repeat)
A. Propofol/fentanyl B. Propofol / nitrous
C. Other options with volatiles c. all of the above
59) In anaesthetised patients undergoing controlled ven-
tilation, release of a lower-limb arterial
tourniquet after sixty minutes will
A. cause no change in end-tidal CO2 tension
B. decrease central venous pressure for more than 30 minutes before re-
turning to baseline
C. decrease oxygen consumption
D. increase cerebral blood flow
E. increase core body temperature transiently
60) A forty year old carpenter falls three metres from a lad-
der onto his left leg. He complains of shooting pains down
his left leg when walking and aching pain over his lower
back. A diagnosis of L5-S1 nerve root compression on the
left would be supported by all except;
a. straight leg raising limited to 40 degrees from supine on the left
b. weakness of toe dorsiflexion
c. depressed Achilles’ tendon reflex on the left, compared to the right
d. diminished sensation over the medial aspect of the left calf and ankle

227
61) A patient with a central dislocation of the hip following
a motor car accident
is noted to be shocked on admission, one hour after the accident. The
most likely cause is
A. Ruptured bladder B. Fat embolism
C. Ruptured urethra D. Neurogenic shock
E. None of the above
62) A 13 year old injures his leg at football, presents three
weeks later with a painful lump on the medial aspect of the
lower end of the femur. Xray shows elevated periosteum and
some new bone formation. The most probable diagnosis is:
A. Osteochondroma B. Osteosarcoma
C. Osteoclastoma D. Haematoma
E. Osteomyelitis
63) A 12 year old presents having knocked his knee 3
weeks ago and now has a painful knee. An xray shows the
periosteal lifting with irregular bone growth under the peri-
osteum. The diagnosis is:
A. Osteomyelitis B. Osteogenic sarcoma
C. Osteogenesis imperfecta D. Bony metatases
E. Haemophilia

64) A 24 year old male is admitted to hospital with facial


and scalp lacerations, and fractured femur following a motor
vehicle accident. On the second day he becomes restless,
confused, tachypnoeic with an oral temperature of 38C. The
most likely diagnosis is:
A. Delayed concussion B. Fat emboli syndrome
C. Secondary haemorrhage D. Pneumonia
E. Pneumothorax

228
65) Patient for total knee replacement under spinal anaes-
thetic. Continous femoral nerve catheter put in for post op
pain relief. Good analgesia and range of motion 18hrs post
op. At 24hrs post op, patchy decreased sensation in leg and
unable flex knee.
What is the cause?
A. Compression neurapraxia (i think it said due to torniquet)
B. DVT
C. Muscle ischaemia
D. Damage to femoral nerve
E. Spinal cord damage.
66) A 15 year old boy who injures his knee playing soc-
cer. An MRI shows that the medial meniscus is torn. Damage
to one of the following ligaments must be excluded. Which
one?
a. Anterior cruciate ligament
b. Fibular (lateral) collateral ligament
c. Patellar ligament
d. Posterior cruciate ligament
e. Tibial (medial) collateral ligament
67) A 48 year old man complains of episodes of joint pain
for over 10 years. The pain has occurred mostly in his hands
and feet; it improves and then relapses again. On physical
examination there are deformities of his hands involving
the fingers with swan-neck deformities and ulnar deviation.
Which of the following serologic tests is most likely to be
positive in this man?
a. Anti-double stranded DNA b. Borrelia burgdorferi antibodies
c. Rapid plasma reagin d. Rheumatoid factor
e. HLA-B27

229
68) A 44 year old man with no prior medical history has had
increasing back pain and right hip pain for the past ten years.
The pain is worse at the end of the day. On physical examina-
tion he has enlargement of the distal interphalangeal joints. Ra-
diographs show prominent osteophytes involving the vertebral
bodies and narrowing of the joint space at the right acetabulum.
Which of the following diseases is he most likely to have?
a. Gout b. Lyme disease
c. Osteoarthritis d. Osteomyelitis
e. Rheumatoid arthritis
69) A pathologic study is performed involving patients
with a history of arthritis. The femoral heads of these pa-
tients were submitted to surgical pathology examination
following hip resection and placement with prostheses. The
pathologic findings on microscopic examination of these
specimens are evaluated. Which of the following findings
is most likely to be seen most frequently in these patients?
a. Bone marrow fibrosis b. Crystal deposition
c. Loss of articular hyaline cartilage d. Osteomyelitis of bone
e. Proliferative synovitis

70) A 45 year old man presents with a BMI of 28, making


him overweight. He works in an office, and is mostly seden-
tary during the working day. He does no physical activity.
For health benefits, your exercise prescription should
include advice on gradually increasing physical activity.
Which of the following would be the target for this man?
a. accumulate less than 150 minutes of moderate exercise per week
b. accumulate 150 minutes of vigorous exercise per week
c. accumulate 150 minutes of moderate exercise per week
d. accumulate 70 minutes of vigorous exercise per week
e. accumulate 70 minutes of moderate exercise per week

230
71) A muscle must be overloaded in order to gain strength.
Which one of the following does not overload the muscle?
a. Altering the form of exercise b. Eccentric exercise
c. Increasing the speed of movement d.Increasingtheresistance
e. Stretching the muscle
72) A 62 year old patient with longstanding steroid-de-
pendent asthma presents with lower thoracic back pain. His
wife has noticed that he has developed a stooped posture
over the last year. What is the most likely diagnosis?
a. ankylosing spondylitis b. congenital kyphoscoliosis
c. osteoarthritis d. osteoporosis
e. spinal stenosis
73) A 53 year old women complains of nocturnal pain in
the right hand and wrist that radiates up the arm and that
is relieved by flicking the wrist. On examination there mild
wasting of the thenar eminence. What is the most likely di-
agnosis?
a. base of thumb osteoarthritis b. carpal tunnel syndrome
c. De Quervain’s tenosynovitis d. rheumatoid arthritis
e. tetany
74) A 30-year-old man underwent an anterior lumbar dis-
cectomy and fusion at L4-L5 and L5-S1 through an anterior
retroperitoneal approach 1 month ago. He now reports that
he is unable to obtain and maintain an erection. The most
likely cause of this condition is:
a) Disruption of the sympathetic nerves during anterior lumbar exposure
b) Traction on the parasympathetic nerve at the L4-L5 level
c) Not related to the surgical dissection
d) Injury to the pudendal nerves in the anterior sacral region during dis-
section at the L5-S1 level
e) Sexual dysfunction secondary to retrograde ejaculation

231
75) A patient presents to the emergency department with
a segmental tibia fracture as a result of a gunshot injury with
a 1-cm entrance wound. After appropriate irrigation and de-
bridement, the wound measures 4 cm, there is no arterial
injury, and the wound edges are easily approximated. Based
on the Gustillo-Anderson classification, this wound is grad-
ed as:
(A) Type I (B) Type II (C) Type IIIA
(D) Type IIIB (E) Type IIIC
76) Six weeks after open reduction and internal fixation
of a talar neck fracture, an anteroposterior radiograph of the
ankle reveals a lucency deep to the subchondral surface of
the talar dome. This indicates:
(A) Malreduction of the fracture
(B) Osteonecrosis of the talus
(C) Collapse of the dome of the talus
(D) Associated tibial plafond impaction
(E) Revascularization of the talus

77) An 18-year-old woman arrives in your office 3 years


after sustaining a comminuted right femur fracture treated
with intramedullary (IM) nail fixation. She is complaining
of moderate low back pain. On physical examination, you
note that she has an 8-cm leg length discrepancy, and radio-
graphs confirm that the right femur has healed 8 cm short.
The most appropriate treatment at this point is:
A) Shoe lift on the right
B) Limb lengthening of the right femur
C) Limb shortening of the left femur
D) Observation
E) Left leg epiphysiodesis

232
78) A 39-year-old man sustained a grade II open diaphy-
seal tibia fracture that was treated with irrigation and de-
bridement, external fixation, and delayed wound closure
12 weeks ago. At 9 weeks, the patient’s weight bearing sta-
tus was increased to partial weight bearing and the patient
has since reported slight leg pain. Radiographs indicated a
comminuted diaphyseal fracture of the tibia with no signs
of callus formation. The radiographs also show no signs of
loosening of the external fixator pins. Treatment at this point
should include:
A) Removal of external fixator and placement of a patellar tendon bearing
(PTB) cast, and progression to weight bearing as tolerated
B) Irrigation and debridement
C) Removal of external fixation and intramedullary nailing D) Autogenous
bone grafting
E) Placement of coralline hydroxyapatite into the fracture site
79) A young patient is diagnosed with torticollis. However,
radiographs do not confirm a skeletal cause for the patient’s
head tilt and rotation. Nothing suggests a neoplastic origin
for the disorder. What other tests/examinations should be
performed:
A. Gastroenterologic examination B. Stretching exercises
C. Ophthalmologic examination D. Laboratory tests
E. A and C
80) In an otherwise healthy 57-year-old patient, which of
the following represents the most pressing relative indica-
tion for surgical treatment?
a. A history of chronic, severe axial neck pain
b. A large herniated disk on MRI of the neck
c. Cervical radiculopathy with neurologic findings
d. Cervical myelopathy
e. All the above require surgical treatment

233
81) A 6-year-old boy has neck pain and stiffness following
an upper respiratory tract infection. He presented with his
head tilted to the right and turned to the left 3 weeks ago,
but a soft cervical collar has not been beneficial. There is no
known history of trauma. A computerized tomography scan
shows rotatory subluxation of C1 on C2. The next step in the
treatment of this child is:
a. Observation
b. Open reduction and C1-C2 fusion through an anterior approach
c. In situ C1-C2 fusion posteriorly
d. Cervical traction
e. Hard cervical collar
82) A 34-year-old man involved in a motor vehicle acci-
dent sustains a pneumothorax, a closed femur fracture, and
closed, displaced fractures of the ipsilateral humerus, radi-
us, and ulna. Management of his femur fracture is open re-
duction internal fixation. Management of his humerus frac-
ture should consist of:
a) Skeletal traction
b) Closed reduction and plaster immobilization
c) Open reduction and internal fixation of all fractures
d) External fixation of all fractures
e) Internal fixation of the humeral fracture and immobilization of the fore-
arm fracture
83- A 45 year old was given steroids after renal transplant.
After 2 years he had difficulty in walking and pain in both
hips. Which one of the following is most likely cause?
a. Primary Osteoarthritis b. Avascular necrosis
c. Tuberculosis d. Aluminum toxicity
e. stress fractures
84- A 30 year old man had a road traffic accident and sus-
tained fracture of femur. Two days later he developed sud-

234
den breathlessness. The most probable cause can be:
a. Pneumonia b. Congestive heart failure
c. Bronchial asthma d. Fat Embolism
e. Pulmonary embolism
85- A 55-year-old woman reports a spontaneous onset
of severe pain in her ribs. AP and lateral chest radiographs
show severe osteopenia, two rib fractures, and three ver-
tebral compression fractures. Laboratory studies show a
hemoglobin level of 9.0g/dL and a monoclonal spike on se-
rum protein electrophoresis. Which of the following imaging
studies would be most helpful in establishing the diagnosis?
Skeletal survey B-Technical bone scan
c- Bone density determination d- MRI scan of the thoracic spine
e- CT scan of the chest and abdomen
86- An otherwise healthy 45-year old man has an intraos-
seous low-grade chondrosarcoma of the distal femur with
no dedifferentiation or metastatic disease. Treatment should
consist of;
surgical resection only. b- radiation therapy only.
c- radiation therapy and surgical resection. d- chemotherapy only.
e- chemotherapy and surgical resection
87- - A 28-year-old man with a history of osteogenic sar-
coma 10 years ago was treated with standard multi-agent
chemotherapy that included doxorubicin. Currently, he is
disease free and needs to undergo revision of a distal femur
replacement. What test should be performed as part of his
pre op work-up?
a.Liver ultrasound b- Echocardiography
c- Bone densitometry d- Bone marrow aspiration
e- All of the above

235
88- A12-year-old patient has an epiphyseal lesion. What is
the most likely diagnosis?
Giant cell tumor b- Aneurysmal bone cyst
c-Eosinophilic granuloma
d- Dysplasia epiphysealis hemimelica e Chondroblastoma
89- Patients with multiple hereditary osteochondromas
frequently have loss of forearm rotation and ulnar shorten-
ing. What procedure is most likely to improve forearm rota-
tion in these patients?
a.Tendon transfer
b- Ulnar lengthening
c- Radial head excision
d- Distal ulnar resection
e- Simple excision of the osteochondroma
90- An otherwise healthy 60 year old man reports the sud-
den inability to extend his middle finger. Examination reveals
that the metacarpophalangeal joint of the finger will not ac-
tively or passively extend beyond 45 degrees of flexion. Prox-
imal interphalangeal jointmotion is unrestricted, and compos-
ite finger flexion is to the distal palmar crease. Radiographs
are normal. Treatment of this condition should consist of; 
a. Release of A1 pulley
b. Relocation of the common extensor tendon
c. Reconstruction of the radial sagittal band
d. Exploration of the metacarpophalnageal joint radial collateral
ligament
e. Radical flexor tenosynovectom
91- A 25 year old professional boxer reports pain and
persistent swelling over the metacarpophalangeal (MCP)
joint of his middle finger. Radiographs, including Brewerton
views, are normal. Nonsurgical management, consisting of
a 3-month course of activity modification, extension splint-

236
ing, and anti-inflammatory drugs, has failed to provide re-
lief. Management should now consist of:
a. reassurance and continued nonsurgical care
b. debridement of the MCP joint
c. cortisone injection into the flexor sheath
d. curettage and bone grafting of the third metacarpal head
e. repair of the extensor hood
92- A 40-year-old woman has a chronic boutonniere defor-
mity of the proximal interphalangeal (PIP) joint of her middle
finger with a preserved joint space. She lacks 45 degrees of
active extension but has full passive extension of the PIP
joint. Treatment should consist of 
a. central slip tenotomy b. volar plate release
c. lateral band relocation d. arthrodesis of the PIP joint
e. arthroplasty of the PIP joint

93- A 4-year-old girl undergoes repair of both flexor ten-


dons in zone II. Initial postoperative physical therapy should
consist of
a- passive mobilization only.
b- active extension and passive flexion mobilization.
c- active place and mold mobilization.
d- immobilization.
e- unrestricted active mobilization.
94- A 45-year-old man underwent a zone II flexor tendon
repair. What type of splint will allow for the greatest safe ex-
cursion of the repaired tendons in the first 6 weeks?
a- Splint with continuous passive motion device
b- Static progressive splint
c- Dynamic splint

237
d- Dynamic splint with a palmar pulley
e- Dynamic splint with a hinge allowing for motion at the wrist
95- An otherwise healthy 60-year-old woman has had in-
termittent pain, numbness and tingling in the thumb, index
and middle fingers of her right hand for the past 6 months.
She reports that these symptoms awaken her from sleep
nightly. Two cortisone injections into the carpal tunnel tem-
prorarily resolve her symptoms for 2-4 weeks after each in-
jection. Electrodiagnostic studies are normal. What is the
best course of action?
a.Repeat electrodiagnostic studies
b. MRI of the cervical spine
c.Carpal tunnel release
d.Release of the median nerve in the proximal forearm
e. Referal to a pain clinic
96- A patient sustains a closed dorsal dislocation of the
proximal interphalangeal joint of the middle finger without
an associated fracture. Closed treatment results in a concen-
tric stable reduction. The finger is not immobilized. Which of
the following conditions may appear one year later?
a- Triggering b- Lateral instability
c- Swan-neck deformity d- Boutonniere deformity
e- Loss of distal interphalangeal joint flexion
97- A 60-year-old woman who has had pain and numb-
ness in her thumb, index, and long fingers underwent an
uneventful open carpal tunnel release. Postoperatively, she
reports complete resolution of her symptoms but now notes
progressive thenar atrophy that was not present prior to
surgery. What is the most likely cause of this finding?
a- Incomplete release of the transverse carpal ligament
b- Cervical radiculopathy
c- Unrecognized Martin-Gruber anastomosis

238
d- Unrecognized transligamentous motor branch of the median nerve
e- Median artery pseudoanuerysm
98- A 19-year-old man has hand pain after striking a wall
with a clenched fist. Examination reveals swelling and ten-
derness at the dorsum of the hand. AP and lateral radio-
graphs do not show any obvious pathology. Which of the
following imaging studies should be ordered next?
a- CT scan b- 30 degree pronated view
c- 45 degree supinated view d- Carpal tunnel view
e- Clenched fist view
99- A 25-year-old amateur player sustained a dorsal fx-
dislocation of the PIP joint of his long finger. He underwent
closed reduction with post-reduction radiographs showing
the joint is well reduced with a fracture fragment involving
35% of the volar articular surface of the middle phalanx.
Management should now include
a- ORIF.
b- buddy taping to the adjacent index finger.
c- early motion with application of a dynamic banjo splint.
d- application of a cast with the hand in the “safe position” for 3 weeks.
e- dorsal extension block splinting
100- A 35 year old man has pain in the antecubital fossa
after lifting a couch. Examination reveals limited active flex-
ion and supination secondary to pain. Palpation reveals sig-
nificant pain over the antecubital fossa without a palpable
defect. Management should consist of;
a. Immobilization in a long arm cast at 90 degrees of flexion for 6 weeks
b. Immobilization in a sling for 3 weeks, followed by occupational therapy
c. open exploration of the biceps tendon
d. immediate active-assisted range of motion
e. MRI to evaluate the distal biceps tendon

239
101-A 78-year-old man has long-standing ankle stiffness
and pain that is aggravated by walking. Corticosteroid injec-
tion has not relieved his pain. He has a painful 10-degree arc
of ankle motion. Radiographs reveal end-stage degenerative
arthritis. Which of the following devices will be most effec-
tive?
a- Heel lift
b- Single rocker sole shoe modification
c- Double rocker sole shoe modification
d- Medial heel flare shoe modification
e- Lateral heel flare shoe modification
102-A 55-year-old man who is an avid golfer undergoes a
total hip arthroplasty. What effect would this procedure have
on his golf game?
a- A likely return to play with a minimal change in handicap
b- Return to play is not recommended because of the risk of prema-
ture wear
c- Return to play with a significant increase in average drive distance
d- No need to use a golf cart following hip arthroplasty
e-More pain during the golf swing associated with left-sided surgery
compared with the right side
103-A 55-year-old woman has metastatic breast cancer. She
is given a nitrogen-containing bisphosphonate to inhibit bone
absorption. What is the mechanism of action for this therapy?
a- Disruption of isoprenylation
b- Reduction of carbonic anhydrase activity
c- Tumor inhibition due to tumor cell apoptosis
d- Interference with osteoclast adhesion
e-Interference of tumor cells ability to absorb bone
104- A 3-year-old boy has a septic hip. You suspect that

240
Kingella kingae is the causative organism. What culture me-
dium should be requested?
a- Agar plates b- Thayer-Martin plates
c- Blood culture medium d- Egg-based mediums
e- Luria Bertani medium
105- 16-A 35--year-old man sustains an isolated, high-
energy right pilon fracture. The brake travel time returns to
normal (presumed ability to drive) how many weeks after the
patient begins weight bearing?
a-0 (when weight bearing begins) b-2 c-4 d-6 e-8
106-A 42--year-old man undergoes direct repair of an
acute patellar tendon rupture. To permit early knee range of
motion while protecting the integrity of the repair, which of
the following methods is recommended?
a- Standing squats
b- Closed chain kinetic quadriceps exercises
c- Active knee flexion in a prone position
d- Active range of motion in a seated position without resistance
e- Active-assisted range of motion in a seated position without re-
sistance
107- In addition to an appropriate course of antibiotics, which
of the following is the most appropriate definitive treatment for
a child who has acute hematogenous osteomyelitis and a 2-cm
by 2-cm abscess within the distal femur metaphysis?
a-Percutaneous biopsy and culture
b-Percutaneous biopsy, culture, curettage
c-Percutaneous biopsy, culture, curettage, and bone grafting
d-Open biopsy, culture, and debridement
e-Open biopsy, culture, and en bloc resection of the distal femur
with application of external fixation

241
108-A 56-year-old woman sustained a left distal radius
fracture when she fell getting out of a chair. In addition to
fracture care, the next step in assessment and management
should include which of the following?
a-A bone scan
b-Pelvis and spine radiographs
c-Ultrasound bone density testing
d-Dual-energy x-ray absorptiometry
e-Urinary pyridinolines and N-telopeptides
109-A 75-year-old man has pain in the proximal thigh after
a fall. He appears to be in good health, and reported mild
thigh pain prior to his fall. The radiograph reveals a trans-
verse fracture of the proximal shaft of the femur with some
lateral cortical thickening at the fracture site. What aspect of
the patient’s history most likely explains this fracture?
a-Smoking b-IV drug use
c-Long-term steroid use d-Chronic bisphosphonate use
e-Family history of rickets
110-In a 2-year-old child, what neurovascular structure is
most likely to be injured while performing a trigger thumb
release?
a-Princeps pollicis artery b-Ulnar digital nerve
c-Radial digital nerve
d-Thenar motor branch of the median nerve
e-Palmar cutaneous branch of the median nerve
111- A-3-year-old girl with an L4 level myelomeningocele
is ambulatory with a walker and ankle-foot orthoses. Radio-
graphs reveal bilateral dislocated hips. On the right side, the
acetabulum is quite shallow; however, on the left side it is well
formed. What is the most appropriate treatment for both hips?
a- Continued nonsurgical management

242
b- Open reduction on the left side and closed reduction on the right
side
c- Bilateral open reduction with external oblique transfer to the
greater trochanter on the right ide only
d- Bilateral open reduction of both hips with bilateral transfer of the
iliopsoas to the greater trochanter
e- Bilateral open reduction of both hips with iliopsoas transfers to
the greater trochanter bilaterally and right pelvic osteotomy
112-A 135-lb, 11-year-old boy sustained a closed commi-
nuted midshaft femur fracture. What is the best instrumenta-
tion option?
a-Bridge plating b-Flexible titanium nails
c-Closed reduction and spica cast
d-Solid retrograde intramedullary rod
e-Solid antegrade piriformis fossa entry intramedullary rod
113-A 57-year-old woman sustains a low-energy fracture
of the distal radius. In evaluating for metabolic disorders,
what is the most likely serum abnormality?
a-Elevated PTH b-Decreased CTX
c-Decreased NTX
d-Decreased alkaline phosphatase
e-Low 25-hydroxy cholecalciferol (25 OH vitamin D)
114-A 60-year-old woman underwent total knee arthro-
plasty. Her twin brother underwent the same procedure last
year.What gender-related differences could be expected?
a-Lower outcome scores in women
b-Improved range of motion in women
c-Improved implant survivorship in women
d-No difference as long as gender-specific implants are used
e-Higher incidence of postoperative patellofemoral pain in women

243
115-A 12-year-old boy has a slipped capital femoral epiph-
ysis. He can bear weight with assistive devices. What is the
likelihood of development of osteonecrosis after in situ fixa-
tion?
a-Less than 10% b-10% to 35% c-36% to 65%
d-66% to 90% e-Greater than 90%
116- A patient undergoing rehabilitation after anterior cru-
ciate ligament reconstruction experiences postoperative
arthrofibrosis. What clinical finding is most expected to be
present?
a-Patella alta b-Large knee effusion
c-Increased posterior drawer d-Decreased patellar translation
e-Resolution of the extensor lag by postoperative day one
117-A 45-year-old man sustained a right sacroiliac joint
dislocation and a symphysis rupture in a motorcycle acci-
dent. He was initially hypotensive and tachycardic and was
given fluid resuscitation. Which of the following findings is
the best indicator that he is fully resuscitated and ready for
definitive skeletal stabilization?
a-Hemoglobin is normal b-Heart rate has decreased
c-Blood pressure is normal d-Basedeficitis0-3mmol/L
e-Urine output is greater than 30 mL/h
118-Which of the following scenarios involving treatment
options for specific pediatric elbow fractures has the high-
est rate of developing a nonunion or fibrous union?
a-A Gartland type I supracondylar humerus fracture treated with a
long-arm cast for 3 weeks
b-A Gartland type III supracondylar humerus fracture treated with
closed reduction and percutaneous pinning
c-A 1-cm displaced lateral condyle fracture treated with open re-
duction and pin fixation

244
d-A 1-cm displaced medial epicondyle fracture treated with a long-
arm cast for 3 weeks
e-A flexion-type supracondylar humerus fracture treated with open
reduction and internal fixation
119-A 36-year-old woman sustains an oblique, closed
fracture of the humeral shaft 11 cms proximal to the lateral
epicondyle with associated radial nerve palsy. The fracture
is treated closed with the nerve injury treated expectantly.
Which of the following is the first muscle expected to dem-
onstrate evidence of re-innervation?
a-Supinator b-Brachioradialis c-Extensorpollicislongus
d-Extensor indicis proprius e-Extensor carpi radialis brevis
120-A 70-year-old man hits his head in a fall. He has upper
extremity weakness and dysesthesias. Examination reveals
profound bilateral upper extremity weakness with 4/5 quad-
riceps and 5/5 anterior tibialis and gastrocsoleus strength.
This finding is most consistent with which of the following
syndromes?
a-Brown-Sequard b-Central cord c-Anterior cord
d-Posterior cord e-Cervical myelopathic
121- A 19-year-old football player sustains a direct blow to
the medial side of the tibia while being tackled. MRI reveals
an isolated lateral collateral ligament tear of the knee. Exam-
ination would be expected to reveal which of the following?
a-Increased tibial external rotation at 30 degrees knee flexion
b-Increased varus laxity at 0 degrees
c-Increased varus laxity at 0 degrees and 30 degrees knee flexion
d-Increased varus laxity at 30 degrees knee flexion
e-Increased varus laxity at 0 degrees and increased tibial external
rotation at 30 degrees knee flexion

245
122-A patient allowed no more than touch-down weight
bearing on the right lower extremity could use what type of
assistive device support?
a-Two crutches
b-A single crutch in the left hand
c-A single crutch in the right hand
d-A cane in the left hand and a single crutch in the left hand
e-A cane in the right hand and a single crutch in the right hand
123-A 52-year-old man has difficulty walking 3 months af-
ter undergoing successful transmetatarsal amputation for
ischemic gangrene. Observation of his gait reveals a short-
ened stance phase and abrupt abbreviated toe off on the
surgical side. Which of the following devices would most
likely improve his gait?
a-Solid ankle ankle-foot orthosis
b-Hinged ankle ankle-foot orthosis
c-Three-quarter length steel shank shoe modification
d-Full-length steel shank shoe modification
e-Full-length steel shank and rocker sole shoe modification
124-One year after undergoing medial compression screw
fixation of a talar neck fracture with medial comminution, a
55-year-old man has a painful limp. Eversion is limited and
radiographs reveal a healed fracture. What is the next most
appropriate step in treatment?
a-Lateral column lengthening
b-Medial calcaneal sliding osteotomy
c-Crescenteric osteotomy of the talus
d-Opening medial wedge osteotomy of the talus
e-Closing lateral wedge osteotomy of the talus
125-Moderate or severe pain at 6 months after musculo-

246
skeletal injury can be as high as 38% depending on the type
and extent of injury. Which of the following factors is a sig-
nificant independent predictor of moderate or severe pain
after musculoskeletal injury?
a-Male gender b-Female gender
c-Age of younger than 65 years d-High school graduate
e-History of pain interfering with activities
126-Minimally invasive total hip arthroplasty using a Wat-
son-Jones approach as compared with a traditional trans-
gluteal (anterolateral) approach results in which of the fol-
lowing?
a-Decreased blood loss
b-Decreased surgical time
c-Similar early gait kinematics
d-Increased risk of posterior dislocation
e-Increased risk of heterotopic bone formation
127-Fixation of a displaced two-part pertrochanteric hip
fracture is performed with a compression screw and side plate
(dynamic hip screw). In contrast with a right hip fracture, a left
hip fracture is more inclined to what form of malreduction?
a-Varus b-Valgus c-Axial shortening
d-Proximal fragment flexion e-Proximal fragment extension
128-A 67-year-old woman with painful rheumatoid arthri-
tis presents for metacarpophalangeal joint arthroplasties of
all four fingers. What is the expected functional outcome 1
year after surgery?
a-Increased flexion but no improvement of the ulnar drift
b-Decreased extensor lag and improvement of the ulnar drift
c-Decreased total range of motion but improvement of the ulnar
drift

247
d-Improvement in total range of motion but increased extensor lag
e-No change in total range of motion or extensor lag
129-A morbidly obese, diabetic 50-year-old woman underwent
knee arthroplasty and did well initially. Three weeks after surgery
she has a sudden gush of fluid from the knee. She is seen in
the emergency department and placed on antibiotics. Two days
later clear fluid is still draining from her incision. Her C-reactive
protein and erythrocyte sedimentation rate are normal. The knee
is not red, hot, or painful. Range of motion is 0 to 120 degrees.
What would be the next most appropriate step in management?
a-Arthroscopic debridement b-Ten days of oral
antibiotics
c-Six weeks of IV antibiotics
d-Irrigation and debridement with possible polyethylene exchange
e-Placement in a knee immobilizer and follow up in 3 days for re-
evaluation
130-A patient has a subtrochanteric femoral fracture in
which both the greater and lesser trochanters remain in
continuity with the proximal fragment. What is the resultant
deformity of the proximal fragment?
a-Abduction, flexion, internal rotation
b-Abduction, extension, internal rotation
c-Abduction, flexion, neutral rotation
d-Abduction, flexion, external rotation
e-Adduction, extension, external rotation
131-A 72-year-old man has thigh discomfort. A radiograph
reveals a mid-diaphyseal solitary femoral lesion with miner-
alization. He has a history of prostatic carcinoma diagnosed
10 years ago. What is the next most appropriate step in man-
agement?
a-Biopsy

248
b-Intramedullary stabilization
c-Intramedullary stabilization with cement augmentation
d-Presurgical embolization and intramedullary stabilization
e-Local irradiation followed by intramedullary stabilization
132-A 13-1/2-year-old boy of average height, whose skel-
etal age matches his chronologic age, is predicted to have a
3.4-cm limb-length inequality at skeletal maturity. The differ-
ence is primarily tibial. Which of the following is considered
the best treatment to equalize limb length?
a-Immediate tibial epiphysiodesis
b-Immediate femoral epiphysiodesis
c-Immediate combined femoral and tibial epiphysiodesis
d-Tibial epiphysiodesis at age 15 years
e-Combined femoral and tibial epiphysiodesis at age 15 years
133-Locking plate fixation is employed to manage a dis-
placed two-part proximal humerus surgical neck fracture in
a 68-year-old woman. What method best ensures preserva-
tion of the reduction?
a-Deliberate varus reduction
b-Insertion of a laterally positioned intramedullary fibular strut graft
c-Insertion of inferomedial locking (“calcar”) screws within the hu-
meral head
d-Insertion of nonlocking “pull” screws subsequent to locking
screws within the humeral head
e-Inclusion of suture fixation within the osseous portions of the tu-
berosities
134-A female patient walks with an “antalgic gait”. What
gait abnormality does the term antalgic describe?
a-Unsteady wide-based gait b-Pelvic tilt at
midstance

249
c-Shortened stance on the affected limb
d-Advancing leg lifted high to clear toes
e-Decreased quadriceps moment in midstance
135-A 1-month-old girl has weakness of the right upper
extremity that has been present since birth. In the work-up,
which of the following findings would suggest a favorable
outcome?
a-Ptosis b-Twitch biceps activity
c-Periscapular muscle atrophy
d-Motor activity on electromyography
e-Meningoceles on magnetic resonance imaging
136- A deep, 5-cm soft-tissue mass is removed without
prior radiographic evaluation, expecting it to be a lipoma.
The mass was intracompartmental with good hemostasis.
The final pathology report lists the diagnosis as a high-grade
pleomorphic sarcoma. Treatment should include which of
the following?
a-Amputation
b-Radiation therapy without more surgery
c-Chemotherapy without more surgery
d-Close observation for the next 2 years
e-Radical re-resection of the tumor bed and radiation therapy
137- A 15-year-old girl with acetabular dysplasia requires
a pelvic osteotomy. The hip is concentric with a center edge
angle of 5 degrees. Which of the following is the most ap-
propriate type of osteotomy?
a-Chiari b-Dega c-Ganz d-Salter e-Pemberton
138- During cemented bipolar hip for fracture NOF in
82yrs old woman. She suddenly became hypotensive and
hypoxic. What is the most likely reason?

250
a.Pulmonary embolism caused by venous trauma during reduction
b.Pulmonary embolism caused by preexisting DVT
c.Intramedullary fat embolism
d.Inadequate preoperative fluid replacement
e. Vasodilatation caused by cement
139- A 26yrs old man has R. knee pain due to RTA. X-rays
reveal a distal femoral fracture with metaphyseal comminu-
tion. What is the most common missed injury diagnosed by
CT scan?
a.Coronal fracture of medial femoral condyle
b.Coronal fracture of the lateral femoral condyle
c.Sagittal fracture of the med. Femoral condyle
d.Sagittal fracture of the lat. Femoral condyle
e.ACL injury
140- A 4yrs old boy has fever, in ability to stand for two
days, O/E tenderness over the distal aspect Rt femur with
no joint effusion. His WBC count 13,300/mm, ESR 55mm/h
.Which of the following x-rays finding is most likely to be
present?
a.Well circumscribed lytic lesion b-Periosteal reaction
c-Sequestrum d-Deep soft tissue swelling
e-Cortical erosion
141- In a healthy 38yrs old woman she develops numb-
ness over the medial border of her Lt. forearm and hand fol-
lowing a low impact RTA a month ago. O/E; difficulty with
fine motor function in the Lt. hand with weakness in long
flexor function of all her digits and thumb.What is the most
likely diagnosis?
a.C8 radiculitis
b.Incomplete spinal cord injury

251
c.Ulnar nerve palsy at elbow
d.Ulnar nerve palsy at Guyon’s canal
e.Secondary problem because the find do not fit an anatomical pat-
tern
142- On examination of 25yrs old man with hereditary
motor sensory neuropathy you find a cavo-varus hind foot
correctable with Coleman block testing. Treatment should
consist of:
a.Plantar fascial release, dorsal closing wedge osteotomy of 1st MT
and peroneous longus to pernoeous brevis tendon transfer
b.Split anterior tibialis tendon transfer
c.1st.MP arthrodesis and ETA
d.Dwyer’s calcaneal osteotomy and T. posterior tendon transfer
e.Triple arthrodesis
143- A patient sustained a distal fibular fracture treated
conservatively. Current x-rays reveal an increased talo-cru-
ral angle. What is the most likely long term abnormality re-
lated to his malunion?
a.Medial Talr tilt b-Lateral talar tilt
c-Anterior talar subluxation
d-Posterior talar subluxation
e-Medial osteochondral defect
144- A 18yrs old footballer twists his knee , unable to pas-
sively extend it beyond 25degrees. MRI shows displaced
bucket-handle tear lateral meniscus. Arthroscopy confirms
that with a 3-mm peripheral rim. What is the best line of
treatment at this stage?
a.Partial Meniscetomy
b.Reduction of the meniscus and inside-out repair with vertical mat-
tress sutures

252
c.Reduction of the meniscus and repair with bioabsorbalbe arrows
d.Reduction of the meniscus and repair with non absorbable stitches
e.Reduction of the meniscus and repair with mulberry knots
145- A27yrs old boxer has pain and swelling over the MCP
of middle finger, x-rays are normal. He was treated conser-
vatively by a course of activity modification, splintage in
extension and NSAIDs with no improvements. His manage-
ment should consist of;
a.Reassurance and continued the same
b.Debridement of the MCP joint
c.Steroid injection into flexor sheath
d.Curettage and bone grafting of his finger
e.Repair of the extensor hood.
146- A12 yrs boy sustained a knee injury after a fall of his
bike, the pain was better a day after the injury. X-rays reveal
a lesion in the distal femur, management should include;
a.Open biopsy and intralesional excision
b.CT guided needle biopsy and antibiotic therapy
c.Curettage and autologus bone grafting
d.NSAIDs only
e.Observation and follow up
147- A 8yrs old boy underwent MUA & pining of a SC frac-
ture elbow 7 hrs ago, He requires an increasing dose of pain
killer, his fingers are warm and pink with intact sensation
and palpable distal radial pulse, but he is not able to extend
his fingers actively with painful passive extension. His man-
agement should include;
a.Elevation, ice packing and reassessment the next day
b.Increase dosage of his pain killer
c.Repeat x-rays for fear of displacement

253
d.Measurement of compartment pressure
e.Measurement of PT and PTT
148- A 29 yrs old sportsman has gradual worsening plan-
tar heel pain that is most pronounced with first few steps
getting out of bed, examination shows tenderness over the
medial calcaneal tuberosity. What is the most likely diagno-
sis?
a.Calcaneal stress fracture
b-Entrapment of lateral plantar nerve
c-Plantar fasciitis
d-Posterior ankle impengment
e-FHL tendinitis
149- A 24 yrs old man with grade III isthmic spondylolesthe-
sis of L5-S1 planned for surgery , the procedure should include;
a.Posterior lumbar fusion
b-L5 corpectomy
c-Only laminectomy
d-Stand-alone anterior inter body fusion
e-Complete reduction of the deformity
150- An obese 73yrs old woman with isolated medial knee
OA with 110* of passive motion , she is going for UCKA.
Which of the following is considered the greatest advantage
of unicompartmental knee arthroplasty compared to TKA?
a.Improved ROM post operatively
b.Greater prothetic longevity
c.Greater relief of postoperative knee pain
d.Faster early rehabilitation
e.Better correction of postoperative deformity

254
151- A 77yrsold man with history of smoking underwent
a successful Lt. THR 10yrs ago, He now complains of 2
months of progressive pain of his Rt. Hip. Ap views and CT
scan shows fracture actabulum and metastasis spots of the
pelvis. What is the next most appropriate step in his man-
agement?
a.Immediate cemented Rt. THR
b.Open reduction and IF of the acetabular fracture
c.Activity modification, IV biphospanate and follow up
d.Tc99 and CT scan of the chest. Abdomen and pelvis
e.Radiotherapy
152- A25yrs old woman who had excision of lateral sesa-
moid for intractable kerastosis, now developed painful hy-
pertrophic callus with no benefit of all types of conservative
treatment, her surgeon advised excision of the remaining
sesamoid. What would be the most common complication
of this procedure?
a.Claw toe deformity b-Painful neuroma
c-Painful scar d-Hallux varus
e- Hallux valgus
153- A 22 yrs old man sustained a pivoting injury to his Lt.
knee few days back, Examination reveals effusion and lim-
ited extension/ flexion to only 45*. MRI showed acute ACL.
Initial management should consist of;
a.Immobilization with POP cast
b.Immediate surgical construction
c.Preoperative physio for mobilization
d.Prophylactic bracing and return to activity
e.Arthrocentesis and rest

255
154- A 10yrs old girl sustains fracture dislocation of her
Rt. Hip. What is the most likely complication following open
reduction& IF of this fracture?
a.Sciatic nerve palsy b-Non union
c-Osteonecrosis d-Chondrolysis e-Coxa valga
155- While you are performing quadriceps sparing MIS for
TKA, the patellar tendon starts to peel off the tibia tubercle.
Reconstruction is stopped before the integrity of the tendon
is compromised. What is the best course of action?
a.Transfixation pins to the tibial tubercle to protect the tendon
b.Repair the peeled portion of the tendon with suture anchors
c.Augmentation of patellar tendon with semi tendenosus autograft
d.Convert the technique to a mid-vastus approach
e.Covert to the standard medial para patellar arthrotomy
156- A 2-week-old girl has hematogenous septic arthritis
of her right knee. There are no open lesions and no other
sites of infection. The most common infecting organism in
this setting include S. aureus and;
a. Group B Strep and Gm(-) Bacilli
b. Kingella kingae and H. influenza B
c. Neisseria gonorrhoeae and Borrelia burgdorferi
d. Yesinea species and H. influenza B
e. Kingell kingae and Mycoplasma variants
157- . After repair of a torn quadriceps tendon, which of
the following rehabilitation protocols provides range of mo-
tion and minimal stress on repair?
a. Active open chain extension and active open chain flexion
b.Active open chain extension and active closed chain flexion
c.Active-assisted extension and active open chain flexion

256
d.Passive extension and active open chain flexion
e.Passive extension and active closed chain flexion
158- A healthy, active 26-year-old woman sustained a dis-
placed two-piece calcaneal fracture with subluxation of a
posterolateral fragment into the subfibular recess. Recom-
mended treatment should consist of
a.Open or percutaneous reduction and internal fixation
b.Excision of the posteriolateral fragment
c.non-weight-bearing and early mobilization
d.Casting for 3 months
e.posterior arthroscopic debridement
159- A 67 year-old man with diabetes mellitus has under-
gone initial casting for a displaced midfoot neuroarthropath-
ic collapse. Examination reveals an unstable midfoot with
blanching of the skin with weight bearing. What is the most
appropriate step in management?
a.Charcot restraint orthotic walker (CROW)
b.Triple arthrodesis
c.reduction of the midfoot and application of an external fixator
d.midfoot arthroplasty
e.Rigid foot orthosis with posting of the lateral midfoot
160- A 58 yo woman with rheumatoid arthritis reports
progressive neck pain and difficulty with fine motor move-
ments, including playing cards and handling coins. Exami-
nation revealed hyperreflexia without objective weakness.
What is the most important radiographic factor that may pre-
dict neural recovery after decompressive surgery?
a.Basilar invagination of less than 1cm
b.Atlanto-dens interval of greater than 14mm
c.Posterior atlanto-dens interval of less than 5mm

257
d.Sublaxial subluxation of less than 4mm
e.Rotary subluxation of less than 10*
161- If the posterior condylar axis is used for determining
the rotation of the femoral component, which of the follow-
ing errors is expected when 3˚ of external rotation are built
into the jig that controls femoral rotation?
a.Internal rotation of the femoral component if the lateral femoral
condyle is hypoplastic
b.Internal rotation of the femoral component in a varus knee
c.Internal rotation of the femoral component when there is wear of
the posterior aspect of the medial femoral condyle
d.External rotation of the femoral component if the lateral femoral
condyle is hypoplastic
e.Femoral component rotation parallel to the epicondyle axis in all
knees
162- A 15 month old boy requires treatment with a halo
device. What is the preferred pin configuration?
a. Four pins, with a torque of 6 to 8 in-lb
b.Four pins, with a torque of 2 to 4 ft-lb
c.Four pins, with a torque of 6 to 8 ft-lb
d. Six to twelve pins, with a torque of 2 to 4 in-lb
e.Six to twelve pins, with a torque of 2 to 4 ft-lb
163- A 35-year-old man has pain, limited motion, and
swelling of his middle finger after sustaining a volar punc-
ture wound 3 days ago. Examination reveals a flexed resting
posture, pain with passive stretching, and tenderness along
the flexor tendon sheath. Appropriate management should
consist of
a.Outpatient management with oral antibiotics.
b.IV antibiotics alone.
c.Surgical drainage over the entire length of the finger.

258
d.Surgical drainage via small incisions and intraoperative sheath ir-
rigation with a catheter.
e.Needle aspiration of the flexor tendon sheath with continuous ir-
rigation and oral antibiotics.
164- A 37-year-old man is undergoing an anterior cervi-
cal discectomy and fusion for a large central disk hernia-
tion. Following placement of an interbody strut into the disk
space, motor-evoked potentials are lost in the right and left
upper extremities. What is the next most appropriate step in
management?
a. Close the wound and perform a laminectomy.
b.Close the wound and obtain an emergent MRI.
c.Perform a wake-up test.
d. Remove the interbody strut.
e. Insert an anterior cervical plate.
165- A 35-year-old runner who has had intermittent calf
pain for the past 4 months now reports cramping, coolness
of the leg, and occasional paresthesias of the foot. Symp-
toms are worse with walking and relieved with running. Ex-
amination reveals that dorsiflexion and plantar flexion of the
ankle accentuate the patient’s symptoms. What diagnostic
study will best confirm the diagnosis?
a.Electromyography and nerve conduction velocity studies
b-Vascular studies
c- Compartment pressure measurement
d- Noncontrast MRI
e- Noncontrast CT
166- Examination of a 52-year-old man with a 4-week his-
tory of hip pain reveals a destructive, lytic proximal femoral
lesion. He has not history of malignancy. What is the most
likely diagnosis?

259
a.Metastasis
b-Brodie’s abcess
c-Hydatid cyst
d-Hyperparathriodism
e-Simple bone cyst
167- A 44-year-old woman has a symptomatic bunion and
a painful plantar callus under the second metatarsal head
that continues to limit her activity and shoe wear despite the
use of shoe modifications. Radiographs show a intermetar-
sal ankle of 18 degrees, a hallux valgus angle of 38 degrees,
and a first metatarsal that is shorter than both the second
and third metatarsals. When considering surgical options,
each of the following first metatarsal procedures are appro-
priate except ;
a. Z osteotomy (Scarf )
b. oblique proximal osteotomy (Ludloff )
c. distal chevron osteotomy
d. proximal crescentic osteotomy
e. Lapidus procedure
168- A 30-year-old man sustained a calcaneal fracture
4 years ago that was treated nonsurgically. Posttraumatic
arthritis has now developed. He elects to be treated with a
subtalar bone block distraction arthrodesis. This procedure
attempts to correct all of the following abnormalities com-
monly seen after calcaneal fractures except;
a. anterior ankle impingement
b- subfibular peroneal impingement
c- hindfoot valgus
d- limb shortening
e- subtalar arthritis

260
169- When patients with rheumatoid arthritis are being
treated with the disease-modifying antirheumatic drug adali-
mumab, what is the optimal time for surgical intervention?
a.Immediately after infusion
b.Approximately 10 days after injection
c.Only after a year of continuous therapy
d.When drug levels are at their highest
e.When the pharmacokinetic levels reach equilibrium
170- A 60-year-old patient who has diabetes mellitus with
peripheral neuropathy has a plantar ulcer on the plantar sur-
face of the second metatarsophalangeal joint. Examination
reveals no fluctuance or purulent drainage, pulses are pal-
pable, and the ulcer does not involve bone. Range of motion
of the ankle reveals dorsiflexion to neutral and 15° of plantar
flexion. Which of the following treatments offers the lowest
recurrence rate of ulceration?
a. Early débridement and excision of the metatarsal head
b-Second ray resection
c- IV antibiotics with ulcer debridement
d-Total contact casting
e- Total contact casting with gastrocnemius recession
171- A 43-year-old woman who is right-hand dominant fell
onto her outstretched arm while rollerblading 1 day ago. She
reports a painful wrist. Examination reveals swelling and
tenderness dorsally. Radiographs reveal a non displaced
transverse fracture of the distal radius. She is placed in a
short arm cast. What can be done to reduce the risk of type1
complex regional pain syndrome?
a.Transcutaneous electrical nerve stimulation
b. Occupational therapy treatment for finger dexterity
c.Strict elevation above the heart for 72 hours

261
d.Alpha adrenergic blockers for 2 weeks after injury
e. Daily oral vitamin C for 2 months
172-42-year-old health care professional has had knee
pain for the past 2 months. An MRI scan of the knee reveals a
large effusion with loculations and synovial thickening, and
results of an open biopsy and culture are consistent with
tuberculosis. Sensitivity tests show no resistance to antibi-
otics. Following debridement and synovectomy, appropriate
antibiotic therapy should include
a. rifampin and pyridoxine.
b-rifampin and ethambutol hydrochloride.
c- isoniazid. d- isoniazid and pyridoxine.
e- isoniazid, rifampin, pyrazinamide, and pyridoxine
173- Improvement in hip range of motion following ce-
mented total hip arthroplasty in patients with ankylosing
spondylitis has been found to be limited by
a. infection. b- neurologic involvement.
c- heterotopic ossification. d- soft-tissue contractures.
e- ankylosis of the lumbar spine
174- A 76-year-old woman has had generalized muscle
weakness and arthralgias for the past 2 years. Radiographs
show generalized osteopenia and a pseudofracture (Loos-
ers zone) in the inferomedial aspect of the femoral neck.
Laboratory studies show normal serum calcium and hemo-
globin levels and a mildly elevated alkaline phosphatase
level. What is the most likely diagnosis?
a. Osteomalacia b- Osteoporosis c- Multiple myeloma
d-Leukemia e- Pagets disease of bone
175- An otherwise healthy 37-year-old man has had bilat-
eral posterior heel pain for the past year. Examination re-
veals fullness, warmth, and tenderness over the posterior
aspect of the heels. Radiographs are normal. Laboratory

262
studies show a normal CBC and an erythrocyte sedimenta-
tion rate of 50 mm/h (normal up to 20 mm/h), and an HLA-
B27 is positive. What is the most likely diagnosis?
a. Rheumatoid arthritis b- Ankylosing spondylitis
c- Lupus erythematosus d- Reiter syndrome e- Lyme disease
176- A 21-year-old runner who is 160cm tall and weighs
52kg reports the onset of left leg pain when she increased
her training program. History reveals that her last menstrual
period was 18 months ago. Bone density of the spine tested
by DEXA is 1.8 standard deviations below the mean for age.
A bone scan is normal. Management should include
a. cessation of running and 10 mg of alendronate per day.
b. nutritional counseling and a stretching program.
c. an ultrasound of the heel and 500 mg of calcium per day.
d. calcitonin, a short leg cast, and multivitamins.
e. evaluation of the amenorrhea, achievement of nutritional bal-
ance, and cross- training
177- A patient undergoing primary total knee arthroplasty
with a valgus knee remains tight laterally in both full exten-
sion as well as deep knee flexion. Release of what structure
will help balance the joint?
a- Iliotibial band b- Popliteus
c- Posterior lateral capsule d- Posterior cruciate ligament
e- Lateral collateral ligament
178- 39-year-old male presents in the emergency room
after a high-speed motor vehicle accident. The patient has
been intubated by paramedics at the scene and is on as-
sisted ventilation. He is unconscious. Physical examination
reveals a distended abdomen, and initial screening x-rays
reveal a displaced fracture of the pelvic ring. Initial evalua-
tion should include all of the following except?
a. Fluid resuscitation and establishment of venous access.

263
b. Diagnostic peritoneal lavage.
c. Thorough physical examination, including evaluation of the urinary and
lower gastrointestinal tract.
d. Application of external fixation.
e. CT of the abdomen.
179- 60 yrs old male with bony abnormality at upper tibia
associated with sensorineural hearing loss. On laboratory
examination serum alkaline phosphatase levels are (440
mU/l) elevated and serum Ca and PO4 are normal. Skeletal
survey shows ivory vertebrae and cotton wool spots in x-ray
skull. Diagnosis is :
a) Fibrous dysplasia
b) Paget disease
c) Osteosclerotic metastasis
d) Osteoporosis
e.Metastasis
180-The best outcome following reverse total shoulder
arthroplasty has been reported for what preoperative etiol-
ogy?
a- Fracture sequelae b- Rheumatoid arthritis
c- Rotator cuff tear arthropathy d- Failed hemiarthroplasty
e- Conversion of proximal humerus resection
181- A 23 young male with gunshot wound to L wrist
from handgun. Muzzle velocity is 1500 ft/s. (Around500m/
sec) Dorsal and volar wounds measure 3mm in diameter are
present but not bleeding. He is NVI fully at the hand, tendons
all work. What of these statements is true?
a. local wound care in ED with preoperative antibiotics for 3 days
b. Gunshot wounds of this type is of a high velocity
c. Even table neuro deficits needs exploration

264
d. Shotguns of this type are considered low energy
e. all of the above
182- A 47-year-old woman presents to an orthopaedic sur-
geon with a 3-month history of progressive edema in her
right medial foot and ankle. There is no history of trauma,
but the woman was a long- distance runner in college. The
initial pain has sub-sided, but she now has difficulty walking
because of an acquired progressive flatfoot. Which of the
following findings would best support your diagnosis?
(a) Equal strength in both feet for the single heel raise test
(b) More toes seen lateral to the heel from a posterior view
(c) Decreased posterior tibiocalcaneal angle
(d) Swelling below the lateral malleolus
183. A 22-year-old ballet dancer presents with pinpoint
pain in her left forefoot in the third intermetatarsal space,
which radiates to her toes when she walks. Manipulation of
the third and fourth metatarsal heads elicits a slight click
(positive Muldor sign), which confirms a diagnosis of Mor-
ton’s neuroma. All of the following are considered helpful
therapies for this condition EXCEPT
a) Customized foot orthotics and wide-toed athletic shoes to avoid
compression
b) Dorsal or plantar surgical neurectomy upon failure of conserva-
tive treatments
c) Increased heel height to improve forefoot circulation and space
redistribution
d) Local anesthetics and corticosteroid injections to reduce pain and
inflammation
e) Metatarsal pad with ice treatment to reduce neural irritation
184. A 28-year-old man in a motor vehicle accident sus-
tains a hyper dorsiflexion injury to his foot. A radiograph
reveals a Hawkins type II talar neck fracture with dislocation

265
of the subtalar joint. Open reduction with internal fixation is
performed, and the patient returns for follow-up radiographs
8 weeks later. A radiograph shows well-placed hardware and
a radiolucent band in the dome of the talus (Hawkins sign).
Based on these findings, which of the following is LEAST
likely to occur in this patient?
a) Ankle stiffness
b) Avascular necrosis (AVN) of the talus
c) Subtalar and talonavicular arthritis
(D) Talus nonunion or delayed union
e) Varus malunion
185. Which of the following statements regarding appro-
priate therapy for tarsal tunnel syndrome (TTS) is correct?
a) There is no role for the use of local tarsal canal and anti-inflam-
matory agent injections in the symptomatic treatment of TTS
b) If the bifurcation of the tibial nerve beneath the lacinate ligament
is enlarged, surgically cutting the ligament provides space for nerve
expansion
c) It is important to preserve the fibrous tethers of tibial nerve
branches to the fascia of the abductor hallucis muscle during surgical
release of the tarsal tunnel
d) Resection of the flexor retinaculum is associated with poor surgi-
cal outcome
e) Wide-cushioned shoes and arch supports have not been shown
to be effective for relieving compression
186. A 63-year-old man lands awkwardly while jumping
and hears a loud pop in his left ankle. He cannot bear weight
on the ankle, and edema ensues soon after the injury. Which
of the following fractures would be missed if only radio-
graphs of the ankle are taken without knee examination or
radiographs of the entire lower leg?
a) Calcaneal fracture b) Maisonneuve fracture

266
c) Pilon fracture d) Pott fracture
e) Talar neck fracture
187- A 62yrs old man with type diabetes presents with
swelling in his Lt ankle, effusion but minimal pain. X-rays;
sever osteopenia, bone destruction and loose bodies. The
managements of this man include all BUT one of the follow-
ing;
a. Resting and splinting b. Aspiration
c. Total ankle replacement d. Akle arthrodesis
e. Arthroscopic lavage
188- A 16yrs old post Polio patient there is grade IV pow-
er of tibials anterior, grade III in his pernoeal muscles and
grade II in his gastrocnemius and soleus muscles the most
likely deformity is:
a. Talipes equino varus b. Talipes equino vulgus
c. Pes cavus d. Calcaneovalgus e. Equinus only
189- 1-A 27-year-old chef sustains a traumatic injury to
the index finger in an electric mixing machine. Examination
reveals an isolated 2cm x 2cm loss of palmar skin over the
proximal phalanx with exposure of the flexor tendon. Cover-
age of this defect is best accomplished with;
a. an axial flag flap from the long finger
b. split-thickness skin grafting
c. full-thickness skin grafting
d. a moberg advancement flap
e. a thenar flap
190- A 40-year-old woman has a chronic boutonniere de-
formity of the proximal interphalangeal (PIP) joint of her
middle finger with a preserved joint space. She lacks 45 de-
grees of active extension but has full passive extension of
the PIP joint. Treatment should consist of;

267
a- central slip tenotomy b- volar plate release
c- lateral band relocation d- arthrodesis of the PIP joint
e- arthroplasty of the PIP joint
191- A 28 year old woman who is recently postpartum has
radial sided wrist pain. The treatment that will most likely re-
solve her symptoms is injection of corticosteroids in which
of the following locations?
a. Intersection of the EPB and radial wrist extensors
b. 1st extensor compartment
c. 1st carpometacarpal joint
d. Scaphotrapezial joint
e. Superficial radial nerve as it exits from beneath the brachioradia-
lis
192- A 52yo F sustained a nondisplaced distal radius
fracture 6mo ago and was treated w/ short arm casting. She
now reports acute inability to extend her thumb. What is the
treatment of choice?
a. observation and reassurance
b. primary repair of EPB
c. primary repair of EPL
d. transfer of brachioradialis to the EPB
e. transfer of the EIP to the EPL
193- A 45 year old laborer sustains an extra-articular frac-
ture of the distal radius with 30 degrees of dorsal tilt and
some dorsal comminution. Closed reduction and splinting
using finger traps and local anesthesia yields an anatomic
reduction with normal volar tilt and radial length. At 1 week
follow up, dorsal tilt is now neutral (0 degrees). At 2 weeks
follow up, dorsal tilt is now at 5 degrees with some loss of
radial length. Management should consist of

268
a. continued observation in the current splint
b. repeat closed reduction and recasting
c. conversion to a removable splint and range of motion exercises
d. surgical stabilization in situ with external fixation
e. surgical reduction and stabilization with internal or external fixa-
tion
194- A 32 year-old woman fell onto her outstretched hand
4 weeks ago. Initial radiographs revealed no evidence of
fracture; therefore, management consisted of a wrist splint.
She now reports increased swelling of the entire hand, in-
tense burning pain, increased sensitivity to touch and a
mottled skin color appearance. Repeat radiographs are nor-
mal. What is the most likely diagnosis?
a – compartment syndrome of the forearm
b – carpal tunnel syndrome
c – complex regional pain syndrome – type I
d – allergic reaction to the splint material
e – rupture of the extensor pollicis longus tendon
195- A 35-year-old man has pain, limited motion, and
swelling of his middle finger after sustaining a volar punc-
ture wound 3 days ago. Examination reveals a flexed resting
posture, pain with passive stretching, and tenderness along
the flexor tendon sheath. Appropriate management should
consist of
a. outpatient management with oral antibiotics.
b. IV antibiotics alone.
c. surgical drainage over the entire length of the finger.
d. surgical drainage via small incisions and intraoperative sheath ir-
rigation with a catheter.
e. needle aspiration of the flexor tendon sheath with continuous ir-

269
rigation and oral antibiotics.
196- A 35-year-old meat cutter sustains a fingertip injury
to his ring finger in a meat slicing machine. Examination
reveals an oblique wound beginning at the distal interpha-
langeal joint flexion crease with loss of volar tissue only.
There is no exposed bone or tendon. Primary coverage is
best achieved with a
a- V-Y advancement flap b- thenar flap
c- axial flag flap d- cross-finger flap
e- split tickness skin graft
197- A 45-year-old man with a history of rheumatoid ar-
thritis reports that he is unable to extend his ring finger.
Examination reveals that with active extension, he has a 60
degree extensor lag at the MP joint, but when the finger is
passively extended he can maintain it in a fully extended
position. Treatment should consist of
a – sagittal band reconstruction
b – exploration of the posterior interosseous nerve
c – flexor tenosynovectomy and resection of one slip of the flexor
digitorum sublimis tendon
d – extensor tenodesis to the EDC tendon of the middle finger
e – crossed intrinsic transfer
198- A 24-year-old golf player has persistent, deep hypo-
thenar palm pain after falling with his arm extended behind
him. Plain radiographs and special radiographic views fail
to show any abnormalities. What study will provide the most
cost-effective diagnosis?
a- Bone scan b- Ultrasound c- CT
d- Electrodiagnostic studies of the median and ulnar nerves
e- MRI of the wrist

270
199- A 41 yo male laborer reports the insidious onset of
weakness in his dominant right arm. He cannot recall a spe-
cific traumatic event. He notes that he can no longer extend
his fingers or thumb. He can extend his wrist in a radial di-
rection, but it is weak. Biceps and triceps strength is normal.
Electromyography after 3 monts of observation reveals no
reinnervation potentials. What is the next step in manage-
ment?
a. Continued observation
b. Decompression of the radial tunnel
c. Transfers of the flexor carpi radialis to the finger extensors and
the Palmaris longus to the extensor pollicis longus
d. Four-tailed tendon transfer of the extensor carpi radailis longus
to the extensor hoods f all four fingers.
e. Oberlin nerve transfer using ulnar nerve motor fascicles
200- A 38-year-old construction worker with no history of
trauma has had a painful swelling in the hypothenar emi-
nence of his dominant hand for the past 4 weeks. He also re-
ports numbness in the ulnar two digits and cold intolerance.
Which of the following studies is most useful for diagnosis?
a- CT scan b- Bone scan
c- Arteriogram
d- Doppler ultrasound e- Electrodiagnostic study

271
272
ANSWERS:

1-d 2-b 3 4-d 5-c 6-c 7-e 8-d 9-b 10-c 11-b
12-d 13-d 14-c 15-b 15-a 16-d 17-a 18-b 19-e
20-c 21-a 22-c 23-c 24-c 25-c 26-b 27-b 28-d
29-b 30-d 31-c 32-b 33-b 34-b 35-d 36-b 37-c
38-b 39-b 40-b 41-c 42-a 43-b 44-a 45-c 46-b
47-a 48-b 49-b 50-b
51-b 52-d 53-d 54-a 55-c 56-c 57-b 58-a 59-d
60-d 61-e 62-b 63-b 64-b 65-a 66-e 67-d 68-c 69-
c 70-c 71-e 72-d 73-b 74-c 75-c 76-e 77-b 78-d
79-e 80-d 81-d 82-d 83-b 84-d 85-a 86-a 87-e
88-e 98-c 90-e 92-c 93-d 94-e 95-c 96-c 97-d
98-b 99-e 100-e
101-c 102-a 103-c 104-a 105-d 106-d 107-d 108-d
109-d 110-e 111-e 112-b 113-e 114-d 115-c 116-b
117-e 118-d 119-b 120-e 121-a 122-a 123-e 124-d
125-d 126-c 127-b 128-a 129-e 130-d 131-e 132-d
133-e 134-c 135-c 136-e 137-a 138-c 139-b 140-d
141-a 142-a 143-b 144-b 145-e 146-e 147-d 148-c
149-a 150-c
151-d 152-a 153-c 1 54-c 155-e 156-a 157-e 158-a
159-a 160-b 161-a 162-d 163-d 164-d 165-b 166-a
167-c 168-c 169-b 170-e 171-d 172-e 173-c 174-a
175-d 176-e 177-e 178-e 179-b 180-a 181-a 182- b
183-c 184-b 185-b 186-b 187-c 188-d 189-a 190-c

273
274
Enhanced Or extended matching
questions MCQs ( EMCQs)

Increased number of options to chose from 7-10 to in-


crease search space
Allowing for the choice of two or more options , without
specifying how many required for any given question al-
location of full marks to combination of two or more options
and different options
Each set of three EMCQs will deal with an orthopaedic
topic , each question has the same instruction.
1-surgical approaches to pelvis?
a-sartorius and tensor fascia lata
b-internal oblique
c-rectus femoris and Sartorius
d-gluteus maximus and gluteus medius
e-posterior division of obturator nerve
f-inguinal ligament
g-inferior gluteal
h-adductor brevis and adductor magnus
i-rectus abdominis
From the list of options above, choose the most appropriate option
for each of the following statements. Each option may be used once,
more than once or not at all.
1-these muscles are elevated in the kocher-langenbeck approach
to the acetabulum.

275
2-in the anterior approach to the pubic symphysis these muscles
are divided but not denervated.
3- This structure may be damaged if the anterior superior iliac spine
is taken during bone graft harvesting.

2-pelvic osteotomies
a-granz b-shantz c-pemberton
d-innominate/triple e-salter f-steel
g-chiari h-shelf
From the list above, the most appropriate option for each of the
following statement. each option may be used once, more than once
or not at all.
1-relies on formation of fibrocartilage under the new acetabular
roof.
2-layers of bone graft used to build up deficient lateral acetabulum.
3-osteotomies close to the acetabulum to redirect the orientation
of the acetabulum.
3-Special test during knee examination?
a-lachman test b-anterior drawer test
c-pivot shift test d-clark’s test
e-j sign f-apprehension test
g-mcmurray’s test h-apley’s test
i-dial test j-quadriceps active test
Which of the options above is best described in each of the follow-
ing statement? each option may e used once, more than once or not
at all.
1-this test would be positive in an isolated posterior cruciate liga-

276
ment injury without injury to the posterolateral corner.
2-this is the most sensitive test for anterior cruciate ligament defi-
ciency.
3-this test is reliant on an intact medial complex and an intact ilio-
tibial band.

4-rediological features of the knee?


a- patella baja b-Segond sign
c-Pellegrino-stieda sign d-patella alta
e- Blumensaat’s line f-Insall and Salvati method
g-Blackburn and peel method
h- squared-off lateral femoral condyle
i-appears on three sagittal MRI images
j- Double posterior cruciate ligament (PCL) sign on MRI
Which of the options above is best described in each of the following
statement?each option may be used once,more than once, or not at all
1- a radiographic feature of a discoid lateral meniscus.
2-small radio-opacity adjacent to the medial femoral condyle
3-the ratio between the perpendicular distance from the lower ar-
ticular margins pf patella to the tibial plateau and the length of articu-
lar surface of patella
5-nerves in the foot
a-common personal nerve b-deep peroneal nerve
c-superficial peroneal nerve d-tibial nerve
e-lateral plantar nerve f-medial plantar nerve
g-medial calcaneal nerve h-saphenous nerve
i-sural nerve
which of the option above is best described in each of the following

277
statement each option may be used once, more than once or not at all
1-this nerve travels between flexor digitorum brevis and abductor
hallucis
2-this nerve would need to be intact for a successful tendon trans-
fer for a foot drop
3-entrapment of this nerve’s first branch is a cause of medial heel pain
6-surgical procedures in the foot and ankle
a-modified brostrom b-scarf c-jones
d-girdlestone-taylor
e-watson-jones f-lapidus g-evans
h-chrisman-snook i-modified mcbride
j-keller
k-mitchell’s l-akin
Which of the options above is best described in each of the following
statement? each option may be used once,more than once or not at all;
1-a 28-year old lady presents with a bunion.clinically she has a hy-
permoble first ray ,and radiographs reveal an intermetarsal angle of 22
degree
2-a 31-year old man presents with ankle instability and varus stress
radigraphs of the ankle show a varus tilt
3-A 54-year old lady present with a second toe deformity. there is
excessive flexion at the proximal interphalangeal joint, but this cor-
rects passively
7-osteochondroses;
a- Freiberg’s disease b- Kienbock’s disease
c-Kohler’s disease
d- Sinding-larsen-johansson syndrome
e-Van neck’s disease f-Sever’s disease
g- Thiemann’s disease h- Panner’s disease

278
Which of the options above is best described in each of the follow-
ing statement? each option may be used once ,more than once or not
at all;
1-a 9-year old girl presents with posterolateral elbow pain and click-
ing
2-a 12-year old boy presents with a limp after a fall from height.ex-
amination reveals tenderness on the planter aspect of the calcaneum
3-a 5 year old boy presents with pain on walking.examination re-
veals tenderness on the medial aspect of his foot
8-inherited conditions;
a- fibroblast growth factor receptor 3 (FGFR3)
b-x-linked recessive c-PEX
d- Type I collagen e- fibrillin
g-core binding factor alpha-1 (CBFA1)
h- Type II collagen
i=cartilage oligomeric matrix protein (COMP)
j- Type x collagen
which of the options above is best described in each of the follow-
ing statement? each option may be used once, more than once or not
at all;
1-associated with an x-linked dominant condition
2-associated with a condition characterized by absent clavicles
3-associated with a conditon presenting with gower’s sign
9-paediatric fractures;
a-salter-harris I b-greenstick c-salter-harris II
d-torus e-salter-harris III f-avlusion
g-salter-harris IV h-non-accidental injury
i-salter-harris VI j-salter harris-V

279
Which of the options above is best described in each of the follow-
ing statement? each option may be used once, more than once or not
at all
1-injury to the perichondral ring
2-atype of acetabular injury with the worst outcome
3-supination-inversion injury to the medial malleolus
10-cervical spine injuries;
a- type I odontoid fracture b-pseudosuluxation
c-bilateral facet dislocation
d- -c2 rotatory subluxation e-hangman’s fracture
f-unilateral facet dislocation
j- type II odontoid fracture h-jefferson’s fracture
i-type III odontoid fracture
Which of the options above is best described in each of the following
statement? each option may be used once, more than once or not at all
1-a 19-year-old presents with a very painful neck following an axial
compression injury
2-a 7-year old boy falls down some stairs. he has mild pain. his lat-
eral cervical spine x-ray shows 4mm translation of c2 on c3
3-an 18-year old falls down some stairs. he has mild pain. his lateral
cervical spine x-ray shows a ‘bow tie’ sign
11-consedering spinal cord injuries;
a- brown-sequard syndrome b-central cord syndrome
c- Complete transection of the cord
d-conus medullaris syndrome
e- Anterior cord syndrome f-tabes dorsalis
g- Tethered cord syndrome h-posterior cord syndrome
i- None of the above

280
Which of the options above is best described in each of the follow-
ing statement? each option may be used once, more than once or not
at all
1- Only 10-15% of patients demonstrate functional recovery follow-
ing this
2-this injury is associated with the best prognosis
3-this produces contralateral muscle paralysis and ipsilateral hyper-
aesthesia to pain and temperature
12-corrosion
a-passivation b-uniform attack c-galvanic
d-crevice e-fretting f-pitting
g-intergranular h-inclusion
i-fatigue
Which of the options above is best described in each of the follow-
ing statement? each option may be used once, more than once or not
at all
1- this is when two dissimilar metals become electrically coupled
2-this describes combination of wear and crevice corrosion
3-this process is characterized by an area of oxygen depletion pre-
venting passivation
13-metabolic bone disease;
a-primary hyperparathyroidism b-hyperthyroidism
c-malignancy with boe metastases
d-pseudohypoparathyroidism
e-vitamin d deficiency f-renal osteodystrophy
g-paget’s disease h-hypophosphotasia
i-hypophosphataemic rickets
Which of the options above is best described in each of the follow-

281
ing statement? each option may be used once, more than once or not
at all
1-an autosomal recessive disorder with increased urinary phospho-
ethanolamine
2-the test results in this include a raised alkaline phosphatase and
urine hydroxyproline
3-the test results in this include a markedly raised serum phospho-
rus and parathyroid hormone with radiographs revealing a ‘rugger-
jersy’ spine
14-primary tumours;
a-osteoid osteoma b-osteoblastoma
c-aneurysmal bone cyst d-haemangioma e-eosinophilic
granuloma f-giant cell tumour g-plamacytoma
Which of the options above is best described in each of the follow-
ing statement? each option may be used once, more than once or not
at all
1-an 8-year-old boy presents with mid-thoracic back pain.radio-
graphs of his spine showed a vertebra plana at T10
2-a 40-year-old man sustained a wege fracture of L1 follwing a fall.
jailhous striations of the vertebra were also seen
3-a high recurrence rate is reported following surgical excision of
this tumour
15-spinal cord injury;
a-C4 b-C5 c-C6 d-C7 e-C8 f-T1 g-T2-T12
Which of the functional levels above is best described in each of the
following statement? each option may be used once, more than once
or not at all
1-a 20-year-old male sustained a fracture dislocation of the cervical
spine. he can eat and dress up but is unable to actively straighten his
arm

282
2-pateints at this level can cut meat but are unable to grasp
3-patients at this level can transfer independently. manual wheel-
chairs and the use of flexor hinge wrist-hand orthosis can be operated
at this level
16-structures around the shoulder;
a-glenoid labrum
b-superior glenohumeral ligament (SGHL)
c-middle glenohumeral ligament (MGHL)
d-anterir band of the inferior glenohumeral ligament (IGHL)
e-posterior band of the inferior glenohumeral ligament(IGHL)
f-coracohumeral ligament(CHL) g-infraspinatus h-
subscapularis
Which of the options above is best described in each of the following
statement? each option may be used once, more than once or not at all
1-this is highly variable and poorly deined in up to 40% of the popu-
lation
2-this is the primary restraint to anterior/inferior translation of the
head with the shoulder abducted to 90° and in maximum external ro-
tation
3-with the shoulder in external rotation this is an important dynam-
ic stabilizer to posterior subluxation
17-treatment of shoulder injuries;
a- collar and cuff b-manipulation under anesthesia( MUA)
c- MUA and percutaneous K-wire fixation
d-oped reduction and internal fixation (ORIF)
e-ORIF and bone grafting f-hemiarthroplasty
g- shoulder resurfacing
h-unconstrained total shoulder replacement

283
i- reverse total shoulder replacement
Which of the options above is best described in each of the following
statement? each option may be used once, more than once or not at all
1-two-part proximal humerus fractuer with 6mm displacement of
the greater tuberosity fragment
2-four-part head splitting proximal humerus fracture in a 55-year-
old
3-four-part vagus impacted proximal humerus fracture in a 45-year-
old
18-wound management;
a-cross finger flap b-heterodigital island flap
c-terminalization d-haematoma evacuation
e-nail bed repair and splint f-V-Y plasty (advancement)
g-distal replant h-full thickness skin grafting
Which of the options above is best described in each of the follow-
ing statement? each option may be used once, more than once or not
at all
1-a 26-year-old chef cuts the tip of his finger with a sharp knife.
there is no bone exposed, the wound is volar favourable and >1cm
2-a 42-year-old housewife minced the pulp of her index finger and
there is a volar unfavourable wound with exposed bone >1cm
3-a gardener put his hand under the blade of a lawnmower. there
is very little bone in the distal phalanx with an exposed ragged flexor
digitorum profundus (FDP) tendon and no nail visible
19-hand deformities;
a- intrinsic plus hand b-boutonniere deformity
c-pseudo-boutonniere deformity d- swan neck deformity
e-intrinsic minus hand f-quadrigia effect
g- lumbrical plus finger h-caput ulnae wrist

284
Which of the options above is best described in each of the following
statement? each option may be used once, more than once or not at all
1-on metacarpophalangeal joint (MCPJ) extention there is less
flexion of the proximal interphalangeal joint (PIPJ) than with passive
flexion of the MCPJ
2-tendons adjacent to an injured finger (ususally flexor tendon)
don’t flex owing to a shared muscle belly
3-central slip ruptures leads to this condition
20-planter layers of the foot;
a-first layer b-between first and second c-second layer
d-between second and third e-third layer
f-between third and fourth g- forth layer
Which of the options above is best described in each of the follow-
ing statement? each option may be used once,more than once or not
at all
1-amuscle which is not supplied by the tibial or its branches
2-agroup of muscles supplied by both the medial and lateral plantar
nerves
3-the muscles related to the knot of henry
21-injuries around the hip;
a- stress fracture of femoral neck b-adductor strain
c-iliotibial band syndrome
d- labral tear e-hip dislocation
f-hamstring avulsion
g- meralgia paraesthetica h-greator trochanteric bursitis
i-avascular necrosis
Which of the options above is best described in each of the follow-
ing scenarios ? each option may be used once,more than once or not
at all

285
1-a 25-yaer-old male presents with groin pain and clicking with no
history of trauma
2-a 19-year-old sprinter collapsed during a race and has diffuse but-
tock pain
3-a 22-year-old female long distance runner describes her hip as ‘re-
peatedlly dislocating’ with lateral pain around the greater trochanter
22-anticoagulants;
a-unfractioned heparin b-asprin c-warfarin
d-low-molecular-weight heparin e-rivaroxaban
f-dabigatran g-dextran
Which of the options above is best described in each of the follow-
ing statement? each option may be used once, more than once or not
at all
1-the dose of this drug may need to be increased when adminstered
concomitantly with rifampicin
2-orally active,direct factor xa inhibitor
3-thrombin inhibitor
23-bone tumours and tuour like conditions;
a-osteoid osteoma b-ewing’s sarcoma
c-rhabdomyosarcoma d-leukaemia
e-osteosarcoma f-fibrous dysplasia
g-non-ossifying fibroma h-adamantinoma
i-neuroblastoma j-langerhans’s cell histocytosis
Which of the options above is best described in each of the follow-
ing statement? each option may be used once, more than once or not
at all
1-a condition associated with genetic translocation t (11:22)
2-when associated with soft tissue tumours is termed Mazabraud’s
syndrome

286
3-has a lesion typically <2cm diameter
24-hip disorders;
a-avascular necrosis (AVN) b-septic arthritis
c-slipped upper femoral epiphysis(SUFE)
d-femoroacetabular impingment
e-developmental dysplasia of the hip(DDH)
f-snapping fascia lata g-perthes disease
h-coxa vara
i-epiphysiolysis j-transient synovitis
which of the options above is best described in each of the follow-
ing statement? each option may be used once, more than once or not
at all
1-may be associated with positive ober test
2-a condition where damage occurs at the chondrolabral junction
3-may be diagnosed with trethowan’s sign
25-classification of fractures
a-Mayfield b-Tile c-Garden
d-Young-burgess e-Damschen f-Essex-lopresti
g-Mason h-AO i-Bado
j-Frykman k-Gustillo
Which of the options above is best described in each of the follow-
ing statement? Each option may be used once, more than once or not
at all
1-a force-vector mechanistic classification of pelvic injuries
2-a proposed classification for scapula-thoracic dissociation
3-a classification of radial head fractures

287
26-genetic defects
a-achondroplasia b-Duchenne’s muscular dystrophy
c-osteogenesis imperfecta d- Marfan’s
e-multiple hereditory exostoses f-pseudoachondroplasia
g-multiple epiphyseal dysplasia h-hypophosphataemic rickets
i-diastrophic dysplasia
Which of the options above is best described in each of the follow-
ing statement? Each option may be used once, more than once or not
at all
1-a condition associated with a defect in fibrillin
2-a condition associated with a defect in the EXT1/EXT2 genes
3-a condition associated with a defect in dystrophin
27- INSERTION OF TENDONS;
a-Lateral malluollus b-medial malleolus c-talus
d- calcaneum e-cuboid f-navicular
g- lateral cuneiform h-intermediate cuneiform
i-first metatarsal j- fifth metatarsal
Which of the options above is best described in each of the follow-
ing statement? each option may be used once, more than once or not
at all
1 – The peroneus brevis
2 – The peroneus longus
3 – Tibialis anterior
28-paediatric elbow;
a-capitellum b-medial epicondyle c-radial head
d-lateral epicondyle e-lateral condyle f-medial condyle
g-trochlea h-coronoid i-olecranon

288
Which of the options above is best described in each of the follow-
ing statement? each option may be used once, more than once or not
at all
1-non-union of which can result in tardy unlar palsy
2-can be affected by osteochondritis dissecans
3-ossifies at around the age of 8 years
29-surgical options;
a-wrist fusion b-lunotriquetral fusion
c-ulnar shortening d-radial shortening
e-proximal row carpectomy f-lunate excision
g-capitate lengthening
Which of the options above is best described in each of the follow-
ing statement? each option may be used once, more than once or not
at all
1-a patient with dorsal wrist pain and an unlar minus variance with
lichtman stage 3A disease
2-a patient with dorsal wrist pain and an ulnar minus variance with
lichtman stage 3B disease
3- a patient with dorsal wrist pain and an ulnar minus variance with
lichtman stage 4 disease
30-choice of knee replacement design
a-medial unicompatmental b-lateral unicompartmental
c-patellofemoral d-total,posterior cruciate retaining
e-total,posterior cruciate substituting
f-total .constrained non-hinged
g-total,constrained hinged
Which of the options above is best described in each of the follow-
ing statement? each option may be used once, more than once or not
at all

289
1-a 69-year-old lady with rheumatoid arthritis and a painful knee
with radigraphs showing diminished joint space in both lateral and me-
dial compartments and neutral alignment
2- a 67-year-old man with a painful knee and tricompartmental
osteoarthritis on radiographs. Thirty years ago he sustained a signifi-
cant sporting injury when he hyperflexed his knee with his foot plan-
tarflexed but did not receive any specific treatment
3- a 61-year-old man with medial knee pain and slight varus de-
formity. radiographs show reduced joint space in the medial compart-
ment and a valgus stress radiographs show reduced joint space in the
lateral compartment
31-the popliteal fossa
a-popliteal artery b-popliteal vein c-semitendinous
d-tibial nerve e-semimembranosus
f-common peroneal nerve g-biceps femoris
h-popliteal fascia i-medial head of the gastrocnemius
j-femoral artery k-lateral head of the gastrocnemius
Which of the above is best described in each of the following state-
ment. each option may be used once, more than once or not at all
1-this forms the superolateral boundary
2-this forms part of the roof
3-this neurovascular structure is anteriormost
32-a patient undergoes hip arthrodesis
a-0-5% b-20-30% c-50-60% d-40-50% e-90-100%
From the list of options above,choose the most appropriate option
for each of the following 
Scenarios. each option may be used once,more than once or not at
all
1-the proportion of patients that can be expected to experience sig-
nificant back pain

290
2-the proportion of patient that can be expected to experience sig-
nificant ipsilateral knee pain
3-the proportion of patients that can be expected to experience sig-
nificant contralateral hip pain
33-synovial pathology;
a-pigmented villonodular synovitis 
b-synovial chondromatosis
c-juvenile rhumatoid arthritis
d- haemophilia 
e-lipoma arborescens F-gout g- pseudogout
Which of the above is best described in each of the following state-
ment.each option may be used once,more than once or not at all
1-this results in a haemosiderotic synovitis
2-this results in a synovium which has a bright yellow, nodular, pap-
illary appearance
34-painful foot in an adult
a-halux rigidus b-mallet toe
c-tibialis posterior tendon insufficiency
d-achilles tendonosis  e-planter fasciitis
f-flexor hallucis longus impingement g-gout
h-morton’s neuroma i- midfoot osteoarthritis
j- haglund’s deformity k-ankle osteoarthritis
l-talar osteochondral lesion
Which of the above is best described in each of the following state-
ment.each option may be used once,more than once or not at all
1-a 23-year-old professional dancer complains of pain in her foot
particularly when en pointe 
2-a 21-year old complains of pain at the back of his foot after exer-

291
cise and has a bump on the back of his heel
3-a 41-year old man complains of heel pain which is worse in the
morning better during the day and bad again in the evenings
35-examination of the lower limb;
a-Thomas’ b-Faber  c-Ely’s
d-Barlow’s e-Ort’olani’s  f-Galleazi’s
g-Trendelenburg h- fader’s
Which of the above is best described in each of the following state-
ment. each option may be used once, more than once or not at all
1-this test detects a tight rectus femurs by flexing the knee while
prone
2-this test involves attempting to reduce a dislocated hip
3-a positive test suggests either developmental dysplasia of the hip
or a leg length discrepancy
36-head injury;
a-concussion b-cerebral haemorrhage
c-epidural haemorrhage
d-subdural haemorrhage e-subarachnoid haemorrhage 
f-cerebral contusion g-diffuse axonal injury
h-depressed skull fracture i-basilar skull fracture
Which of the above is best described in each of the following state-
ment.each option may be used once, more than once or not at all
1-a 41-year old man sustains a head injury.clinical examination re-
veals battle’s sign
2-a 31-year old sustains a head injury and his scan reveals bleeding
from the middle meningeal artery 
3-a 31-year old sustains a head injury and his scan rebeals bleeding
from the bridging veins

292
37-Mechanism of action of antibiotic used in orthopae-
dics;
a-aminoglycosides b-penicillins c-cephalosporine
d-clondamycin e-imipenem/meropenem f- vancomycin
g-tetracyclin h-macrolides i-fluroquninolones
j-rifampicin k-trimethoprim l-linezolid
Which of the above is best described in each of the following state-
ment.each option may be used once, more than once or not at all
1-inhibits DNA dependent RNA ploymerase and can give red-orange
urine
2-blocks the attachment of aminoacyl-tRNA to the acceptor site on
the 30S ribosomal 
Subunit thus preventing prokaryotic protein synthesis
3-works as an antimetabolite by inhibiting dihydrofolate reductase
38-neuromuscular conditions 
a- cerebral palsy b-duchenne’s muscular dystrophy 
c-charcot-marie-tooth disease d-friedreich’s ataxia 
e-arthrogryposis f-myelodysolasia
g-spinal muscular atrophy h-dejerine-sottas disease 
i-poliomyelitis j-myaesthenia gravis
Which of the above is best described in each of the following state-
ment. each option may be used once, more than once or not at all
1-a contraindication to the use of botulinum toxin
2- associated with genetic defect of PMP22
3-a condition caused by mitochondrial dysfunction
39-infections of the hand;
a-Orf virus b-staphylococcus aureus
c-clostridia and group A β-streptococci

293
d-herpes simplex virus type I
e-candida albicans f-mycobacterium marinum
g-eikenella corrodens h-pasteurella multocida
Which of the above is best described in each of the following state-
ment. each option may be used once,more than once or not at all
1-a 26-year old presents to the emergency department with a small
laceration over his index metacarpal of his right hand . It is swollen
with pus draining. History relates to a punching incident 
2- a dentist presents with recurrent vesicles on the finger and pain
3-a 65-year old woman was bitten by a cat on the dorsum of the left
wrist and now can’t move the hand and has sever swelling and pain
40-examination of the foot and ankle;
a-Coleman block test b-Stlfverskiold test
c-grind test d-Simmond’s test
e-Thompson test f-anterior drawer
g-Mulder’s click h-single leg heel raise
i-talar tilt j- Homan’s sign k-the squeeze test
Which of the above is best described in each of the following state-
ment. each option may be used once, more than once or not at all
1-this test the integrity of the anterior talofibular ligament
2-this test the integrity of the ankle syndesomosis
3-this test is useful for assessing an equinus deformity
41-spinal tracts;
a-dorsal column b-lateral spinothalamic tract
c-anterior spinothalamic tract d-lateral corticospinal tract
e-ventral corticospinal tract f-dorsal spimocerebellar tract
g-ventral spinocerebellar tract

294
Which of the above is best described in each of the following state-
ment. each option may be used once, more than once or not at all
1-this tract conveys temperature sensation
2-this tract conveys proprioceptive information of the lower limbs
from muscle spindle
3-this tract is also known as the crossed pyramidal tract
42-hip bearing surfaces; 
a-cobalt-chrome socket
b-cobalt-chrome acetabular shell with ceramic liner
c-polyethylene socket
d-cobalt-chrome acetabular shell with polyethylene liner 
e-Stainless steel acetabular shell with polyethylene liner
F-cobalt-chrome head
g-stainless steel head h-titanium head
Which of the above is best described in each of the following state-
ment. each option may be used once, more than once or not at all
1-this component or combination has the highest wear rate when
used as a bearing surface
2-this component or combination is associated with ‘strip wear’
3-this component or combination has the lowest 10-year revision
rate when used in the acetabulum
43-apophyseal conditions and. Associated traction inju-
ries;
a-medial epicondyle  b-inferior patellar 
c-calcaneum d-ischial tuberosity
e-anterior superior iliac (ASIS) f-tibial tubercle
g-greater trochanter h-iliac crest 
i-lesser trochanter j-anterior inferior iliac spine (AIIS)

295
Which of the above is best described in each of the following state-
ment. each option may be used once, more than once or not at all
1-the affected area in Sinding-Larsen-Johansson syndrom
2-site of potential apophyseal avulsion during tennis serves
3-apophysis appearing age 5 years
44-compartments; 
A-1 B-2 C-3 D-4 E-5 F-6
G-7 H-8 i- 9 J-10 K-11 L-12
Which of the above is best described in each of the following state-
ment. each option may be used once, more than once or not at all
1-the number of compartments in the lower leg
2-the number of compartments in the hand
3-the number of compartments in the forearm
45-Evolution of cementing techniques;
a-porosity reduction using a vacuum b-antibiotic impregnated
c-tapered polished stem d-finger packing
e-cholophylladdition f-pulsatile lavage
g-collarless stem h-carbon dioxide jet preparation of femur
Which of the above is best described in each of the following state-
ment. each option may be used once, more than once or not at all
1-this is typically a third generation cementing technique
2-this is typically a second generation cementing technique
3-this is typically a first generation cementing technique
46-scoliosis treatment;
a- Boston brace b-Milwaukee brace
c-posterior instrumented fusion of the thoracic curve
d-anterior or posterior fusion of the lumbar curve
e-anterior release followed by posterior instrumentation of the tho-
racic curve

296
f-growth rods application
g-spinal fusion down to pelvis
h-posterior instrumented fusion of the thoracic and lumber curve 
i-hemivertebrectomy
Which of the above is best described in each of the following state-
ment. each option may be used once, more than once or not at all
1-a 15-year old girl presents with a right thoracic curve of 55° And a
lumber curve of 40°.on
Holding views the thoracic curve corrects to 33° And the lumber to
18° With no significant apical vertebral rotation
2-a 12 year old presents with a right low thoracic curve (apex at
T10) of 35°. She is still premenarchal
3-a 15 year old wheelchair user boy with cerebral palsy presents
with a 70° Lumber curve with increased pelvic obliquity
47- arthroscopic findings in the shoulder;
a- SLAP type I b-SLAP type II c-SLAP type III
d-SLAP type IV e-rotator cuff articular side tear
 f-biceps tendon fraying g-peel-back lesion
h-anterior labral periasteal sleeve avulsion (ALPSA) lesion
i-biceps tendon subluxation
Which of the above is best described in each of the following state-
ment. each option may be used once, more than once or not at all
1-the most common clinically significant type of SLAP lesion identi-
fied at arthroscopy
2-describes a bucket handle tear of the labrum with a longitudinal
tear extending into the biceps
3-identifying this pathology at arthroscopy should raise the suspi-
cion of a subscapularis tear

297
48-Upper limb trauma 
a-Galleazzi fracture  b-Monteggia fracture
c-Barton’s fracture d-Ronaldo fracture 
e-Bennett’s fracture f-Smith’s fracture
g-Colles fracture  h-Essex-Lopresti injury
i- Steiner’s lesion
Which of the above is best described in each of the following state-
ment. each option may be used once, more than once or not at all
1-this is an injury that is classified by Bado
2-this is a Y or T shaped intra-articular fracture described in1910
3-this involves dislocation of the distal radioulnar joint and fracture
of the radial head
49-The diabetic foot :
a-Eichenholtz stage 0 b-Eichenholtz stage 1
c-Eichenholtz stage 2 d-Eichenholtz stage 3
e-Wagner and Meggitt grade 0 f-Wagner and Meggitt grade 1
g-Wagner and Meggitt grade 2 h-Wagner and Meggitt grade 3
i- wagner and Meggitt grade 4 j-Wagner and Meggitt grade 5
Which of the above is best described in each of the following state-
ment. each option may be used once, more than once or not at all
1-a 53 year old diabetic lady presents with an ulcer on the sole of
her foot with a tendon visible
2-a 66 year old diabetic man presents with an abnormally shaped
foot and his radiographs suggest absorption of osseous debris and fu-
sion of larger fragments
3-a 55 year old diabetic man has a very prominent first metatarsal
head on the sole of his foot with haemosiderin deposits in the skin 

298
50-position of hip arthodesis;
a- 15° b-25-30° c-0-5° d-5-10° e-50° 
f-75° g-100° h-35°
Which of the above is best described in each of the following state-
ment. each option may be used once, more than once or not at all
1-the desired position of flexion
2-the desired position of adduction 
3-the desired position of internal/external rotation

299
ANSWERS:
1) 1-d 2-i 3-f 2) 1-g 2-h 3-a 3) 1-j 2-a 3-c 4) 1-h 2-c 3-g 5) 1-f
2-d 3-e 6) 1-f 2-a 3-d 7) 1-h 2-f 3-c
8) 1-c 2-g 3-b 9) 1-i 2-j 3-g 10) 1-h 2-b 3- f 11) 1-b 2-a 3-I 12)
1-c 2-e 3-d 13) 1-h 2-g 3-f
14) 1-e 2-d 3-f 15) 1-c 2-d 3-c 16) 1-c 2-d 3-h 17) 1-d 2-d 3-d
18) 1-f 2-a 3-c 19) 1-a 2-f 3-b
20) 1-g 2-c 3-c 21) 1-d 2-f 3-c 22) 1-c 2-e 3-f 23) 1-b 2-f 3-a
24) 1-f 2-d 3-c 25) 1-d 2-e 3-g
26) 1-d 2-e 3-b 27) 1-j 2-i 3-I 28) 1-e 2-a 3-g 29) 1-d 2-e 3-a
30) 1-e 2-e 3-d 31) 1-g 2-h 3-a
32) 1-c 2-d 3-b 33) 1-d 2-e 3-f 34) 1-f 2-j 3-e 35) 1-c 2-e 3-f
36) 1-i 2-c 3-d 37) 1-j 2-g 3-k
38) 1-j 2-c 3-d 39) 1-g 2-d 3-h 40) 1-f 2-k 3-b 41) 1-b 2-f 3-d
42) 1-h 2-b 3-c 43) 1-b 2-h 3-a
44) 1-d 2-j 3-c 45) 1-a 2-f 3-d 46) 1-c 2-a 3-g 47) 1-b 2-d 3-I
48) 1-b 2-d 3-h 49) 1-g 2-c 3-e 
50) 1-b. 2-c 3-c

300
BIOMATERIAL & BIOMECHANICS
MCQs
1) Which of the following biomaterials is most likely to
undergo pitting and crevice corrosion/
a . cobalt-cromium-molybdenum alloy b . stainless steel
c. titanium d. polyethylene
e. polymethylmethaacrylate
2) which of the following materials has the highest stiff-
ness?
a . Bone b. Tendon c. Titanium
d. Ceramics e. stainless steel
3) Which of the following terms describes materials that
are able to undergo a large amount of plastic deformation
prior to failure/
a . Brittle b. Viscoelastic c. Anisotropic
d. Isotropic e. Ductile
4) In relation to biomaterials, which of the following is a
definition of stress?
a . A measure of internal force b. A measure of deformation
c. A measure of stiffness d. A measure of energy
e. A measure of rigidity
5) On a stress-strain curve, which of the following is a
definition of the yield point?
a . The initial point at which the material begins to deform
b. The maximum strength achieved by material
c. The transition point from elastic to plastic deformation
d.The point where the material fractures
e. The point at which sicoelastic deformation ocuurs

301
6) Which of the following statements about Titanium is
NOT true?
a . Titanium has poor resistance to wear
b. Titanium particles may initiate a histocytic response
c. Titanium alloy generates less wear debris than cobalt-chrome al-
loy for THR
d. Titanium has a high yield strength
e. Titanium undergoes self-passivation to form adherent oxide coat-
ing
7) Which of the following statements relating to polyeth-
ylene is NOT true?
a . polyethylene displays viscoelastic properties
b. It displays thermoplastic properties
c. It generates free radicals in response to gamma irradiation
d. Its wear damage is most often the result of third-body inclusions
e. its wear debris has no effect on the longevity of THR
8) Which of the following statements relating to bone ce-
ment( Polymethylmethacrylate) is NOT true”
a Cement function as an adhesive between bone and implant
b. It has poor tensile strength
c. Polymerisation of cement is an exothermic reaction that occur in
three stages
d. Insertion of cement can lead to a significant drop in blood pres-
sure
e. Reduction of the number of pores in cement can increase tensile
strength
9) What is the reason for adding barium sulphate to bone
cement?
a . It acts as an accelerator b. It acts as an inhibitor

302
c. It acts as an colouring agent d. It acts as an radio-opacifier
e. It reduces cement wear debris
10) Which of the following properties do ceramic materi-
als such as aluminium oxide have?
a . Low compressive strength b. Low modulus elasticity
c. poor wear characteristic d.Lowfractureresistance
e. Low conductiveness of elasticity
11) Which of the following statements about the mechani-
cal properties of bone is NOT true?
a . Bone is strongest in compression
b. The mineral content of bone is the main determinant of stiffness
c. Ageing results in a decrease in the cortical diameter of bone
d. Implants may lead to osteoporosis of adjacent bone
e. Bone is anisotropic and viscoelastic
12) Which of the following statements about mechanical
properties of tendons is true?
a . Ruptured tendons show histological evidence of inflammation
b. Ruptured tendons show histological evidence of degeneration
c. Tendons are strongest in compression
d. They consist mainly of type II collagen
e. They can withstand up to a 25% strain prior to rupture
13) Which of the following statements is TRUE regarding
the use of metal plates for I.F.?
a . Plates are load-sharing devices
b. A neutralization plate applies static compression across the fracture
c. Plates are most effective when applied to the compression side
of the bone
d. Plate and screw removal after union does not weaken the bone

303
e. Doubling the thickness of a plate increases rigidity by eightfold
14) Which of the following statements is TRUE regarding
the use of intramedullary nails?
a . Doubling the radius of an IM nail increases rigidity by more than
eightfold
b . IM nail are load-bearing devices
c. IM nail resists bending forces better than rotational forces
d. Material properties do not influence the rigidity of IM nail
e. There is no difference in torsional stiffness between closed-sec-
tion and slotted nails
15) When using Ex. Fix, which of the following factors is
most important for stability?
a . Anatomical reduction of the fracture
b. Use of large diameter pins
c. Decrease bone –rod distance
d. Using rods and pins in different plans
e. A near-near and far-far pin configuration in relation to the frac-
ture site
16) Which of the following statements about THR is cor-
rect?
a . The femoral stem should be placed in slight varus
b. Increasing the femoral component offset reduces the abductor
moment arm
c. A smaller head size increases range of motion and stability
d. A large head size increases friction and volumetric polyethylene wear
e. The wear rate of ultra-high-molecular Wt. polyethylene in the
acetabulum is about 1 mm per year.
17) Which of the following measures does NOT help to
optimize patellar tracking during TKR?
a . Medial placement of the femoral component

304
b. Lateral placement of the tibial component
c. Medial placement of the patellar component
d. slight external rotation of femoral component
e. Avoiding malrotation of tibial component
18) Which of the following factors is most important in
determining the joint reaction force?
a . Joint congruence b. joint lubrication
c. joint contact area d. The muscle acting about the joint
e. Articular cartilage thickness
19) What is the optimal position for arthrodesis of the Hip?
a . 10 degrees of flexion and 25 degrees of abduction
b. 25 degrees flexion and Zero abduction
c. Zero flexion and 25 abduction
d. Zero flexion and Zero abduction
e. 10degrees extension and 25 degrees abduction
20) Which of the following statements about movements
of the knee joint is true?
a . A normal knee cannot achieve any recruvatum
b. The centre of rotation remains constant throughout the range of
motion
c. The femur externally rotates during the last 15degrees of exten-
sion
d. The patella helps to increase the power of extension
e. ACL controls femoral roll-back
21) Which of the following defines the mechanical axis of
the lower limb?
a . A line drawn from the centre of gravity to the ground
b. A line drawn from the centre of femoral head to the centre of the
knee

305
c. A line drawn from the centre of femoral head to the centre of
ankle
d. A line drawn along the shaft of femur and tibia
e. A line drawn along the centre of tibial plateau to the centre of
ankle
22) In relation to mechanical axis of lower limb, in which
of the following is the anatomical axis of the femur?
a . 3 degrees of varus b. 6 of varus c. Zero
d. 3 valgus e. 6 valgus
23) Which metatarsal bears the most Wt during gait?
a . First b. Second c. Third
d. Forth e. Fifth
24) What amount of shoulder abduction is achieved by
glenohumeral joint?
a . 30 degrees b. 60 degrees c. 90 degrees
d. 120 degrees e. 145 degrees
25) Regarding cervical spine, the injury centre of rotation
in a flexion-distraction fracture:
a) is located anterior to the centre of rotation of the spine
b) is located within the centre of rotation of the spine
c) is located posterior to the centre of rotation of the spine
d) may be located in all three of the above
e) None of the above
26) About force all are true except:
a. is a vector quality
b. it has a magnitude, line of action, and point of impact.
c. The unit of force is Newton (N)
d. 1 Newton (1N) is defined as the force that causes a 1-kg mass

306
e. A force may be compressive, tensile, bending, shear, or rotator
27) Deformation these statements all are true except;
a. If an axial force is applied to a ligament, it will stretch, an axial
force applied to a long bone, it will cause it to elongate or shorten.
b. Torsion applied to a ligament or bone causes twisting.
c. If a force applied perpendicular to the long axis of a long bone it
will cause bending.
d. A localized force applied to the surface of articular cartilage will
cause compression above the point of contact.
e. It is a change in the shape of a structure in response to force or
moment.
28) The amount of deformation depends on all are true
except:
a. Magnitude of the force or moment.
b. The shape of the material of the structure (area of contact).
c. Stiffness of the material of the structure.
d. Time of application of the force or moment
e. It is measured by Gibbes’s law
29) Forces acting on hip In the coronal plane, these state-
ments are true except:
a. Body wt applied to a lever arm extending from the body’s cen-
tre of gravity (axis) to the centre of the femoral head.
b. Abductors force applied to a lever arm extending from the lateral
aspect of greater trochanter to the centre of the femoral head.
c. During lifting from a chair running or jumping, the load may ap-
proach 10 times body wt.
d. the estimated load on the hip during the stance phase of the step
cycle is at least three times the body weight
e. the force of abductors must equal body wt to maintain pelvis

307
level when standing on one leg
30) In Total Hip Arthroplasty , these statements are true
except;
a. The forces acting on the stem in both coronal and saggital planes
produce torsional effect on the stem
b. fatigue fracture of the stem start on the antero-medial aspect of
stem and progress postero-medially
c. axial bending and torsional forces produce tensile effect on the
antero-lateral aspect and compressive on postero-medial aspect of
the stem.
d. Centralization of the head and lengthening abductor leaver arm
is to centralize the head
e. lengthening the abductor leaver arm by lateral reattachment of
the greater trochanter, in this way the moment produced by the body
wt is decrease
31) Stem failure all are true except;:
a. Usually occurs at the medial aspect i.e the area of maximum
tensile stress.
b. Stem design (curve) determines point of maximum tensile stress
(more desirable to be on surface of the stem).
c. Direction of load applied in both coronal and saggital planes (de-
termine torsional stress).
d. Varus/valgus orientation of the stem in the canal determine the
length of bending lever arm
Level of firm support by cement.
32) The diameter of the head & neck of femoral prosthesis
these statements are true except;
a. The diameter of the neck of small heads is thicker which may ap-
proach the diameter of the head to be strong enough.
b. Impingement of the neck on the cup during small arc of motion

308
especially if the cup is deep.
c. Neck impingement on cup transfers force to cup& stem causes
loosening tendency &dislocation.
d. Cups for larger heads are shallower; the depth of the cup is equal
to the radius of the head.
e. The smaller the head the chance of polyethyelene wear
33) About Tribology all are true except;
a. Deals with interfacial conditions of two contacting objects in rela-
tive motion to each other
b. Molecules in the lubricant that adhere chemically to the surfaces
increase friction and wear
c.. Friction force: is present wherever one object slides over another
d. Adhesive wear occurs when a local welding point breaks during
motion releasing a small fragment
e. Abrasive wear is caused by cutting of a hard surface in a softer
one OR when free particles are entrapped between the two surfaces
34) Factors which determine wear all are true except:
a. Coefficient of friction of the material.
b. Finishing of surfaces (polishing).
c. Boundary lubrication
d. Torque
e. Distance travelled in each cycle (depends on head diameter in
THR).
true except:
a. It produces a stimulus to maintain bone mass and prevents disuse
osteoporosis.
b. Decreased modulus of elasticity of the stem
c. It transfer stress to bone to increase bone quality.

309
d. If metallic cup is used with a plastic liner, stress is distributed
evenly.
e. Peak stress decreases if subchondral bone is removed.
36) About the principle of “Elastic Stability” these state-
ments are true except;
a. This elastic deformation creates a bending moment within the
long bone which will tend to angulate the fracture in the direction and
the plane of the concavity of the curved rod, as the rod wants to return
to its initial curved state.
b. The moment is counteracted by a second rod of matched diam-
eter and curve, which balances the first rod with an equal but opposite
moment.
c. The two intramedullary nails act complimentarily to stabilize the
fracture.
d. The biologic fixation is a rigid one that sufficiently stable against
angular, translational and torsion deforming forces and is associated
with early formation of exuberant callus.
e. No additional external immobilization is required
37) About the principle use of “Elastic Stability” these
statements are true except;
a. Elastic stable intramedullary nailing is ideally suited for mid-di-
aphyseal transverse,
b. Short oblique or short spiral fractures of the femur with minimal
comminution, in growing children.
c. The use of flexible nails can be extended to more proximal sub-
trochanteric fractures,
d. The use of flexible nails can be extended to Intercondylar frac-
tures
e. some multifragmentary fractures by modifying the technique
38) The working length of a bicortical screw is defined as
the;

310
a-diameter of bone traversed by the screw
b-number of threads in the screw
c-when tapping been used
d-self tapping
e-compressed near the fracture side
39) The advantage of using blocking screws during Intra-
medullary nailing of metaphyseal tibial fractures include all
of the following except
a. facilitate reduction b-maintain reduction
c-enhance construct stiffness d-neutralize translational forces
e-diminish the risk of nail failure
40) What design feature of cementless femoral stems lim-
its osteolysis of distal femure when used in THR?
a. Fills the diaphysis b-Fills the metaphysic
c-Collared d-Circumferentially coated
e-Distally fluted
41) What is the most significant disadvantage of ceramics
in orthopaedic implants?
a-Lower wear resistance b-Low elastic modulus
c-Low toughness d-Weak under compressive load
e-Poor wet ability
42) Which of the following features improves fluid film lu-
brication in a metal on metal THA?
a. Smaller diameter head, completely fit between the socket and
the head with sufficient roughness to allow for microseparation be-
tween head and socket
b. Smaller diameter head, slight clearance between socket and head
with no surface roughness

311
c. Larger diameter head, completely congruent fit between socket
and head with no surface roughness
d. Larger diameter head, slight clearance between socket and head
with minimal surface roughness
e. All of the above
43) What complication occurs more frequently with cur-
rent resurfacing arthroplasty compared with THA?
a. Osteolysis b-Infection
c-Sciatic nerve palsy periprosthetic fracture d-Dislocation
44) What type of nonunion is most likely to unite by in-
creasing biomechanical stability?
a. Atrophic b-Oligotrophic c-Hypertrophic
d-Infected e-Synovial pseudarthrosis
45) Which of the following devices provides the best pro-
phylactic stabilization of an impending pathologic fx caused
by a lytic defect in the mid-diaphysis of the femur?
a. plate and screws alone b- plate and screws with cement
c- Rush rod with cement d- Ender’s rods with cement
e- locked intramedullary nail
46) Which of the following is considered the most sen-
sitive method for measuring wear or migration in total hip
arthroplasty?
a- Acetabular teardrop technique
b- Plain radiographic overlay technique
c- Martell method (Hip Analysis Suite)
d- Radiostereometric analysis (RSA)
e- EBRA (Einzel-Bild-Roentgen-Analysis) single image radiographic
analysis

312
47) For upper extremity amputees, what is the advantage
of using a myoelectric prosthesis compared with a body-
controlled prosthesis?
a- Enhanced sensory feedback b- Less maintenance
c- Lighter weight d- Limited therapy time for training
e- Moderate or no harnessing
48) Which of the following osteoconductive bone graft
substitutes has the highest compressive strength?
a- Coralline hydroxyapatite b- Collagen-based matrix
c- Calcium phosphate d- Calcium sulfate
e- Tricalcium phosphate
49) What form of joint rehabilitation results in decreased
joint moments and joint contact forces?
a- Walking in water b- Low speed walking
c- Open chain strengthening d- Closed chain strengthening
e- Electrical stimulation during active motion
50) What type of amputee is the most ideal candidate for
a myoelectric prosthesis?
a- Transtibial b- Transfemoral
c- Transhumeral d- Transradial
e- Knee disarticulation
51) Which of the following statement is correct concern-
ing total joint replacement arthroplasty?
a. Non cemented prosthesis have a greater half life than cemented
ones
b. Total knee and hip prostheses have a life expectancy of approxi-
mately 10 years
c. The major failure of total joint arthroplasty is aseptic mechanical
loosening at the interface between the bone, cement, and implant

313
d. Biologic tissue ingrowth into a prosthesis worsens long-term re-
sults
e. Rigid fixation at the time of implantation is important to delay
tissue ingrowth
52) Which of the following statement is NOT true concern-
ing bone remodelling?
a. Remodelling occur by the same rate in all types of bone
b. Remodelling can occur not only on the surface of trabeculi
c. The remodelling process takes approximately 120 days in an adult
d. Trabecular bone remodelling occurs up to 10 times faster than
cortical bone remodelling
e. Bone remodelling involves bone formation without resorption
53) Which of the following statement is true concerning
types of bone found in the human body?
a. The Bone marrow is formed by bone cells
b. Trabecular and cortical bone differ in their chemical, molecular
and cellular components
c. Primary bone must be formed on existing surfaces
d. Woven bone reflects a highly organized microstructural organization
e. Primary osteonal bone is the primary constituent of adult cortices
54) The most important structural component of connec-
tive tissue is collagen. Which of the following statement is
NOT true concerning types of collagen?
a. Collagen is prime factor for tissue healing
b. All collagen is fiber forming
c. Type 1 collagen is the most abundant in the human body
d. Type 2 collagen is found in cartilage
e. The basement membrane collagens, type 4 and 5, do not form
regular fibers

314
55) Which of the following statement is true concerning
the biologic mechanisms of fracture repair?
a. Osteons causes union by callus formation
b. The mechanisms involved depend primarily on the stability of
the fracture
c. The last material formed by osteoblasts at the fracture site is
woven bone
d. Callus increases the cross-sectional area of the injury therefore
weakening the structure
e. Woven bone provides a permanent microstructure in the area
of a fracture
56) Continuous passive mobilization following flexor ten-
don repair of Zone II injuries produces:
a. delays the soft tissues healing
b. Increased total arc of digital range of motion.
c. Enhance the incidence of poor results.
d. Increased incidence of postoperative tendon rupture.
e. Increased incidence of infection.
57- A joint is innervated by the articular branches of the
nerves which supply the muscles which move the joint. This
law is known as:
a. Hilton’s law b. Andry’s law
c. Wolff’s law d. Cushing’s law
e. McBride’s law
58- Which is not a deep heat therapy?
a. Short wave diathermy b. Ultrasound therapy
c. Infrared therapy d. Microwave therapy
e. ECSW

315
59- Bone Graft all are true Except;
a-osteoconductive, b- osteoinductive,
c- osteogenic properties d-Stem cells
e-Mineralized bone matrix
60- Platelet rich plasma (PRP) consists of plasma sample
all are true except;
a. one’s own blood b-blood bank transfusion
c. enriched with autologous platelet
d- It stimulate soft tissue healing
e- calcium chloride used to initiate platelet activation in the pre-
pared sample of PRP 
61- Brittle material all are true except:
a- that exhibits linear stress stain relationship up until the point of
failure
b- undergoes elastic deformation only,
c- with little to no plastic deformation
d- under goes plastic deformation
e- ceramics is a good example
62- Anisotropic materials
a- That possess different mechanical properties depending on the
direction of the applied load
b- That possess the same mechanical properties in all directions
c- Ligaments are an example
d- Bone is an example
e- Tendons are an example
63- Titanium has these advantages except;
a- very biocompatable

316
b- forms adherent oxide coating through self passivation
c- corrosion resistant
d- low modulus of elasticity makes it more similar to biologic ma-
terials as cortical bone
e- high resistance to wear (notch sensitivity)
64- Stainless Steel all are true except;
a- primarily iron-carbon alloy with lesser elements of chromium,
molybdenum
b- Contain high amount of manganese
c- fracture resistant
d- susceptible to corrosion
e- stress shielding of bone due to superior stiffness
65- Cobalt alloy all are true except;
a-The main components is cobalt and chromium
b-It is very strong
c-generates more metal debris than cobalt chrome
d-It is thermoplastic
e-better resistance to corrosion than stainless steel
66- Ultra-high-molecular-weight polyethylene all are true
except;
a-Tough b-Non-ductile
c-resilient d-resistant to wear
e-wear usually caused by third body inclusions
67- Forces at the Shoulder joint all are true except;
a-120° Abduction
b- Deltoid Extremity Weight (70% BW)
c- GH Joint EW (90% BW)

317
d- Rotator Cuff EW (85% BW)
e- Load Bearing (Approximately 1BW)
68- The shoulder is a complex of 5 joints, these state-
ments are true except;
a- Any disturbances at any of these joints are likely to interfere
with the smooth rhythm observed in movements of this complex.
b- The overall ratio of scapulothoracic to glenohumeral move-
ment of 1:2 is made possible by a clockwork mechanism that involves
movements at these 2 articulations
c- The coronal movements of the clavicle connect the shoulder
girdle to the torso.
d- Stability of the scapulothoracic, glenohumeral joints and acro-
mio clavicular joints rely not only on the intrinsic capsular ligaments,
bony architecture and musculature, but also on extrinsic ligaments
that need to be addressed in management of instability at these joints
e- Premature degeneration and tears of the rotator cuff tendons
pose special clinical problems by disturbance of the delicate balance
between the cuff and the deltoid muscle
69- The Q-angle is often measured when examining the
knee,
a- The Q-angle is formed from a line drawn from the ASIS to the cen-
ter of the patella and from the center of the patella to the tibial tubercle.
b- The Q-angle is measured by subtracting from 180 degrees.
c- A normal Q-angle in men is 14 degrees
d- A normal angle in women is 7 degrees.
e- An increase in Q-angle can mean a higher risk of patellar prob-
lems including patellar subluxation and patellar dislocation.
70- Actions that decrease joint reaction force include  excep;
a- increase in ratio of A/B (shift center of rotation medially)
b- moving the acetabular component as far medial, inferior, ante-

318
rior shifting body weight over affected hip this results in Trendelen-
burg gait
c- increasing offset of femoral component 
d- Short stem prosthesis
e- Increase shear across joint
71- The use of cane in contralateral hand will help in all
except;
a- Reduces abductor muscle pull
b- Decreases the moment arm between the center of gravity and
the femoral head
c- Carrying load in ipsilateral hand
d- Produces additional downward moment on same side of rota-
tional point
e- Increases the adductors load
72- Actions that increase joint reaction force include ex-
cept;
a- Valgus neck-shaft angulation
b- Decreases shear across joint
c- Lateralization of greater trochanter
d- Varus neck-shaft angulation
e- Short stem prosthesis
73- Patello-femoral joint force all are true except;
a- The patello-femoral joint reaction force, depending only upon
the quadriceps muscle force
b- the angle of knee flexion, is always smaller than the quadriceps
muscle force during level walking,
c- During stair walking the patello-femoral joint reaction force at-
taineda level of 3.3 body weight or seven times the value of the PFJR
force during level walking.

319
d- Comparison of the values for PFJR force during level and stair
walking explains why a patient with patello-femoral joint derange-
ment is more disabled for stair walking.
e- The straight leg raising exercise against the same resistance re-
sults in a PFJR force at only 0.5 body weight
74- Which of the following best describes the relationship
of the subtalar and transverse tarsal joints during the phas-
es of gait?
a- Eversion of the subtalar joint locks the transverse tarsal joint b.
b- Transverse tarsal (Chopart) joint axes are parallel during heel strike
c- The calcaneus is in inversion throughout stance phase
d- Tibialis anterior concentrically contracts during stance phase
e- During push-off the foot becomes flexible due to eversion of
the calcaneus
75- Which phase of gait is affected most in a patient with
quadriceps atrophy?
a. Terminal swing b. Preswing c. Initial swing
d. Midstance e. Midswing
76- Which of the following descriptions of muscle activity
during normal gait is correct?
a. Gastrocnemius-soleus contracts eccentrically during heel
strike  24% (345/1435)
b. Gastrocnemius-soleus contracts concentrically during heel strike
c. Gastrocnemius-soleus contracts concentrically during swing
phase
d. Tibialis anterior contracts concentrically during toe-off
e. Tibialis anterior contracts eccentrically at heel strike
77- which of the following does NOT occur during the nor-
mal push-off stance of the gait cycle?

320
a. subtalar joint inversion
b. transverse tarsal joint locks
c. external rotation of the lower extremity
d. tightening of the plantar fascia
e. loosening of the spring ligament
78- About the wrist joint is all are true except;
a- Carpal stability is derived from numerous intra-and intercarpal lig-
aments in addition to closely approximated wrist flexors and extensors.
b- Motion occurring at the carpus is predominantly biplane--radi-
al ulnar deviation and palmar flexion and extension.
c- The center of motion for these planes of movement is located
within the proximal and palmar pole of the capitate.
d- Deformity can arise as a result of the inherent motor imbalance
noted between the 6 wrist motors.
e- The flexor carpi radialis is the dominant wrist motor with the least
significant force being supplied by the extensor carpi radialis longus.
79- About elbow joints, these statements are true except;
a- Positioning the hand in space for fine movements, powerful
grasping and serving as a fulcrum for the forearm.
b- Biomechanics of the elbow joint can be divided into, kinemat-
ics, stabilizing structures in elbow stability and force transmission
through the elbow joint.
c- The active stabilizers are the bony articular geometry and the
soft tissue stabilizers.
d- The active stabilizers are the muscles that provide joint com-
pressive forces and function
e- The elbow joint as ct as the mechanical link in the upper ex-
tremity between the hand, wrist and the shoulder.

321
80- Biomechanics of spine during walking, all are true except;
a- People walking with heavier backpack load adopted a compen-
satory trunk lateral flexion posture.
b- Kinematic gait parameters such as walking speed and stride
length remained unchanged with the increasing loads.
c- Walking with backpack load of 15%BW resulted in correspond-
ing increase in lumbosacral force of 26.7%
d- People walking with backpack load of 30%BW resulted in cor-
responding increase in lumbosacral force of 64%
e- People walking without backpack load have no increase in the force.
81- The main signs of an antalgic or painful gait all are
true except;
a- The Increased amount of time spent in the stance phase.
b- This is because people do not want to spend any more time
than necessary on a foot that is causing them pain.
c- The stance phase is usually divided equally between the two legs.
d- Decreased stride length, which results from patients not want-
ing to push off from their painful foot as powerfully as normal.
e- One stride tends to be much longer than the other
82- The difference between walking and running all state-
ment are true except;
a- The toe off stage of gait begins as the toes leave the ground.  This
represents the start of the Stance phase
b- During running there is a period of time when both feet are off
the ground (the “float” phase).  
c- Running is associated with greater speeds the forces that go
through the foot when it lands can be substantially greater than dur-
ing walking.
d- Often 4-5x body weight during running
e- Often up to 6-7x body weight during sprinting

322
83- Lumbar spine movements during walking all are true
except;
a- Global lumbar spine ROM during walking does not exceed one
fourth of maximal lumbar ROM during elementary trunk movements.
b- Frontal and transverse plane ROM and velocity significantly in-
crease with increasing walking velocity.
c- The same increase occurs in sagittal plane parameters.
d- The direction and magnitude of rotation and lateral bending
coupling displayed individual variations during walking.
e- Observations suggesting the existence of a preferred, individu-
ally varying, walking velocity, complete our findings.
84- Causes of walking asymmetry in people with idiopath-
ic scoliosis all are true except?
a- The capability to remain in upright standing position is funda-
mental to develop the ability to walk.
b- The stability in quiet standing is determined by keeping the
vertical axis of the body
c- proper muscle activity, joints mobility, balance,
d- Feet structure
e- Proprioception.
85- Gait pattern in subjects with scoliosis all are true ex-
cept;
a- There are two basic periods in every gait stride: a stance and
swing phase.
b- The proportion of the two phases is about 60% to 40% of the
time between consecutive heel strikes of the same
c- The stance is more demanding phase due to the need of shock
absorption at the time weight acceptance and recovery of the stabil-
ity during single limb support.
d- The stability during single and double limb support phases de-

323
pends the alignment of the spine and lower limbs, muscle strength,
joint mobility, coordination or balance of the body.
e- Walking have nothing to do with the reasons for scoliosis progression.
86- About Hyaluronic Acid, all are true except;
a- is a carbohydrate, more specifically a mucopolysaccharide oc-
curring naturally throughout the human body
b- When not bound to other molecules, it binds to water giving it
a stiff viscous quality similar to “Jello”
c- Its function in the body is, amongst other things, to bind water
and to lubricate movable parts of the body
d- Its consistency and tissue-friendliness allows it to be beneficial
in skin-care products as an excellent moisturizer
e- HA is one of the most nonphydrophilic molecules in nature
87- Dual-energy X-ray absorptiometry (DXA) all are true
except;
a- is a means of measuring bone mineral density (BMD).
b- Two X-ray beams with different energy levels are aimed at the
patient’s bones.
c- When soft tissue absorption is subtracted out, the BMD can be
determined from the absorption of each beam by bone.
d- Dual-energy X-ray absorptiometry is the most widely used and
most thoroughly studied bone density measurement technology.
e- Far superior to the nuclear bone scan, as it is sensitive to certain
metabolic diseases of bones in which bones are attempting to heal
from infections, fractures, or tumors.
88- Extracorporeal shock wave therapy (ESW) all are true
except;
a- ESWT delivers focused shock waves to the body.
b- both forms of shock wave therapy can be used in the treatment
of plantar fasciitis.

324
c- Low-energy shock wave treatments are given as a series of three
or more treatments.
d- high-energy shock wave treatments are given as three sessions.
e- Shock wave therapy is thought to work by inducing microtrau-
ma to the tissue that is affected by plantar fasciitis.
89- Navigation in Orthopaedics, all are true except;
a- Navigation System is a computer-aided, image-free navigation system
b- Radiation is the disadvantage
c- Enhance the surgical workflow and achieve optimal implant
alignment.
d- Optional modules for TKA, ACL, HTO
e- Every joint is unique and should be treated accordingly
90- Computer-assisted orthopedic surgery [CAOS]-based
technologies represent a spectrum of devices including all
but one:
a- 3-D image-guided and nonimage-based navigation systems,
b- intraoperative fluoroscopic navigation,
c- robotic-assistive tools, and new intraoperative visualization devices.
d- CAOS tools couple simulations with real time evaluations of sur-
gical performance.
e- Cuts the real time for preoperative or intraoperative imaging
91- About navigation in orthopaedics, all are true except
a- The first phase is data acquisition preoperatively or during surgery,
b- Including preoperative images, fluoroscopic images.
c- The second and third steps are tracking and registration, which
is the ability to relate images such as x-rays, CTs or MRIs and a pa-
tient’s 3-D anatomy to the anatomical position on the surgical field.
d- Tracking as we need sensors and measurement devices that can
provide feedback during surgery on the orientation and relative po-

325
sition of tools to bony anatomy
e- the most important factors in the adoption of the CAOS tech-
nologies will be diffeculties of use for surgeons and OR staff
92- Ultrasound technique in orthopedic practice all are
true except;.
a- Muscoskeletal sonography is safer and more informative than
X-rays for evaluating soft tissues pathology.
b- Compared with MRI, it is less informative
c- Musculoskeletal ultrasonography is indicated for evaluation of
soft tissue damage, particularly in sports injuries.
d- the assessment of articular structures and diseases.
e- Ultrasound should be performed when investigating rotator cuff
tears, inflammation, calcific tendinitis and impingement syndrome.
93- Stem cells all are true except;
a- Mesenchymal stem cells are obtained from living adult tissue.
b- Bone marrow stromal cells are mesenchymal
c- stem cells that, in the proper environment, can differentiate into
cells that are part of the musculoskeletal system.
d- Like Stem cells, progenitor cells can become cells with more
specialized functions
e- They can help to form trabecular bone, tendon, articular carti-
lage, ligaments and part of the bone marrow
94- About Ozone therapy, all are true except;
a- ozone injection is an effective treatment for herniated discs.
b- Ozone has been suggested for use in dentistry,
c- use by athletes in an attempt to increase performance
d- When infused into human blood, ozone produces reactive oxy-
gen species (ROS) or free radicals
e- Ozone caused a an over production of the lower lung membranes

326
95- Biomechanics of the Subtalar joint ,all are true except;
a- The motion generated at the joint is that of flexion-supination-
adduction or extension-pronation-abduction.
b- The subtalar motion is generated by the contour of the articular
surfaces and is guided by the intrinsic ligaments.
c- Further support is provided by the extrinsic ligaments
d- Under tibiotalar vertical loading with internal rotation, the subta-
lar joint complex is in a close-pack position with maximum talus head.
e- The posterior talocalcaneal surfaces are interlocked medially.
96- The goal of the subtalar joint arthroereisis is
a- To reduce the pronation range of motion of the subtalar joint
(STJ)
b- To create a positive therapeutic change in the alignment and
function of the foot and lower extremity during weightbearing ac-
tivities.
c- Is a surgical procedure to prevent excessive pronation and pre-
serve varus range of motion within the STJ.
d- It is also known as Triple arthrodesis.
e- It is done by bone graft anterior to the lateral process of the
talus on the floor of the sinus tarsi of the calcaneus to block excessive
STJ pronation.
97- Decay of force transients following active stretch of
muscles, all are true except
a- Following active lengthening of muscle, force reaches an iso-
metric steady-state above that which would be achieved for a purely
isometric contraction at the same muscle length.
b- Residual force enhancement (RFE), cannot be predicted by the
force-length relationship,
c-All due to unexplained by the cross-bridge theory of muscle
contraction.

327
d- Older adults experience lower RFE than young for the ankle
dorsiflexors
e- Primarily owing to a greater reliance on passive force enhance-
ment (PFE)
98- Cryopreservation Affects the Nanoscale Material Prop-
erties of Trabecular Bone,all are true except;
a- Tissues such as bone are often stored via freezing, or cryopreser-
vation.
b- For whole bone, the mechanical properties (strength and mod-
ulus) does significantly changed
c- Material properties at the trabecular and lamellar scales are dis-
tinct from whole bone properties,
d- The impact of freeze-thaw cycling at this scale is unknown.
e- The effect of repeated freezing on viscoelastic material proper-
ties of trabecular bone was quantified via dynamic nanoindentation.
99- Whole-body angular momentum during stair walking
with passive and powered lower-limb prostheses, all are
true except;
a- Individuals with a unilateral transtibial amputation have a great-
er risk of falling compared to able-bodied individuals, and falling on
stairs can lead to serious injuries.
b- Individuals with transtibial amputations have lost ankle dorsi-
flexor muscle function.
c- This is critical for regulating whole-body angular momentum to
maintain dynamic balance.
d- Powered prostheses have been designed to provide active
ankle power generation with the goal of restoring biological ankle
function. However,
e- The effects of using a powered prosthesis on the regulation of
whole-body angular momentum are unknown.

328
100- To reduce ACL injury risk, all are true except;
a- One option is to improve muscle strength and/or the activation
of muscles crossing the knee under elevated external loading.
b- Elevated gastrocnemii forces corresponded with increased joint
compression and lower ACL forces.
c- The elevated quadriceps and gastrocnemii forces during land-
ing may represent a generalized muscle strategy to increase knee
joint stiffness,
d- The use of knee cap support will protect the knee and ACL from
external knee loading and injury risk.
e- Understanding muscle‫׳‬s function during single-leg jump land-
ing and should serve as the foundation for novel muscle-targeted
training intervention programs aimed to reduce ACL injuries in sport.

ANSWERS:

1-b 2-d 3-e 4-a 5-c 6-c 7-e 8-a 9-d 10-d 11-
c 12-b 13-e 14-c 15-a 16-d 17-a 18-d 19-b 20-d
21-c 22-e 23-b 24-d 25-a 26-a 27-d 28-e 29-e
30-b 31-a 32-e 33-b 34-d 35-e 36-d 37-d 38-a
39-e 40-d 41-c 42-d 43-c 44-c 45-e 46-d 47-c
48-e 49-c 50-a
51-c 52-a 53-c 54-b 55-b 56-b 57-a 58-c 59-e
60-b 61-d 62-b 63-e 64-b 65-d 66-b 67-a 68-c
69-d 70-d 71-e 72-d 73-a 74-b 75-d 76-e 77-e
78-e 79-c 80-a 81-a 82-a 83-c 84-e 85-e 86-e 87-
e 88-d 89-b 90-e 91-e 92-b 93-d 94-e 95-e 96-d
97-d 98-b 99-b 100-d

329
330
SPORTS MEDICINE MCQs
1: Which sport causes the most head injuries?
Football b. Cycling c. Soccer d.Baseball
2: Where is the largest (and often injured) tendon in the
body?
a- Back b. Ankle c. Neck d. Shoulder
3: What’s the exact cause of muscle cramps?
Injury b. Vitamin deficiency
c. Dehydration d. Unknown
4: A sprain is an injury to:
A ligament b. A muscle
c.A tendon d.A bursa
5: What causes the most emergency room visits?
a.Bruises and scrapes b. Cuts
c. Broken bones d. Sprains and strains
6: What is R.I.C.E. treatment?
a- Rest, Ice, Crutches, Elevation
b- Rest, Ice, Compression, Elevation
c- Rest, Ibuprofen, Crutches, Exercise
d- Reinforcement, Immobilization, Cryotherapy, Electrostimulation
7: What’s the most common runner’s injury?
a- Shin splints b. Runner’s knee
c. Achilles tendinitis d. Plantar fasciitis
8: Shin splints are caused by:
a-Tiny fractures b. Torn ligaments

331
c. Inflammation d.All of the above
9: What do golfers and weightlifters injure most often?
a-The knee b. The hips
c. The lower back d. The elbow
10: Tennis elbow is usually caused by:
a-Trauma b. Infection c. Underuse d. Overuse
11: A good warm-up will:
a. Increase your blood flow b. Warm your muscles
c. Boost your breathing d. All of the above
12: What’s the best time to stretch your muscles?
a-After warming up b. During exercise
c. After exercise d. None of the above
13: What is most often injured in downhill skiing?
a- The head b. The knee c. The ankle d. The wrist
14: If you have plantar fasciitis, what part of your body is
hurt?
a-The knee b. The foot
c. The ankle d. None of the above
15: According to current research, the safest stretching
technique appears to be ;
a- ballistic
b. proprioceptive neuromuscular facilitation (PNF)
c. static d. isotonic
16: Tennis elbow and golfer’s elbow are both examples of what?
a-Tendonitis b. Muscle Strains c. Ruptures d. Nephritis

332
17: Sports physiologists classify the two types of muscle
fibers as what?
a-smooth and rippled b. fast and slow twitch
c. aerobic and anaerobic d. Fsciollation
18: Pain in the anterior compartment of the tibia (which
sometimes occurs from prolonged running on a hard sur-
face) is called what?
a-Tendonitis b. Subluxation
c. Shin Splints d. cramps
19: What is the focus of rehabilitation for the repaired
knee immediately following ACL-reconstruction surgery?
a-Strengthening b. Restoring Range of Motion
c. Gait Training d. Prevent stiffness
20: Iliotibial band syndrome (ITBS) is characterized by
pain at which area of the knee?
a-Lateral b. Anterior c. Posterior d. medial
21: According to the American College of Sports Medi-
cine guidelines, patients who have diabetes should engage
in aerobic activity:
a- 3 to 4 days per week, 10 minutes per session
b- 3 to 5 days per week, 20 to 60 minutes per session
c- only if they are younger than age 50 years
d- Once weekly
22: What shoulder injury is common in throwing sports?
a-Contusion b. Low Back Strain
c. Rotator Cuff Injury d. dislocation
23: A partial dislocation of a joint is referred to as a what?
a-subluxation b. separation
c. malocclusion d. sprain

333
24: What should you apply to an acute (recent) ankle
sprain
a-Heat b. Ben-Gay c. Ice d. Hot spray
25 : The pre-participation physical examination is con-
ducted for the purpose of what?
a- Determine in condition or defect which exist and put the ath-
lete at risk
b- Bring the athlete’s attention to any muscle weakness or imbalance
c- Determine whether an athlete can participate in the event safely
d- All of the above
26: When the pre-participation physical examination is
conducted ideally?
a- One month before the beginning of the season
b- Two weeks before the season begin
c- One week before
d- Six months before
27: The following are extrinsic factors for sports injuries
except;
a- Sports equipments used
b-Sports environmental conditions
c- Type of sporting activities
d- Physical characteristic of the athlete
28: The lack of which muscle strength is considered as a
risk in overuse injuries?
a-Isometric muscle strength
b. Dynamic muscle strength
c. Ballistic muscle strength
d. All of the above

334
29; which one of the following is considered a significant
risk factor in acute sports injuries?
a- Increased dynamic muscle strength
b- Decreased dynamic muscle strength
c- Increased static muscle strength
d- Decreased ballistic muscle strength
30: Anecdotally, fatigue affects the physical and mental
performance of a player during which time in the game?
a-First quarter b. Second quarter
c. Middle of second quarter d. End of the game
31: According to the body mass index (BMI), which of the
following athlete is considered obese?
a-Body builder with BMI of 32.9
b. Short athlete with BMI of 24.6
c. Basketball player with BMI of 25
d. Power lifter with BMI of 29.4
32: Which of the following is an early sign of acclimatiza-
tion of high altitude?
a-Decrease tidal volume b. Decrease respiratory rate
c. Increase stroke volume d. Increase cardiac output
33: The highest duration of training should be performed
during which following sports season?
a-Post-season b. Pre-season
c. In-season d. Off-season
34: Which somatotype of an individual has large efficient
abdominal viscera and tends to be obese?
a-Endomorphs b. Mesomorphs
c. Viscerotonoic d. Ectomorph

335
35: Which intrinsic feedback mechanism is used by the
sports person to monitor their own ability to maintain stabil-
ity of functionally unstable joint?
a-Biofeed and feedback technique
b. Proprioceptive re-training
c. Sensory re-education
d. All of the above
36: The level of strength can be determined by using the
following device except:
a-Cable tensiometer b. Cybex isokinetic dynamometer
c. Nautilus machine d. Dynamometer
37: In individuals with asthma, regular physical training
leads to improvements in:
a. Forced expiratory volume in one second
b. Vital capacity
c. Peak expiratory flow rate
d. maximal oxygen uptake
e. bronchial hyper-responsiveness.
38: The increased rate of spondylolysis with certain sports
is believed to be related to which of the following?
a- Participation of smaller athletes
b- Repetitive extension, flexion, and rotational forces in the lumbar spine
c- Significant axial loading associated with landing after a jump or dismount
d- High velocity collision e- Excessive rest
39: Osteoporosis is:
a- Defined as a bone density 2·5SD below the mean age matched
reference range

336
b- Defined as a bone density 2·5SD below the mean young refer-
ence range
c- Defined as a bone density between 1–2·5SD below the mean
young reference range
d- Characterised by a low mineral content in bone
e- Approximately four times more common in women than men
40: Bone mineral density can by improved:
a- In the spine by swimming
b- In the total body by moderate intensity walking
c- By high impact exercise
d- More when exercise is combined with Hormone replacement
therapy(HRT)
e- More by strength training than endurance training
41: Exercise-induced amenorrhoea:
a- Is due to a defect in the ovary leading to reduced oestrogen secretion
b- Is often associated with a late puberty
c- Results in low hip bone mineral density
d- Results in an accelerated bone resorption
e- Can affect up to 80% of some athletic groups
42: What is the common adverse reaction to corticoste-
roid injections?
a- Post injection flare and tendon rupture
b- Extra pain or discomfort and facial flushes
c- Headache and abnormal menstrual bleeding
d- Hypersensitivity reactions and extra pain or discomfort
e- Skin atrophy, depigmentation, and facial flushes

337
43: Which statement correctly reflects currently available
evidence for the effectiveness of corticosteroid injections
for shoulder pain?
a- There is insufficient evidence to support or refute the effective-
ness of corticosteroid injections for shoulder pain.
b- The effectiveness of injections is superior to other conservative
treatments at any moment of follow up.
c- Injections are no better than physiotherapy for the painful
shoulder, but superior to placebo treatment or analgesics.
d- Injections show beneficial short-term effects compared to oth-
er conservative treatments, but there is little evidence for long term
benefits.
d- The available evidence shows little or no beneficial effects of
corticosteroid injections for shoulder pain.
44: Osteitis pubis
a- presents in males and females in equal numbers
b- is a result of overuse rather than acute injury
c- is characterised by pain radiating into the medial thigh and scrotum
d- is always bilateral
e- None of the above
45: Sportsman’s hernia
a- refers to a tear of the rectus abdominis muscle
b- may be seen on ultrasound examination
c- usually affects the medial end of the inguinal canal
d- is an indirect incipient inguinal hernia
e-responds well to surgical repair in athletes
46: Imaging in groin pain
a- may involve peritoneal imaging in the diagnosis of sportsman’s
hernia

338
b- is best done with MRI rather than CT scan
c- may use ultrasound imaging for the diagnosis of bursitis
d- None of the above
e- All of the above
47: The diagnosis of patellar tendinopathy is ideally
based on
a-palpation tenderness b. changes on imaging
c. history of the condition d. functional tests
e. all of the above
48: Anti-inflammatory medication in the treatment of
overuse patellar tendinopathy is
a- indicated because of the nature of the pathology
b- rarely indicated
c- best applied topically
d- effective in long-term tendon pain
e- none of the above
49: Conservative treatment of patellar tendinopathy is
a-based on good scientific evidence
b. best based on exercise
c. best based on massage d. ineffective
50: Factors resist joint mobility all true except;
a-Warm-up
b.Neural receptors of the surrounding muscles
c. High altitude d. Age
51: In dynamic muscle exercises the contraction is named;
a-Eccentric b. Isometric c. Isokinetic
d. Isotonic e. Isoinertial

339
52: Competitive anxiety affects the performance of athlete in:
a- Anxiety as an objective feeling of apprehension
b-The major problem in competition is letting your mind work
against you rather than for you
c- Competition can cause athletes to react both physically and men-
tally in a manner which can positively affect their performance abilities
d- Sometimes accompanied by heightened physiological arousal
e- Anxiety can be recognized on three levels Cognitive - by partic-
ular thought process, Somatic - by physical response and Behavioral
- by patterns of behavior
53: Cognitive anxiety consists of except;
a-Stomach upset b. Sweating c. Bradycardia
d. Negative concern about performance and other unpleasant feel-
ing
e. Somatic anxiety includes symptoms of autonomic reactions
54: Preseason planning, the team physician should coor-
dinate:
a- Development of policy to address pre-season planning
b- Pre-participation evaluation of athletes
c- Participation of the administration in medical issues
d- Implementation of the strategies
d- All of the above
55: The medical bag items should include except;
a- Alcohol and povidone iodine swabs
b- Bandages, scissors
c- Tracheostomy set
d- Local anesthetic agents
e- Sutures set

340
56: Return-to-play decision should address all TRUE ex-
cept;
a- Safety of athlete
b- Potential risk to the safety of other participants
c- Functional capability of the athlete
d- Functional requirements of the athlete’s sport
e- Coach and trainer decision
57: The team physician duties during bad environments
should include all except;
a- Develop a modification or cancelation policy for the event in
hazardous conditions
b- In hot conditions determine an ambient temperature and rela-
tive humidity appropriate for the type of sport, abilities and age of
the athlete
c- In cold conditions, determine an ambient temperature and
wind-chill , altitude and latitude
d- Suspend activities when lightning and thunder conditions
e- Approximately 2 days per time zone crossed is needed to reset
circadian
58: Female athlete triad all are true except:
a-Amenorrhea b. Osteoporosis c. Eating disorder
d. Calcium and vit. D are needed e. Tachycardia
59: Doping all are true except;
a- Substance to gain a competitive advantage
b- Tests for doping in Olympic games firstly used in Mexico 1968
c- Diuretics are listed as doping agent
d- Erythropiotin is a doping drug
e- Blood tests are the diagnostic tools

341
60: Exercises related changes to the menstrual cycle all
are true except:
a-Diminished progesterone secretion
b. Increased basal body temperature
c. Increase luteal phase
d.Normal cycle length
e. Endometrial hyperplasia
61; Physiological changes that may affect exercise
throughout pregnancy include except:
a- Musculoskeletal changes including weight gain
b- Medical changes including increased heart rate,
c-Decrease cardiac output
d-increase blood volume
e- increase respiratory rate
62: The goals of exercise throughout pregnancy are true except;
a- maintain or improve preexisting levels of maternal fitness with-
out undue risk to the mother or the developing fetus
b- Pregnancy increases nutritional needs for calories, iron, calci-
um, and folic acid
c- Exercise in the supine position after 16 wk should be avoided
due to potential great vessel compression
d- Unplanned pregnancy indicates high-risk behavior
e- The presence of sexually transmitted diseases (STDs) indicates
low-risk behavior
63: The following statements all are true except;
a- Intense exercise alone does not necessarily cause menstrual
dysfunction, provided there is adequate caloric intake for the energy
needs

342
b- Consequences of menstrual dysfunction may include lower lev-
els of estrogen and/or progesterone
c- Higher BMD
d- Higher incidence of stress fractures
e- Infertility
64: The following statements are true except;
a- The female is at a low risk of ACL injury in multiple sports and
activities
b- Noncontact ACL injury rate is 2–10 times higher in female ath-
letes than in their male counterparts
c- Causes of noncontact ACL injuries may be multifactorial; pro-
posed risks include environmental, anatomical, hormonal, biome-
chanical, and neuromuscular factors
d- Noncontact ACL injuries occur commonly during deceleration,
landing or cutting.
e- At-risk positions during these maneuvers include knee exten-
sion, flat foot, and off-balance body position
65: It is essential that treatment of the injured or ill athlete
should include:
a. Follow an individualized plan, which may include consultations
and referrals Include a rehabilitation plan, Include equipment modi-
fication, bracing, and orthoses as necessary
b. Address psychosocial issues
c. Provide a realistic prognosis as to the safe and timely return to
practice or competition
d. Include documentation
e. All of the above
66: Events should have fluids (and food if indicated) avail-
able for safe participation.
a- Fluids should be easily accessible and strategically placed

343
b- 180–360 mL of fluid should be available for every 15–20 min of
continuous activity
c- Excessive fluid intake may result in hypernatremia
d- Provide additional fluid choices containing carbohydrate and
sodium for events involving continuous activity lasting more than 1
h, It is desirable to Cool fluids to 59c*
e- Encourage participants to replace sweat losses during activity
and replace weight loss post-event
67: It is essential to provide on-site:
a- Basic first aid and CPR
b- Early fracture fixation
c- Advanced cardiac life support and advanced trauma life support
d- Intravenous fluid administration for non life–threatening illness
e- Hyper- and hypothermia evaluation and initial care
68: The team physician must have all are true except;
a- An unrestricted medical license that is responsible for treating
and coordinating the medical care of athletic team members.
b- The principal responsibility of the team physician is to provide
for the well-being of individual athletes — enabling each to realize
his/her full potential.
c- The team physician should possess special proficiency in the
care of musculoskeletal injuries and medical conditions encountered
in sports.
d- The team physician also must actively integrate medical exper-
tise with other healthcare providers, including medical specialists,
athletic trainers, and allied health professionals.
e- The team physician must ultimately assume responsibility sole-
ly for making medical decisions that affect the athlete’s safe partici-
pation.
69: Popping Knee syndrome is:
a- due to anatomy of semi tendinosus in pes anserine

344
b- snapping popliteus
c- discoid menisci
d- medial tibial plateau cupping
e- giving way knee
70: All of the following statements are true except;
a- quadriceps tendon ruptures more common than patellar tendon
b- patellar tendon ruptures common in younger patients with di-
rect or indirect trauma
c- quadriceps tendon ruptures commonly in elderly patients with
indirect trauma
d- in patellar tendinitis the pain is worse in flexion than extension
71: Jumper’s knee is:
a-quadriceps tendinitis
b. patellar tendinitis
c. prepatellar bursitis
d. Iliotibial tract band friction syndrome
72: Iliotibial tract band friction syndrome more common in :
a-Tennis players b. Volleyball players
c. Cyclists d. Footballers
73: Hamstring strain which is a common injury in sprint-
ing to prevent recurrence, the return to play should be de-
layed till;
a-Six weeks b. Eight weeks
c. Strength of scarotuberous ligament d. It is pain free
74: Athletic pubalgia is due to:
a- Sprain of pubosacral ligament
b- Sprain of scartuberous ligament
c- Strain and repetitive microtrauma of rectus femorus
d- Strain and repetitive microtrauma of adductors

345
75: Piroformis syndrome is:
a- Wasting and chronic muscle fatigue of piroformis
b- Sciatic nerve entrapment
c- Obturator nerve entrapment
d- Reflex sympathetic dystrophy of piroformis
76: Skier’s hip is:
a-Snapping hip syndrome
b. Proximal femoral fracture
c. Trendelenbeg hip
d. Bone marrow oedema syndrome
77: Anterior tarsal tunnel syndrome is due to the com-
pression of:
a-Deep peroneal nerve b. Superficial peroneal nerve
c. Tibial nerve d. Lateral plantar nerve
78: Tarsal tunnel syndrome is due to compression of:
a-Medial plantar nerve b. Deep peroneal nerve
c. Superficial peroneal nerve d. tibial nerve
79: Joggers foot is:
a-plantar fasciitis b. calcaneal spur
c. medial plantar nerve entrapment
d. lateral plantar nerve entrapment
80: Tennis leg is:
a-Strain of tendoachilis b. Tibialis posterior strain
c. Calf muscles strain d. Plantar tendon strain
81: The most commonly involved ligament in ankle strains is:
a-Anterior tibio-fibular ligament
b. Posterior tibio-fibular ligament
c. Deltoid ligament d.Calcaneo-fibularligament

346
82: Turf toe is;
a- Due to sever dorsiflexion of metatarso-phalangeal joint of big toe
b- Hallux valgus
c- Seen in duplication of big toe
d- Avulsion of extensor expansion
83: During throwing maximum torque is generated with
action of:
a-Late preparation b. Deceleration
c. Acceleration d. Middle of deceleration
84: Calcifying tendinitis predominantly affects:
a-Supraspinatus tendon b. Infraspinatus tendon
c. Teres minor tendon d. Subscapularis tendon
85: Minor traction and compression injuries of the bra-
chial plexus are commonly known as burners or strings by:
a-Boxers b. Wrestlers
c. Handball players d. Football players
86: The players should be removed from competition till
x-ray of cervical spine is done if burners occur:
a-For the first time
b. More than once
c. Three times continuously in a gap of 2hours
d. Minimum of five times
87: Little leaguer’s elbow is:
a-Stress fracture of medial epicondyle
b- Subluxation of radial head
c- Partial rupture of annular ligament
d- Partial rupture of anterior band of ulnar collateral ligament
88: Jersey finger is:
a- Avulsion injury of flex degitorum profundus

347
b- Avulsion injury of terminal extension tendon
c- Avulsion injury of flex. Digitorum superficialis
d- Rupture of A1&A2 pulley
89: Gamekeeper’s thumb includes:
a-Radial collateral ligament injury
b. Ulnar collateral ligament injury
c. Both of the ligament injuries
d. Bennett’s fracture
90: Steener’s lesion is associated with:
a-Jersey finger b. Boxers knuckle
c. Drummer’s wrist d. Gamekeeper’s thumb
91: Spear tackler’s spine is seen in :
a-Gymnastics b. Football players
c. Ballet dancers d. Basketball players
92: In young athlete, sudden cardiac death is usually due to;
a. Syncope b. Coronary artery disease
c. Cardiac temponade d. hypertrophic cardiomyopathy
93: All are adverse effect of abuse of anabolic steroids
except:
a. Irreversible alopecia b. Blindness
c. Hypercholesterolemia d. gynecomasia
94: The second leading cause of fatality in football play-
ers is:
a-Exercise induced bronchospasm b. Chest injury
c. Heat stroke d. Splenic ruture
95: In acute knee injury with swelling of haemoarthrosis
with muscle spasm, which is thew most sensitive test?
a-Lachman’s test b. Pivot shift test
c. Apley’s grinding test d. Mcmurray’s test

348
96: Which of the following components of fitness is de-
fined as the body›s ability to sustain prolonged exercise?
a-Cardiorespiratory endurance b. Body composition
c. Flexibility d. Muscle endurance
Muscle strength
97: Which of the following is a benefit of physical fitness?
a-Decrease in lean body mass
b. Reduced risk of type 1 diabetes
c. Impaired immune system
d. Increased risk of cardiovascular disease
e. Increased bone density
98: What is the primary energy source during high-inten-
sity exercise?
a-Fat Equal mixture of carbohydrate b. Protein, and fat
c. Alcohol d. Carbohydrate e. Protein
99: What nutrient should be consumed immediately be-
fore exercise to give muscles an immediate source of glu-
cose and spare glycogen stores?
a-Vitamin Supplement b. Carbohydrate
c. Fat d. Alcohol e. Protein
100: Which of the following forms of exercise is predomi-
nately anaerobic?
a-Stair climbing b. Swimming
c. Brisk walking d.weight lifting e. jogging

349
ANSWERS;

1-c 2-b 3-d 4-a 5-b 6-b 7-c 8-d 9-d 10-d 11-
d 12-c 13-b 14-b 15-c 16- a 17-b 18-c 19-b 20-a
21-b 22-c 23-a 24-c 25-d 26-a 27-d 28-a 29-a
30-d 31-a 32-d 33-d 34-d 35-b 36-a 37-a 38-a
39-e 40-e 41-c 42-d 43-a 44-d 45-b 46-a 47-c
48-a 50-c
51-d 52-c 53-c 54-e 55-c 56-e 57-e 58-e 59-
e 60-e 61-c 62-e 63-c 64-a 65-e 66-c 67-b 68-e
69-d 70-d 71-b 72-c 73-c 74-d 75-b 76-b 77-a
78-d 79-c 80-c 81-a 82-a 83-c 84-a 85-d 86-d 87-
a 88-a 89-c 90-d 91-b 92-d 93-b 94-c 95-a 96-a
97-e 98-d 99-b 100-d

350
Entrance Exam of Orthoaedics
ONE Single Answer
1) Calcitonin is secreted by:
a.Thyroid gland b.Parathyroid gland
c.Adrenal glands d.Ovaries
2) Which of the following is an intraarticular tendon?
a. Sartorius b. Semitendinosus
c. Anconeus d. Popliteus
3) which of the following conditions is least likely to pres-
ent as an eccentric osteolytic lesion:
a. Aneurysmal bone cyst. b. Giant cell tumor
c. Fibrous cortical defect d.Simple bone cyst
4 ) Joint erosions are not a feature of:
a. Rheumatoid arthritis c. Psoriasis
b. Multicentric reticulohistiocytosis
d. Systemic lpus erythematosus
5 ) Which of the following is not a feature of hypercalce-
mia:
a. Diarrhoea b. Polyuria c. Depression d. Vomiting
6 ) Physical quality of life index is measured by all except:
a. Infant mortality b. Life expectancy at age one
c. Literacy d. Per capita income
7) When a drug is evaluated for its usefulness in con-
trolled conditions, it is termed as a trial signifying:
a.Efficacy b. Effectiveness
c. Efficiency d. Effect modification
8) Which of the following is not an indication for endotra-

351
cheal intubation?
a. Maintenance of a patent airway
b. Pulmonary toilet
c. To provide positive pressure ventilation
d. Pneumothorax
9 ) In adults, the spinal cord normally ends at;
a-. Lower border of L1.
b. Lower border of S1.
c. Lower border of L3.
d. Lower border of L5
10) The common cause of subarachnoid hemorrhage is :
1. Arterio-venous malformation.
2. Cavernous Angioma.
3. Aneurysm.
4. Hypertension
11) Mycotic abscesses are due to:
a. Bacterial infection b.Fungal infection
c. Viral infection d . Mixed infection.
12) Total score in Glasgow Coma Scale of a conscious
person is :
a. 8 b. 3 c. 15 d. 10
13) Which of the following is not a component of Glasgow
Coma Scale?
a. Eye opening. b. Motor response.
c. Pupil size. d. Verbal response
14) The most common malignancy found in Marjolinâ s
ulcer is:
a. Basal cell carcinoma.
b. Malignant fibrous histiocytoma.

352
c. Squamous cell carcinoma.
d. Neurotrophic malignant melanoma
15 ) Which one of the following imaging techniques gives
maximum radiation exposure to the patient?
a. Chest X-ray. b. MRI
c. CT scan. D. Bone scan
16 The most common location of hypertensive intracra-
nial haemorrhage is :
a.Subarachnoid space. b.Basal ganglia.
c.Cerebellum. d.Brainstem.
17) Expansile lytic osseous metastases are characteris-
tic of primary malignancy of :
a. Kidney. b. Breast
c. Bronchus. d. Prostate
18) The maintenance of posture in a normal adult human
being depends upon:
a. Integrity of reflex are.
b. Type of muscle fibres.
c. Muscle power.
d. Joint movements in physiological range.
19) Which of the following organs is not involved in cal-
cium homeostasis?
a. Kidneys. b. Intestines.
c. Skin. d. Lungs.
20) All of the following are major complications of mas-
sive transfusion, except:
a. Hypokalemia. b. Hypomagnesemia
c. Hypothermia d. Hypocalcemia.
21) Disc Prolapse is Common in
a. L4, L5, b. L5, S1 c. T12, L1 d. S1, S2

353
22) Non Union Is more Common In
a. Fracture Lateral Condyle Of Humerus
b. Colle’s Fractured.
c . Supracondylar fracture of Humerus
d. All of the above
23) Common Complication of Septic Arthritis of Hip is
a. Hypermobile Joint b. Both
c. Bony Ankylosis d. None
24) Posterior Dislocation Of HIP-
a. Adduction, b. Internal Roatation,
c. Flexion, d. All of the above
25) Osteoporosis Seen in all Except-
a. After Menarchae b. Phenobarbitone therapy
c. Treatment with Steroids d. After menopause
26) Genu varum means;
a. Bowing of Knee b.. Bowing of Femur
c. Bowing of Tibia d. Injury to pelvis
27) Mallet finger is
a. Fracture of Proximal phalanx
b. Rupture of flexor tendon.
c. Capsular rupture of PIP Join
d. Rupture of extensor digitorum profundus slip
28) Which of the following is a Tumour of bone origin
a. Osteoma b. Myeloma
c. Leukemia d. all of the above
29) Most common complication of Anterior dislocation of
shoulder is
a. Recurrent dislocation b. Osteoarthritis

354
c. Myositis ossificans
d. Avascular necrosis of head of humerus
30) Function of Menesci all are true Except:
Helps the knee stability c. circulating synovial fluid
Locking and unlocking occurs in the Lateral meniscus d.The whit
zone highly capsular structures
31).concerning the radial nerve all are true except
a. it ends at the superior border of the supinator
b. it is mainly motor than sensory.
c. its root value is from c3 to c7.
d. it divides into posterior interossus
32) Trendelenberg test is positive in the following except::
a- Leg Calve-Perth’s disease b- Coxa vara d. coxa vulgus
c. Paralysis of Abductors
33) Total blood volume in an adult who weighs 70 kilo-
grams is
a. 2600 ml b. 3600 ml
c. 4600 ml d. 5600 m
34) Myosin is involved in ;
a. Muscle Contraction b. Secretion of Insulin
c. Conduction of nerve impulse d. None of the above
35) Prussian blue is used for staining
a. Haemosiderin b. Bone c. Muscle d. Fat
36) Which of the following disorders is concerned with
pathology in Collagen and connective tissue
a. Myasthenia Gravis d. Phaeochromocytoma
c. Marfans’ Syndrome d. Sickle Cell Anemia
37) Hypertrophy of muscle is due to
a. Increase in number of mitochodnria

355
b. Decrease in number of mitochondria
c. Increase in size of mitochondria
d. Abnormal shape of mitochondria
38) Most common complication of Extra Capsular Frac-
ture in a female patient aged 30 years is;
a. Malunion b. Non Union
c. Fracture of Radius d. None of the above
39) in compartment syndrome all are true except;
a.Sustained increase of pressure
b.Volkmann’s ischemic contracture is the end result
c. loss of sensation is the prime symptom
d. pain the prime symptom
40). Origin of bone is from:
a. Ectoderm. b. Mesoderm
c. Endoderm. d. All of the above.
41) Acute osteomyelitis is commonly caused by:
a. Staph aureus. b. S. pyogenes.
c. H. influenzae. d. Salmonella.
42) Acute osteomyelitis usually begins at:
a. Epiphysis. b. Metaphysis.
c. Diaphysis. d. Any of the above
43) A greenstick fracture:
a. Occurs chiefly in the elderly. b. Does not occur in children.
c. Is a spiral fracture- of tubular bone.
d. Is a fracture where part of the cortex is intact and part is crum-
pled or cracked.
44) Spiral fracture is due to:
a. Blunt trauma. b. Axial compression.
c. Twist. d. Direct impact.

356
45) The single most important factor in fracture healing is:
a. Correct bone alignment. B. Immobilization.
c. Accurate reduction. d. Organization of clot.
46 ) Immobilization is not required in fracture involving:
a. Scapula. b. Wings of ilium.
c. Rib. d. All of the above.
47) Perthes’ disease is common to age group of:
a. 1-5. b. 6-10. c. 11-15. d. 16-20.
48) First bone to ossify in foetal life is:
a. Femur .b. Tibia. c. Clavicle. d. Sternum.
49) Radial nerve palsy may occur in fracture of humerus
involving:
a. Surgical neck. b. Shaft.
c. Lower end. d. At all of the above locations.
50) Which nerve is compressed in carpal tunnel syn-
drome:
a. Ulnar. b. Median. c. Radial. d. All of the above.

ANSWERS:
1-a 2-d 3-b 4-a 5-c 6-b 7-a 8-b 9-a 10-d 11-b
12-c 13-c 14-c 15-a 16-a 17-b 18-c 19-d 20-a 21-a
22-a 23-c 24-d 25-a 26-a 27-d 28-a 29-a 30-a 31-c
32-d 33-d 34-a 35-a 36-c 37-a 38-a 39-c 40-b 41-a
42-b 43-b 44-c 45-d 46-d 47-b 48-c 49-b 50-b

357
358
Libyan Board of Orthopaedic (First
Part Paper)

1-Abnormal lateral curvature of the spine of which a sta-


bilization procedure may be done:
A. scoliosis B. osteoporosis
C. exostosis D. herniated nucleus pulposa
2-Death of bone tissue due to the lack of circulation:
A. arthritis B. avascular necrosis
C. malunion D. osteomyelitis
3-Equipment used to debride and and clean infected
wounds and joints:
A. periosteal elevator B. drill/reamer
C. suction irrigator D. impactor
4-The most likely site for autogenous bone graft is the:
A. femur B. ischium C. scapula
D. ilium
5-In orthopedic surgery, absorbable suture would most
likely be used on:
A. tendons B. periosteum
C. ligaments D. bone
6-The patient would be placed in prone position for which
of these procedures:
A. Excision of a Baker’s Cyst
B. AK Amputation
C. Knee Arthroscopy
D. Triple Arthrodesis of the Ankle

359
7-When assisting with the application of a plaster cast,
you would do all of the following except:
A. secure a bucket of hot water
B. prepare only one plaster bandage or strip at the time
C. gently squeeze excess water from the plaster bandage after bub-
bles have ceased to rise
D. clean plaster from the patient and equipment as soon as possible
8-The type of fracture that includes splinter or crushed
bones:
A. Colles’ B. greenstick
C. compound D. comminuted
9-General term for joint reconstruction:
A. osteotomy B. arthrodesis
C. osteoarthritis D. arthroplasty
10-All of the following are guidlines to be used when us-
ing bone cement except:
A. mix immediately before use B. use a fume evacuator
C. mix with a gloved hand
D. supply to the surgeon in a syringe for injection into the medullary
canal
11-Heavy suture is used to reattach a weakend anterior
capsule to the glenoid fossa to correct recurrent dislocation
of the shoulder in which procedure:
A. Excision of Ganglion B. Keller Procedure
C. Bankhart Procedure D. Kyphoplasty
12-Removal of a fluid filled sac protruding through wrist
joint for cosmesis or discomfort is called:
A. Excision of Ganglion B. Patellectomy
C. McBride Procedure D. Metacarpal Arthroplasty

360
13-Use of prosthetics to replace arthritic joints in the
hand:
A. Triple Arthrodesis B. Carpal Tunnel Release
C. Bankhart Procedure D. Metacarpal Arthroplasty
14-Excision of kneecap for degenerative arthritis:
A. Bristow Procedure B. Patellectomy
C. Putti-Platt Procedure D. Rotator cuff correction
15-Bone retrieved from the patient for bone grafting is
called:
A. cadaveric B. autogenic
C. xenogenic D. allogenic
16-The placement of fixation pins through the skin into
bone proximal and distal to the fracture site and then con-
nected to stabilizing bars, is called:
A. external fixation B. skeletal fixation
C. internal fixation D. compression fixation
17-Fixation device most commonly used for intratrochan-
teric fractures of the femur:
A. compression plates and screws
B. hinged replacement prosthesis
C. rods/nails D. Kirschner wires
18-When a tourniquet is used the surgeon must be noti-
fied which time intervals:
A. at the half hour, and every fifteen min- utes thereafter
B. at the half hour, and every half hour thereafter
C. at the hour, and every fifteen minutes thereafter
D. at the hour, and every half hour there- after
19-Screws come in which types:
A. cortical only B. cortical and Kirschner
C. cancellous and cortical D. trochanteric and Kirschner

361
20-Polymethylmethacrylate is more commonly known as:
A. a material implants are made from
B. material used to coat implants when placing a noncemented
prosthesis
C. a material soft tissue allografts are made of
D. material used to hold the prosthesis when placing a cemented
prosthesis
21-The implant used on small bone fragments or as part
of skeletal traction is called:
A. cancellous screw B. Steinmann pin
C. cortical screw D. nail
22-When performing a skin prep for surgery on the tibia,
the skin prep would extend:
A. from above the hip joint to the toes
B. from above the knee joint to the toes
C. from the incision site down to the toes
D. from the hip joint to the knee joint
23- The dressing applied at the end of an orthopedic pro-
cedure which gives com pression to the wound is a/an:
A. kerlix fluff B. xeroform
C. ace or elastic bandage D. fiberglass or plaster cast
24-The sterile tourniquet cuff is applied:
A. before the skin prep begins
B. during draping but before other drapes are placed over the upper
extremity
C. at the end of the draping procedure
D. never; tourniquet cuffs do not come sterile
25-The x-ray views usually obtained during orthopedic
surgery include all of the following except:
A. sagittal B. oblique

362
C. anterior/posterior D. lateral
26-After shoulder surgery:
A. the unaffected arm is immobilized against the torso of the patient
B. the affected arm is not immobilized, as the patient will not move
it due to postoperative discomfort
C. the affected arm is not immobilized, as the patient needs to begin
range of motion in the joint immediately
D. the affected arm is immobilized against the torso of the patient
and padding is placed between skin surfaces which will contact one
another
27-Casting material can be made from all of the following
materials except:
A. hydroxyapatite B. gypsum
C. anhydrous calcium sulfate impregnated crinoline
D. fiberglass
28-An open fracture, where bone protrudes through the
skin, is also referred to as a fracture:
A. greenstick fracture B. closed traumatic fracture
C. comminuted fracture D. compound fracture
29-A fracture where the tendonous attachments of the
bone are torn away is called a/an:
A. tendon fracture B. epiphyseal separation
C. avulsion D. compound fracture
30-The stage of osteogenesis where there is a prolifera-
tion of cells from the fracture site into the fibrin to form fi-
brous connective tissue and cartilage is called:
A. osteoid stage B. blood clot formation stage
C. remodeling stage D. callus stage

363
31-The method of fracture treatment where a pulling force
is exerted directly on the bone itself via a pin in the bone us-
ing a system of weights and pulleys is called:
A. Buck’s traction B. external fixation
C. open reduction internal fixation (ORIF)
D. skeletal traction with balanced suspension
32-Principles of joint reconstruction include all of the fol-
lowing except:
A. joint function depends on the quality of its structures
B. bones are held loosely in place by ligaments and the joint capsule
C. the synovial membrane lining the joint capsule secretes synovial
fluid to lubricate the joint
D. articular cartilage covers the ends of the bones where they meet
to form a joint
33-A loosening of a joint with some displacement of the
articular surface from the joint capsule is called a/an:
A. dislocation B. avulsion
C. fracture D. subluxation
34-Reconstruction of a joint to restore or improve range
of motion and stability or to relieve pain is referred to as an:
A. arthrotomy B. arthroplasty
C. arthrogenesis D. arthroscopy
35-Concepts concerning polymethylmethacrylate use on
the surgical field include all of the following except:
A. room and component temperature will affect the setting time
B. contact of the polymer and your skin may cause a dermatitis re-
action
C. avoid excessive exposure to the vapors produced during mixing
of the polymer and the powder
D. polymethylrnethacrylate should be prepared at the beginning of
the case so it is ready at the desired time of use

364
36-Visualization of a joint for diagnosis and conservative
treatment of joint disease is called:
A. arthroplasty B. arthrotomy
C. arthroscopy D. arthrorrhaphy
37-An overgrowth of bone is referred to as an:
A. exostosis B. osteoblast
C. osteocyte D. osteoclast
38-The “freeing up” of an entrapped nerve is referred to
as a/an:
A. neuropexy B. kyphosis
c. neurolysis D. kyphoplasty
39-Excision of a tendon sheath and its associated synovi-
um is referred to as a:
A. tenorrhaphy B. tenotomy
C. tendopexy D. tenosynovectomy
40-The yellow, elastic tissue connecting the lamina is
called the:
A. nucleus pulposa B.anteriorcruciateligament
C. ligamentum flavum D. coraco-acromion ligament
41-A rotator cuff tear occurs in which region of the body:
A. knee B. shoulder
C. vertebrae D. ankle
42-A fracture where healing has not taken place in the
usual amount of time is referred to as a/an:
A. nonunion B. malunion
C. delayed union D. subluxation
43-The patient position for a Bristow Procedure is:
A. semi-Fowler B. prone
C. Sims D. lithotomy

365
44-An Austin Moore prosthesis is used at which joint:
A. knee B. hip
C. shoulder D. ankle
45-A Colles’ fracture occurs in which bone(s)?
A. radius B. humerus
C. tibia/fibula D. clavicle/acromion
46-Positioning for procedures include all of the following
combinations except:
A. supine in traction for a Compression Hip Nailing
B. lateral for a Total Hip Arthroplasty
C. semi-Fowler for a Bristow Procedure
D. prone for a Total Knee Arthroplasty
47-The bony process which can be involved in an ankle
fracture is the:
A. medial malleolus B. medial meniscus
C. lateral trochanter D. calcaneous
48-The procedure which consists of excision and fusion
of the middle phalangeal joint of the foot with incision of the
flexor tendon is:
A. Bunionectomy B. Triple Arthrodesis
C. Correction of Hammer Toe Deformity
D. Metacarpal Arthroplasty
49-The fracture commonly treated with angled blade
plates or a long cancellous screw would be:
A. patella fracture B. tibial plateau fracture
C. humeral shaft fracture D. femoral shaft fracture
50-A Triple Arthrodesis does not include fusion of which
bones:
A. calcaneocuboid joint B. metacarpophalangeal joint
C. talocalcaneal joint D. talonavicular joint

366
51-Which position is the patient placed in for an Anterior
Cervical Discectomy and Fusion?
A. supine B. prone
C. Sims D. Kraske
52-Long cancellous screws placed with a gliding hole for
the shaft provide:
A. compression of bone fragments
B. approximation of bone fragments
C. distraction of bone fragments
D. subluxation of bone fragments
53-A potential complication of ischemia of the femoral
head is:
A. intermittant claudication B. intussusception
C. avascular necrosis D. chondromalacia
54-A fracture commonly found in children is a:
A. comminuted B. greenstick
C. Colles’ D. longitudinal
55-A fracture requiring debridement and closure prior to
casting would be a
A. comminuted B. greenstick
C. compound D. spiral
56-A surgical procedure designed to stiffen or fuse a joint
is called:
A. Arthropexy B. Arthroplasty
C. Arthrotomy D. Arthrodesis
57-A hemostatic agent used to control bleeding from an
open bone is
A. heparin sodium B. protamine sulfate
C. methylene blue D. bone wax

367
58-The power drill or saw used to divide bone with a “side-
to-side” cutting action is a/an:
A. reciprocating saw B. rotary drill
C. oscillating saw D. drill/reamer
59-A tear in the lateral or medial knee cartilage is operated
by performing a/an:
A. Synovectomy B. Menisectomy
C. Patellectomy D. Arthrodesis
60-Chronic dislocation of the patella may predispose the
patient to a condition called:
A. osteomyelitis B. chondromalacia
C. rheumatoid arthritis D. osteoporosis
61-Baker’s cysts are found in the:
A. popliteal fossa B. interdigital fossa
C. intercarpal fossa D. olecranon fossa
62-Which one of the following is the correct order for
screw placement:
A. drill, measure, tap, place B. drill, tap, measure, place
C. tap, drill, measure, place D. tap, measure, drill, place
63-When implanting plates and screws, what medication
is used on the surgical field?
A. heparin sodium for anticoagulation
B. polymixin for decreasing edema
C. Hypaque for hemostasis
D. Bacitracin for antimicrobial prophylaxis against osteomyelitis
64. What differentiates a tendinitis from a tenosynovitis?
A. A tenosynovitis involves a synovial sheath .
B. A tendinitis does not have an active inflammatory process.
C. They are the same.

368
D. A tenosynovitis affects only tendons of the upper extremity.
65.  Choose the appropriate parameters for a transverse
friction massage in treating tendinitis:
A. 3/10 intensity, 4x/week, 10 minutes, 3 weeks
B. 5/10 intensity, 3x/week, 4-5 minutes, 1-2 weeks
C. ,3/10 intensity, 3x/week, 4-5 minutes, 1-2 weeks
D. ,5/10 intensity, 4x/week, 10 minutes, 3 weeks
66. A patient comes to you with articular cartilage pathol-
ogy at the knee. The patient has chondral fibrillations that
are about 1.55 mm. What Outerbridge classification applies
to this patient?
A. ,I B. ,II C. ,III D. ,IV
67.  ,What is Davis› law?
A. ,Bone applies to the stresses placed upon it.
B. ,In order for Osteogenesis to occur, we must apply 60% of the
tolerable stresses.
C. ,Soft tissue applies to the stresses placed upon it.
D. ,The stress-strain curve is shifted to the left when we apply forces.
68.  ,When we are training, we should follow what rule to
avoid overloading the musculoskeletal system?
A. ,5% B. ,10% x C. ,7% D. ,14%
69.  ,Which of the following is not true about the likeli-
hood to develop RSD?
A. ,UE > LE B. ,Women > Men
C. ,#1 cause is fracture D. ,non-work injury > work injury
,70.  ,Same force is termed:
A. ,Isometric B. ,Isotonic C. ,Isokinetic
71. ,How often should we see a subacute patient?
A. daily B. 2-3 x/week C. 1x/week

369
72. Where is the most common fracture location of the
proximal humerus?
A.Anatomical Neck B. Lesser tuberosity
C. greater tuberosity D. surgical neck
73. Neer›s classification is based on the following struc-
tures, except:
A. Greater tuberosity B. Surgical neck
C. Head D. Lesser tuberosity
E. Proximal Shaft
74. Neer›s is classified based on displacement and an-
gulation ;
A. 1 cm, 45 degrees B. 2 cm, 30 degrees
C. 1 cm, 30 degrees D. 1.5 cm, 45 degrees
E. None of these are correct
75. About the percentage (%) of proximal humerus frac-
tures are classified as Neer 1-part.
A. 75% B. 10% C. 80% D. 25%
76. Neurovascular injuries are likely to occur with all of
the following except one:
A. Anterior dislocation B. Inferior dislocation
C. Posterior dislocation D. 4-part fracture
77. The following are all reasons to do a hemiarthro-
plasty, except:
A. 2 part fracture in the elderly
B. 3 part fracture in the elderly
C. 4 part fracture D. head splitting
78. Which is not true of primary healing?
A. Least common. B. External callus formation .
C. Compression plates used. D. Early motion is possible.

370
79. Which is not a sign of ischemia?
A. puffiness B. paralysis
C. Pinkness D. pain E. pulselessness
80. With healing, callus forms at about what week?
A. 2 B. 3 C. 4 D. 5
81. When can we apply stresses over a fracture site?
A. When the patient can tolerate the pain.
B. When the cast is removed.
C. When the bone is clinically healed
D. When there is radiographic evidence that the fracture line is
completely closed.
82. Which is NOT a criteria for return to function?
A. 20% strength deficit or less
B. normal ROM
C. independent in all ADLs
D. normal kinesthetic awareness
83. The determination of a disease or clinical problem by
systematically comparing and contrasting clinical findings
between two or more disease is:
A. Diagnosis B. Differential Diagnosis
C. Prognosis D. Clustering
84. A patient has diffuse, aching, poorly localized pain in
the low back area. Which is a probable cause for the pain?
A. gallbladder B. nerve entrapment
C. multifidus strain D. ovarian cancer
85. Which is NOT included in the upper quarter screen?
A. TMJ B. UE
C. Upper Thoracic Spine D. Shoulder Complex
E. Thoracolumbar spine

371
86. What is selective tissue tension?
A. Isolate a tissue and apply tension to record the tissue›s response
B. A technique that is used to assess arthrokinematic motion.
C. Selecting tissue and applying the most tension that it can pos-
sibly take to get plastic deformation.
87. Radiculopathy is differentiated from radiculitis how:
A. Radiculitis is when neurological signs present.
B. Radiculopathy is when neurological signs are present.
C. Only a radiculopathy involves nerve root irritation.
D. Radiculitis is only present in the lumbar spine.
88. what occurs by reflex inhibition or muscle weakness.
A. Functional instability B. Pseudolocking
C. Voluntary Instability D. Giving Way
89. Which signs will be produced by a myelopathy ?
a-UMNL b-LMNL
c-All of the above d-Non of the above
90. Which is not a sign of a UMN lesion?
A. Positive pathologic reflexes
B. Negative superficial reflexes
C. Hyporeflexia
D. Spastic gait
91. If the patient had pain that followed the L5 derma-
tome, where would the pain end?
A. Heel B. Back of calf
C. Front of calf D. Big Toe
92. Passive ROM testing includes:
A. End-feel assessment
B. Sequence of pain and resistance
C. identification of capsular pattern D. all of these

372
93. You are assessing the passive range of a patient›s
shoulder. The patient presents with pain that is synchro-
nous with tissue resistance. The patient also has marked
limitations in the following pattern: ER > Flex > IR > Abduc-
tion. From this, what can you infer about the stage of the
healing process, acuity, and pattern?
A. Acute, inflammatory stage, non-capsular
B. subacute, reparative, non-capsular
C. acute, reparative stage, capsular
D. subacute, remodeling, capsular
94. You have just completed resistive movement testing
on a patient›s shoulder abductors. The patient tells you that
there is pain and you note weakness. Which is not a pos-
sible cause?
A. fracture B. complete avulsion
C. dislocation D. acute arthritis
95. You test the knee flexors isometrically. You find that
the quads on the left are weak and painless. What can you
infer?
A. Rupture of the muscle B. Dislocation
C. The patient is a malingerer- these don›t happen together.
D. Subluxation
96. You are grading the point tenderness along a patient›s
posterior deltoid. The patient complains of pain and winces.
What grade?
A. I B. II C. III D. IV
97. The patient has a Biceps DTR of 2. What does that
mean?
A. hyporeflexia B. Normal
C. hyperreflexia E. clonus

373
98. The patient has a positive Babinski test. What does
that look like?
A. Pointer finger and thumb flick toward each other
B. There is a «bouncing» after contraction
C. Big toe goes into extension
D. All toes go into extension
99. Which is best to show a stress fracture?
A. MRI B. CT
C. Osteoscintigraphy D. Arthrography
100. Which is true about joint mobility testing?
A. The joint becomes more painful as you approach the open pack
position.
B. Movements are not under voluntary control.
C. You must have normal osteokinematrics before you have normal
arthrokinematics.
D. Joint play movements are performed in the closed pack position.

374
ANSWERS:

1-a 2-b 3-c 4-d 5-b 6-a 7-a 8-d 9-d 10-c 11-c 12-a
13-d 14-b 15-b 16-a 17-a 18-c 19-c 20-b 21-b 22-b
23-c 24-b 25-c 26-d 27-a 28-d 29-c 30-d 31-d 32-b
33-d 34-b 35-d 36-c 37-a 38-c 39-d 40-c 41-b 42-a
43-a 44-b 45-a 46-d 47-a 48-c 49-b 50-b
51-a 52-a 53-c 54-b 55-c 56-d 57-d 58-c 59-b 60-b
61-a 62-a 63-d 64-a 65-c 66-c 67-c 68-b 69-d 70-b
71-b 72-d 73-b 74-a 75-c 76-b 77-a 78-b 79-c 80-b
81-c 82-a 83-d 84-c 85-e 86-a 87-b 88-d 89-a 90-c
91-d 92-d 93-b 94- 95-a 96-b 97-b 98-c 99-c 100-b

375
376
Libyan Board of Orthopaedics (FINAL
EXAM Paper ONE)

1 -All are TRUE re; brachial plexus injury EXCEPT;


a- Preganglionic lesion have a better prognosis than postganglionic
ones.
b- Erb’s plasy causes paralysis of abductors and external rotators of
the shoulder
c- Kumpke plasy , Horner syndrome may be present on the ipsilat-
eral side
d- Histamin test is useful to differentiate between the pre and post
ganglionic lesions.
2-Meralgia parasthetica is due to involvement of which
nerve?
a- Medial Cutaneous nerve of the thigh
b- Lateral cutaneous nerve of the thigh
c- Sural nerve d- Femoral nerve
3-Volkmann’s Ischemia most commonly involves which
muscle?
a- Pronator teres
b- Flexor carpi radialis
c- Flexor digitorum profundus
d- Flexor digitorum superfecialis
4- Bone growth is influenced maximally by which hor-
mone?
a- Thyroxin b- Growth hormone
c- Testosterone d- Estrogen

377
5- Complete transaction of spinal cord at the C 7 level pro-
duces all of the following EXCEPT:
a- Hypotension b- Limited respiratory effort
c- Anesthesia below that level d- Areflexia below that level
6- All true about haemophelia except:  
a. is not caused by an absolute deficiency of factor VIII,
b.  sex-linked recessive
c. is confirmed by abnormal PTT & low factor IX activity
d. patients may present with a recurrent knee hemarthrosis
7- The nerve root which is at risk for an injury when plac-
ing percutaneous screws at sacro-iliac joint is :
a- L4 b- L3 ,L4 c- L5 d- S1
8- Thompson test is diagnostic for:
a- Fixed flexion deformity of the hip b- PCL rupture
c- Rotator cuff tear d- Tendo-achilis rupture
9- Which of the following views gives most information
about post column of pubis?
a- AP view b- Iliac oblique
c- Obturator oblique d- Inlet view
10-The nerve most commonly injured with extension type
of supracondylar fracture of the humerus is
a- Posterior interosseous b- Anterior interosseous
c- Ulnar nerve d- Radial nerve
11- The most important pulley in the thumb is :
a- Oblique pulley b- A1 pulley
c- C1 pulley d- A2 pulley
12- All true about chondrocalcinosis except:
 a. chondrocalcinosis of the meniscus occurs only  in knee joints

378
b. occurs in association with several distinct metabolic disorders; 
c.  the majority of cases of chondrocalcinosis occur from trauma;
d. Calcium pyrophosphate dihydrate crystals are visualized under
polarized light Microscopy
13- The most common nerve involved in Volkmann’s isch-
emic contracture is :
a- Median Nerve b- Radial nerve
c- Ulnar nerve d- Posterior interosseous nerve
14- Windswept deformity is seen in which of the follow-
ing?
a- Ricketts b- Rheumatoid arthritis
c- Scurvy d- Ankylosing spondylitis
15- Tendon transfer for wrist drop is:
a- Pronator teres b- Flexor carpii radialis
c- Flexor carpii ulnaris d- Palmaris longus
16- Cozen’s test is positive in which of the following of :
a- Tennis elbow b- De-quervaine’s disease
c- Carpal tunnel syndrome d- Ulnar bursitis
17- Calcification of the menisci of the knee strongly sug-
gests a diagnosis of :
a- Acromegaly b- Hyperparathyroidism
c- Renal osteodystrophy d- Pseudogout
18- Action of K-wire fixation provides which of the follow-
ing?
a-Two-point fixation b- Three-point fixation
c- Compression d- Wt. concentration
19- Bone graft with maximum osteogenic potential is:
a- Fresh autograft b- Fresh cortical autograft
c- Osteoperoosteal graft d- Vascularized bone graft

379
20- All are TRUE EXCEPT:
a- Osteoblasts form osteocytes
b- Osteoblast secret alkaline phosphatase
c- Osteoblasts are produced by local mesenchymal precursors
d- Osteoblasts line thelacunae of bone and form osteoclasts
21) which of the following is true regarding superior
mesenteric artery (SMA)syndrome?
a. the condition often occurs in overweight female patients
b. this syndrome is also known as a cast syndrome
c. this condition occurs following curve correction as a result
of an increase in the angle between the aorta and the superior
mesenteric artery
d. the condition is due to an ischaemic event of the SMA
22) in central canal stenosis of the lumbar spine, all
of the following are true exept ?
a. it is more common in men
b. acquired stenosis is the more common type
c. tension sign are usually positive
d. can occur as a result of paget’s disease
23) all the following are true for hangman’s fracture
except?
a. it is a traumatic spondylolisthesis of c2 on c3 as a result of
bilateral fracture of c2 pars or pedicles
b. the mechanism of injury is a primary hyperflexion of the
neck
c. type IIA fracture has a significant angulation without displace-
ment
d. type U fracture has a significant angulation without displace-
ment

380
24) the central cord syndrome is due to?
a. a fall on a flexed neck
b. hyperextension injury on a background of a herniated disc
c. a hyperextension injury in a patient with a facet joint hyper-
trophy and thickened ligament flavum
d. an anterior spinal artery lesion
25) injury to which artery is most likely to cause un-
controllable bleeding during the posterior approach to
the hip?
a. inferior gluteal
b. superior gluteal
c. pudendal
d. ascending branch of lateral circumflex femoral
26) with regard to closed suction drains used in sur-
gery, which of the following is true?
a. wound infection is higher with use of drains
b. haematoma formation is lower with use of drain
c. wound dehiscence is more likely without the use of drains
d. blood transfusion is more likely with the use of drains
27) What is the predominant source of femoral head
perfusion?
a. lateral circumflex artery
b. obturator artery
c. medical circumflex artery
d. descending branch of lateral circumflex artery
28) in which situation following deep infection of a
total hip replacement could single-stage revision be con-
sidered?
a. the erythrocyte sedimentation rate (ESR) and C-reactive pro-
tein (CRP) are not raised

381
b. it is less than 6 months since primary surgery
c. there is a known organism from preoperative aspirate and
the patient has commenced antibiotics
d. the organism is seen and identified via fresh frozen specimen
sent during surgery, and the patient is commenced on antibiotics
29) what is meralgia paraesthetica due to?
a. compression of the obturator nerve at the obturator foramen
b. haematoma around part of the sciatic nerve
c. compression of the lateral cutaneous nerve of thigh
d. diabetes
30) which of the following is the greatest risk factor
for heterotopic ossification following elective total hip
replacement?
a. previous formation of heterotopic ossification
b. hypercalcaemia
c. male
d. head injury within the previous 48 hours
31) which of the following statements regarding ante-
rior cruciate ligament (ACL) grafts is false?
a. the maximum load to failure of a patellar tendon graft is
approximately 2600 newtons
b. the use of an autologous hamstring graft results in a 50%
loss of hamstring strength
c. the maximum load to failure of a quadruple hamstring graft
is approximately 4500 Newton’s
d. allograft processing does not always alter the mechanical
properties of the graft
32) which of the following is considered to be the
primary stabilizer of knee to external rotation?
a. anterior cruciate ligament (ACL)

382
b. posterior cruciate (PCL)
c. lateral collateral ligament (LCL)
d. medical collateral ligament (MCL)
33) which of the following should be avoided during
total knee arthroplasty to avoid lateral patellar sublux-
ation?
a. internal rotation of the tibial component
b. external rotation of the femoral component
c. lateral placement of the tibial component
d. lateral placement of the femoral component
34) Regarding total knee replacement, which of the fol-
lowing is incorrect?
a. the joint line can be safely lowered 4mm without an adverse
effect on motion and joint instability
b. the minimum recommended thickness of an ultra-high molec-
ular-weight polyethylene
c. a deficient extensor mechanism is a relative contraindication
to a total knee replacement
d. there is a poorer implant survivorship in patients with rheu-
matoid arthritis
35) the following situations preclude the use of a
high tibial osteotomy for the treatment of medical com-
partment osteoarthritis, except?
a. lateral tibial subluxation
b. normal lateral compartment
c. deficient anterior cruciate ligament
d. inflammatory arthropathy
36) the following are all considered predisposing fac-
tors for patellofemoral disorders, except?
a. femoral anteversion

383
b. lateral patella tilt
c. patella baja
d. reduced trochlea sulcus
37) Which of the following statements regarding ante-
rior cruciate ligament (ACL) reconstruction is true?
a. the femoral tunnel should be in the 1 o’clock position in the
straight knee
b. on the lateral radiograph, the femoral tunnel should be on
the anterior half of Blumenssaat’s line
c. on the femoral side, interference screw fixation has been
shown to be superior to suspensory type fixation
d. during hamstring harvesting , the connection between the
semitendinous and the medial gastrocnemius must be divided
38) which of the following statements is true regard-
ing mobile-bearing total knee replacements in compari-
son to fixed –bearing total knee replacements?
a. there is a reduction in the amount of volumetric wear
b. they result in a better post-operative range of motion
c. they do not have a better survivorship
d. they should only be used in patients under 70 years of age
39) The main internal rotators of the hip are?
a. gluteus minimus and tensor fascia lata
b. obturator internus,superior and inferior gemelli
C. piriformis and quadrates femoris
D. piriformis and gluteus medius
40) the surgical approach for the posterior cruciate
ligament insertion site during an open inlay technique
is?
a. a posteromedial approach between medial gastrocnemius and
semimembranosus

384
b. a posertomedial approach between medial gastrocnemius and
semitendinosus
c. a posteromedial approach between semimembranosus and
semitendinosus
d. a posteromedial approach between splitting medial gastroc-
nemius
41) the blood supply to the anterior cruciate ligament
is ?
a. the medial superior genicular artery
b. the lateral superior genicular artery
c. the middle genicular artery
d. the lateral inferior genicular artery
42) a vertical talus is most commonly associated with
which of the following ?
a. oligohydraminos b. arthrogryposis
c. congenital talipes equinovarus d. tarsal coalition
43) the Lisfranc ligament’s attachments are?
a. intermediate cuneiform to base of second metatarsal on
plantar surface
b. medial cuneiform to base of second metatarsal on planter
surface
c. intermediate cuneiform to base of first metatarsal on dorsal
surface
d. medial cuneiform to base of first metatarsal on plantar sur-
face
44) which of the following is the most common pri-
mary soft tissue malignancy of the foot?
a. malignant melanoma b. osteosarcoma
c. squamous cell carcinoma d. synovial cell sarcoma

385
45) which of the following is true of talar neck frac-
tures?
a. if a displaced talar neck fracture is reduced and stably fixed,
the chance of subtalar arthritis is 25%
b. a type III fracture is associated with tibiotalar dislocation
c. a Hawkins sign seen on a radiograph 6 weeks post-injury is
poor prognostic sign
d. type I and II fracture may be treated non-operatively
46) which of the following best describes a low
transverse fibular fracture and vertical medial malleolar
fracture?
a. supination- external rotation b. supination- adduction
c. pronation-external rotation d. pronation-abduction
47) which of the following is the optimal position for
ankle arthrodesis?
a. 5° varus, 10° external rotation, 0° flexion, 5mm anterior talar
translation
b. 5° valgus, 10° external rotation, 10° dorsiflexion, 5mm anterior
talar translation
c. 5° valgus, 10° external rotation, 0° flexion, 5mm posterior talar
translation
d. 5° varus, 10° internal rotation, 0° flexion, 5 mm posterior talar
translation
48) Which of the following is true of calcanal fractures?
a. they are typically associated with an increased Bohler›s angle
b. they are extra-articular in the majority of cases
c. the Sanders classification is based upon the number and
location of articular fragments on sagittal CT scan
d. the wound complication rate is 5% following open reduction
and internal fixation

386
49) which of the following is not true of articular car-
tilage composition in severe osteoarthritis?
a. increased water content
b. decreased chondroitin 4-sulphate concentration
c. decreased collagen content
d. decreased keratin sulphate:chondroitin sulphate ratio
50) vitamin D-resistant rickets is inherited as which of
the following traits?
a. autosomal dominant b. autosomal recessive
c. X-linked dominant d. X-linked recessive
51) the diagnostic criteria for ankylosing spondylitis
include all of the following except?
a. HLA B27 positivity
b. limitation of motion of the lumbar spine
c. history of pain in the lumbar spine
d. limited chest expansion to 2.5 cm or less
52) which of the following tumours is least likely to
involve the posterior elements of the spine?
a. osteoid osteoma b. aneurismal bone cyst
c. osteoblastoma d. osteochondroma
53) all of the following principles must be adhered to
when performing a biopsy of a bone tumor except?
a. the selection of the biopsy path should made in consultation
with the surgeon who will perform the definitive excision
b. the biopsy tract should be marked to allow excision at the
time of definitive surgery
c. the biopsy should ideally be performed at the centre where
the definitive excision is likely to be carried out
d. the tumor should be approached through normal tissue be-
fore entering the reactive zone

387
54) curettage and grafting is acceptable treatment for
all of the following lesions except?
a. osteoblastoma b. aneurismal bone cyst
c. osteofibrous dysplasia d. chondromyxoid fibroma
55) all the following are features of giant cell tumour
of bone except?
a. the most common site in the axial skeleton is the sacrum
b. cross the physis c. metastasize to lung
d. metaphyseal-epiphyseal location
56) which of the following is true regarding Ollier’s
disease
a. autosomal dominant
b. sarcomatous degeneration occurs in 30% of patients
c. hand involvement is an uncommon feature
d. incidence of conversion to chondrosarcoma is similar Ma-
fucci’s syndrome
57) which of the following is the most sensitive clini-
cal sign for detection of developmental dysplasia of the
hip (DDH) in a baby aged 6 months?
a. galeazzi test
b. asymmetric skin folds in the thights
c. limited hip abduction in flexion
d. ortolani’s test
58) when treating slipped upper femoral epiphysis
(SUFE), which of the following might be the sole indica-
tion for subcapital osteotomy?
a. metaphyseal blanch sign b. Southwick angle >60°
c. avascular necrosis (AVN) d. femoral retroversion

388
59) A supraondylar fracture of the distal humerus with
posterolateral displacement should be reduced by per-
forming reduction manoeuvres in the following order?
a. valgus- flexion- pronation
b. varus- extension- supination
c. valgus- extension- pronation
d. traction- pronation- flexion
60) following clubfoot surgery , which of the following
is the the commonest residual deformity?
a. forefoot adduction b. internal tibial torsion
c. forefoot supination d. equines
61) The ligament which is injured the most in an ankle sprain is
a) Spring ligament b) Deltoid ligament
c) Anterior talofibular ligament
d) Posterior talofibular ligament

62)Galeazzi’s Fracture (Adults) all false except:


a.  usually the head dislocation is palmar; 
b. fracture is almost always located just above proximal border
of pronator quadratus
c.  usually there is posteriorr angulation with  a spiral configurations
d ;  brachioradialis  causes shortening & rotation of proximal RU joint 
e. adults tend to have good results with closed reduction
63) Complete transection at the C7 cervical vertebral level
will show all the features Except:
a) Anaesthesia below the level of the lesion
b) Areflexia below the level of the lesion
c) Diminished respiratory rate d) Hypotension

389
64) Which of the following conditions is not associated
with an increased risk of developing Achilles tendinopathy?
a) Fluoroquinolone antibiotics
b) Diabetes mellitus
c) Obesity
d) Oestrogen deficiency
65)Tibial Plafond Fracture:
a. term pilon  fracture was introduced to describe compression in-
juries by Appley in 1973
b. combination of calcaneal & distal tibial diaphyseal  fractures  with
comminution
c. fibula may remain intact
d. ankle & distal tibial metaphyseal fractures usually with intra ar-
ticular comminution
66)Syndesmotic Injuries of the Ankle:
a. syndesmosis is made up of anterior-inferior tibiofibular ligament,
interosseous ligament, and posterior-inferior fibular ligaments, inferior
transverse tibiofibular ligament, ligament; 
b. the above ligaments stabilize the mortise by opposing the talus
in the fibular notch
c. syndesmotic injuries are unusual in displaced Weber A fractures
d. managed by cast  immobilization in plantigrade position
67) In reviewing complications of anterior dislocation of
the shoulder, one finds that
a) Rotator cuff damage is rare in patients over 40 years
b) Vascular damage is generally seen in younger patients
c) Checking skin sensation over the deltoid insertion is a reliable
way of determining axillary nerve damage
d) Both transient and complete axillary nerve injuries recover prac-
tically completely

390
68) What is the main functional deficit after distal biceps
tendon ruptures?
a) Loss of extension strength only
b) Loss of flexion strength only
c) Loss of extension strength and supination strength
d) Loss of flexion strength and supination strength69) A small
well demarcated avascular fragment of bone and overlying
cartilage sometimes separates and appears as a loose body
in the joint (OCD). Which is the commonest site in the knee?
a) Medial femoral condyle b) Lateral femoral condyle
c) Patella d) Upper part of tibia
70) 1- Anterior dislocation of the proximal interphalangeal
joint in the hand is associated
with a great risk of permanent impairment because of:
a. Rupture of the volar plate
b. Development of a Boutonniere deformity
c. Damage to the digital nerve
d. Damage to the digital artery
71)Burst Fractures  of Spine; all true except:
a. burst  fractures involve only compressive failure of vertebral body both
anteriorly & posteriorly, with failure of both anterior & middle columns
b. axial loading applied to intravertebral disc results in increased
nuclear pressure and hoop stresses in the annulus;
c. it is essential that the patient be log roled off the trauma board
as soon as possible
d.  ileus is not commonly  associated condition
72) The most common organism responsible for an epi-
dural abscess is:
a. Streptococcus species b. Enterobacter species
c. Staphylococcus d. Haemophilus influezae

391
73) in the rheumatoid hand with an intact extensor tendon
mechanism, replacement of the metacarpophalangeal joints
might be considered if there is?
a. Palmar and proximal dislocation of the base of the phalanx
b. MARKED ULNAR DRIFT COMBINED WITH (A) AND (B)
c. All of these
d. None of these
74)Dislocations of the Elbow:
a. is usually closed and posterior; 
b. usually result from fall onto flexed elbow. 
c. central angle of semilunar notch is significantly smaller in group
of pts who had dislocation of the elbow compared to normal
d.  lateral collateral ligament provides 54% valgus stability
75) Which of the following statements concerning hemi-
arthroplasty of the hip for fracture is not correct?
a. Cemented implants are associated with a lower overall reopera-
tion rate
b. Cemented implants are associated with a lower peri-operative
fracture rate
c. Cemented implants carry the same overall complication rate as
uncemented implants
d. Cemented implants have the same survivorship as uncemented
implants
76) Which of the following organisms is currently least
likely to be responsible for septic arthritis of the hip in a
child?
a. Enterobacter species
b. Haemophilus infleunzae
c. Methicillin resistant Staphylococcus aureus
d. Staphylococcus aureus
e. Streptococcus group A

392
77) Which of the following quadrants of the acetabulum
are most at risk for injury by screwsDURING fiXATION OF
AN UNCEMENTED TOTAL HIP replacement?
a. Anterior-superior and anterior-inferior
b. Posterior-superior and posterior-inferior
c. Anterior-superior and posterior-superior
d. Anterior-inferior and posterior-superior
78)Snapping hip; all are true except:
a, snapping of the iliopsoas tendon over the iliopectineal eminence 
b. painful snapping sensation over the anterior aspect of the groin; 
c. may be a cause of a painful  total hip replacement 
d. snapping of either the lateral  border of the IT band or the me-
dial   border of the gluteus maximus over the lesser rochanter
79) Eliciting the impingement test in the hip involves
which of the following manoeuvres?
a. Flexion, abduction and external rotation
b. Flexion, adduction and internal rotation
c. Flexion, adduction and external rotation
d. Flexion, abduction and internal rotation
Cross matching MCQ, The options may be used once or more than
once or not at all:
Which of the following options is best described in each of the fol-
lowing statements for shoulder injuries , it can be used ONCE or MORE
or NON at All?
a) Collar &Cuff b) Hemiarthropasty c) open reduction &internal
Fixation d) Reverse shoulder arthroplasty e)open reduction and
bone grafting
80) Two-part proximal humeral fracture with 8mm dis-
placement of greater tubrosity.
81) Four-part head splitting proximal fracture in 55 yrs old

393
82) four-part valgus impacted proximal fracture in 44yrs old.
● Choose the most appropriate option for each of the following
statements in the surgical approach to the pelvis ?
A ) Gluteus maximus and medius b) Sartorius & tensor facial lata
c) Inguinal ligament d) rectus abdominus e) posterior division
of obturator nerve
83) These muscles are elevated in the Kocher-Langen-
beck approach to the acetabulum
84) in the anterior approach to the symphysis pupis these
muscles are divided but not denervated
85) this structure may be damaged if the ASIS is taken
during bone grafting.
● Which of the following options is best described in tumors of
the hand:
a) Synovial b ) epithelioid c) neurofibroma d) schwanoma e)
enchondroma
86) It is the most common soft tissue sarcoma of the
hand
87) This is the most common benign bone tumor and if
multiple carries 30% risk of sarcoma
88) A lesion of the nerves that shells out like a leap from
pod
● Which of the following is best described in upper limb trauma?
a) Galleazzi # b) Monteggia # c) Rolando # d) Essex-Lopresti
injury
e) Stener lesion
89 ) It is an injury described by Bado
90 ) This a Y-or T-shaped intraarticular fracture described
in1910
91 ) It involves dislocation of distal R/U joint and fracture
of radial head

394
Which of the following is best described in surgical approaches?
a) Kocher›s b) Henry›s c) Thompson›s d) Smith-Petersen›s
e) Deltoprctoral
92) Exploits internervous plane between the femoral &
posterior gluteal nerves
93) Exploits internervous plane between radial & median
nerves
94) Will involve encountering the ascending branch of
lateral femoral circumflex.
Which of the following is best described in hip disorders?
a) AVN b) SUFE c ) DDH d) snapping fascia lata e) femoro-ace-
tabular impingment
95) May be associated with +ve Ober test
96) A condition where damage occurs at chondrolabral
junction
97) May be diagnosed with Trethowan›s sign
Which of the following is best described in x-ray of the knee?
a) Patella baja b) Pellegrini-Steida sign c) patella alta d)
squared-off lateral femoral condyle e) Segond sign

98) X-rays showing discoid lateral meniscus


99) X-rays with radio-opacity adjacent to medial femoral
condyle
100) X-rays of TKR on top of HTO

395
ANSWERS:

1-a 2-b 3-c 4-c 5-b 6-c 7-c 8-d 9-b 10-b
11-a 12-a 13-a 14-a 15-a 16-a 17-d 18-b 19-d
20-d 21-b 22-d 23-a 24-d 25-d 26-c 27-d 28-
d 29-d 30-a 31-c 32-b 33-b 34-d 35-d 36-b
37-c 38-d 39-b 40-c 41-a 42-b 43-b 44-c 45-c
46-b 47-c 48-b 49-b 50-a 51-d 52-d 53-a 54-d
55-d 56-c 57-a 58-b 59-d 60-b 61-c 62-b 63-
c 64-d 65-d 66-a 67-d 68-d 69-a 70-b 71-d
72-c 73-c 74-a 75-a 76-b 77-a 78-d 79-b 80-c
81-c 82-c 83-a 84-d 85-c 86-b 87-e 88-d 89-
b 90-c 91-d 92-d 93-b 94-d 95-d 96-d 97-b
98-d 99-b 100-a

396
Libyan Board of Orthopaedics (MCQ
PAPER II)

1) A 45-year old man was given steroids after renal trans-


plant. After 2 years he had difficulty in walking and pain in
both hips. Which one of the following is most likely cause?
a- primary osteoarthritis. b- avascular necrosis.
c- tuberculosis. d- graft rejection.
e- septic arthritis
2) A 10 years old child, presents with predisposition to
fractures, anaemia, hepatosplenomegaly and a diffusely in-
creased radiographic density of bones. The most likely di-
agnosis is:
a- osteogenesis imperfecta. b- pyknodysostosis.
c- myelofibrosis. d- osteopetrosis.
e- secondary’s
3) A 9 years child, presents with scoliosis, hairy tuft in
the skin of back and neurological deficit. Plain x-ray reveals
multiple vertebral anomalies and a vertical bony spur over-
lying lumbar spine on AP view. the most probable diagnosis
is:
a- dorsal dermal sinus. b- diastomatomyelia .
c- tight filum terminale. d-caudal regression syndrome.
e- Brown Tumour
4) a symptomatic male has raised alkaline phosphatase 3
times of normal , liver function tests are (N) .the most likely
diagnosis will be:
a- metastatic bone disease. b- primary hyperparathyroidism.
c- paget’s disease. d- osteomalacia. e- osteoporosis

397
5) a 19- year old patient came with pain in his right fe-
mur. The x-ray showed a lytic lesion surrounded by sclerotic
zone in the diaphysis. The diagnosis is most probably:
a- Ewing’s sarcoma. b- osteoid osteoma.
c- osteoblastoma. d- osteoclastoma.
e- chronic osteomyelitis
6) A 10-year-old boy presents with fractures of humours
x-ray reveals a lytic lesion at upper end. Likely condition
is:
a- unicameral bone cyst. b- osteosarcoma.
c- osteoclastoma . d- aneurismal bone cyst.
e- hydatid cyst
7) a 45 years male presents with an expansile lesion in
the centre of femoral metaphysic . the lesion shows endos-
teal scalloping and punctuate calcifications.
Most likely diagnosis is: a- osteosarcoma.
b- chondrosarcoma. c- simple bone cyst.
d- fibrous dysplasia. e- secondary deposits
8) A 15 years old presents with a mass in the distal femur.
X-ray of the lesion showed features of codman’s triangle
and sun ray appearance. The diagnosis is:
a- osteoclastoma. b- Ewing’s sarcoma.
c- osteosarcoma. d- chondroblastoma.
e- callus formation
9) a 55 years fit gentleman has got recurrent dislocation
of hip joint. After THR, he has abductor deficiency. The ap-
propriate treatment in this case is:
a- trochanteric osteotomy with distal advancement.
b- closed reduction and hip immobilization.
c- resection arthroplasty.

398
d- revision surgery with a larger femoral head and a constrained PE liner.
e- arthrodesis.
10) a 62 years old gentleman who had undergone total
patellectomy previously is now undergoing TKR. The pre-
ferred prosthesis is:
a- posterior retaining prosthesis.
b- posterior cruciate substituting.
c- constrained non hinged.
d- constrained hinged.
e- all of the above
11) a young woman met with an accident and had mild
quadriparesis her lateral x-ray cervical spine revealed C5-
C6 fracture dislocation. Which of the following is the best
line of management?
a-immediate anterior decompression.
b- cervical traction following by instrument fixation.
c- hard cervical collar and bed rest.
d- cervical laminectomy.
e- Minerva Jacket
12) post polio myelitis , a patient has grade II power in
gastrocnemius, grade III in peroneus, grade IV in tibialis an-
terior. He is most likely to have which deformity:
a- calcaneo valgus. b- equinovarus.
c- calcaneovarus. d- genu valgus.
e- pes cavus
13) a previously healthy 45 years old labourer sudden-
ly develops acute lower back pain with right leg pain and
weakness of dorsiflexion of the right great toe. Which of the
following is true?
a- immediate treatment should include analgesic muscle relaxant
and back strengthening exercise.

399
b- appearance of foot drop indicate early surgical intervention .
c- if neurological signs fail to resolve within 1 week lumbar laminec-
tomy and excision of any herniated nucleus pulposus should be done.
d- if the neurological sign resolve within 2-3 weeks but low back
pain persists, the proper treatment would include lumbar fusion.
e- intralesional Steroid injection
14) a patient sustained injury to the upper limb three years
back: he now presents with valgus deformity in the elbow
and paresthesia over the medial border of the hand. The in-
jury is likely to have been:
a- supra condylar fracture humerus.
b- lateral condyle fracture humerus.
c- medial condyle fracture humerus.
d- posterior dislocation of the elbow.
e- fracture head of radius
15) in a 27 year old athlete which of the following is most
sensitive test for diagnosing ACL laxity :
a- anterior drawer test. b- pivot shift test.
c- lachman test. d- stein man test.
e- stress radiography test
16) a 44 year old man presents with acute onset of low
back ache and pain radiation to right lower limb . examina-
tion showed SLR restricted to 40° on right side. Cross SLR
was negative, weakness of EHL was present on the right
side, sensory loss in the first web space of right side. what
is the most likely diagnosis?
a- prolapsed intervertebral disc L 4-5
b- spondylolysis L 5-S1
c- lumbar canal stenosis
d- spondylolisthesis.
e- cauda equine syndrome

400
17) an 11 years old child weighting 70 kg presents with
restriction of abduction and internal rotation. There is ten-
derness in scarpa›s triangle . On flexing the hip limb is ab-
ducted. The most likely diagnosis is;
a- perthes disease. b- slipped capital femoral epiphysis
c- femoral hernia d- tuberculosis hip
e- saphena varesis
18) a 35 years old carpenter has pain around her radial
styloid , Finkesteins test is positive . treatment involves all
except :
a- rest in splint b- local hydrocortisone injection
c- oral steroids d- surgical release
e- non of the above
19) a 9 years old boy presents with a swelling in the upper
end of the tibia since 2 month . the swelling is mildly tender
and the local temperature is slightly raised .the patient is fe-
brile and leucocyte count is mildly elevated . the x-ray reveal
a radiolusency in the upper tibia with periosteal new bone
formation . the next investigation should be :
a- C.T scan of the lower limb b- technicium 99 bone scan
c- M.R.I d- bone biopsy e-bloodfilm
20) a 8 years old male child presents with mild pain and a an-
talgic gait following a trivial fall . the most likely diagnosis is:
a- fracture of the neck b- perthe›s disease
c- SCFE d- septic arthritis e- transient synovitis
21) a 4 year old male child presents with bilateral cavus
feet. General examination reveals a swelling and hairy ne-
vus on the low back. the most likely diagnosis :
a- spinal dysraphism b- cerebral palsy
c- charcot marie tooth disease d- friedreichs ataxia
e- residual poliomyelitis

401
22) an 18 years old boy while playing football, suddenly
twisted his knees and fell down. He got up after 10 minutes
and started playing again. Next day his knees were swollen
and he could not move it .the most probable cause is:
a- medial meniscus. b- ACL tear
c- MCL tear d- PCL tear
e- all of the above
23) a patient who presents with a burning pain presents
over the entire planter aspect of the foot , not relieved by
rest is most likely to suffer from:
a- peripheral vascular disease b- tarsal coalation
c- tarsal tunnel syndrome d- plantar fibro matosis
e- gout
24) a 40 years old man was repairing his wooden shed on
friday morning . by afternoon , he felt that the hammer was
becoming heavier . He has pain in the lateral side of elbow
and also found that squeezing water out of sponge hurt his
elbow. Most likely muscle group to be involved is :
a- biceps brachi and supinator
b- flexor digitorum superficials
c- extensor carpi radialis longus and brevis
d- triceps brachi and anconeus
e- brachioradialis and supinator
25) you are commencing a repair of acute rupture of Achil-
les tendon that occurred 8 days previously in a 32 years old
recreational tennis player . fibrillation of the tendon end is
noted. The following is most important to maximize the ulti-
mate outcome of the repair:
a- resection of the frayed end, and end-to-end apposition
b- interposition of plantaris tendon in the repair
c- repair of the tendon with the foot in slight equinvus

402
d- repair with tendon ends at normal resting tension
e-immobilization only with equines position
26) 22 yrs old footballer with recurrent shoulder disloca-
tion. What is the most likely cause?
a-Large engaging Hill-Sach lesion
b-Subscapularis deficiency
c-Labrum nonunion
d-Lateral capsule avulsion
e-Congenital glenoid
27) a 45 years old male presents with a medial compart-
ment osteoarthritis . he has 10° varus deformity and no in-
stability on clinical examination . after an adequate trail of
conservative treatment , the surgical procedure of choice is:
a- high tibial osteotomy
b- unicondylar knee arthroplasty
c- total knee arthroplasty
d- arthrodesis
e- Arthroscopic shaving
28) a 68 years old man presents with rib fracture , osteo-
penia and multiple soft tissue swelling . bone scan reveals
multiple hot spots in skull and vertebrae. most likely diag-
nosis is:
a- multiple myeloma b- metastasis
c- neuroblastoma d- myelofibrosis
e- Paget disease
29) an 8 years old boy presents with a progressively in-
creasing swelling and pain over the upper end of tibia of 6
month duration . radiographs show a lytic lesion with scle-
rotic margins of the upper tibial metaphysic. Most likely di-
agnosis is:
a- osteogenic sarcoma b- ewing’s sarcoma

403
c- brodies abscess d- osteoid osteoma
e- granulocytoma
30) a 15 years old boy has a destructive lesion in the dis-
tal femur with soft tissue extension. Needle biopsy shows a
high grade O.G.S CT scan of the chest is normal and techne-
tium scan shows involvement of only distal femur. What is
the surgical stage according to Ennekings clasific:
a- stage I b- stage II a c- stage II b
d- stage III e.stageIV
31)- A 22 – year –old medical student was slightly intoxi-
cated and fell onto his extended wrist while his forearm was
pronated. He has pain and clicking sensation on the ulnar
side of his wrist. X-ray and nerve conduction studies are
normal. The most likely diagnosis is?
a- Scapholunated dissociation.
b- Hook of hamate fracture.
c- Triangular fibrocartilage complex (TFCC) tear.
d- Piso-triquetral subluxation.
e- Extensor carpi ulnaris (ECU) subluxation.
32)- If a 28-year old male motor biker had a complex dis-
tal radius fracture (volar fixation required) and acute severe
carpal tunnel syndrome, which of the following surgical ap-
proaches would be correct?
a- Perform a Henry’s approach and a separate, very ulnar carpal
tunnel incision.
b- Observe the carpal tunnel syndrome for 48 hours after surgery.
c- Perform a Henry’s approach and a separate carpal tunnel inci-
sion.
d- Continue Henry’s approach across the wrist with an S curve and
decompress the carpal tunnel.
e- Continue Henry’s approach across the wrist and decompress
the carpal tunnel.

404
33)- A 56 year-old obese man presents with a painless
deterioration in bilateral hand function. Initially it was the
metacarpophalangeal (MCP) and proximal interphalangeal
(PIP) joints that were involved and now it is the distal inter-
phalangeal (DIP) joints. He has thick tight skin and a positive
prayer sign. The most likely disease is?
a-Gout. b-Osteoarthritis.
c-Rheumatoid arthritis. d-Scleroderma.
e-Diabetic cheirarthropathy.
34)- A 38-year-old man presents with dorsal wrist pain. He
has a stiff wrist with very limited range of motion and can’t
work as a mechanic. Plain films reveal grade IV Kienbock’s
disease. He should be treated with?
a-Proximal row carpectomy. b-Wrist replacement.
c-Curettage and vascularized pronator quadrates graft.
d-Radial shortening. e-Wrist arthrodesis.
35)- A 41-year old man sustained a distal radius fracture
whilst hiking in the Green Mountin. He was treated in plaster
by a local doctor and went on to malunion. He presents with
ulnar sided pain and on examination he impacts on the ulnar
side, with a negative grind test at the distal radioulnar joint
(DRUI). His best treatment would be?
a-Ulnar shortening osteotomy.
b-Darrach procedure.
c-Sauve-Kapandji procedure.
d-Arthroscopic debridement of DRUI.
e-Distal ulnar head implant arthroplasty.

405
36)- A 17-year-old snowboarder fell onto his outstretched
pronated hand. He presents with on going ulnar-sided wrist
pain. He is tender over the ulnar fovea and has no click. The
distal radioulnar joint (DRUI) is stable. Plain films are normal
and a MR arthrogram show a triangular fibrocartilage com-
plex (TFCC) defect adjacent to the ulna. How is this classi-
fied according to the Palmer classification?
a- Class 2A lesion. b-Class IA lesion.
c-Class 2B lesion d-Class IB lesion.
e-Class IC lesion.
37)- A 16-year-old girl had multiple fracture in her forearm
and hand. One year later after fracture healing she presents
with trouble gripping things. When the metacarpapophalan-
geal (MCP) joint is extended you cannot passively flex the
proximal interphalangeal (PIP) joint. When the MCP joint is
flexed it is possible to passively flex the PIP joint. Her exten-
sors are at a good length. Which of the following is incor-
rect?
a- She has an intrinsic plus hand.
b- She has a claw hand.
c- She has a positive Bunnell test.
d- She has a likely positive Bouviere effect.
e- There is an imbalance between the intrinsic and extrinsic muscles.
38)- A 23-year-old Goalkeeper had an avulsion of the
flexor digitorum profundus (FDP) tendon of his ring finger.
This was diagnosed early and despite proximal migration he
had it reinserted with a button technique. Six months later
he complains that he can’t close his fingers tightly over the
ball. This problem is?
a- Lumbrical plus effect. b-Swan neck deformity.
c-Quadrigia effect. d-Intrinsic tightness.
e-Chronic mallet finger.

406
39)- A 23-year-old was intoxicated at a wedding and fell
through a glass window. He presents to the emergency de-
partment with a radial wrist laceration with arterial bleeding.
With regards to the timing of surgery the major blood supply
to the hand is provided by which of the following?
a-Deep branch of the radial artery. b-Radial artery.
c-Deep palmar arch. d-Superficial palmar arch.
e-Interossous artery
40)- A41-year-old man presents with a swelling at the level
of his distal interphalangeal ( DIP ) joint on his right middle
finger. What is the most likely diagnosis?
a-Epidermoid cyst. b-Keratoacanthoma.
c-Mucoid cyst. d-Epithelioid tumour.
e-Sebaceous cyst.
41)- All of the following contribute to the wrist and hand
deformity in rheumatoid arthritis except?
a- Volar subluxation of the extensor carpi ulnaris ( ECU ).
b- Radio-scapho-capitate ligament failure.
c- Scaphoid extension.
d- Supination of the carpus on the forearm.
e- Distal radioulnar joint ( DRU ) destruction.
42)- A 13- year-old boy is referred to you after a trivial fall
onto his elbow. Radiographs reveal a dislocated radial head.
He does not have much pain.His mother says she has al-
ways had joint pains with abnormal knee caps. She keeps
pointing to her knees in an excited manner with long fake
nails. The most likely diagnosis is?
a- Marfan syndrome. b-Monteggia Bado injury.
c-Generalized ligamentous laxity.
d-Ehlers-Danlos syndrome. e-Nail patella syndrome.

407
43)- A 43-year-old woman presents with decreased digital
flexion and an injury in Zone 2 of her left hand. On explora-
tion what percentage laceration of the flexor tendon would
you repair.
a-40% b-25% c-45% d-50% e-35%
44)- A football player sustains a suspected acromiocla-
vicular joint ( ACJ ) separation.
Which of the following is the most appropriate radio-
graphic view to evaluate the ACJ?
a-Stryker notch view. b-Serendipity view.
c-Zanca view. d-Supraspinatus outlet view.
e-Garth view.
45)- A 55- year- old patient has chronic pain over the later-
al aspect of the elbow, exacerbated when playing backhand
tennis stroke. On examination she has pain with resisted
middle finger extension. Which muscle attachment is most
likely involved?
a- Brachioradialis. b-Extensor digitorum communis.
c-Extensor carpi radialis longus.
d-Extensor carpi radialis brevis. e-Anconeus.
46)- A 31- year-old weightlifter reports right shoulder pain
with cross-body adduction
a- As well as point tenderness at the acromioclavicular joint ( ACJ
).X-rays show osteopaenia of the distal clavicle. Initial treatment
should include?
b- Glenohumeral joint injection.
c- Periscapular muscle strengthening.
d- Axtivity modification.
e- Capsular release.
f- Arthroscopic resection of the distal clavicle.

408
47)-A patient is known to have a SLAP tear. An MRI shows
a large cyst in the spinoglenoid notch. What additional find-
ing on examination is the patient likely to display?
a- Weakness in external rotation.
b- Weakness in forward elevation.
c- Weakness in internal rotation.
d- Medial scapular winging.
e- Lateral scapular winging.
48)-A 50-year-old male complains of acute shoulder pain
and an inability to lift his arm over his head after an anterior
shoulder dislocation. Examination reveals active forward el-
evation to 30ْ and grade 3/5 external rotation strength. An ar-
throgram shows extravasation of the dye into the subacro-
mial space with no evidence of arthritis. What is the most
appropriate treatment option?
a- Shoulder hemiarthroplasty.
b- Rotator cuff repair.
c- Reverse total shoulder arthroplasty.
d- Total shoulder arthroplasty .
e- Physiotherapy.
49) A 35-year-old woman sustains an elbow fracture dis-
location which includes a coronoid fracture involving more
than 50% and a comminuted radial head fracture. What is
the most appropriate treatment?
a- Closed reduction and early range of motion.
b- Radial head resection and coronoid open reduction internal
fixation.
c- Radial head arthroplasty and coronoid open reduction internal fixation.
d- Radial head resection and lateral collateral ligament reconstruction.
e- Radial head arthroplasy, coronoid open reduction internal fixa-
tion and lateral collateral ligament repair.

409
50) A 46-year old man presents to the clinic with severe
back pain. All of the following are consistent with non-or-
ganic signs except?
a- Discrepancy between findings on supine and sitting straight leg
raising tests.
b- Disproportionate facial expressions or tremor during examina-
tion.
c- Low back pain on passive rotation of shoulders and pelvis in the
same plane.
d- According to Waddell et al, non-organic signs should be re-
quired with malingering or the presence of a psychological problem.
e- Non-dermatomal sensory loss.
51) A 32-year-old man presents with a 2 month history of
back and right-sided leg pain. He walked with a right Tren-
delenburg gait. The most likely diagnosis is?
a- An ipsilateral paracentral disc herniation at L3-L4.
b- An ipsilateral paracentral disc herniation at L5-S1.
c- An ipsilateral far lateral disc herniation at L4-L5.
d- An ipsilateral far lateral disc herniation at L5-S1.
e- An ipsilateral forminal disc herniation at L4-L5.
52) A 58-year-old lady with rheumatoid arthritis (RA) pres-
ents with neck pain and occipital headache. Which of the
following is true regarding her condition?
a- Atlantoaxial subluxation occurs in 25% of cases of RA as the re-
sult of pannus formation.
b- An anterior atlantodens interval (ADI) of more than 3.5 mm on
flexion-extension views indicates instability and an absolute indica-
tion for surgery.
c- A space available for the cord (SAC) of less than 14 mm or an ADI
of more than 9-10 mm is an indication for spinal stabilization.
d- A Ranawat (CI-C2) index of <17 mm is consistent with basilar invagination.
e- A subaxial subluxation of >4 mm is not indicative of cord compression.

410
53) A 12-year-old girl with scoliosis was found to have a
fluid cavity within the spinal cord on a routine preoperative
MRI scan. All of the following are true regarding the spinal
cord finding except?
a- Craniocervical junction abnormalities are predisposing factors.
b- Scoliosis is reported in 25-85% of Syringomyelia cases.
c- Scoliosis with syringomyelia has been characterized by 44-50%
incidence of left thoracic curves.
d- Proprioception and vibration sensation are usually not affected.
e- Decompression of the syrinx in patients above the age of 10
years will improve or stabilize the scoliotic curve.
54) A 33-year-old male is involved in a road traffic acci-
dent sustaining a fracture dislocation of the cervical spine.
He has absent motor function absent sensation and tone.
The bulbocavernous reflex is intact. Which of the following
best describes this spinal cord injury pattern?
a-Central cord syndrome. b-Incomplete spinal cord injury.
c-Complete spinal cord injury. d-Neurogenic shock.
e-Spinal shock.
55) A 20-year-old cyclist was hit by a car sustaining a spi-
nal cord injury. He has an MRC 5 in his deltoid and biceps,
MRC 0 in his wrist extensors, flexors and triceps. He has an
absent anal tone and perianal sensation. He has absent tone
and power in his lower limbs. How would you define his pa-
tient’s neurological injury?
a-Incomplete C5. b-Complete C5.
c-Complete C6 d-Incomplete C6.
e-Incomplete C7.
56) A 19-year-old medical student presents with a Scheuer-
mann’s kyphosis in the thoracic spine with a cob angle of 85*
between T5 and T12. All of the following are correct except?
a- The condition is often accompanied by a lumbar hyperlordosis.

411
b- This is the most common cause of thoracic back pain in adolescents.
c- An MRI scan should be part of a routine preoperative workup.
d- A posterior instrumentation should stop at the distal most tilted
vertebra.
e- At surgery, flavectomy (ligamentum flavum excision) should be
performed at the apex of the curve.
57) A 29-year-old restrained front seat passenger was in-
volved in a road traffic accident, sustaining a flexion distrac-
tion injury of L1. Which of the following is true regarding this
fracture?
a- Gastrointestinal injuries occur in 25% cases.
b- A bony chance fracture has a high non-union rate compared to
the ligamentous injury.
c- A ligamentous chance fracture should be treated using a distrac-
tion construct with three levels above and two levels below the fracture.
d- A ligamentous chance fracture should be treated using a compres-
sion contruct with one level above and one level below the fracture.
e- It is commonly associate with neurological deficit.
58) A 10-year-old girl presents with a painful scoliosis.
Radiographs showed a 35* right thoracic scoliosis with a ra-
diolucent nidus <1 cm at the apex of the curve. Which of the
following is incorrect regarding this spinal lesion?
a- Pain is usually due to prostaglandin found in the nidus.
b- CT scan is the investigation of choice to identify the nidus.
c- Bone scan is always hot with intense uptake.
d- Medical management with one-steroidal anti-inflammatory
drugs (NSAIDs)is a satisfactory treatment modality in this case.
e- The natural history of this condition is that of spontaneous reso-
lution.
59) A50-year-old man presents with difficulty mobilizing
and clumsiness buttoning his shirt. He had a fixed cervical
kyphosis of 15%. An MRI scan showed a central disc hernia-

412
tion at C5-C6 with signal changes within the cord. What is
the next appropriate management step?
a- Observation.
b- Posterior decompression.
c- Posterior decompression and fusion.
d- Anterior cervical decompression and fusion.
e- Posterior osteotomy to correct the kyphosis followed by poste-
rior fusion.
60) A patient is to have primary total hip replacement and
takes methotrexate for rheumatoid arthritis. Methotrexate
should be?
a- Stopped 24 hours before surgery.
b- Stopped 1 week before surgery.
c- Reduced to half the dose but not stopped.
d- Continue as usual.
e- Gradually reduced to stop 24 hours before surgery and then
restarted 2 weeks following surgery.
61) A 72 year-old patient is suspected to have an infected
total hip replacement, rather than aseptic loosening, 8 years
following surgery. Which of the following would be a useful
investigation to differentiate between them?
a-Radio-labelled white cell scan. b-Radionuclide bone scan.
c-Plain X-ray. d-Hip aspiration.
e-Ultrasound scan of the hip joint.
62) A 27-year-old patient presents with groin pain and
clicking. He has a history of mild developmental dysplasia
of the hip (DDH) as a child. Which of the following is the
most likely finding on a plain radiograph?
a-Acetabular osteophyte.
b-Acetabular cyst.
c-Femoral head subarticular cysts.

413
d-Femoral head/neck junction prominence.
e-Femoral neck stress fracture.
63) a 74-year-old patient has developed degenerate
change in her hip requiring total hip replacement. She has
previously had a pertrochanteric femoral fracture fixed with
a dynamic hip screw device. What is the correct surgical
management?
a- Do not perform total hip replacement due to poor outcome.
b- Remove metalwork, use strut allograft and cable plate to rein-
force femur, then use fully coated uncemented stem.
c- Remove the sliding screw only, and perform hip resurfacing to
prevent risk of diaphyseal fracture.
d- Remove metalwork, use strut allograft and cable plate to rein-
force femur, then use cemented stem.
e- Remove metalwork, insert cemented stem passing two cortial
thicknesses below lowest screw hole.
64) A 40-year-old patient is developing avascular necrosis
of the femoral head. The contour is normal (i.e, no collapse).
Although structural changes are evident on MRI. What is the
preferred treatment?
a- Hip resurfacing arthroplsty.
b- Non-vascularized fibular graft.
c- Low dose steroid for 18 weeks.
d- Core decompression.
e- Vascularized fibular graft.
65) After cementing in a total knee replacement for a val-
gus knee, you find that it remains tight laterally in extension.
The next most appropriate step is to?
a- Carry out a medical release.
b- Carry out a medical release and increase the size of the polyeth-
ylene insert.

414
c- Release the iliotibial band.
d- Release popliteus.
e- Decrease the size of polyethylene insert.
66) A 34-year-old man presents with a 3 month history of
knee pain, with an inability to squat. The most likely diagno-
sis is?
a- Primary osteoarthritis
b- Osteochondral defect.
c- Loose body.
d- Posterior horn meniscal tear.
e- Pigmented villonodular synovitis.
67) A 71 year-old patient presents with increasing knee
pain 6 weeks after having a total knee replacement. Which
of the following would be most reliable in the diagnosis of
infection?
a- Erythrocyte sedimentation rate.
b- C-reactive protein.
c- Microscopy and culture of joint aspirate.
d- Triple phase isotope bone scan.
e- Magnetic resonance imaging.
68) An 81-year-old lady is reviewed 6 months after a total
knee replacement. Her only complaint is some numbness on
the outer aspect of her midline scar. The most likely cause
of this is?
a- Injury to the femoral nerve from the tourniquet.
b- Division of the lateral femoral cutaneous nerve.
c- Injury to the anterior femoral cutaneous nerve from the toqniquet.
d- Division of the infrapatellar branch of the saphenous nerve.
e- Injury to the lateral femoral cutaneous nerve from the tourniquet.

415
69) A 62-year-old man presents with a painful snapping
sensation when extending his knee, 6 months after a poste-
rior stabilized total knee replacement. The most appropriate
treatment is?
a- Bracing.
b- Topical anti-inflammatory ges.
c- Revision of patellar component.
d- Arthroscoppic or open debridement.
e- Revision of femoral component.
70) A 58-year-old man is listed for a total knee replace-
ment. He underwent a closing wedge high tibial osteotomy
10 years prior. The most likely problem one would encounter
during the total knee replacement is?
a- Difficult surgical exposure.
b- Lateral ligament laxity.
c- Difficult tibial stem replacement.
d- Non-union of the osteotomy.
e- Patella baja.
71) A 22-year-old man has an arthroscopy 1 year after
microfracture treatment for a full-thickness chondral defect.
The defect has filled and biopsy is taken. This is most likely
to show?
a-Hyaline cartilage. b-Fibrocartilage. c-Cancellous bone.
d-Cortical bone. e-Elatin.
72) An active 66-year-old man is reviewed 1 year after a to-
tal knee replacement. He complains that it does not feel right
and clinical examination identifies an incompetent medical
collateral ligament. The most appropriate treatment is?
a- Anti-inflammatory gel and tablets.
b- Hinged knee brace.
c- Open repair of the medial collateral ligament.

416
d- Reconstruction of the medial collateral ligament.
e- Revision to a constrained knee prosthesis.
73) A 32-year-old man sustains a Lisfranc fracture dislo-
cation. Which of the following is the most important factor in
predicting a satisfactory outcome?
a-Severity of initial injury.
b-The state of the articular cartilage.
c-The age of the patient.
d-The smoking status of the patient.
e-Whether or not a compensation claim is involved.
74) A 13-year-old girl who enjoys dancing presents with
a painful big toe whilst performing. The likely diagnosis is?
a-Hallux valgus. b-Hallux rigidus.
c-Sesamoid fracture. d-Turf toe.
e-Extensor hallucis longus rupture.
75) A 27-year-old banker injures his foot and sustains a
displaced divergent Lisfranc fracture-dislocation. The opti-
mal management would consist of?
a- Below knee plaster cast.
b- A bridging external fixator.
c- Closed or open reduction and k-wire stabilization.
d- Closed or open reduction and screw stabilization.
e- Closed or open reduction and combined screw and K-wire sta-
bilization.
76) A 22-year-old man injures his ankle after a twisting
injury whilst snowboarding. He has been diagnosed with a
ligament injury, but 8 weeks after injury has continued lat-
eral pain. The most likely diagnosis is?
a- Lateral malleolus fracture.
b- Fracture of the lateral process of the talus.

417
c- Fracture of the body of the talus.
d- Fracture of the neck of the talus.
e- Peroneus longus rupture.
77) An 11-year-old boy presents with a wound on the sole
of his foot after stepping on a nail in the garden, whilst wear-
ing training-shoes. Which organism would you be most con-
cerned about?
a-Staphylococcus aureus. b-Staphylococcus epidermidis.
c-Pseudomonas aeruginosa. d-Pasteurella multocida.
e-Beta-haemolytic streptococci.
78) A 29-year-old male tennis player presents after an an-
kle injury. He describes feeling as if someone kicked him in
the back of the ankle. What is the most important benefit of
surgical treatment for this patient?
a-Quicker return to sport.
b-Decreased risk of chronic pain.
c-Decreased risk of re-injury.
d-Decreased risk of nerve damage.
e-Decreased immobilization time.
79) A 23-year-old presents with on-going pain and stiff-
ness four months after a severe ankle sprain. A radiograph
shows a Berndt and Harty type IV lesion of the lateral talar
dome. The optimal management would be?
a- Below knee weight bearing plaster cast or protective boot.
b- Below knee non-weight bearing plaster cast or protective boot.
c- Arthroscopy, excision and microfracture.
d- Arthroscopy and internal fixation.
e- Open reduction and internal fixation.

418
80) A 12-year-old boy presents with a painful flatfoot. Lat-
eral radiographs show an “antreater” sign. The most likely
diagnosis is?
a-Plantar fasciitis. b-Accessory navicular
c-Tibialis posterior rupture. d-Talocalcaneal coalition.
e-Calcaneonavicular coalition.
81) A 12-year-old boy presents with a 2 month history of
right knee pain after a fall. He has lost 3 kg. in weight but
is otherwise well. He is pale, apyrexial and his right knee
is slightly swollen and warm on examination. Plain radio-
graphs reveal areas of dense sclerosis admixed with areas
of radiolucency in the distal femoral methaphysis. Aggres-
sive periosteal new bone formation is also noted. Which of
the following is the most likely diagnosis?
a- Parosteal osteosarcoma.
b- Periosteal osteosarcoma.
c- High- grade intramedullary osteosarcoma.
d- Telangiectatic osteosarcoma.
e- Osteomyelitis.
82) A 28-year-old female presents to your clinic with pro-
gressively increasing pain in her left wrist. She has also re-
cently been having repeated episodes of abdominal discom-
fort, nausea and vomiting. A plain radiograph of the wrist
reveals an eccentrically placed lytic lesion in the metaphys-
ic and epiphysis with thinning of the cortex. You suspect a
giant cell tumour of bone. What is the most appropriate next
step in the management of this patient?
a- Perform a bone biopsy.
b- Curettage alone.
c- Curettage and phenolization.
d- Currettage, high-speed burr, cement and bone graft.
e- Check serum parthyroid hormone (PTH) and calcium.

419
83) A 55-year-old male is diagnosed with a dedifferenti-
ated chondrosarcoma of the femur which appears on MRI to
have a significant extraosseous component. Distant staging
has not revealed any metastases. What surgical stage would
be assigned to this tumour according to the system of the
Musculoskeletal Tumour Society (MSTC)?
A- IA. B- IB. c- IIA. d- IIB. E- IIIB.
84) A 17-year-old boy presents with a haemarthrosis in the
left knee. There is no history of trauma. His parents mention
that they have recently noted their son having protracted
bleeding from minor cuts. The patient also reports a long
history of abdominal pain and diarrhea. On examination, the
patient is of short stature and appears emaciated. Abdomi-
nal examination is unremarkable but an anal fissure is not-
ed. Blood tests show a microcytic anaemia with a prolonged
prothromben time (PT) and a prolonged activated partial
thromboplastin time (APTT). Further evaluation of clotting
factors is likely to show low levels of which of the following?
a-Factors II, V, VII, IX and X. b-Factor VIII.
c-Factor IX. d-Von Willebrand factor.
e-Factors II, VII, IX and X.
85) A 10-year-old boy presents with difficulty rising from a
crouching position. On examination he is noted to be obese,
hypertensive and has multiple small bruises on his limbs.
Which of the following investigations is most appropriate to
yield the likely diagnosis?
a-Thyroid function tests.
b-Serum creatine phosphokinase and muscle biopsy.
c-Urine cortisol. d-Serum glucose.
e-MRI pituitary gland.
86) A 17-year-old boy sustains multiple fractures of his
limbs duting his first fill season playing in the youth team for
a premier league football club. On examination, he is noted

420
to have hepatomegaly. Radiographs of his lower limbs show
dense sclerosis of his femora and tibiae with enlarged flask-
shaped metaphyses. Which one of the following statements
is true with regard to the underlying diagnosis?
a- The autosomal dominant form can lead to death in infancy.
b- The number of osteoclasts are usually reduced.
c- The disorder may result from a defect in the thyroid.
d- The lacunae are characteristically empty.
e- Albers-Schonberg disease is the autosomal recessive form.
87) A 27-year-old male presents with a 1 year history of
progressive pain and swelling in the tibia. Examination re-
veals anterior bowing of the tibia. Radiographs show mul-
tiple well-circumscribed lucent lesions separate by sclerotic
bone in the tibial diaphysis. Histology shows epithelial-like
cells in aglandular pattern on a background of fibrous stro-
ma. What is the recommended treatment for this condition?
a- Close observation.
b- Internal fixation and bone grafting.
c- Chemotherapy, wide surgical resection and limb salvage.
d- Wide surgical resection and limb salvage.
e- Extracorporal radiotherapy.
88) A 30-year-old male present with a bony growth arising
from the proximal phalanx of his left middle finger which ap-
peared 3 months ago and is steadily increasing in size, and
now causing discomfort. He denies a history of trauma. Ra-
diographs demonstrate an irregular bony mass arising from
the dorsolateral surface of the proximal phalanx. The matrix
of the lesion contains mature bone. What is the diagnosis?
a-Osteochondroma. b-Nora’s disease.
c-Periosteal chondroma. d-Parosteal osteosarcoma.
e-Myositis ossificans.

421
89) A 70- year-old male presents with weight loss, fatigue
and back pain. Examination reveals hepatomegaly and axil-
lary lymphadenopathy. Blood tests show a raised calcium and
erythrocyte sedimentation rate. Serum protein electrophoresis
reveals an M-protein spike, with an elevated serum IgM level.
Urinary Bence-Jones proteins are detected. Radiographs show
lytic lesions in the T6, T8 and T9 vertebrae. These are cold on
bone scan. Biopsy reveals intensely eosinophilic plasma cells.
Which of the following is the most likely diagnosis?
a- Multiple myeloma.
b- Plasmacytoma.
c- POMS ( polyneuropathy, organomegaly, endocrinopathy,
monoclonal gammopathy and skin changes ) syndrome.
d- Waldenstrom macroglobulinaemia.
e- Monoclonal gammopathy of undetermined significance.
90) A 31-year-old male presents with stiffness and pain in
his back and hips. Examination reveals a bluish-grey discol-
ouration of his ear cartilage and sclera and decreased range
of movement in his hips. His urine is noted to be black. Ra-
diographs of jis lumbar spine show multilevel disc degen-
eration. This patient most likely has a deficiency of which of
the following enzymes?
a-Muscle phosphorylase. b-Hexosaminidase A.
c-Cystathione B-synthase.
d-Hemogentisic acid oxidase
e-B-galactosidase.
91) A 2-year-old infant presents with seizures and hair
loss. He is noted to have a positive Chvostek’s sign and an
electrocardiogram shows a prolonged QT interval. His par-
ents say that he has also suffered from multiple infection
since birth due to a T-cell deficiency. This child’s syndrome
is associated with failure of the development of which of the
following embryonic structure?

422
a-Neural crest. b-Third and fourth pharyngeal pouches.
c-Rathke’s pouch. d-Foramen caecum.
e-Urogenital ridge.
92) A supracondylar fracture of the distal humerus with
posterolateral displacement should be reduced by perform-
ing reduction manoeuvres in the following order?
a-Valgus – Flexion - Pronation.
b-Varus – Extension – Supination.
c-Valgus – Extension – Pronation.
d-Traction – Pronation – Flexion.
e-Varus – Extension – Pronation.
93) a previously healthy 41-year-old man suffers a mini-
mally displaced distal radius fracture and is treated in a cast
for 4 weeks. He presents 14 weeks later with dorsal wrist
pain. What is the most likely diagnosis?
a- Rupture of the extensor pollicis longus (EPL) tendon.
b- Rupture of the extensor indicis proprius (EIP) tendon.
c- Missed scaphoid fracture.
d- De Querrvain’s tenosynovitis.
e- Arthritis of the first carpometacarpal joint.
94) An 86-year-old man falls and sustains a minimally dis-
placed proximal humerus fracture. What is the best way to
manage him?
a- Physiotherapy and passive range of motion, 10 days following
the injury.
b- Immobilization for 4 weeks in a sling.
c- Physiotherapy the following day, with range of motion exercises.
d- Intramedullary nail fixation.
e- Urgent open reduction and locking plate fixation.

423
95) A 30-year-old woman is involved in a road traffic ac-
cident and is found to have a pelvic symphysis separation
of 4 cm and a sacral fracture. She undergoes a normal sec-
ondary survey and is haemodynamically stable. Definitive
fixation should involve which of the following?
a- Skeletal traction for 3 months.
b- Internal fixation of the symphysis pubis with anterior external
fixation.
c- Internal fixation of the symphysis pubis and internal fixation of
the sacrum.
d- Posterior only external fixation.
e- Anterior only external fixation.
96) An 11-year-old girl sustains a closed femoral shaft
fracture., which is then treated with an anterograde intra-
medullary nail via a piriformis fossa entry point. In follow-up
she is noted to have collapse of the femoral head. This is
most likely due to ?
a- Missed concurrent subcapital fracture.
b- Infection.
c- Pressure changes within the capsule of the hip joint.
d- Perthes disease.
e- Injury to the lateral ascending vessels of the femoral neck.
97) A 31-year-old woman has fracture-dislocation at C5-
C6. A clavicle fracture is noted on an otherwise normal chest
X-ray. Her pulse is 45, blood pressure is 83/40 mmHg. And
respiratory rate is 28. An abdominal ultrasound is negative.
What type of shock is most likely in this patient?
a-Haemorrhagic. b-Septic. c-Neurogenic
d-Spinal. e-Tension pneumothorax.

424
98) A 6-year-old has a posteriorly displaced supracondy-
lar fracture, with absent pulses, but a warm, pink hand. What
is the optimal management?
a- Immediate reduction of the fracture in the emergency department.
b- Open reduction with immediate exploration by a vascular sur-
geon’s.
c- Closed reduction in theatre, with reassessment of the vascularity.
d- Urgent brachial angiography.
e- Exploration by a vascular surgeon followed by external fixation.
99) A 20 yrs old lady presented with painful high arched
foot and hammer toes . Wt bearing x-rays of the foot lateral
view showed Meary’s angle of 20degrees and calcaneal pich
angle is 45 degrees. Which type of pes cavus is this?
a- physiological pes cavus b- Plantaris
c- cavo-varus d- Calcaneus
e- roker-buttom
100) A 43yrs old man was diagnosed to have osteochon-
dritis dessicans medial femoral condyle. He requires scru-
tiny of the other knee also because of the chance of bilateral
lesion is :
a- 5% b- 10% c- 20% d- 25% e-
non of the above

425
ANSWERS:

1-b 2-d 3-b 4-c 5-a 6-a 7-b 8-a 9-a 10-b 11-
b 12-c 13-b 14-b 15-c 16-a 17-b 18-c 19-c 20-e
21-a 22-a 23-c 24-c 25-a 26-a 27-a 28-a 29-c 30-
d 31-c 32-c 33-e 34-e 35-a 36-d 37-b 38-c 39-d
40-c 41-c 42-e 43-d 44-c 45-d 46-c 47-a 48-b
49-e 50-d 51-d 52-c 53-e 54-c 55-b 56-d 57-d
58-d 59-d 60-d 61-d 62-d 63-e 64-e 65-c 66-d
67-c 68-d 69-d 70-e 71-b 72-e 73-e 74-d 75-e 76-
b 77-c 78-c 79-c 80-e 81-c 82-e 83-d 84-e 85-c
86-d 87-d 88-b 89-d 90-d 91-b 92-b 93-a 94-a
95-c 96-e 97-c 98-c 99-c 100-d

426
SICOT DIPLOMA EXAMINATION:

1-Risk factors for Developmental Dysplasia of the Hip


(DDH) include all of the following except:
a-Breech positioning b. Male sex
c-Positive family history d. Being first-born child
e-Decreased intra uterine space
2-In newborns and infants up to 6 months of age diag-
nosed with developmental dysplasia of the hip, which of the
following treatment options is most appropriate?
a-Application of a Pavlik Harness
b-Closed reduction and application of a Pavlik Harness
c-Closed reduction and application of a hip spica
d-Open reduction
e. Open reduction and application of a Pavlik Harness
3-What is the major risk associated with both open and
closed reductions in developmental dysplasia of the hip?
a-Vascular injury b. Nerve injury
c. Pain d. Stiffness e. Infection
4- Which zone of the growth plate is affected in Achondro-
plasia?
a-Zone of Maturation b. Zone of Provisional Calcification
c-Reserve Zone d. Hypertrophic Zone
e. Proliferative Zone
5-Which zone of the growth plate is affected in Ricketts?
a-Zone of Maturation b. Zone of Provisional Calcification
c-Reserve Zone d. Zone of Degeneration
e. Proliferative Zone

427
 6-Which zone of the growth plate is affected in Acute
Haematogenous Osteomyelitis?
a-Primary Spongiosa b. Hypertrophic Zone
c-Reserve Zone d. Secondary Spongiosa
e. Proliferative Zone
 7-The Centre Edge Angle (CEA) of Wiberg measurement
is useful for the assessment of which condition?
a-Slipped upper femoral epiphysis b. Perthes disease
c-Developmental dysplasia of the hip
d. Adolescent idiopathic scoliosis
e. Supracondylar fracture of the humerus
8-Which of the following congenital hand anomalies re-
quires removal of supernumerary digits and is commonly
called mirror hand?  
a-Triphalangeal thumb b. Ulnar dimelia
c-Apert syndrome d. Postaxial polydactyly
e-Camptodactyly
​ 9-Which of the following congenital hand anomalies re-
quires removal of supernumerary digits, rarely occurs as a
solitary deformity, and is usually associated with complex
syndactyly?
a-Central polydactyly b. Bifid thumb
c-Clinodactyly d. Preaxial polydactyly
e- Congenital ring syndrome
10-Which of the following congenital hand anomalies require
removal of supernumerary digits and is further subdivided into
3 groups according to Stelling and Turek Classification?
a-Ulnar dimelia b. Apert syndrome
c-Postaxial polydactyly d. Camptodactyly
e. Bifid thumb

428
​11-Congenital short femur is associated with the follow-
ing, except:​
a-Anterior cruciate deficiency b. Anterolateral femoral bow
c-Fibular hemimelia
d. Varus deformity of the knee and patella subluxation
e-Femoral Retroversion
12-The most common cause of in-toeing in the pre-school
age group is:
a-Developmental Dysplasia of the Hip b. Metatarsus Adductus
c-Internal Tibial Torsion d. Excessive Femoral Anteversion
e-Clubfoot
13-Southwick angle (epiphyseal-shaft angle) measure-
ment in case of a slipped capital femoral epiphysis (SCFE)
is used for which of the following?
a- For prognosis for AVN
b- Establish the presence/absence of a slip
c- To find out the aetiology of a slip
d- Determine the severity of the slip
e- Duration of the slip
14-A 4-year-old healthy boy who is the product of normal ver-
tex pregnancy and delivery presents to your office with a chief
complaint of bilateral in-toeing. On prone positioning, he has a
thigh foot axis of 10 degrees and bilateral internal rotation of fe-
murs of 80 degrees and external rotation of 20 degrees. Which of
the following is the next step in the management of this patient?
a- AP pelvis radiograph to rule out acetabular dysplasia
b- Educate the family that the natural history is favourable and re-
assess as needed
c- Show the family proper sitting posture in order to avoid W-sitting
d- Femoral derotational osteotomies
e- Twister cables for internal tibial torsion

429
15-Marfan syndrome is most likely associated with de-
fects in which of the following structural proteins?
a-Fibronectin b. Elastin c. Type IX collagen
c-Type VIII collagen e. Fibrillin
16-A 5-year-old male child presents with a pathologic
fracture of the proximal humerus. Which of the following tu-
mours are most likely to be the cause?
a-Aneurysmal bone cyst b. Ewing sarcoma
c-Fibrous cortical defect d. Osteosarcoma
e-Unicameral bone cyst
17-Non-union following a paediatric lateral condyle frac-
ture has been associated with which of the following?
a-Restriction in pronation at elbow
b. Median nerve palsy
c-Radial nerve palsy
d. Ulnar nerve palsy
e-Cubitus varus
18-In patients with clubfeet treated with soft-tissue re-
lease, which of the following variables shows the greatest
correlation with long-term functional impairment?
a- Frequency of cast application
b- Tibialis anterior muscle function
c- Subtalar joint function
d- Extent of soft-tissue release
e- Duration of cast treatment
19-The cavus deformity in Charcot-Marie-Tooth disease is
caused by what muscular imbalance?
a- Peroneus longus overpowering tibialis anterior
b- Peroneus brevis overpowering peroneus tertius
c- Tibialis anterior overpowering tibialis posterior

430
d- Achilles overpowering tibialis posterior
e- Extrinsic toe flexors overpowering intrinsics
20-A 6-year-old female sustains a distal femoral physeal
fracture that is reduced, pinned and eventually heals after 6
weeks in a cast. In your follow-up radiographs of this patient,
you notice that a portion of the physis is no longer visualized.
You order a CT scan to delineate the problem and plan your
surgical approach. Which of the following scenarios is true?
a- A central bony bar making up 70% of the physis is appropriate
for resection
b- A peripheral bony bar making up 70% of the physis is appropri-
ate for resection
c- A central bony bar making up 10% of the physis with femoral
valgus of 20 degrees is appropriate for resection
d- A peripheral bony bar making up 20% of the physis with 5 de-
grees of valgus is appropriate for resection
e- Children less than 8 years old can usually overcome a physeal
bar of less than 15%

Answers:

1-b 2-b 3-a 4-e 5-b 6-b 7-c 8-b 9-a 10-
c 11-d 12-c 13-d 14-b 15-e 16-e 17-d 18-d 19-a
20-d

431
REFFRENCES AND FURTHER READINDS

1-American Academy of Orthopaedics Association(AOAA)


MCqs Books 1995-2013.
2- Exam corner, Journal of Bone &joint Surgery (JBJS)
different volumes.
3- Orthopaedics Quick Review, Apurv Mehra. Second Edi-
tion 2013
4- Orthopaedic In Training Examination-OITE Review
Questions 2006-2013.
4- Postgraduate Orthopaedics MCQ for FRCS (TR&ORTH),
Kesavan Sri-Ram.
5- Jaypee’s Review of Orthopaedics, Nadeem Ashraf.
6-Australian Fellowship examinations , Sydeny.

ACKNOWLEDGEMENTS

First of all I must say ( Al-HAMDUOLILLAH) with God’s


help to fulfill my ambition to write this book. I offer my sincer
thanks to all of my collegues in Orthopaedic Departement in
Hadba-Khadra Hospital,for their continous support and en-
couragement. My inimitable thanks to my dougther Dr. Huda
for All the work she did for this book.
Special thanks to Al-Qabas Al-Jadid Co. for Media & Ad-
vertising Services - Al-Dahra - Tripoli

432

You might also like